Está en la página 1de 136

LENGUA CASTELLANA

Y LITERATURA
3.º ESO

Solucionario

Juan Carlos Pantoja Rivero


Laura Espí Jimeno
Beatriz González Gallego
Esperanza Mateos Donaire
Emilio Sales Dasí

MADRID · BUENOS AIRES · CARACAS · GUATEMALA · LISBOA · MÉXICO · NUEVA YORK


PANAMÁ · SAN JUAN · BOGOTÁ · SÃO PAULO · AUCKLAND · HAMBURGO · LONDRES
MILÁN · MONTREAL · NUEVA DELHI · PARÍS · SAN FRANCISCO · SÍDNEY · SINGAPUR
SAINT LOUIS · TOKIO · TORONTO
LENGUA CASTELLANA Y LITERATURA
Solucionario 3.º ESO

No está permitida la reproducción total o parcial de este libro, ni su tratamiento informático, ni la


transmisión de ninguna forma o por cualquier medio, ya sea electrónico, mecánico, por fotocopia,
por registro u otros métodos, sin el permiso previo y por escrito de los titulares del Copyright.
Diríjase a CEDRO (Centro Español de Derechos Reprográficos, www.conlicencia.com) si necesita
fotocopiar o escanear algún fragmento de esta obra.

Nota: Este libro se atiene al artículo 32 del derecho de cita de la Ley de Propiedad Intelectual de 1996 (RDLeg
1/1996 de 12 de Abril).

Derechos reservados © 2015, respecto a la primera edición en español, por:

McGraw-Hill/Interamericana de España, S.L.


Edificio Valrealty, 1.ª planta
Basauri, 17
28023 Aravaca (Madrid)

Código: 978-00-094-2045-0
Depósito legal: M-18149-2015

Dirección general: Álvaro García


Equipo editorial: Marilia Blanco y Cristina Núñez
Diseño de cubierta: Paula Requena
Diseño interior: Equipo de diseño de McGraw-Hill
Fotografías: 123RF y archivo McGraw-Hill
Maquetación: Diseño y Control Gráfico, S.L.U.
Impreso en:

IMPRESO EN ESPAÑA-PRINTED IN SPAIN


ÍNDICE

UNIDAD PÁGINA

Unidad 1. Los cimientos de tu lengua .................................................................................................................................................... 4

Unidad 2. El mágico poder de las palabras ......................................................................................................................................... 13

Unidad 3. Última hora ................................................................................................................................................................................... 21

Unidad 4. Libertad de expresión ............................................................................................................................................................. 30

Unidad 5. Háblame de ti .............................................................................................................................................................................. 40

Unidad 6. En la variedad está el gusto .................................................................................................................................................. 49

Unidad 7. Amar, luchar, rezar… ................................................................................................................................................................. 59

Unidad 8. Melibeo soy… .............................................................................................................................................................................. 69

Unidad 9. Coged de vuestra alegre primavera… ............................................................................................................................. 80

Unidad 10. Aquellos locos tan ingeniosos ........................................................................................................................................... 89

Unidad 11. Entre pícaros y poetas ............................................................................................................................................................ 98

Unidad 12. Todos a una ................................................................................................................................................................................. 107

Anexo. Las tipologías textuales en la lengua oral ............................................................................................................................ 117

Guías de lectura ............................................................................................................................................................................................... 118

Lengua castellana y Literatura. 1.º ESO. Solucionario 3


1 LOS CIMIENTOS DE TU LENGUA

APERTURA DE UNIDAD punto de vista: la felicidad de este mundo es artificial, una


mentira del gobierno, que controla a sus habitantes. Bernard
1. Este fragmento explica en qué consiste el cruel espec- acaba exiliándose y John finalmente sucumbe a los placeres
táculo que da nombre a la obra. También en este fragmen- del «Mundo Feliz» y acaba suicidándose.
to se explican las razones por las que ha surgido dicho Entre otros temas, el libro ataca la producción del ensamblaje
espectáculo y su finalidad. ¿Quién ha ideado los Juegos en línea como humillante, la liberación de la moral sexual
y para qué? ¿Conoces algún castigo semejante que se calificándola como una afrenta contra el amor y la familia, el
haya llevado a cabo en el mundo real en alguna época? uso de eslóganes, el concepto de un gobierno centralizado, y
El Capitolio, que representa a los opresivos gobernantes de el uso de la ciencia para controlar los pensamientos y acciones
los doce distritos. Estos distritos son los diferentes sectores de la gente. Huxley ataca a la sociedad consumista y capita-
en los que se divide y organiza una sociedad clasista. La élite lista, una sociedad vacía y frívola como la de Los Juegos del
de la sociedad vive en el Capitolio. Hambre, que se deja controlar y anular por pura comodidad
Los gobernantes del mismo han establecido estos macabros y por una terrible falta de valores.
juegos (que de lúdicos no tienen nada) para recordar a los dis- El show de Truman es una película estadounidense dirigida
tintos distritos que todo intento de sublevarse será frenado por Peter Weir en la que Jim Carrey encarna al protagonista
por el Capitolio, como ya ocurrió en otro momento de la histo- y que se estrenó en 1998.
ria. El intento frustrado de los distritos de emanciparse del El argumento de esta película gira en torno al tema de un
Capitolio es un tema tabú pero tiene un recordatorio simbólico programa de telerrealidad titulado The Truman Show, su pro-
en los Juegos del Hambre. tagonista, Truman Burbank, vive ajeno a su condición de
Hay algunos ejemplos históricos reales en que los invasores protagonista de este reality show. Las personas que él ve,
han oprimido al pueblo invadido como, por ejemplo, los roma- incluso sus familiares, no son quienes él piensa, sino actores
nos que sometieron al martirio a los cristianos. También eran de televisión. Ha sido creado para entretener al público (nació
juegos macabros en los que los cristianos eran arrojados a las de una madre de alquiler y destinado a vivir para esta serie).
bestias para divertir al pueblo romano. Los autores del guion han manejado siempre los hilos de su
2. Te proponemos que veas algunos fragmentos de la versión vida, hasta que un hecho accidental hace sospechar a Tru-
cinematográfica de esta obra, por ejemplo, el principio, man de todo cuanto le rodea. Finalmente descubre la verdad
y, gracias a su astucia, logra escapar del tiránico mundo de
hasta el momento en que comienzan los Juegos propia-
cartón-piedra en el que ha vivido desde siempre.
mente dichos. Después puedes entablar un debate con tus
compañeros en torno a estas preguntas: VIVE LA LECTURA
a) Analiza el tipo de sociedad que retrata la escritora.
¿En qué se parece y en qué se diferencia de la tuya Comprensión lectora
propia? 1. ¿Quién protagoniza este fragmento? ¿Te parece original
b) ¿Qué te parece el planteamiento de esta obra? el planteamiento del cuento? ¿Por qué?
c) ¿Qué opinas sobre la adaptación al cine de esta parte Un insecto, concretamente una mosca con la peculiar carac-
de la obra? ¿Crees que existe algún aspecto mejorable? terística de que tiene inteligencia y ha sido instruida por un
sabio.
d) ¿Qué te parece la idea de emplear un reality show
como castigo? 2. ¿En qué persona se narra la historia? Indica en qué párra-
Respuesta libre.
fos habla el personaje principal.
3. Busca más información sobre la autora de esta obra y su Se narra en primera persona, pero no se trata de una primera
persona autobiográfica sino de un narrador testigo. La mosca
producción novelística. La idea de una sociedad futura que
habla en los párrafos dos, tres y cuatro, y en estos párrafos
plantea su novela recuerda a la que retrata Aldous Huxley es la mosca la que utiliza la primera persona autobiográfica.
en Un mundo feliz y a una película que aborda los reality
shows: El show de Truman. Busca información sobre ambas 3. Resume su historia vital.
y establece similitudes y diferencias. La mosca nació en la biblioteca del sabio Macrocéfalo, quien la
Un mundo feliz es la novela más brillante del escritor británico capturó en su infancia y la sometió a múltiples experimentos.
Aldous Huxley. La obra recrea una sociedad del futuro en la Los resultados de todos ellos contradecían las teorías del sabio
que la tecnología ha avanzado sobre todo en dos direcciones: y este quiso matar a la mosca por despecho, pero no encontró el
la reproducción humana y la hipnopedia. La combinación de valor necesario y, finalmente, le permitió la vida y la aleccionó.
ambas da como resultado una sociedad de autómatas deshu- 4. Explica todos los datos que se ofrecen en la lectura sobre
manizados. El ser humano vive en un mundo mediocre y sin el sabio Macrocéfalo y describe cómo te lo imaginas.
pasiones, en donde la química ayuda a controlar los estados de Don Eufrasio Macrocéfalo se describe por la mosca más como
ánimo y donde no hay contacto físico, ni siquiera para procrear. un filósofo que como un sabio. Lo que parece claro en este
Esta forma de vida puede parecer en principio una utopía, pero fragmento es que es un hombre indeciso y algo orgulloso.
al final resulta una caricatura de un mundo excesivamente tec-
nológico y frío. Es, en realidad, una burla y una crítica al hom- 5. ¿Crees que este fragmento constituye una crítica o una
bre moderno. Bernard Max, por su inteligencia, no responde al alabanza a los sabios? Aporta alguna afirmación del texto
condicionamiento psicológico a que son sometidos todos los que sirva de ejemplo a tu razonamiento.
humanos, de manera que empieza a pensar por sí mismo y se Es una crítica a algunos sabios que parecen tener muchos co-
convierte en un inadaptado social (un antisistema). nocimientos pero su ciencia no va más allá del puro discurrir.
La trama se complica cuando Bernard visita la reserva, junto De hecho el apellido Macro- («grande») -céfalo («cabeza») es
con su frívola compañera, Lenina, lugar en el que habitan se- un apelativo burlesco. Se aprecia en esta cita: Y de cuestión
res vivos en estado salvaje (sin condicionar) y conoce a John en cuestión, don Eufrasio llegó al punto de partida necesario
el Salvaje, nacido por el método reproductivo natural. Este para dar un solo paso en firme. Todo esto le ocupó muchos
personaje visita el «Mundo Feliz» de Bernard y plantea su meses, que fueron dilatando el plazo de mi muerte.

4 Lengua castellana y Literatura. 3.º ESO. Solucionario


LOS CIMIENTOS DE TU LENGUA 1
6. Señala si las siguientes afirmaciones te parecen verda- La epopeya es un subgénero de la narrativa escrito en verso
deras o falsas: que cuenta la historia de pueblos y naciones. Las más antiguas
son la mesopotámica (Epopeya de Gilgamesh) y las grecola-
a) Macrocéfalo es el nombre de la mosca que protago-
tinas (Ilíada, Odisea y Eneida).
niza este relato.
La Ilíada es una epopeya griega atribuida al poeta Homero,
b) Don Eufrasio enseña a hablar a la mosca porque se que vivió hacia el siglo VIII a. C. Es un extenso poema inte-
siente muy solo. grado por más de quince mil versos y cuya acción desarrolla
c) La mosca nació en libertad, pero don Eufrasio la atra- una serie de acontecimientos en torno al asedio de Troya por
pó para hacer experimentos con ella. parte de los griegos.
d) La mosca ha leído muchos de los libros de la biblioteca El héroe que protagoniza el relato es Aquiles, que abandona el
y es políglota. combate por estar en desacuerdo con el reparto de un botín.
Este solo vuelve a la lucha para vengar la muerte de su amigo
e) Don Eufrasio pretendía atravesar la mosca con una agu- Patroclo, al que ha matado el príncipe troyano Héctor.
ja, pero sus pensamientos filosóficos le apartaron de
12. En este cuento, Clarín nos relata cómo el narrador (un es-
tal decisión.
tudioso que había ido a consultar la biblioteca del sabio Eu-
a) Falsa. c) Verdadera. e) Verdadera.
frasio Macrocéfalo) saca a la mosca de excursión al campo.
b) Falsa. d) Verdadera. Inventa los acontecimientos que tienen lugar durante dicha
7. ¿En qué consiste la teoría cartesiana que Macrocéfalo que- excursión y construye así un final de unas veinte líneas para
ría comprobar? ¿Por qué no logra demostrar dicha teoría? la historia.
¿Cómo repercute el hecho de que no pueda comprobar Respuesta libre.
dicha teoría en la vida de la protagonista? 13. ¿Crees que este cuento pertenece al subgénero narrativo
En que los animales son como máquinas. Porque la propia de la fábula? Busca información sobre dicho género y
inteligencia que empieza a mostrar la mosca, aprendiendo
ofrece tu respuesta razonada.
todo lo que Macrocéfalo le enseña, corrobora lo contrario. Su
inteligencia la aleja de la máquina y la acerca al ser humano. No. Aunque está protagonizada por un animal y este comparte
Lo que la salva es que, al parecerse a los humanos en este con los seres humanos la capacidad para el diálogo y la inte-
aspecto, a don Eufrasio le aborda una duda moral: si tiene o ligencia, no tiene una finalidad ejemplar o ejemplarizante y
no derecho a matar a la mosca. carece de una moraleja.

ESTUDIO DE LA LENGUA
Vocabulario
1. Señala en el siguiente microrrelato qué fonemas están
8. ¿Qué quiere decir la mosca cuando afirma que es «políglo-
representados por varias letras:
ta»? Busca la etimología y el significado de esta palabra.
Estás en casa, y es de noche, apagas la última luz. Qué extra-
La palabra políglota viene del griego poliglotos formada por
ño: de pronto todo desaparece.
polis («muchas») y glotos («lengua»). Se refiere a una persona
Pedro UGARTE
–en este caso una mosca– que habla y domina varios idiomas.
/k/  c (+ a, o, u), k, qu (+ e, i)
9. Explica el significado de la expresión «meter baza» y aporta
/g/  (+ a, o, u), gu (+ e, i), w
sinónimos tanto cultos como coloquiales. Luego, escribe
una oración con cada uno. /z/  c (+ e, i), z
Meter baza significa intervenir en una conversación ajena de 2. En este otro microrrelato, se menciona una clepsidra y un
modo inesperado. Sinónimos cultos: discrepar, interrumpir, cíclope. Identifica el significante, el significado y el referen-
intervenir o entrometerse. Sinónimo coloquial: meter las te (aporta una imagen) de cada signo lingüístico.
narices. Perseguido por tres libélulas gigantes, el cíclope alcanzó el
10. Busca en el diccionario la definición de estas palabras. centro del laberinto, donde había una clepsidra. Tan sediento
Escribe con ellas un pequeño texto fantástico cuya pro- estaba que sumergió irreflexivamente su cabeza en las aguas
tagonista sea una hormiga. de aquel reloj milenario. Y bebió sin mesura ni placer. Al apu-
rar la última gota, el tiempo se detuvo para siempre.
quinqué canija baldada
Javier P UCHE
ascendientes suplicio entrañas Significante: c-l-e-p-s-i-d-r-a.
Quinqué: lámpara de mesa alimentada con petróleo y provista Significado: reloj de agua.
de un tubo de cristal que resguarda la llama.
Referente:
Ascendientes: padre, madre o cualquiera de los abuelos, de
quienes desciende una persona.
Canija: débil y enfermiza.
Suplicio: lesión corporal, o muerte, infligida como castigo.
Baldada: cansada, fatigada.
Entrañas: órganos contenidos en las principales cavidades del
cuerpo humano y de los animales. Lo más íntimo y esencial
de un asunto. Lo más oculto y escondido.

Investigación y redacción
11. Busca información sobre el género de la epopeya y sobre
la Ilíada, a la que hace referencia la mosca. Explica el para-
lelismo que se establece entre esta y la Batracomiomaquia
y la Mosquea, que aluden a los batracios y las moscas.

Lengua castellana y Literatura. 3.º ESO. Solucionario 5


1 LOS CIMIENTOS DE TU LENGUA

Significante: c-i-c-l-o-p-e. c) bastante decepcionado


Significado: gigante de la mitología griega con un solo ojo. d) después de las siete
Referente: e) llueve torrencialmente
a) Nominal. c) Adjetival. e) Verbal.
b) Adjetival. d) Adverbial.
6. Di a qué tipo de sintagma pertenecen los que aparecen
en negrita en los siguientes refranes:
a) Más rápido se coge al mentiroso que al cojo.
b) Al que a buen árbol se arrima buena sombra le cobija.
c) Más vale pájaro en mano que ciento volando.
a) Adverbial y preposicional.
b) Nominal.
c) Preposicional.
7. Indica la modalidad de las siguientes oraciones:
a) Dile a tu hermana que venga a verme.
b) ¿Podríamos comer lentejas hoy, por favor?
3. Lee las siguientes frases célebres y realiza el análisis mor- c) Los libros son el mejor amigo de los aventureros.
fológico de las palabras resaltadas en negrita. Aporta
todos los datos que conozcas (tipo de palabra, género, d) No toques esa piedra de ahí.
número, grado, tiempo, etc.). e) ¿Quién de vosotras tiene el retrato? ¿Quién?
a) La diferencia entre estupidez y genialidad es que la a) Apelativa, exhortativa.
genialidad tiene sus límites (Albert Einstein). b) Apelativa, exhortativa. Aunque tiene forma de interroga-
b) Recordar es fácil para el que tiene memoria. Olvidar tiva, en el fondo es un ruego.
es difícil para el que tiene corazón (Gabriel García c) Enunciativa afirmativa.
Márquez). d) Apelativa, exhortativa.
c) La raza humana tiene un arma verdaderamente efi- e) Interrogativa.
caz: la risa (Mark Twain). 8. Escucha la canción de Pablo Alborán Solamente tú y ana-
d) No entres allá de donde no puedas libremente salir liza la modalidad de cada oración.
(Mateo Alemán). • Regálame tu risa (exhortativa).
e) La educación es el arma más poderosa que puedes • Enséñame a soñar (exhortativa).
usar para cambiar el mundo (Nelson Mandela).
• Con solo una caricia me pierdo en este mar (enunciativa
a) Diferencia: sustantivo común, abstracto, femenino, singu- afirmativa).
lar. Genialidad: sustantivo común, abstracto, femenino,
• Regálame la estrella, la que ilumina esta noche, llena de
singular. Tiene: verbo de la segunda conjugación, tercera
paz y de armonía, y te regalaré mi vida (exhortativa).
persona del singular del presente de indicativo. Voz activa.
Sus: determinante posesivo, tercera persona del plural. • Haces que mi cielo vuelva a tener ese azul, pintas de colores
b) Para: preposición. El: determinante artículo determina- mis mañanas solo tú (enunciativa afirmativa).
do, masculino, singular. Difícil: adjetivo calificativo de • Navego entre las olas de tu voz y tú, y solamente tú, haces
una terminación, singular. Que: conjunción. Corazón: que mi alma se despierte con tu luz (enunciativa afirmativa).
sustantivo, común, concreto, contable, individual, mascu- • Enseña tus heridas y así las curará. Que sepa el mundo
lino, singular. (Aquí está usado en sentido figurado, así entero que tu voz guarda un secreto. No menciones tu
que es abstracto porque se refiere a sentimientos, que es nombre que en el firmamento se mueren de celos (exhor-
abstracto). tativa).
c) Humana: adjetivo calificativo, femenino, singular. Ver- • Tus ojos son destellos, tu garganta es un misterio (enuncia-
daderamente: adverbio de modo. Eficaz: adjetivo cali- tiva afirmativa).
ficativo de una terminación, singular.
9. Escribe una carta a alguien a quien aprecies mucho. Uti-
d) Allá: adverbio de lugar. No: adverbio de negación. Sa- liza oraciones de todas las modalidades.
lir: verbo de la tercera conjugación. Forma no personal:
infinitivo. Respuesta libre.
e) Educación: sustantivo común, abstracto, femenino, sin-
gular. Poderosa: adjetivo calificativo, femenino, singular. EL TALLER DE LAS PALABRAS
4. Crea un consejo para esquivar la soledad en forma de 1. Descompón en monemas las siguientes palabras:
oración contundente como las de la actividad 3. Realiza reinserción idealizar pájaros juguetón
su análisis morfológico completo.
arboledas lavaremos abrasador aniñado
Respuesta libre.
• re (morfema derivativo, prefijo) -inser- (lexema) -ción
5. Indica de qué tipo son los siguientes sintagmas: (morfema derivativo, sufijo)
a) el lunes de la próxima semana • idea- (lexema) -al- (morfema derivativo, sufijo) -izar (mor-
b) cansado de todo fema derivativo, sufijo)

6 Lengua castellana y Literatura. 3.º ESO. Solucionario


LOS CIMIENTOS DE TU LENGUA 1
• pájar- (lexema) -o-s (morfemas flexivos) • Puleva (acrónimo de Pura Leche de Vaca)
• juguet (lexema) -ón (morfema derivativo, sufijo aumen- • AMPA (siglas de Asociación de Madres y Padres de Alum-
tativo) nos)
• árbol- (lexema) -eda (morfema derivativo, sufijo) • ICONA (acrónimo de Instituto Nacional para la Conserva-
• lav- (lexema) -aremos (morfemas flexivos) ción de la Naturaleza)
• a- (morfema derivatvo, prefijo) -bras- (lexema) -ador (mor- • FM (siglas de Frecuencia Modulada)
fema derivativo, sufijo) • Malena (acrónimo de Magdalena o María Elena)
• a- (morfema derivativo, prefijo) -niñ- (lexema) -ado (mor- • BBVA (unión de las siglas BBV, Banco Bilbao-Vizcaya y
fema derivativo, sufijo) Argentaria)
2. Forma nuevas palabras a partir de las que te damos aña- • láser (acrónimo compuesto por las siglas: light amplifica-
diendo estos sufijos: -nte, -ero, -dor, -azo, -on. tion by stimulated emision of radiation, es decir, amplia-
ción de la luz por emisión estimulada de radiación)
gol pesca hablar comer ropa
• OMS (siglas de Organización Mundial de la Salud)
bala acoger casa presidir sal
• sida (siglas de síndrome de inmunodeficiencia adquirida)
• golazo • pescador • hablador • comedor
• Feve (acrónimo de Ferrocarriles de Vía Estrecha)
• ropero • balazo • acogedor • casero
• presidente • salero EL RINCÓN DE LA NORMA
3. ¿Qué sufijos de la actividad anterior cambian la categoría 1. Añade tilde donde sea necesario. Si dudas, consulta el
gramatical de la palabra a la que se añaden? Pon otros diccionario.
ejemplos.
Corrige
-dor (orar, orador), -nte (pender, pendiente).
4. Añade uno de estos prefijos a las siguientes palabras para dijeron medula damelo margen arbol
formar su antónimo: i-, in-, anti-, des-. telegrafo jamas Oscar iran cogi
responsable congelante descriptible habitado carcel sali ambar patena ambiguo
empleo oportuno confiado lógico • dijeron • médula • dámelo • margen • árbol
• irresponsable • anticongelante • indescriptible • telégrafo • jamás • Óscar • irán • cogí
• deshabitado • desempleo • inoportuno • cárcel • salí • ámbar • patena • ambiguo
• desconfiado • ilógico 2. Lee detenidamente el siguiente fragmento y añade tilde
5. Forma palabras con los lexemas y morfemas derivativos donde corresponda:
siguientes, indicando qué procedimiento has seguido en
cada caso. Corrige
boca- compra- -gras- -legal -ista —Es enorme… —murmure—. ¿Qué hay al otro lado?

des- para- -alm- -tensión -ado —Unos dicen que el oceano no se acaba nunca —repuso
Erik—; otros aseguran que al final del horizonte se en-
hipo- a- -caid- -añ- -ar cuentra la magica isla de Hy Brasil, y hay quien afirma
cumple- en- -grand- -venta -ada que la Tierra es redonda como una naranja y que más alla
del mar se encuentran las Indias, Catay y Cipango. Pero
i- perr- -calle -ito -era lo unico cierto es que nadie lo sabe, porque nadie ha ido
alli para comprobarlo.
• hipotensión • ilegal • desalmado Sonrei ante la absurda idea de que la Tierra fuera redon-
• compraventa • perrito • bocacalle da —de ser asi, los que estan abajo se caerían, ¿no es
cierto?—.
• cumpleañera • agrandar • paracaidista
César M ALLORQUÍ: La catedral.
• engrasada
6. Escribe juntos o separados los siguientes términos unidos —Es enorme… —murmuré—. ¿Qué hay al otro lado?
por composición: —Unos dicen que el océano no se acaba nunca —repuso
cartón + piedra para + choques Erik—; otros aseguran que al final del horizonte se encuentra
la mágica isla de Hy Brasil, y hay quien afirma que la Tierra
perro + lobo recién + casado
es redonda como una naranja y que más allá del mar se en-
guía + catálogo coche + patrulla cuentran las Indias, Catay y Cipango. Pero lo único cierto es
medio + día sofá + cama que nadie lo sabe, porque nadie ha ido allí para comprobarlo.
• cartón piedra • perro lobo Sonreí ante la absurda idea de que la Tierra fuera redonda
—de ser así, los que están abajo se caerían, ¿no es cierto?—.
• catálogo guía • mediodía
• parachoques • recién casado
3. Explica la regla de acentuación que se cumple en las pa-
labras acentuadas:
• coche patrulla • sofá cama
a) Déjame que te diga una cosa: así no llegarás a ninguna
7. Explica qué significan estas siglas y acrónimos: parte, Óscar.
Puleva AMPA ICONA FM Malena b) Los íberos llegaron a la península ibérica antes de la
BBVA láser OMS sida Feve invasión romana.

Lengua castellana y Literatura. 3.º ESO. Solucionario 7


1 LOS CIMIENTOS DE TU LENGUA

a) Déjame (palabra esdrújula) que te diga una cosa: así (agu- es la consecuencia final de un daño causado por hepatopatía
da acabada en vocal) no llegarás a ninguna parte, Óscar crónica. Las consecuencias dependen fundamentalmente del
(llana que acaba en consonante diferente a n o s). grado de funcionalidad que el hígado pueda conservar a pesar
b) Los íberos (esdrújula) llegaron a la península (esdrújula) de la alteración histológica.
ibérica (esdrújula) antes de la invasión (aguda acabada En este texto falla la adecuación. El tema es lo suficientemen-
en n) romana. te serio para ser tratado con un médico especialista, no con
4. Busca errores de acentuación en tu entorno: carteles, anun- un auténtico desconocido. Además sería recomendable tratar
cios, rótulos o luminosos. Anota cinco de ellos y ponlos en el asunto en persona, no en un chat. El contexto situacional
no es el más adecuado porque las tecnologías nos acercan
común con tus compañeros.
la información, pero en muchos casos esa información no es
Respuesta libre. fiable y hay que filtrar los datos que obtenemos. Por otro lado,
5. Separa en sílabas estas palabras y coloca la tilde donde emisor y receptor no comparten el mismo conocimiento del
sea necesario: caso, pues la información se presenta sin una contextualiza-
ción previa. No sabemos los síntomas, la edad del paciente del
Corrige que se habla, qué pruebas se le han realizado ni los resultados
de las mismas. Es imprudente adelantar un diagnóstico en una
huesped sombrio aeronave doscientos situación tan inconcreta.
buho ruido vagais acuatico Receptor y emisor no comparten el mismo registro y segura-
heroico Luis cooperacion llovia mente no tienen el mismo grado de conocimientos médicos.
conociais cacatua Uruguay cienaga Por eso la primera persona que interviene escribe trankilos
y dice que el término cirrosis hepática le suena muy feo (ad-
nauseabundo mienteme saliesen construid verbialización del adjetivo). Se ve que es una persona con un
nivel cultural bajo y ningún conocimiento de medicina. No es
• hués-ped • som-brí-o • ae-ro-na-ve
adecuado que la persona que contesta use tantos tecnicismos
• dos-cien-tos • bú-ho • rui-do del campo de la medicina. Además no es coherente porque
• va-gáis • a-cuá-ti-co • he-roi-co si la persona que pregunta no entiende lo que es cirrosis
• Luis • co-o-pe-ra-ción • llo-ví-a hepática tampoco sabrá qué significa tejido hepático, he-
patopatía crónica o funcionalidad, por no decir alteración
• co-no-cí-ais • ca-ca-tú-a • U-ru-guay
histológica.
• cié-na-ga • nau-se-a-bun-do • mién-te-me
Tampoco es coherente por parte del emisor que pida que
• sa-lie-sen • cons-truid alguien (pronombre indefinido) conteste a su duda, dando a
6. Conjuga en tu cuaderno estos tiempos de los verbos ver, entender que da igual quién sea y el grado de conocimiento
oír, aplaudir: condicional simple, pretérito imperfecto de que tenga sobre el tema. Y resulta incoherente que pida esta
indicativo y presente de subjuntivo. información a quien quiera contestarla para ganar tranqui-
lidad. Por último, tampoco es coherente que si la persona
Condicional simple: vería, oiría, aplaudiría.
que pregunta busca tranquilidad, la persona que contesta lo
Pretérito imperfecto de indicativo: veía, oía, aplaudía. haga con tecnicismos incomprensibles para cualquier perso-
Presente de subjuntivo: vea, oiga, aplauda. na ajena a la medicina, pues es normal que se intranquilice en
lugar de encontrar la serenidad que busca. La última incohe-
7. Subraya y clasifica los diptongos, triptongos e hiatos de rencia de este «médico» es que recomienda calma (aunque
este texto: no contribuye decisivamente a ella), hasta que un médico
Apelo al lector, para que él diga si hay indicios de demencia en profesional interprete los resultados. Da a entender con ello
el estilo o en el método de la historia que ha estado leyendo. que él no es el más indicado para contestar porque no es ese
Soy un hombre joven encerrado en el cuerpo de un viejo. Pero profesional.
a todo el mundo le resulta increíble la innegable realidad de
2. Busca en chats, foros, blogs o mensajes de texto de móvi-
este hecho.
les otros textos que no posean alguna de las propiedades.
H. G. WELLS: El caso del difunto señor Elvesham.
Respuesta libre.
Apelo al lector, para que él diga si hay (diptongo) indicios (dip-
tongo) de demencia (diptongo) en el estilo o en el método de la 3. Explica por qué canal crees que se difundiría este peque-
historia (diptongo) que ha estado leyendo. Soy (diptongo) un ño texto y cuál es su intención comunicativa:
hombre joven encerrado en el cuerpo de un viejo (diptongo). Con motivo de la jornada de desinfección prevista para hoy,
Pero a todo el mundo le resulta increíble (hiato) la innegable las instalaciones de la piscina cubierta permanecerán cerra-
realidad (hiato) de este hecho. das hasta mañana jueves. Muchas gracias y disculpen las
molestias.
LA FACTORÍA DE TEXTOS Canal oral o escrito. Según la intención comunicativa es
1. Observa el siguiente texto extraído de un chat y señala informativo.
cuál de las propiedades del texto crees que falla aquí. 4. Escribe un texto que posea adecuación, coherencia y co-
—A mi suegro le han hecho una serie de pruebas y en una de hesión y que se adapte a las siguientes características: su
ellas han determinado que tiene cirrosis hepática. ¿Alguien forma será la de una narración, su intención comunicativa
podría explicarme qué es eso para que nos quedemos tranki- consistirá en informar, debe pertenecer al ámbito periodís-
los? Me suena muy feo. tico y su canal de comunicación debe ser escrito.
—En todo caso hay que mantener la calma hasta que un médi- Respuesta libre. Debe tenerse en cuenta que lo que se está
co profesional interprete los resultados. Os explico en qué con- pidiendo al alumno es que redacte una noticia de un suceso
siste la enfermedad: la cirrosis que afecta al tejido hepático con la finalidad de informar, no de opinar.

8 Lengua castellana y Literatura. 3.º ESO. Solucionario


LOS CIMIENTOS DE TU LENGUA 1
ACTIVIDADES FINALES b) Respuesta libre.
Repasa lo que has aprendido c) Encima (adverbio), de (preposición), todo (pronombre in-
definido), había (verbo), un (determinante artículo inde-
1. Lee el siguiente texto y resuelve las cuestiones que se terminado, masculino, singular), papel (sustantivo común,
proponen a continuación. concreto, contable, masculino, singular), enrollado (par-
Encima de todo había un papel enrollado. Cuando lo desen- ticipio funcionando como adjetivo). Cuando (adverbio), lo
rollaron, vieron que estaba escrito con una caligrafía difícil (pronombre), de senrollaron (verbo), vieron (verbo), que
de descifrar. Sin embargo, Annika dijo que podría entenderlo. (conjunción) estaba (verbo), escrito (participio, adjetivo)
—Léenoslo —le pidió David. con (preposición) una (determinante artículo indetermi-
nado, femenino, singular) caligrafía (sustantivo común,
Y ella empezó:
concreto, incontable, femenino, singular) difícil (adjetivo
«Hoy es 30 de junio de 1763. Acabo de oír al reloj de abajo dar calificativo, femenino, singular) de (preposición) desci-
las ocho». frar (verbo). Sin embargo (locución conjuntiva), Annika
Annika enmudeció y miró a David. También hoy era 30, y aca- (sustantivo propio) dijo (verbo) que (conjunción) podría
baban de oír, igualmente, dar las ocho en el reloj de abajo… entender (verbo) lo (pronombre enclítico).
Jonás abrió unos ojos como platos. ¡Eso sí que era sorprenden- d) • desenrollaron: des- (morfema derivativo, prefijo) -en-
te! Conectó el magnetofón. ¡Había que leer el escrito en voz (morfema derivativo, prefijo) -roll- (lexema) -aron (mor-
alta para grabarlo! El papel temblaba en las manos de Annika. femas flexivos).
Lo agarró fuertemente y prosiguió leyendo: • descifrar: des- (morfema derivativo, prefijo) -cifr- (le-
«Estoy sentada frente a la ventana. Es el tiempo en que flo- xema) -ar (morfema derivativo, sufijo).
recen los tilos, y quisiera abrir la ventana y sentir el olor de • caligrafía: cali- (lexema griego, hermoso) -grafía (le-
las flores… Pero ya no me quedan fuerzas. Sé que no viviré xema griego, escritura).
mucho tiempo. Aunque eso no me preocupa.
• enmudeció: en- (morfema derivativo, prefijo) -mud- (le-
»Delante de mí, encima de la mesa, tengo el estuche que hizo
xema) -ec- (morfema derivativo, sufijo) -ió (morfemas
para mí Andreas cuando cumplí catorce años. Cuando con-
flexivos).
templo mis ojos y veo mi cara en el espejo que hay en la parte
interior de la tapa, pienso y deseo que se queden ahí y que • sorprendente: sorprend- (lexema) -ente (morfema deri-
se encuentren con los ojos de aquel que un día encontrará vativo, sufijo).
este estuche y lo abrirá. ¡Cómo me gustaría mirar esos ojos, y • florecen: flor- (lexema) -ec- (morfema derivativo, sufijo)
conocer el corazón de esa persona y sus sentimientos!». -en (morfemas flexivos).
María GRIPE: Los escarabajos vuelan • sentimientos: senti- (lexema) -miento- (morfema deri-
al atardecer (Editorial SM). vativo, sufijo) -s (morfema flexivo).
a) Indica a qué niveles de la lengua pertenecen las si- e) Están representadas el verbo (lee) y los pronombres (nos
guientes unidades: y lo).
• La novela de la que se ha extraído este fragmento. f) Enunciativas: Encima de todo había un papel enrollado.
Cuando lo desenrollaron, vieron que estaba escrito con
• Los fonemas que componen la palabra encima. una caligrafía difícil de descifrar.
• Este fragmento de la novela. Exclamativas: ¡Eso sí que era sorprendente! ¡Había que
• Annika. leer el escrito en voz alta para grabarlo!
• Léenoslo. Desiderativa: ¡Cómo me gustaría mirar esos ojos, y cono-
• Unos ojos como platos. cer el corazón de esa persona y sus sentimientos!
g) Sí, es un verdadero texto. Es adecuado porque tiene en
b) Busca en el texto un sintagma de cada tipo.
cuenta al lector y va distribuyendo la información de ma-
c) Analiza morfológicamente todas las palabras del pri- nera que se genere el suspense propio de una novela de
mer párrafo. misterio, como es esta.
d) Descompón en monemas las siguientes palabras: de- También es adecuado el registro culto y el léxico que utili-
senrollaron, descifrar, caligrafía, enmudeció, sorpren- za para el canal escrito propio de la transmisión de obras
dente, florecen, sentimientos. literarias.
e) ¿Qué categoría o categorías gramaticales están re- La coherencia se ha conseguido desarrollando la narra-
presentadas en la expresión Léenoslo? Justifica tu res- ción en torno a un tema y unas claves. Además el texto
puesta. está dotado de una estructura clara, en donde las ideas se
suceden de una forma lógica, progresiva y cohesionada.
f) Busca en el texto dos oraciones enunciativas, dos ex- Esto último se consigue gracias al uso de los pronombres,
clamativas y una desiderativa. que evitan la repetición de conceptos, el uso de determi-
g) ¿Constituye este un buen ejemplo de texto? ¿Reúne las nantes posesivos y la sustitución léxica.
propiedades necesarias? Razona tu respuesta y aporta 2. Explica la regla de acentuación que se cumple en las si-
ejemplos claros. guientes palabras: difícil, léenoslo, empezó, enmudeció,
a) • Es un texto. Jonás, sí, magnetofón.
• Son fonemas. • difícil: es llana y acaba en consonante distinta de n o s.
• Es un texto. • léenoslo: sobresdrújula (se acentúa siempre).
• Es una palabra. • empezó: es aguda y acaba en vocal.
• Es una oración. • enmudeció: ídem.
• Es un sintagma. • Jonás: es aguda y acaba en s.

Lengua castellana y Literatura. 3.º ESO. Solucionario 9


1 LOS CIMIENTOS DE TU LENGUA

• sí: es una palabra monosílaba. Lleva tilde para diferenciar- 8. Inventa frases como eslogan para promocionar los siguien-
se de la conjunción homónima si. tes productos.
• magnetofón: es aguda y acaba en n. Respuesta libre.
3. Define con tus palabras cada una de las unidades de la 9. Indica a qué tipo de sintagma corresponden los siguientes
lengua representativas de cada nivel y pon un ejemplo y marca su núcleo:
tomado del texto.
a) algunas tardes de verano
Respuesta libre.
b) felizmente casados
4. Escucha atentamente una noticia en un informativo de
c) bastante lejos del parque
televisión. Después resume su contenido e indica de qué
tipo de texto se trata según su forma, intención comuni- d) cada día de lluvia
cativa, ámbito de uso y canal razonando tus afirmaciones. e) caminó lentamente
Haz lo mismo con el texto de Los Juegos del Hambre que f) en la estación de tren
abre la unidad.
g) tus ojos azules
Respuesta libre.
a) algunas tardes de verano (SN)
5. Lee el siguiente poema y responde a las cuestiones que
se formulan sobre él: b) felizmente casados (SAdj)

La palabra c) bastante lejos del parque (SAdv)

La palabra pregunta y se contesta d) cada día de lluvia (SN)


tiene alas o se mete en los túneles e) caminó lentamente (SV)
se desprende de la boca que habla f) en la estación de tren (SPrep, no tiene núcleo propio, pero
y se desliza en la oreja hasta el tímpano estación es núcleo del SN que hay dentro).
la palabra es tan libre que da pánico g) tus ojos azules (SN)
divulga los secretos sin aviso
e inventa la oración de los ateos 10. Escribe los siguientes tipos de textos haciendo uso de la
es el poder y no es el poder del alma modalidad indicada en cada caso:
y el hueso de los himnos que hacen patria […] a) Una pequeña nota para hacer un pedido en la que
la palabra es correo del amor uses el modo imperativo.
pero también es arrabal del odio b) Una reclamación a Papá Noel porque te ha regalado
golpea en las ventanas si diluvia un objeto absurdo en la que uses la modalidad inte-
y el corazón le abre los postigos rrogativa.
y ya que la palabra besa y muerde c) Unas instrucciones de uso de un exprimidor en las que
mejor la devolvemos al futuro
uses el modo imperativo.
Mario BENEDETTI: Inventario cuatro.
d) Una pequeña narración o microrrelato en el que uses
a) ¿Cómo describe el autor la naturaleza de las palabras? la modalidad enunciativa.
¿Te da la sensación de que las siente como algo débil
e) Una pequeña carta expresando tus deseos de que un
o como algo poderoso?
enfermo se recupere en la que uses oraciones desi-
b) Inventa tu propia definición en prosa o en verso sobre derativas.
la palabra.
f) Un mensaje publicitario en el que uses la modalidad
c) Busca en el poema tres sustantivos, cinco verbos, dos exhortativa.
adjetivos, dos preposiciones y dos conjunciones. Respuesta libre.
a) La palabra se define como algo vivo e inquieto, como una
realidad contradictoria pero con mucho poder. Es algo má- MIRA A TU ALREDEDOR Y…
gico. Se siente en todos los versos como algo poderoso,
capaz de cualquier imposible. … ve más allá
b) Respuesta libre. 1. El siguiente texto es una exposición literaria sobre cómo
c) Sustantivos: palabra, alas, túneles.
subir escaleras. Léelo con atención y observa si se cumplen
en él y de qué manera las propiedades textuales (adecua-
Verbos: pregunta, contesta, tiene, mete, desprende.
ción, coherencia y cohesión).
Adjetivos: libre, ateos.
Las escaleras se suben de frente, pues hacia atrás o de costado
Preposiciones: en, de. resultan particularmente incómodas. La actitud natural con-
Conjunciones: o, y. siste en mantenerse de pie, los brazos colgando sin esfuerzo,
la cabeza erguida aunque no tanto que los ojos dejen de ver
6. Observa las siguientes imágenes extraídas de la prensa los peldaños inmediatamente superiores al que se pisa, y res-
y formula en un sintagma nominal un título que resuma la pirando lenta y regularmente. Para subir una escalera se co-
información: mienza por levantar esa parte del cuerpo situada a la derecha
Respuesta libre. abajo, envuelta casi siempre en cuero o gamuza, y que salvo
excepciones cabe exactamente en el escalón. Puesta en el pri-
7. Los sintagmas que has formulado en la actividad anterior
mer peldaño dicha parte, que para abreviar llamaremos pie, se
¿constituyen enunciados u oraciones? Justifica tu res-
recoge la parte equivalente de la izquierda (también llamada
puesta. pie, pero que no ha de confundirse con el pie antes citado), y
Respuesta libre. llevándola a la altura del pie, se le hace seguir hasta colocarla

10 Lengua castellana y Literatura. 3.º ESO. Solucionario


LOS CIMIENTOS DE TU LENGUA 1
en el segundo peldaño, con lo cual en este descansará el pie, transmitirse por escrito y su intención comunicativa es la
y en el primero descansará el pie. (Los primeros peldaños de producir belleza.
son siempre los más difíciles, hasta adquirir la coordinación b) Ni el texto ni la guitarra reflejan la realidad de una manera
necesaria. La coincidencia de nombre entre el pie y el pie hace transparente, es decir, no imitan la realidad tal y como es,
difícil la explicación. Cuídese especialmente de no levantar sino que descomponen dicha realidad y la reinventan, de
al mismo tiempo el pie y el pie). Llegando en esta forma al se- manera que no parece existir una asociación directa entre
gundo peldaño, basta repetir alternadamente los movimientos el poema o la pintura y la realidad a la que se refiere. En
hasta encontrarse con el final de la escalera. Se sale de ella ambas creaciones parece romperse la coherencia porque
fácilmente, con un ligero golpe de talón que la fija en su sitio, se transgrede la lógica, pero la originalidad de ambas
del que no se moverá hasta el momento del descenso. obras de artes consiste en seguir expresando sentimientos
Julio CORTÁZAR: Historias de cronopios y famas. y emociones a pesar de esa transgresión.
Aparentemente no se cumplen todas las propiedades textua- c) Uno de los movimientos europeos de vanguardia que se
les. La adecuación parece no cumplirse por el hecho de que el dieron en el periodo de entreguerras (1910-1940) y que fue
emisor (el autor) no tienen en cuenta los conocimientos sobre menos pasajero que el resto. Este movimiento artístico
el mundo que posee el receptor (el lector), pues todo el mundo (primero pictórico y luego literario) supuso un cambio
sabe subir una escalera. Parece no tener en cuenta que cual- radical en la concepción del papel del arte y del trabajo
quier lector sabe que eso que va envuelto en cuero o gamuza es del artista. El paso decisivo en su creación lo dará André
un pie, etc. ¿Pero es cierta esa falta de conciencia sobre lo que Breton, tras leer a Sigmund Freud.
el lector sabe? Evidentemente, representa un juego literario al El surrealismo no se presenta solo como una renovación
que el autor recurre en busca de provocación y originalidad. estética, sino que pretende ser una revolución integral. Su
Lo mismo ocurre con la coherencia. En principio no parecen lema sería la frase de Rimbaud: «transformar la vida», y en
coherentes las recomendaciones absurdas (por ser demasia- ese sentido se produce un «encuentro» entre las doctrinas
do cotidianas) para subir las escaleras, como hacerlo de pie, de Freud y las de Marx.
subiendo primero el pie derecho a un peldaño y luego el iz- Para los surrealistas, la vida es la cara más gris de la rea-
quierdo, etc. Pero la literatura goza de una libertad especial lidad; hay que conquistar una verdadera vida, acceder a
en este aspecto, igual que el arte. Pues de estas incoherencias una realidad más alta y para ello es necesaria también una
(coherentemente hiladas unas a otras) se desprende el humor liberación del lenguaje, hay que sacarlo de la lógica. En un
que pretende el texto. poema surrealista, se entremezclan objetos, conceptos y
Por otro lado, dichas incoherencias hacen que un objeto co- sentimientos que la razón mantendría separados; aparecen
tidiano resulte mágico y recién creado ante nuestros ojos. asociaciones libres e inesperadas de palabras, metáforas
Sobre todo ayuda a este efecto el final del mismo: Se sale de insólitas, imágenes oníricas y hasta delirantes. Ante un
ella fácilmente, con un ligero golpe de talón que la fija en su poema así, el lector no «comprende» pero puede recibir
sitio, del que no se moverá hasta el momento del descenso. fuertes impactos que modifican su estado de ánimo y sus-
A pesar de todo ello, la cohesión está perfectamente conse- citan en él oscuras emociones.
guida por el uso de varios mecanismos, entre ellos, los mar- Son cultivadores del surrealismo en España autores como
cadores textuales que dotan al texto de un orden y una lógica Juan Larrea, Salvador Dalí, Luis Buñuel y a su influjo se de-
interna de la que parecía carecer en principio. ben libros fundamentales como Sobre los ángeles de Rafael
2. Los textos literarios incumplen a menudo las propiedades Alberti o Poeta en Nueva York de Federico García Lorca.
del resto de los textos en beneficio de la expresividad. 3. A veces, la naturaleza y el arte se combinan. Observa la
Tratan con ello, como el texto de la actividad anterior, de siguiente imagen y escribe una pequeña historia fantástica
crear sorpresa y belleza. Lee el siguiente texto, y explica con ella que cumpla las tres propiedades textuales. Pon a
si te parece un mensaje coherente y por qué. trabajar a tu imaginación.
Guitarra Respuesta libre.
Habrá un silencio verde
todo hecho de guitarras
… encuentra la clave
destrenzadas. 1. El siguiente texto es un fragmento de un rap de Chojin
La guitarra es un pozo en el que colaboran otros famosos raperos como Kase O,
con viento en vez de agua. Lírico, El Langui o Nach. Léelo o escúchalo en YouTube
Gerardo DIEGO: Imagen. tecleando: El Chojin. «Rap contra el racismo», y realiza las
siguientes actividades que se te plantean:
a) Analiza de qué tipo de texto se trata desde el punto
Kase O
de vista de su forma, su intención comunicativa y el
canal de comunicación que utiliza. De acuerdo con Alto, bajo, feo, guapo, negro, blanco, ¿qué más da? / Dentro
todo ello, explica si existe la propiedad de adecua- de cien años todos calvos bajo tierra, ¿va? / ¿No has probado
nunca a conocer a un extranjero? / Fíjate en los niños:
ción en el texto.
ellos saben de qué va este juego / y es que la raza humana
b) ¿Qué relación observas entre el poema y la imagen es un crisol. / El que no pueda ver belleza en esto no mere-
de la guitarra en cuanto a su parecido con la realidad? ce ver el sol. / Paso el relevo al compañero, para este mundo
c) Investiga sobre el surrealismo y sus manifestaciones nuevo / es el triunfo del amor contra el miedo.
literarias y pictóricas. ¿Qué papel cumple la coheren- Nach
cia en los textos surrealistas? Cuando la bestia racista siente rabia y muerte; / cuando la
a) Desde el punto de vista de su forma, se trata de un poema. fobia se contagia y hierve acusándote de no ser igual; / cuan-
Por su intención comunicativa se trata de un texto poético do en un mundo global / el buscar comida en otra tierra te
y, según el canal que utiliza, se trata de un texto escrito. convierte en ilegal; / cuando la ley de extranjería te atrapa
Sí hay adecuación, porque los textos poéticos suelen sin motivo / y la hipocresía tapa sus ojos y sus oídos; /

Lengua castellana y Literatura. 3.º ESO. Solucionario 11


1 LOS CIMIENTOS DE TU LENGUA

racismo y marginación, cuando solo ven la piel / y se olvidan 4. Inventa enunciados y oraciones con mensajes en contra
de mirar al corazón. del racismo.
a) ¿Este texto es una manifestación de la lengua o del Indica cuáles de esos enunciados son oraciones y cuáles
lenguaje? Explica tu respuesta. son frases.
b) Explica por qué llevan tilde las palabras marginación Respuesta libre.
y acusándote. Luego, escribe tres palabras llanas que 5. Ahora escribe un texto en el que mantengas el tema de
necesiten tilde. los enunciados del ejercicio anterior. Luego, crea un cartel
c) Explica cuál es el significado, el significante y el refe- apropiado que acompañe al texto. Te ofrecemos este de
rente de la palabra relevo. ¿Está usada esta palabra en Nelson Mandela para que te sirva de inspiración.
sentido literal? Justifica tu respuesta apoyándote en la
información que encuentres en el diccionario.
d) Indica la modalidad de las oraciones que aparecen en «NADIE nace odiando a otra persona por el color de
negrita. su piel, o su origen, o su religión.
e) Este texto es pura poesía. Señala en él algunos recursos La gente tiene que aprender a odiar, y si ellos pueden
literarios (metáfora, comparación, etc.) que conozcas. aprender a odiar, también se les puede enseñar a amar;
a) De la lengua, ya que se trata de una manifestación con- el amor llega más naturalmente al corazón huma-
creta del lenguaje (capacidad lingüística que poseen todos no que su contrario».
los seres humanos). Se trata de una manifestación de la
lengua castellana un tanto especial, pues, en oposición
a lo que ocurre con el lenguaje, que todos poseemos, no Respuesta libre.
todos los seres humanos poseemos la capacidad de usar la 6. Busca información sobre la figura de Nelson Mandela y
lengua para hacer un texto literario. Es una manifestación
elabora un resumen de su biografía con los datos más inte-
de lengua, que equivale al idioma, porque el lenguaje tiene
una naturaleza social; entendemos el texto los hablantes
resantes que hayas encontrado.
de una misma lengua, en este caso, el castellano. Nelson Mandela (Umtata, Transkei, 1918-Johannesburgo,
b) Marginación: es aguda y acaba en n. Acusándote: es es- 2013). Político sudafricano que lideró los movimientos con-
drújula y las palabras esdrújulas se acentúan siempre. tra el apartheid. Prisionero durante 27 años (1963-1990)
en penosas condiciones, el gobierno de Sudáfrica recha-
c) Significante: r-e-l-e-v-o. Significado: acción de reemplazar
zó todas las peticiones para ponerlo en libertad. Nelson
un corredor a otro de su mismo equipo en el momento de
Mandela se convirtió en un símbolo de la lucha contra el
recibir el relevo.
apartheid dentro y fuera del país, en una figura legendaria
Referente: que representaba la falta de libertad de todos los negros
sudafricanos.
En 1984 el gobierno intentó acabar con tan incómodo mito,
ofreciéndole la libertad si aceptaba establecerse en uno de los
bantustanes a los que el régimen había concedido una ficción
de independencia; Mandela rechazó el ofrecimiento. Durante
aquellos años, su esposa Winnie simbolizó la continuidad de
la lucha, alcanzando importantes posiciones en el Congreso
No. Está empleado en un sentido metafórico. Pasarle el Nacional Africano. El ferviente activismo de Winnie no estu-
relevo al compañero quiere decir que le cede el turno de vo libre de escándalos; años después, ya en los 90, se vería
palabra porque ambos están juntos en esta causa. Le cede envuelta en un polémico juicio en el que fue acusada de ase-
la palabra para que él también luche contra el racismo. sinato, si bien salió absuelta.
d) • ¿No has probado nunca a conocer a un extranjero? Finalmente, Frederik De Klerk, presidente de la República por
(interrogativa). el Partido Nacional, cedió ante la evidencia y decidió abrir el
• Fíjate en los niños: ellos saben de qué va este juego camino para desmontar la segregación racial, liberando a
(exhortativa y enunciativa). Mandela en 1990 y convirtiéndole en su principal interlocutor
para negociar el proceso de democratización. Mandela y De
• El que no pueda ver belleza en esto no merece ver el sol Klerk compartieron el Premio Nobel de la Paz en 1993.
(enunciativa).
Las elecciones de 1994 convirtieron a Mandela en el primer
• La hipocresía tapa sus ojos y sus oídos (enunciativa).
presidente negro de Sudáfrica; desde ese cargo puso en mar-
• Se olvidan de mirar al corazón (enunciativa). cha una política de reconciliación nacional, manteniendo a
2. Busca otros temas musicales que transmitan este mismo De Klerk como vicepresidente y tratando de atraer hacia la
mensaje y anota la letra de dos. participación democrática al díscolo partido Inkhata, de ma-
yoría zulú. Una película muy recomendable del cineasta es-
Respuesta libre.
tadounidense Clint Eastwood, Invictus (2009), reflejaría con
3. Inventa señales de prohibición en contra del racismo y bastante fidelidad el Mandela de aquellos años; su apoyo a
traduce su significado en forma de oraciones imperativas. una selección nacional formada por blancos durante la Copa
Ejemplo: Mundial de Rugby de 1995, celebrada en Sudáfrica, muestra su
empeño en integrar la minoría blanca y la mayoría negra sir-
Prohibidas las conductas racistas que dañen el corazón viéndose de aquel acontecimiento deportivo y su firme volun-
de las personas. tad de construir una nación para todos los sudafricanos, sin
Respuesta libre. distinción de raza.

12 Lengua castellana y Literatura. 3.º ESO. Solucionario


EL MÁGICO PODER DE LAS PALABRAS 2
APERTURA DE UNIDAD rumores y supersticiones sobre los riesgos que entraña su uso:
puede electrocutar a sus portadores o atraer los rayos.
1. Los integrantes de la «Granja Animal» escribieron sobre un
5. ¿Dónde se puede adquirir el Baby H. P.?
muro las normas que todos deberían acatar tras expulsar
a su dueño. En las buenas tiendas.

a) ¿Cómo intentaron hacer bien visible su mensaje? 6. Comprueba si los siguientes enunciados son verdaderos
(V) o falsos (F):
b) ¿Crees que el rudimentario procedimiento que ellos em-
plearon podría tener algún tipo de relación con los usa- V F
dos por los modernos medios de comunicación de
masas? ¿Cuál? a) El Baby H. P. lo pueden utilizar todos
a) Sobre una pared negra escribieron en letras blancas muy los miembros de una familia.
grandes su mensaje. b) Se recomienda que los niños se lo pongan
b) Aunque los modernos medios de masas emplean recursos para dormir.
más sofisticados, en el fondo los animales hicieron uso
de procedimientos con un evidente poder visual: así el c) El aparato jamás puede actuar como
contraste cromático entre el color de la pared y el de las un pararrayos.
letras, mientras que el tamaño de estas últimas nos remite
al papel importante que tiene la tipografía, por ejemplo, d) La energía transformada en electricidad
en la prensa. puede venderse.
2. Imagínate por un instante que los habitantes de la «Gran- e) El aparato tiene un modelo único, por lo
ja Animal» tuviesen las mismas habilidades racionales de que no es adecuado para todos los niños.
los humanos y desearan difundir los mandamientos entre
toda su población. a) Falso. c) Falso. e) Falso.
a) Aparte del código lingüístico, ¿a qué otros códigos y b) Verdadero. d) Verdadero.
medios de comunicación podrían acudir para transmi-
tir su mensaje? Vocabulario
b) ¿Podrían conseguir su objetivo fácilmente? ¿Por qué? 7. El kilovatio es una medida de potencia eléctrica. Para
medir otras magnitudes se emplean diferentes unidades.
Respuesta libre.
Relaciona cada unidad con su correspondiente magnitud:
VIVE LA LECTURA intensidad de corriente • • pascal
Comprensión lectora longitud • • kelvin
1. En el relato, Arreola recurre al lenguaje publicitario. ¿Qué masa • • ohmio
marcas típicas de este tipo de discurso reconoces en resistencia eléctrica • • kilogramo
el texto? ¿Quién se supone que es el emisor y quién su presión • • amperio
destinatario?
temperatura • • metro
Elementos típicos del lenguaje publicitario son el contenido
descriptivo del texto y el predominio de la función apelativa • intensidad de corriente  amperio
del lenguaje en un intento de convencer al destinatario. Esta • longitud  metro
instancia se corresponde con cualquier ama de casa, a la que • masa  kilogramo
se dirige con un usted (fórmula de respeto) el emisor que, en
• resistencia eléctrica  ohmio
este caso, se identifica con un anunciante.
• presión  pascal
2. ¿Qué producto se anuncia? ¿En qué lugar del texto se
• temperatura  kelvin
describen sus características físicas? ¿Dónde se explica
su funcionamiento? 8. ¿Por qué dice el texto que el Baby H. P. es lucrativo? Explí-
En el texto se anuncia el Baby H. P., un aparato que actúa calo en tu cuaderno.
como depósito de la energía que «producen» los niños con su El aparato es lucrativo porque permite obtener unas ganan-
actividad física diaria. Sus características físicas y su funcio- cias o beneficios, ya que permite ahorrar en el recibo de la
namiento se detallan en el segundo párrafo. electricidad e incluso vender la energía sobrante.
3. ¿Por qué dirías que la historia tiene un carácter fantástico? 9. En los mensajes publicitarios se busca interesar al consu-
La historia tiene un carácter fantástico porque plantea una midor por un producto cuyas virtudes se destacan me-
posibilidad bastante inverosímil, convirtiendo a los niños en diante adjetivos. Anota las palabras que desempeñen esa
una especie de máquinas. Con ello, el autor parodia no solo función en el texto.
el lenguaje publicitario, sino que lanza sus dardos contra una Las virtudes del producto se destacan mediante adjetivos
sociedad cada vez más tecnificada. como: maravilloso Baby H. P., cómodos cinturones, preciosa
4. ¿Qué beneficios puede tener para las familias la compra alcancía o fuente generosa; pero también a partir del signi-
del producto descrito? En cambio, ¿en qué momento ad- ficado de formas verbales como: revolucionar, contribuye a,
vierte el autor de los peligros que se derivan de su uso? se economizan y se obsequian.
Con la adquisición del aparato, el agobiante ajetreo de los ni- 10. ¿Qué extranjerismos y términos cultos emplea el autor?
ños deja de ser un quebradero de cabeza para convertirse en Como extranjerismos: Leyden (por Leiden, universidad ho-
fuente de ahorro económico para las familias. En cambio, en el landesa) y récord. Como términos cultos, de los indicados en
penúltimo párrafo, se alude como de pasada a los supuestos el glosario: motriz, alcancía o lucrativo.

Lengua castellana y Literatura. 3.º ESO. Solucionario 13


2 EL MÁGICO PODER DE LAS PALABRAS

Investigación y redacción g) una banalidad de película


11. En este cuento el autor alerta sobre los peligros del desa- h) nuestros primeros días juntos
rrollo tecnológico, pero también sobre cómo la publicidad i) la casa de los espíritus
puede resultar engañosa. A partir de estas ideas: a) Det + N + CN (enlace + término-SN [N])
a) Elabora una redacción dando tu punto de vista sobre b) Det + Ady + N
el tema: «¿Es el hombre actual esclavo de la técnica?». c) Det + N + Aposición especificativa
b) En grupos, seleccionad varios anuncios publicitarios d) Det + N + Ady + CN (enlace + término-SN [Det + N])
para debatir en el aula si la información que transmiten
e) N + aposición explicativa (Det + N + CN [enlace + término-
es fiable o engañosa. SN {N}])
Respuesta libre.
f) Det + Ady + N + CN (enlace + término-SN [N])
ESTUDIO DE LA LENGUA g) Det + N + CN (enlace + término-SN [N])
h) Det + Det + N + Ady
1. Escribe estos sustantivos en masculino y femenino e indi-
ca cómo se marca el género en cada caso: i) Det + N + CN (enlace + término-SN [Det + N])

diablo astronauta serpiente 4. Descubre la palabra sustantivada en las siguientes oracio-


nes e indica si la sustantivación es ocasional o permanente:
padrino héroe emperador
a) Nos intriga el mañana.
escritor tigre yerno
b) Le daremos el pésame a su abuelo.
avestruz equilibrista profeta
c) Era el hazmerreír de la clase.
• diablo: pasa a femenino como diablesa (cambio en termina-
ción). d) Solo los ricos envidian nuestra felicidad.
• astronauta: mismo lexema para masculino y femenino, es e) En la plaza se oyó un olé de júbilo.
el determinante el que precisa el género. a) mañana: sustantivación permanente.
• serpiente: cambia el género añadiendo macho o hembra. b) pésame: sustantivación permanente.
• padrino: pasa a femenino como madrina (cambio en el c) hazmerreír: sustantivación permanente.
lexema). d) ricos: sustantivación ocasional.
• héroe: pasa a femenino como heroína (cambio en termina- e) olé: sustantivación ocasional.
ción).
5. Además de los numerales cardinales que aparecen en el
• emperador: pasa a femenino como emperatriz (cambio en
terminación).
ladillo Recuerda, existen otros tipos de numerales. Escribe
en tu cuaderno un breve listado de ordinales, multiplicati-
• escritor: pasa a femenino como escritora (cambio en termi-
vos y partitivos.
nación).
Ordinales: primero, segundo, tercero, cuarto, quinto, sexto,
• tigre: pasa a femenino como tigresa (cambio en terminación).
séptimo, octavo, noveno, décimo, undécimo, duodécimo…,
• yerno: pasa a femenino como nuera (cambio en el lexema). vigésimo…, trigésimo…, cuadragésimo…, quincuagésimo…,
• avestruz: cambia el género añadiendo macho o hembra. sexagésimo…, septuagésimo…, octogésimo…, nonagésimo…,
• equilibrista: mismo lexema para masculino y femenino, es centésimo, etc.
el determinante el que precisa el género. Multiplicativos: doble, triple, cuádruple, quíntuple, séxtu-
• profeta: pasa a femenino como profetisa (cambio en termi- ple, séptuple, óctuple, nónuplo, décuplo, undécuplo, duodé-
nación). cuplo, terciodécuplo…, céntuplo, etc.
2. Forma el plural de los siguientes sustantivos: Partitivos: medio o mitad, tercio, cuarto, quinto, sexto o sei-
savo, séptimo, octavo, noveno, décimo, undécimo u onceavo,
raíz crisis jueves lápiz
duodécimo o doceavo, decimotercero o treceavo…, vigésimo
árbol tren naranja peral o veinteavo, etc.
reactor ayuntamiento ciudad fórceps 6. Señala el sujeto en las siguientes oraciones:
• ayuntamiento, naranja: ayuntamientos, naranjas (se aña- a) Yo te encontré en la discoteca.
de –s).
b) A tu padre le disgusta mi ropa.
• árbol, tren, peral, reactor ciudad: árboles, trenes, perales,
reactores, ciudades (se añade –es). c) Todas tus libretas las escondieron ellos.
• raíz, lápiz: raíces, lápices (se añade –es y la z pasa a c). d) Hicimos el trabajo entre mis compañeras y yo.
• crisis, jueves, fórceps: misma forma para singular y plural e) El río Pisuerga pasa por Valladolid.
(el número lo señala el determinante). f) Llegaron hasta los corredores más débiles.
3. Analiza los siguientes sintagmas nominales, identificando g) De la pared colgaban varios cuadros.
todos sus elementos constituyentes: h) Trajo el camarero dos botellas de vino.
a) dos botellas de agua i) Usted le hablará con palabras amables.
b) aquellas hermosas pinturas a) Yo.
c) mi primo Arturo b) mi ropa.
d) la casa amarilla de la esquina c) ellos.
e) París, la capital de Francia d) entre mis compañeras y yo.
f) cuatro horribles chaquetas sin solapas e) El río Pisuerga.

14 Lengua castellana y Literatura. 3.º ESO. Solucionario


EL MÁGICO PODER DE LAS PALABRAS 2
f) hasta los corredores más débiles. • río: corriente de agua; ría: desembocadura del río por don-
g) varios cuadros. de penetran corrientes marinas.
h) el camarero. • música: arte de combinar los sonidos; músico: compositor
o intérprete.
i) Usted.
• leño: trozo de un árbol; leña: conjunto de trozos de troncos
7. Reescribe en tu cuaderno el siguiente fragmento, resta- y ramas de árboles.
bleciendo los sujetos que se han omitido en él: • barco: embarcación de mayor tamaño, movido a motor o
Scrooge cerró la ventana e inspeccionó la puerta por la que vela; barca: embarcación pequeña.
había entrado el espectro. Tenía la llave echada con doble • peso: cantidad que pesa algo; pesa: pieza para calcular el
vuelta, tal como la había cerrado él con su propia mano, y peso o para hacer músculos.
los cerrojos estaban intactos. Trató de decir: «¡Paparruchas!», • el coma: estado de inconsciencia; la coma: signo de
pero se detuvo en la primera sílaba. puntuación.
Charles DICKENS: Canción de Navidad.
10. Clasifica los siguientes nombres, atendiendo a su signi-
Respuesta libre. ficado.
8. Identifica el tipo de impersonalidad que se da en los si- Ejemplo: alambrada  concreto, común, contable, colec-
guientes casos: tivo.
a) En mi pueblo no hay piscina. bofetada ajuar Rusia agua
b) Se invitó al presidente a la celebración. libro aceite amor vajilla
c) Se espera un gran porcentaje de aprobados. jauría televisor salto Andes
d) El verano pasado granizó con gran virulencia. control Asturias trampolín equipo
e) Ordenaron un registro de la casa. • bofetada: concreto, común, contable, individual.
f) Me llamaron al teléfono varias veces. • ajuar: concreto, común, contable, colectivo.
g) Es pronto para la comida. • Rusia: concreto, propio, topónimo.
h) Durante la excursión hizo un tiempo apacible. • agua: concreto, común, de materia.
• libro: concreto, común, contable, individual.
i) Habrá mucha colada en la lavadora.
• aceite: concreto, común, de materia.
j) Se trataba de tu amigo.
• amor: abstracto.
k) A partir de mañana anochecerá una hora más tarde.
• vajilla: concreto, común, contable, colectivo.
a) Impersonal gramaticalizada.
• jauría: concreto, común, contable, colectivo.
b) Impersonal con se.
• televisor: concreto, común, contable, individual.
c) Impersonal con se.
• salto: concreto, común, contable, individual.
d) Impersonal de fenómeno meteorológico. • Andes: concreto, propio, topónimo.
e) Impersonal ocasional. • control: abstracto.
f) Impersonal ocasional. • Asturias: concreto, propio, topónimo.
g) Impersonal gramaticalizada. • trampolín: concreto, común, contable, individual.
h) Impersonal gramaticalizada. • equipo: concreto, común, contable, colectivo.
i) Impersonal gramaticalizada. 11. Transforma los siguientes titulares y eslóganes publicita-
j) Impersonal con se. rios en oraciones con sujeto y predicado:
k) Impersonal de fenómeno meteorológico. a) Gran expectación ante la llegada del Real Madrid.
9. Explica la diferencia de significado existente entre los b) Ferrero Rocher, la expresión del buen gusto.
pares de sustantivos que se enumeran (las ilustraciones c) Maduro, preocupado por la caída del petróleo.
de la parte inferior te serán de ayuda):
d) El número de jóvenes: en máximos en el mundo y míni-
cesto-cesta olivo-oliva madera-madero mos en España.
ramo-rama huerto-huerta el corte-la corte e) Opel Astra. Ingeniería alemana de futuro.
río-ría música-músico leño-leña f) Al menos seis muertos en un ataque a una escuela pú-
barco-barca peso-pesa el coma-la coma blica de Roma.
• cesto-cesta: recipientes de mimbre, con mayor altura del g) Móviles con tapa: ¿el regreso?
cesto que la cesta.
h) Un juguete, una ilusión.
• olivo: árbol; oliva: aceituna, fruto del olivo.
a) La llegada del Madrid provocó una gran expectación.
• madera: materia; madero: tronco de un árbol sin las ramas
o tablón. b) Ferrero Rocher es la expresión del buen gusto.
• ramo: conjunto de ramas y flores combinadas ornamental- c) Maduro está preocupado por la caída del petróleo.
mente; rama: cada una de las partes que nacen del tronco d) El número de jóvenes está en máximos en el mundo y en
de las plantas. mínimos en España.
• huerto: terreno pequeño y acotado; huerta: extensión e) La ingeniería alemana de futuro se llama Opel Astra.
de terreno de regadío para cultivo de verduras y árboles f) Un ataque a una escuela pública de Roma ha provocado
frutales. al menos seis muertos.
• el corte: herida o interrupción o vergüenza; la corte: lugar g) Los móviles con tapa regresan a las tiendas.
o población donde vive el rey con sus servidores. h) Un juguete equivale a una ilusión.

Lengua castellana y Literatura. 3.º ESO. Solucionario 15


2 EL MÁGICO PODER DE LAS PALABRAS

12. Identifica los sintagmas nominales que funcionan como aguacero, aguardiente. Di cuáles de ellas se han formado
sujeto en las siguientes oraciones. A continuación, analiza por derivación y cuáles por composición. Puedes usar un
la estructura de dichos sintagmas. ¿Has encontrado algu- diccionario y anotar en tu cuaderno los significados.
na palabra sustantivada actuando como núcleo? Anótala • desagüe: derivada; «conducto por donde sale el agua».
en tu cuaderno. • aguafiestas: compuesta; «persona que estropea una
a) Ayer cumplió años mi tía Julia. diversión».
b) Seis atracadores encapuchados asaltaron la sucursal • aguador: derivada; «persona que transporta o vende agua».
del banco. • aguadilla: derivada; «breve zambullida».
c) Recibirá un trofeo impresionante el ganador de la • aguachirle: compuesta; «alimento líquido sin sustancia».
carrera. • aguamanos: compuesta; «agua que sirve para lavar las
d) En el referéndum ganó por mayoría el sí. manos».
e) A todos nos encanta el parque de atracciones Port- • aguacero: derivada; «lluvia abundante».
Aventura. • aguardiente: compuesta; «bebida alcohólica de mucha
graduación».
f) Dublín, capital de Irlanda, acogerá un gran concierto
de música electrónica. 3. Reescribe las siguientes oraciones sustituyendo las ex-
g) El frotar se va a acabar. presiones o palabras en negrita por alguna palabra que
pertenezca a la familia léxica de caja. Utiliza el diccionario:
a) mi tía Julia: Det + N + Aposición especificativa
b) Seis atracadores encapuchados: Det + N + Ady a) Eduardo estaba enamorado de la chica que cobra a
c) el ganador de la carrera: Det + N + CN (enlace + término-
los clientes que guardan cola en el supermercado.
SN [Det + N]). Sustantivación de ganador. b) No creo que sea posible meter esta vela en el cande-
d) el sí: Det + N. Sustantivación de sí. labro.
e) el parque de atracciones PortAventura: Det + N + CN (en- c) Estoy aprisionado entre la mesa de estudio y la pared;
lace + término-SN [N]) + Aposición especificativa no puedo ni moverme.
f) Dublín, capital de Irlanda: N + Aposición explicativa (N d) Alicia compró un paquete de tabaco, pero luego lo
+ CN [enlace + término-SN {N}]) quemó, porque, en realidad, quería dejar de fumar.
g) El frotar: Det + N. Sustantivación de frotar. e) Eladio se quedó sin dinero y tuvo que ir a sacar más a
13. En las siguientes oraciones, sustituye los sintagmas nomi- la máquina que había en el exterior del banco.
nales por un pronombre e indica de qué tipo es cada uno. a) cajera c) encajonado e) cajero
a) Ningún asistente quiso firmar la petición. b) encajar d) cajetilla
b) Estas rosas huelen estupendamente; aquellas rosas no 4. Lee el siguiente fragmento de un poema de Jorge Man-
tanto. rique (c. 1440-1479) y busca en él palabras de una misma
c) Dame un regaliz a mí y a ella dale dos regalices. familia léxica. Indaga en el significado exacto de cada una
d) ¿Estudiamos en mi habitación o en tu habitación? de ellas dentro del poema:
a) Nadie quiso firmarla. Ningún asistente: nadie (indefini- Diciendo qué cosa es amor
do); la petición: la (personal átono). Es amor fuerza tan fuerte una porfía forzosa
b) Estas huelen estupendamente; aquellas no tanto. Es- que fuerza toda razón; que no se puede vencer,
tas rosas: estas (demostrativo); aquellas rosas: aquellas una fuerza de tal suerte, cuya fuerza porfiosa
(demostrativo). que todo seso convierte hacemos más poderosa
c) Dame uno a mí y a ella dale dos. Un regaliz: uno (numeral en su fuerza y afición; queriéndonos defender.
cardinal); dos regalices: dos (numeral cardinal). • fuerza: como sustantivo en es amor fuerza, pero como ver-
d) ¿Estudiamos en la mía o en la tuya? Mi habitación: la bo («forzar, obligar, presionar») en fuerza toda razón. De
mía (posesivo); tu habitación: la tuya (posesivo). la misma familia léxica, los adjetivos fuerte y forzosa.
• porfía (sustantivo) y porfiosa (adjetivo) con el significado
EL TALLER DE LAS PALABRAS de «lucha obstinada».
1. Con ayuda de un diccionario, forma las familias léxicas 5. Di si las siguientes listas de palabras forman campos se-
de las siguientes palabras. Anota en tu cuaderno el signi- mánticos o asociativos y justifica tu respuesta:
ficado de aquellas que no conozcas: poesía, mano, salvar, a) primavera, verano, otoño, invierno
caballo, figura. b) rosa, florecimiento, verdor, prado, abril
• caballo: caballete, caballico, caballito, caballuelo, caballe- c) lámpara, linterna, faro, foco, flexo
ría, caballero, caballeresco, caballeroso, caballeriza.
d) examen, biblioteca, gimnasio, título
• figura: figuración, figuradamente, figurado, figurante, fi-
gurar, figurativo. e) motocicleta, automóvil, tractor, bicicleta
• mano: manita, manopla, manosear, manido, manecilla. f) nieve, frío, oscuridad, helada
• poesía: poeta, poetisa, poético, poéticamente. g) pared, muro, verja, cerca
• salvar: salvación, salvado, salvador, salvaguardia, salvo, a) campo semántico e) campo semántico
salvavidas, salvedad. b) campo asociativo f) campo asociativo
2. En la familia léxica de agua encontramos, entre otras, c) campo semántico g) campo semántico
las palabras que te ofrecemos a continuación: desagüe, d) campo asociativo
aguafiestas, aguador, aguadilla, aguachirle, aguamanos, Las palabras de los campos asociativos no comparten semas.

16 Lengua castellana y Literatura. 3.º ESO. Solucionario


EL MÁGICO PODER DE LAS PALABRAS 2
6. En los casos en los que has reconocido un campo semán- 5. Elige ahora entre sino y si no para completar las siguientes
tico, elabora una tabla de semas como la que has visto oraciones:
más arriba. a) Puedes quedarte a dormir ________________ te apetece andar
Respuesta libre. hasta tu casa.
b) Mira, ________________ quieres entenderlo, peor para ti.
EL RINCÓN DE LA NORMA
c) Este era mi ________________ : levantarme al amanecer para
1. Pon las tildes diacríticas que faltan en las siguientes ora- cultivar la tierra.
ciones: d) No quiero comer acelgas, ________________ coliflor, que me gus-
ta más.
Corrige a) si no c) sino
a) No se como poner en marcha la tostadora. b) si no d) sino
b) El te era para mi, mas se lo bebió el: ¡que rápido 6. Por último, completa estas oraciones con asimismo, así
fue! mismo o a sí mismo:
c) Si te hubiera hecho caso, aun estaríamos esperan- a) Déjalo ________________ , ya lo recogeremos mañana.
do a Gerardo en la puerta de tu casa. b) La situación es comprometida, pero Ricardo no tiene
d) Mira que cara de imbécil se te ha quedado cuando por qué castigarse ________________ .
has visto que tengo razón. c) Pintaremos la casa; ________________ construiremos un establo
para nuestro caballo.
e) ¿Cuanto hace que no se nada de ti?
d) Todos los días se habla ________________ en voz alta; todos le
f) Tu has dado un grito mas agudo que el. toman por loco.
a) No sé cómo poner en marcha la tostadora. a) así mismo c) asimismo
b) a sí mismo d) a sí mismo
b) El té era para mí, mas se lo bebió él: ¡qué rápido fue!
c) Si te hubiera hecho caso, aún estaríamos esperando a
Gerardo en la puerta de tu casa.
LA FACTORÍA DE TEXTOS
d) Mira qué cara de imbécil se te ha quedado cuando has 1. Clasifica los programas de algún canal televisivo público
visto que tengo razón. según su finalidad. ¿Has encontrado espacios educativos
e) ¿Cuánto hace que no sé nada de ti? y culturales? ¿Cuáles?
f) Tú has dado un grito más agudo que él. Respuesta libre.

2. Teniendo en cuenta lo que acabas de aprender sobre la 2. Compara el lugar que una misma noticia de actualidad
tilde diacrítica, razona por qué no es fiable el corrector ocupa en diversas portadas de la prensa diaria y en los
ortográfico de un ordenador y por qué hemos de revisar espacios informativos de radio y televisión. ¿A qué puede
lo que escribimos. obedecer que los distintos medios le concedan distinta
importancia a un mismo hecho?
Los correctores ortográficos, si no llevan incorporado un co-
Respuesta libre.
rrector gramatical, no advierten de muchos errores. Aun así,
difícilmente detectará los acentos diacríticos, ni tampoco, las 3. Define los conceptos de desinformación, contrainforma-
palabras homónimas o las palabras con doble acentuación. ción y telebasura.
El corrector ortográfico se guía a través del diccionario que • Desinformación: control intencionado y, por tanto, mani-
lleva incorporado. Sin embargo, tal repertorio no distingue pulación de la información que se suministra, omitiendo
cuando dos términos se escriben igual pero pertenecen a datos importantes para mantener a los destinatarios en
categorías gramaticales distintas o incluso se diferencian una situación de desconocimiento e ignorancia.
semánticamente. • Contrainformación: mecanismo contrario a la desinforma-
3. Construye oraciones con las siguientes palabras, respe- ción, consistente en ofrecer una información independien-
tando la tilde diacrítica: adonde/adónde; como/cómo; te y con un enfoque reivindicativo. Es responsabilidad de
cual/cuál; cuando/cuándo; cuanto/cuánto; donde/dónde; grupos opuestos al poder político o económico. También
que/qué; quien/quién. se usa el término de contrainformación para aludir a la
información falsa que difunde un Estado o un grupo de
Respuesta libre. poder para confundir a sus oponentes.
4. En las siguientes oraciones incluye porque, porqué, por • Telebasura: distingue a un tipo de programas televisivos
qué y por que, según convenga: de muy baja calidad y con dominio del sensacionalismo.
a) Terminé cansado ________________ había estado estudiando 4. Lee un fragmento del artículo de M. García Terán: «Los
toda la noche. adolescentes crearon un extraño lenguaje para chatear»,
b) Nunca sabré ________________ tuviste que tomar esa decisión La Nación (25 de julio de 2004):
tan arriesgada. Advertencia: si usted es un purista del lenguaje, lo que sigue
puede ponerle los pelos de punta. Si no lo es, o está dispuesto
c) He aquí la explicación de todo, el ________________ de su extra- a continuar, trate de descifrar este mensaje: «Akabo d ygr a
ño comportamiento. ksa, tki n un rto. slmos mñna? b.:-)». ¿Un traductor? No lo dude,
d) Estuvimos velando toda la noche ________________ tuvierais re- llame a su hijo.
suelto el problema cuando os despertarais. ¿Te parecen descabellados los temores de la autora? Jus-
a) porque c) porqué tifica tu respuesta con datos o argumentos convincentes.
b) por qué d) por que Respuesta libre.

Lengua castellana y Literatura. 3.º ESO. Solucionario 17


2 EL MÁGICO PODER DE LAS PALABRAS

5. Ahora te toca ser corrector. Formados unos grupos de a) El autor del artículo condena e invita a tomar conciencia
trabajo, se elaborará un listado semanal con todas aque- sobre la gravedad de los últimos episodios de violencia
llas incorrecciones ortográficas que se hayan detectado contra personas discapacitadas.
en una selección de periódicos y en diversos espacios b) El registro estándar es el característico de los textos perio-
televisivos, y se propondrán las soluciones convenientes. dísticos y debe su empleo al intento de que los contenidos
sean entendidos por cualquier lector: para llegar a la gran
Respuesta libre. mayoría.
ACTIVIDADES FINALES c) Respuesta libre.
d) Los sustantivos discapacidad, gravedad y violencia son
Repasa lo que has aprendido abstractos. Por su parte, teléfonos y videocámaras son
1. Lee este fragmento del artículo de opinión de Luis C. concretos, comunes, contables e individuales.
Pérez, «La violencia creciente contra la discapacidad», e) Las palabras desaprensivos y merecedor pertenecen a la
aparecido el 25 de enero de 2008 en el diario Levante. categoría de los adjetivos. En el texto, sin embargo, no
En las últimas semanas han saltado a las páginas de los dia- actúan como adyacentes directos del sustantivo, sino que
rios noticias muy inquietantes sobre hechos violentos diri- ocupan una función nuclear dentro de sus respectivos
gidos contra personas con discapacidad, que han tenido la sintagmas. De ahí que hayan experimentado un proceso
circunstancia agravante de ser difundidos y publicitados por sustantivador.
Internet. f) • Ady + N + CN (enlace + término-SN [N]). SPrep: con
Distintas personas con discapacidad en Madrid y en el País discapacidad
Vasco, por citar los casos más recientes, han sido objeto de • Det + N + Ady-SAdj (Det + N)
agresiones, precedidas de humillaciones y tratos crueles, por • N + Ady + CN (enlace + término-SN [Det + N]). SPrep:
parte de grupos de desaprensivos a los que la discapacidad de de la violencia
la víctima anima a atacar. Pero la agresión no acaba ahí, sino g) Sujeto:
que, elevada a la categoría de espectáculo, ha sido grabada • noticias muy inquietantes
por medio de teléfonos móviles y videocámaras, y exhibida
• La discapacidad de la víctima
obscenamente a través de la red.
• La agresión
Estos hechos tan graves indican un estado mental aún vigente
en ciertas personas que siguen considerando la discapacidad h) Impersonal con se. El sujeto de la oración Se hace merece-
como una circunstancia que rebaja al que la presenta, y en dor del desprecio es muy genérico. Alude a la persona que
tanto que inferior, se hace merecedor del desprecio, de la burla padece alguna discapacidad.
y hasta de la violencia. Como su vida, por portar ese elemento 2. A partir de la siguiente imagen, diseña una campaña pu-
de diversidad, vale menos, se convierte a las personas con blicitaria a favor de la integración de las personas con
discapacidad en víctimas propiciatorias de la violencia […]. discapacidad. Crea seis eslóganes diferentes y a conti-
a) ¿Qué finalidad preside el artículo? nuación divídelos en sintagmas y analízalos.
b) ¿Su vocabulario pertenece a un nivel estándar? ¿Por Respuesta libre.
qué? 3. Francisco de Quevedo enriqueció el léxico castellano
c) Imagínate que tú quieres tratar el mismo asunto del con curiosos vocablos, como demostraría en el célebre
artículo en un foro de Internet. Reescribe de forma retrato del dómine Cabra, incluido en el capítulo IV de
resumida el fragmento. su relato picaresco El Buscón y que ahora reproducimos.
d) ¿A qué clase pertenecen los sustantivos discapacidad, Léelo atentamente.
gravedad y violencia? Asimismo, teniendo en cuenta Él era un clérigo cerbatana, largo solo en el talle, una cabeza
su significado, ¿en qué grupos situarías los sustantivos pequeña […]. Los ojos, avecindados en el cogote, que parecía
que miraba por cuévanos; tan hundidos y obscuros, que era
teléfonos y videocámaras?
buen sitio el suyo para tiendas de mercaderes; la nariz, entre
e) En principio, ¿a qué categoría gramatical pertenecen Roma y Francia, porque se le había comido de unas búas de
las palabras desaprensivos y merecedor? Sin embargo, resfriado […]. Mirado de media abajo, parecía tenedor, o com-
¿como qué otra clase gramatical actúan en el texto? pás con dos piernas largas y flacas […]. Traía un bonete los
¿Ante qué fenómeno nos hallamos, pues? días de sol, ratonado, con mil gateras y guarniciones de grasa;
f) Analiza los sintagmas. Luego indica qué sintagmas pre- era de cosa que fue paño, con los fondos de caspa. La sotana,
posicionales aparecen entre ellos: según decían algunos, era milagrosa, porque no se sabía de qué
color era. Unos, viéndola tan sin pelo, la tenían por de cuero de
• distintas personas con discapacidad rana; otros decían que era ilusión; desde cerca parecía negra,
• los casos más recientes y desde lejos, entre azul […]; no tenía cuello ni puños; lacayuelo
• víctimas propiciatorias de la violencia de la muerte. Cada zapato podía ser tumba de un filisteo. Pues
¿su aposento? Aun arañas no había en él; conjuraba los ratones,
g) Localiza el sujeto en las oraciones:
de miedo que no le royesen algunos mendrugos que guarda-
• En las últimas semanas han saltado a las páginas ba; la cama tenía en el suelo, y dormía siempre de un lado, por
de los diarios noticias muy inquietantes. no gastar las sábanas; al fin, era archipobre y protomiseria.
• La discapacidad de la víctima anima a atacar. a) Explica a qué familias léxicas pertenecen las palabras
• La agresión no acaba ahí. avecindados, ratonado, milagrosa y protomiseria.
h) ¿Qué tipo de impersonalidad se da en la oración Se A continuación, escribe al menos dos términos más de
convierte a las personas con discapacidad en víctimas cada familia.
propiciatorias de la violencia? ¿Es impersonal la ora- b) En el texto aparecen palabras pertenecientes a los
ción Se hace merecedor del desprecio? ¿Por qué? campos semánticos del calzado y de los muebles.

18 Lengua castellana y Literatura. 3.º ESO. Solucionario


EL MÁGICO PODER DE LAS PALABRAS 2
Localízalas, descomponlas en semas y haz una lista con b) • Enunciativa afirmativa: El mar está al acecho.
otras palabras de cada campo semántico. Indica los dis- • Enunciativa negativa: Las mil trampas […] no admi-
tintos semas de cada una. ten ninguna demora.
c) En el texto de Quevedo aparecen las palabras el y él. • Interrogativa: ¿No le parece eso bastante?
Localízalas, explica por qué una de las dos lleva tilde • Imperativa: Dispóngase a iniciar el recorrido.
e indica cuál es la función de cada una de ellas. ¿Para c) • Exclamativa: ¡Mi único secreto es el laberinto!
qué sirve entonces la tilde diacrítica? • Dubitativa: Posiblemente mi único secreto es el
a) • avecindados: vecino, vecindad, vecindario. laberinto.
• Desiderativa: ¡Ojalá mi único secreto fuese el laberinto!
• ratonado: rata, ratón, ratonera, ratonar.
d) • firmeza: firm- (lexema) -eza (morfema derivativo
• milagrosa: milagro, milagrero, milagrería, milagrear. sufijo)
• protomiseria: miseria, mísero, miserable, miserear. • conlleva: con- (morfema derivativo prefijo) -llev- (lexe-
b) Respuesta libre. ma) -a (morfema flexivo)
c) La palabra él es pronombre personal tónico. Como tal, en • salida: sal- (lexema) -ida (morfema derivativo sufijo)
Él era, actúa como sujeto. Por su parte, el término el es un • recorrido: re- (morfema derivativo prefijo) -corr- (lexe-
artículo determinado que funciona como determinante, ma) -ido (morfema derivativo sufijo)
acompañando al sustantivo en el talle, el cogote. La tilde • aguardan: a- (morfema derivativo prefijo) -guard- (lexe-
diacrítica sirve para diferenciar entre palabras que se es- ma) -an (morfema flexivo)
criben igual pero que pertenecen a distintas categorías • redobla: re- (morfema derivativo prefijo) -dobl- (lexema)
gramaticales. -a (morfema flexivo)
Recuerda lo que ya sabías • oleaje: ole- (lexema) -aje (morfema derivativo sufijo)
• sobrevivirá: sobre- (morfema derivativo prefijo) -viv-
4. Lee el siguiente fragmento de la novela de Joan Manuel (lexema) -irá (morfema flexivo)
Gisbert El misterio de la isla de Tökland:
e) • táctica: se acentúa como todas las esdrújulas.
El presidente, con la firmeza de quien decide aplicar una tác- • Historia: no se acentúa al ser llana y acabar en vocal.
tica aceptando de antemano los riesgos que conlleva, remató:
• remató: se acentúa como aguda acabada en vocal.
—Mi único secreto es el laberinto, el más grande conglome-
• único: se acentúa como todas las esdrújulas.
rado de misterios de la Historia. ¿No le parece eso bastante?
Solo podrá afirmar que conoce mi secreto quien consiga atra- • está: se acentúa como aguda acabada en vocal.
vesarlo. Todavía está por ver si usted será capaz de dar con • misterios: no se acentúa al ser llana y acabar en -s.
la salida. Basta ya de inútil discurseo. Dispóngase a iniciar el • inútil: se acentúa al ser llana y terminar en consonante
recorrido. Las mil trampas que le aguardan no admiten nin- distinta de n o s.
guna demora. A partir de este momento, cada minuto cuenta. • dispóngase: se acentúa como todas las esdrújulas.
El mar está al acecho, redobla su oleaje, desde aquí lo siento. • iniciar: no se acentúa al ser aguda y acabar en conso-
Si llegamos tarde nada de esto sobrevivirá. nante distinta de n o s.
a) ¿Por qué podría afirmarse que el fragmento que aca- • aquí: se acentúa como aguda acabada en vocal.
bas de leer es un texto? Justifica tu respuesta. • jamás: se acentúa como aguda acabada en -s.
b) Localiza en el texto una oración enunciativa afirmativa
y otra negativa; luego, encuentra una interrogativa y MIRA A TU ALREDEDOR Y…
otra imperativa. … ve más allá
c) Realizando los cambios oportunos, a partir de Mi único 1. El 30 de octubre de 1938 se emitió en la radio, en forma de
secreto es el laberinto, redacta una oración exclamativa, noticiario, la adaptación que Orson Welles había realizado
una dubitativa y otra desiderativa. de la novela La guerra de los mundos. En el programa, de
d) Separa los distintos monemas que integran cada una casi una hora de duración, se narraba una invasión alie-
de estas palabras: firmeza, conlleva, salida, recorrido, nígena. Como muchos oyentes no habían escuchado la
aguardan, redobla, oleaje, sobrevivirá. introducción donde se explicaba que todo era una drama-
tización, en Estados Unidos se desató el pánico colectivo
e) Explica el uso de la tilde en las palabras táctica, Historia,
al darse como cierta la invasión del planeta.
remató, único, está, misterios, inútil, dispóngase, iniciar,
LOCUTOR 1.— La Columbia Broadcasting System y sus emisoras
aquí, jamás.
asociadas presentan a Orson Welles y el Teatro Mercurio
a) El fragmento en cuestión es un conjunto de enunciados que del Aire en… La Guerra de los Mundos, de H. G. Wells.
mantienen entre sí relaciones de adecuación, coherencia
[CONTROL . Sintonía de entrada (versión corta de 22 seg.)
y cohesión, e integra un texto al transmitir un contenido
hasta final].
con significado lógico. Fijémonos, por ejemplo, cómo el
primer párrafo se adecua a las características de la narra- LOCUTOR 1.— Señoras y señores, a continuación el director del
ción mediante el uso de los verbos en pretérito perfecto sim- Teatro Mercurio del Aire y protagonista de esta emisión…
ple. Asimismo, existe una cohesión léxica, pues en un texto Orson Welles.
donde la intriga es lo primordial ya en el primer párrafo se ORSON WELLES.— Sabemos ahora que, en los primeros años
usa el término riesgos, en el segundo se alude al inmediato del siglo XX, nuestro planeta estaba siendo observado muy
inicio de un recorrido y a las trampas que pueden toparse los atentamente por inteligencias superiores a las del hombre,
personajes. De forma similar, la urgencia temporal se revela aunque también tan mortales como las nuestras.
notable a partir de vocablos o expresiones como demora, Sabemos ahora que mientras los hombres se dedicaban afa-
cada minuto cuenta y sobrevivirá. nosamente a sus múltiples ocupaciones y negocios, estaban

Lengua castellana y Literatura. 3.º ESO. Solucionario 19


2 EL MÁGICO PODER DE LAS PALABRAS

siendo examinados y estudiados tan minuciosamente como • Creación de una identidad ficticia gracias al anonima-
el hombre mismo hace con un microscopio cuando examina to, lo cual promueve el aislamiento social al reducir
los microbios que se concentran y multiplican dentro de los contactos en vivo y sustituirlos por los virtuales.
una gota de agua.
• Falta de privacidad. Según la Agencia Española de
[…] Individuos plenamente convencidos de su dominio
Protección de Datos, el 43 % de los usuarios tiene
sobre este pequeño planeta del sistema solar, que, por
casualidad, o mejor dicho, por designio divino, el hombre
configurado su perfil de forma que todo el mundo
ha heredado, escapando, así, de la misteriosa oscuridad del puede verlo; eso hace que la privacidad pueda ver-
tiempo y del espacio. se en peligro. Resulta especialmente preocupante
Sin embargo, a través del inmenso universo, mentes que en el caso de los menores.
son a nuestras mentes como las nuestras lo son a las de • Falta de control de datos. Otras personas pueden
las bestias de la jungla, inteligencias poderosas, frías y apropiarse de lo que publicamos, incluso puede ha-
carentes de sentimientos, contemplaban con envidia nues- ber robos de identidad.
tro planeta Tierra. Seres que lentamente, pero con mucha • Riesgos por interactuar con extraños. Los niños y
seguridad, preparaban un plan contra nosotros.
adolescentes pueden resultar víctimas de pedófi-
Fue en el año 39 del siglo XX cuando llegó la gran desilusión. los, abusadores sexuales, personas antisociales y
A finales del mes de octubre sucedió lo inesperado. Los nego- delincuentes de todo tipo.
cios estaban en su mejor momento y el miedo a la guerra se
había alejado. Gran número de personas encontraba trabajo b) ¿Es posible sentirse solo aunque una persona tenga
nuevamente, y las ventas en los comercios se disparaban. muchos amigos en la cuenta de su red social? ¿Cómo
Era la noche del 30 de octubre. La agencia de noticias explicarías esta aparente contradicción?
Crossley estimó que unos treinta y dos millones de personas, c) Sin embargo, si eres partidario de las nuevas tecno-
en todo el país, tenían, en ese instante, conectada la radio. logías, ¿cuáles consideras sus principales ventajas?
a) Recaba información sobre el argumento de la novela Anótalas en tu cuaderno.
de H. G. Wells La guerra de los mundos y elabora un d) En otro sentido, el uso indiscriminado de la informa-
breve resumen. ción suministrada por Internet puede fomentar prácti-
b) ¿A qué género narrativo pertenece la obra? Enumera cas como la piratería. Define los conceptos de plagio,
otros títulos literarios de la misma índole que se hayan derecho de autor y propiedad intelectual.
adaptado al cine. ¿Dónde y cuándo transcurren tales e) ¿Por qué no resulta ético plagiar el trabajo de otras
historias? personas?
c) Lo sucedido tras la emisión del guion de Orson Welles a), b) y c) Respuesta libre.
demuestra el poder de sugestión de los medios de d) Plagio: copia intencionada de una obra ajena para ofrecer-
comunicación. En cambio, Julio Verne escribió rela- la como propia.
tos cuyo contenido resultaba fantástico en su época y Derecho de autor: en inglés, copyright, hace referencia a
luego se hizo realidad. Ilustra este curioso fenómeno los derechos morales y económicos que le corresponden
con dos novelas del escritor francés. a un autor como artífice de cualquier obra, ya sea litera-
ria, musical, artística o científica. Su importancia es tal
d) A veces, la realidad supera a la ficción. Recuérdales a tus que están contemplados en la Declaración Universal de
compañeros terribles catástrofes que hayan pasado a la los Derechos Humanos.
gran pantalla con un éxito clamoroso. ¿En qué género Propiedad intelectual: la posee el creador sobre cualquier
cinematográfico incluirías cada uno de los títulos citados? obra o invención surgida de su mente, y engloba tanto a obras
a) Respuesta libre. artísticas como a inventos que pasan a ser de uso habitual.
b) La actividad está orientada a que el alumno reconozca el e) Respuesta libre.
papel de la narrativa de ciencia ficción como suministra- 2. Observa la siguiente imagen. Pertenece a una campaña
dora de argumentos al cine. Pero se puede ampliar a otros contra la piratería del Ministerio de Cultura.
géneros narrativos como el fantástico, épico, etc.
a) ¿Qué argumentos esgrime en contra de la piratería?
c) y d) Respuesta libre.
b) ¿Qué medidas crees que se podrían proponer para
… encuentra la clave contrarrestarla?
1. La rápida implantación de las redes sociales y su uso c) ¿Cómo se puede conseguir que la gente cobre con-
masivo han suscitado distintas polémicas que plantean la ciencia sobre la importancia de proteger el trabajo de
necesidad de un control y un uso responsable. creación?
a) De entre los inconvenientes que se enumeran a conti- d) ¿Alguna vez has pirateado algo? ¿Por qué?
nuación, extraídos del blog Psicología Para Vivir Mejor a) La imagen subraya la importancia que tienen las ideas en
(psicologiavivirmejor.blogspot.com.es), indica los que nuestra vida cotidiana, no solo como fuente de placer y
te resultan más preocupantes y argumenta por qué: disfrute (las leemos, las cantamos, bailamos), sino como
instrumento del progreso humano: hacen crecer y vivir y
• Adicción. Se estima que hay un 6 % de adictos a In-
tener nuevas ideas. Si nosotros recurrimos a la práctica
ternet. La red consume una gran cantidad de tiempo de la piratería y robamos, estamos poniendo en riesgo la
y el exceso de información produce sobrecarga, di- supervivencia de la creatividad y con ella de un patrimo-
ficultando la discriminación de contenidos y el equi- nio humano cuyas virtudes se han enfatizado. Con ello,
librio en el tiempo utilizado. la imagen implica al destinatario en una cuestión que nos
• Fracaso escolar entre los adolescentes por el abuso afecta a todos y no solo a los creadores.
de las nuevas tecnologías. b), c) y d) Respuesta libre.

20 Lengua castellana y Literatura. 3.º ESO. Solucionario


ÚLTIMA HORA 3
APERTURA DE UNIDAD 3. Explica la relación entre la muerte anunciada del persona-
je y la percepción de sus vecinos de que «era un tiempo
1. El texto es un fragmento de la famosa novela de terror
fúnebre, con un cielo turbio y bajo y un denso olor de
Drácula, publicada en 1897. Lo que acabas de leer refle-
aguas dormidas». ¿Qué valor simbólico les atribuyes a
ja el momento en que Jonathan Harker, uno de los per-
los elementos enumerados (tiempo fúnebre, cielo turbio
sonajes principales, ve por primera vez a Drácula en el
y bajo, aguas dormidas)? Presta especial atención a los
ambiente lúgubre de la cripta de su castillo. ¿Con qué
adjetivos.
expresiones consigue el autor crear el ambiente terrorí-
fico que domina la escena? Muy a menudo asociamos el tiempo meteorológico con nues-
tros estados de ánimo, por lo que aquí, el tiempo nublado,
Hay dos oraciones dentro de este fragmento que contribuyen
lluvioso y desapacible de esos días parece tener una relación
claramente a crear la atmósfera de terror. La primera es la
con la tristeza, tan cercana a la muerte. Las expresiones que
siguiente: No sabría decir si estaba muerto o dormido: tenía
emplea el autor contribuyen a esa sensación de muerte, sobre
los ojos abiertos y como petrificados, aunque sin la vidriosi-
todo a través de los adjetivos: el primero, fúnebre, remite di-
dad de la muerte. La incertidumbre de si el conde está muerto
rectamente a ella, los demás subrayan la oscuridad y la tris-
o dormido, unida a esos ojos petrificados y a la expresión
teza: cielo turbio, aguas dormidas. No es difícil relacionar
vidriosidad de la muerte generan el espanto en el lector. Del
estas sensaciones con la muerte.
mismo modo, cuando al final del texto leemos: Al ir a regis-
trarle vi que sus ojos sin vida, aunque estaban apagados y 4. ¿Quién crees que puede ser el narrador? ¿Tendrá algún
no se daban cuenta de mi presencia, tenían tal expresión de tipo de relación con Santiago Nasar? Razona tus respuestas.
odio, que hui de aquel lugar, vemos también cómo se produce El narrador parece ser un amigo o conocido cercano de San-
un nuevo momento de terror, ya que los ojos sin vida le mues- tiago Nasar, que cuenta la historia de primera mano, como
tran al narrador el odio que aquel ser a medio camino entre la observador presente en ella. Esto lo podemos deducir de que
vida y la muerte le profesa. afirma haber estado en la misma fiesta que el protagonista
2. De esta novela se han hecho varias versiones cinemato- (Yo estaba reponiéndome de la parranda de la boda…). Ade-
gráficas. Ved en clase alguna escena que recree el mo- más, cuando las campanas tocaron a rebato, él pudo oírlas
mento recogido en el texto y luego, en grupos de cuatro, mientras se encontraba pasando la noche con María Alejan-
comentad las semejanzas y diferencias que hayáis encon- drina Cervantes, por lo tanto vivía en el mismo pueblo que
trado. Para ello, organizaos así: Nasar. También conocía a la madre de este, quien le contó los
sueños premonitorios que no supo interpretar.
a) Poned en común vuestras ideas en el grupo.
b) Elegid un portavoz. Este expondrá en voz alta las con- 5. Explica la expresión «era un atuendo de ocasión», según
clusiones. el contexto en el que se encuentra.
La expresión atuendo de ocasión aparece en el siguiente frag-
c) A continuación, se abrirá un debate sobre las dos ver-
mento, ahora omitido en la lectura: «Santiago Nasar se puso
siones, la de la novela y la de la película.
un pantalón y una camisa de lino blanco, ambas piezas sin al-
d) Valorad ambas versiones y mostrad vuestras prefe- midón, iguales a las que se había puesto el día anterior para
rencias razonadamente y respetando los turnos de la boda. Era un atuendo de ocasión. De no haber sido por la
palabra. llegada del obispo se habría puesto el vestido de caqui y las
Respuesta libre. botas de montar con que se iba los lunes a El Divino Rostro, la
hacienda de ganado que heredó de su padre». Quiere decir que
3. En grupos de cuatro, contad cada uno una historia de
era una forma de vestir ocasional, con la que no solía ataviarse.
terror. A continuación, elegid la que más os ha gustado;
su autor se la contará a la clase. 6. Sabiendo desde el principio del texto que van a matar al
Respuesta libre. protagonista, ¿cómo interpretarías la siguiente oración?:
4. Elige la historia que más te ha gustado de entre las que «Siempre dormía como durmió su padre, con el arma es-
han contado tus compañeros y redáctala con tus palabras condida dentro de la funda de la almohada, pero antes
en tu cuaderno. de abandonar la casa aquel día le sacó los proyectiles y la
puso en la gaveta de la mesa de noche». ¿Tiene un com-
Respuesta libre.
ponente irónico?, ¿marca la fatalidad del destino? Razona
VIVE LA LECTURA tu respuesta.
En efecto, la oración tiene un tono irónico, ya que si no hubiera
Comprensión lectora descargado la pistola, tal vez la hubiera llevado luego en el
1. ¿Por qué los sueños de Santiago Nasar, relatados en el bolsillo cuando salió por la mañana, antes de que lo mataran.
primer párrafo, pueden considerarse un mal presagio? El destino del protagonista parece estar marcado, pues a la
Aparecen una serie de elementos negativos o, al menos, poco incapacidad de su madre de interpretar los sueños que tuvo
agradables, como sentirse salpicado de cagada de pájaros o hay que unir también la casualidad de que, precisamente esa
volar en un avión de papel de estaño. Esto último da una idea noche, no dejara cargada la pistola.
de fragilidad muy marcada, como si fuera imposible no estre-
llarse volando en ese avión. También contribuye a lo negativo la Vocabulario
expresión no había tenido ningún augurio aciago, ya que esta 7. ¿Qué valor tiene en el texto el adverbio tampoco con el
confianza de la madre de Santiago parece subrayar que preci- que comienza el segundo párrafo?
samente sí había algo aciago, terrible en los sueños de su hijo.
El adverbio tampoco sirve para enlazar lo que se cuenta en el
2. ¿A qué atribuye Santiago Nasar el dolor de cabeza con primer párrafo con lo que se cuenta en el segundo, y, desde
el que se levantó aquella mañana? el punto de vista del contenido, suma una circunstancia más
A la fiesta de bodas en la que estuvo la noche anterior, que le que impidió que Santiago Nasar tuviera cuidado en esa ma-
produjo, a su entender, la resaca. ñana que fue terrible para él.

Lengua castellana y Literatura. 3.º ESO. Solucionario 21


3 ÚLTIMA HORA

8. Busca los adjetivos que hay en el primer párrafo y explica g) Los niños estaban hartos de papilla.
luego qué sensaciones producen en el lector (positivas, h) El color azul marino es el preferido de Noelia.
negativas, de esperanza, de inquietud, de alegría, de tris-
i) Alfonso está bastante satisfecho de las notas de su
teza, etc.).
hijo.
Los principales adjetivos son tierna, feliz, ingrato y aciago.
a) Tenían la cabeza llena de fantasías.
Como vemos, mientras los dos primeros comunican una sensa-
ción agradable, placentera y tranquila, los dos últimos contras- Adj + SPrep
tan con ellos por comunicar todo lo contrario: lo desagradable b) La madre de Diana es una mujer muy alta.
y lo terrible. La felicidad que muestra el estado de ánimo del Adv + Adj
protagonista chocará con la inquietud y los malos presagios que
c) Pintaron el coche de amarillo chillón.
parecían indicar sus sueños y que nadie fue capaz de descifrar.
Adj + Adj
9. Busca en el diccionario el significado de las siguientes
palabras del texto y escribe luego una oración con cada d) El café estaba demasiado caliente y nos quemamos la
lengua.
una de ellas: estragos, parranda, destazado, presagio.
Adv + Adj
Estragos: ruina, daño, asolamiento.
Parranda: juerga bulliciosa, especialmente la que se hace e) Me gustan los ojos azules de Sara.
yendo de un sitio a otro. Adj
Destazado: hecho pedazos. f) El pobre anciano estaba muy cansado de la vida.
Aciago: infausto, infeliz, desgraciado, de mal agüero. Adv + Adj + SPrep
10. En el siguiente fragmento cambia las palabras mar- g) Los niños estaban hartos de papilla.
cadas en negrita por otras que tengan el mismo significa- Adj + SPrep
do o equivalente. Si no encuentras una palabra, puedes h) El color azul marino es el preferido de Noelia.
emplear también una expresión que signifique lo mismo: Adj + Adj
Tenía una reputación muy bien ganada de intérprete certera i) Alfonso está bastante satisfecho de las notas de su
de los sueños ajenos, siempre que se los contaran en ayunas, hijo.
pero no había advertido ningún augurio aciago en esos dos
sueños de su hijo, ni en los otros sueños con árboles que él le Adv + Adj + (SPrep + SPrep)
había contado en las mañanas que precedieron a su muerte. 2. En las oraciones anteriores hay algunos cuantificadores.
Tenía una fama muy bien ganada de descifradora certera de Localízalos y escríbelos en tu cuaderno.
los sueños de los otros, siempre que se los contaran antes de Cuantificadores: muy alta, demasiado caliente, muy can-
comer, pero no había percibido ningún presagio aciago en sado, bastante satisfecho.
esos dos sueños de su hijo, ni en los otros sueños con árboles
3. Clasifica los siguientes adjetivos según sean de una o dos
que él le había contado en las mañanas que antecedieron a
terminaciones en cuanto al género:
su fallecimiento.
vulgar anatómico terrible
Investigación y redacción atroz inquieto malvado
11. Busca información acerca de la novela a la que pertenece absurdo medieval maloliente
el texto y sobre su versión cinematográfica. Recoge los
salvaje imprudente inca
datos en tu cuaderno: director de la película, fecha de
estreno, actores principales, resumen del argumento, etc. rapaz indiferente estudioso
Respuesta libre.
Una terminación Dos terminaciones
12. Compara los argumentos de la novela y la película y haz
vulgar anatómico, -a
una lista con las diferencias que hayas encontrado.
terrible inquieto, -a
Respuesta libre.
atroz malvado, -a
13. Investiga sobre las características del realismo mágico e
medieval absurdo, -a
indica si encuentras alguna en el fragmento que acabas
maloliente estudioso, -a
de leer.
salvaje
Respuesta libre.
imprudente
ESTUDIO DE LA LENGUA inca
1. En las siguientes oraciones señala el sintagma adjeti- rapaz
val (SAdj) e identifica su estructura en función de sus indiferente
componentes:
a) Tenían la cabeza llena de fantasías. 4. Forma oraciones con los adjetivos de la actividad ante-
rior, poniendo especial atención a la concordancia entre
b) La madre de Diana es una mujer muy alta.
estos y el sustantivo al que califiquen. Para ello modifica
c) Pintaron el coche de amarillo chillón. el género y el número tantas veces como sea necesario.
d) El café estaba demasiado caliente y nos quemamos la Respuesta libre.
lengua. 5. En las siguientes oraciones identifica la función que des-
e) Me gustan los ojos azules de Sara. empeña el sintagma adjetival:
f) El pobre anciano estaba muy cansado de la vida. a) Los jugadores volvieron derrotados del campeonato.

22 Lengua castellana y Literatura. 3.º ESO. Solucionario


ÚLTIMA HORA 3
b) Contemplábamos el cielo rojizo del atardecer. e) mucho
c) Caminar es bueno para la salud. Adv
d) Las luces de la habitación estaban apagadas. f) bien / ayer
e) En casa comprobamos que habíamos comprado los Adv / Adv
yogures caducados. 9. Analiza las funciones que desempeñan los sintagmas ad-
a) Complemento predicativo (derrotados). verbiales de la actividad 8.
b) Adyacente (rojizo). a) Complemento circunstancial de modo.
c) Atributo (bueno para la salud). b) Complemento circunstancial de modo.
d) Atributo (apagadas). c) Complemento circunstancial de tiempo / Complemento
e) Complemento predicativo (caducados). circunstancial de tiempo / Complemento circunstancial
6. Di cuál es el adjetivo positivo que corresponde a los si- de lugar.
guientes superlativos: libérrimo, nobilísimo, felicísimo, agra- d) No hay SAdv.
dabilísimo, sapientísimo, fortísimo, altísimo, celebérrimo. e) Complemento circunstancial de cantidad.
Puedes ayudarte de un diccionario. f) Complemento circunstancial de modo / Complemento cir-
cunstancial de tiempo.
libérrimo libre
nobilísimo noble 10. Localiza en las siguientes oraciones los adverbios de can-
felicísimo feliz
tidad:
agradabilísimo agradable a) A Julia le gusta bastante el arroz con leche.
sapientísimo sabio b) Tuvieron bastante tiempo para hacer el trabajo.
fortísimo fuerte c) Había muchas ovejas pastando en aquel prado.
altísimo alto
d) Corrieron mucho y acabaron muy cansados.
celebérrimo célebre
e) Se queja demasiado demasiadas veces.
7. Transforma los adjetivos positivos extraídos de la activi- a) bastante
dad anterior en comparativos de inferioridad, de supe- d) mucho / muy
rioridad y de igualdad, y construye una oración con cada e) demasiado
uno de ellos.
11. En las siguientes oraciones, identifica las locuciones ad-
Inferioridad: menos libre que, menos noble que, menos feliz
verbiales, sustitúyelas por un adverbio y di luego qué
que, menos agradable que, menos sabio que, menos fuerte
que, menos alto que, menos célebre que.
función sintáctica desempeñan:
Superioridad: más libre que, más noble que, más feliz que, a) El hijo de Paqui no tiene ni un pelo de tonto.
más agradable que, más sabio que, más fuerte que, más alto b) Los especuladores gastaron el dinero a manos llenas
que, más célebre que. en los años de abundancia económica.
Igualdad: tan libre como, tan noble como, tan feliz como, tan
c) El instituto en el que estudio está donde da la vuelta
agradable como, tan sabio como, tan fuerte como, tan alto
el aire.
como, tan célebre como.
Respuesta libre. d) Los invitados al banquete comían a dos carrillos, sin
hablar con sus compañeros de mesa.
8. Indica la estructura de los sintagmas adverbiales que apa-
recen en las siguientes oraciones: e) Tus problemas con la informática me traen sin cuidado,
querido.
a) Tiraron bien fuerte y consiguieron abrir la puerta.
b) Sospecho que tu amiga Carla me mira mal. f) En el cumpleaños de Sara lo pasamos de maravilla.
c) Mañana será tarde para viajar tan lejos. g) Revisamos a conciencia todo el texto, por si había al-
gún error.
d) Es el payaso más gracioso del mundo.
h) A ti te salen las tortitas de rechupete.
e) Hemos trabajado mucho para superar esta prueba tan
dura. i) Se leyó la historia de cabo a rabo.
f) Los niños se portaron bien en la fiesta celebrada ayer. j) Acabé los deberes de matemáticas en un periquete.
a) bien fuerte a) El hijo de Paqui no tiene ni un pelo de tonto.
Adv + Adv (adjetivo adverbializado). El hijo de Paqui es muy listo.
b) mal La locución compone el predicado, lo mismo que la alterna-
Adv tiva que proponemos. Ni un pelo de tonto es complemento
c) Mañana / tarde / tan lejos directo; muy listo, atributo.
Adv / Adv / Adv + Adv b) Los especuladores gastaron el dinero a manos llenas en
los años de abundancia económica.
d) Es el payaso más gracioso del mundo.
No hay sintagma adverbial; tan solo un adverbio (más) Los especuladores gastaron el dinero abundantemente
complementando como cuantificador a un adjetivo en los años de abundancia económica.
(gracioso). Complemento circunstancial de modo.

Lengua castellana y Literatura. 3.º ESO. Solucionario 23


3 ÚLTIMA HORA

c) El instituto en el que estudio está donde da la vuelta el EL TALLER DE LAS PALABRAS


aire.
1. Averigua, con ayuda de un diccionario, la procedencia de
El instituto en el que estudio está muy lejos.
los siguientes préstamos léxicos y anótala en tu cuaderno
Complemento circunstancial de lugar. junto con su significado: holding, bigote, mejillón, capricho,
d) Los invitados al banquete comían a dos carrillos, sin blues, garaje, pizzicato, alcoba, affaire, pirsin, biombo, alio-
hablar con sus compañeros de mesa. li, paparazzi, abertzale, vigía, zaguán, café, viaje, rueca,
Los invitados al banquete comían mucho, sin hablar con chef, bandeja, grafiti, élite, footing, brindis, huracán, esqui-
sus compañeros de mesa. rol, glamour, beicon, bigudí, chirimbolo, chamizo, acequia,
Complemento circunstancial de cantidad. caramelo, tomate, bajel, albariño, zurrón, guiñol, chao.
e) Tus problemas con la informática me traen sin cuidado, • holding: anglicismo. Sociedad que controla las acciones de
querido. un grupo de empresas.
Tus problemas con la informática me importan poco, • bigote: germanismo. Pelo que crece sobre el labio superior.
querido. • mejillón: lusismo. Un tipo de molusco.
Complemento circunstancial de cantidad (poco). • capricho: italianismo. Determinación que se hace de ma-
f) En el cumpleaños de Sara lo pasamos de maravilla. nera arbitraria, sin razonar mucho.
En el cumpleaños de Sara lo pasamos muy bien. • blues: anglicismo. Música de origen africano propia de los
Estados Unidos.
Complemento circunstancial de modo.
• garaje: galicismo. Lugar cerrado donde aparcar un vehículo.
g) Revisamos a conciencia todo el texto, por si había algún
error. • pizzicato: italianismo. Técnica para tocar el violín y otros
instrumentos de cuerda, tirando con los dedos de estas
Revisamos muy bien todo el texto, por si había algún cuerdas, como si se pellizcaran.
error.
• alcoba: arabismo. Dormitorio.
Complemento circunstancial de modo.
• affaire: galicismo. Negocio o asunto poco lícito.
h) A ti te salen las tortitas de rechupete.
• pirsin: anglicismo. Perforación en una parte del cuerpo
A ti te salen las tortitas estupendamente. distinta de la oreja.
Complemento circunstancial de modo. • biombo: lusismo. Mampara móvil para separar espacios.
i) Se leyó la historia de cabo a rabo. • alioli: catalanismo. Ajo y aceite, un tipo de salsa hecha con
Se leyó la historia completamente. esos ingredientes y huevo.
Complemento circunstancial de modo. • paparazzi: italianismo. Fotógrafo, por lo general de la
prensa rosa.
j) Acabé los deberes de matemáticas en un periquete.
• abertzale: vasquismo. Patriota, nacionalista.
Acabé los deberes de matemáticas inmediatamente.
• vigía: lusismo. Persona que vigila, generalmente desde un
Complemento circunstancial de tiempo.
faro o una atalaya elevada.
12. Explica qué significan las siguientes locuciones adverbiales • zaguán: arabismo. Portal de una casa.
y construye una oración con cada una de ellas. Si lo nece- • café: italianismo. Bebida que se elabora con la infusión de
sitas, puedes utilizar un diccionario; en este caso, busca la la planta del mismo nombre.
palabra más importante de la locución (pierna, ojo, etc.): a • viaje: catalanismo. Acción de desplazarse de un lugar a otro.
pierna suelta, en un abrir y cerrar de ojos, a tontas y a locas,
• rueca: germanismo. Instrumento para hilar.
por un tubo, a la chita callando, de memoria, en mi vida, a
chorros, en un santiamén, a lo bestia, al pie de la letra, de • chef: galicismo. Cocinero, jefe de cocina.
primera mano. • bandeja: lusismo. Recipiente plano que sirve para trans-
portar bebidas o comidas.
• a pierna suelta: con absoluta tranquilidad. Se emplea con
el verbo dormir. • grafiti: italianismo. Letrero o dibujo que, por lo general, se
hace en las paredes.
• en un abrir y cerrar de ojos: con gran rapidez, de manera
inmediata. • élite: galicismo. Grupo selecto de personas.
• a tontas y a locas: sin pensarlo demasiado, hecho de ma- • footing: galicismo de procedencia inglesa. Carrera a velo-
nera irreflexiva. cidad moderada.
• por un tubo: en abundancia. • brindis: germanismo. Acción de brindar, beber expresando
algún deseo.
• a la chita callando: de manera disimulada, procurando que
• huracán: americanismo. Viento muy fuerte.
no se entere nadie.
• esquirol: catalanismo. Persona que trabaja durante una
• de memoria: sin olvidar nada, sin error.
jornada de huelga, con intención de perjudicar esta.
• en mi vida: nunca, jamás.
• glamour: galicismo. Encanto sensual.
• a chorros: de manera abundante, derrochando.
• beicon: anglicismo. Panceta ahumada.
• en un santiamén: inmediatamente. • bigudí: galicismo. Pequeño instrumento utilizado para en-
• a lo bestia: sin límites, de forma desmesurada. sortijar el cabello.
• al pie de la letra: palabra por palabra, sin dejarse nada sin • chirimbolo: vasquismo. Objeto de forma extraña que no
hacer o decir. tiene un nombre concreto.
• de primera mano: sin intermediarios. Se suele emplear • chamizo: galleguismo. Refugio hecho con ramas y hojas
junto al sustantivo información. de árboles.

24 Lengua castellana y Literatura. 3.º ESO. Solucionario


ÚLTIMA HORA 3
• acequia: arabismo. Conducto de agua para el riego de las • michelín: marca de neumáticos. También pliegues que se
huertas. forman en alguna parte del cuerpo, sobre todo la tripa,
• caramelo: lusismo. Golosina hecha con azúcar fundido y por semejanza con la figura que publicita esos neumáticos.
endurecido. Causas psicológicas.
• tomate: americanismo. Fruto de la tomatera. • enchufar: conectar un enchufe a la red eléctrica. También
• bajel: catalanismo. Barco. colocar a una persona en un puesto de trabajo por amis-
tad, familiaridad o cualquier otra causa irregular. Causas
• albariño: galleguismo. Vino blanco afrutado que se elabora psicológicas.
en Galicia.
• zurrón: vasquismo. Bolsa grande hecha de cuero o de otro
4. Busca eufemismos para los siguientes tabúes: negro (raza),
material. ciego, parir, basurero, retrete, morir, huelga, viejo (persona),
mear, vagabundo, criada, culo, gordo, loco, mendigo.
• guiñol: galicismo. Teatro de títeres.
• negro: de color, moreno, afroamericano.
• chao: italianismo. Adiós. se utiliza como saludo.
• ciego: invidente, deficiente visual.
2. Los siguientes anglicismos son innecesarios. Escribe una
palabra española que pueda sustituirlos: business, spon- • parir: dar a luz, alumbrar.
sor, pin, doping, hall, póster, mailing. • basurero: estación de residuos sólidos urbanos.
• business: negocio. • retrete: váter, aseo, baño, servicio.
• sponsor: patrocinador. • morir: fallecer.
• pin: insignia. • huelga: conflicto laboral.
• doping: dopaje. • viejo: anciano, persona mayor.
• hall: vestíbulo, recibidor. • mear: orinar.
• póster: cartel. • vagabundo: sin techo, sin hogar.
• mailing: buzoneo, correo.
• criada: empleada de hogar, asistenta.
3. Con ayuda de un diccionario, explica razonadamente qué • culo: trasero, pompis.
tipos de cambios semánticos se han producido en las si-
• gordo: grueso, rellenito.
guientes palabras: retrete, pringar, portátil, pluma, ven-
tana, afroamericano, satélite, embarcar, centro de salud, • loco: enfermo mental.
cabina, techo, michelín, enchufar. • mendigo: sin techo, sin hogar.
• retrete: pasa de referirse a un cuarto apartado para reti- 5. Las siguientes palabras han experimentado algún tipo de
rarse y estar en soledad, a un cuarto para realizar las ne- cambio semántico. Construye oraciones con ellas en las
cesidades fisiológicas. Causas históricas. que se perciban las diferencias entre sus significados y ex-
• pringar: impregnar algo de una sustancia pegajosa o prin- plica de qué tipo son los cambios producidos: caballero,
gosa. De ahí pasa a significar comprometer a alguien en navegar, azafata, portal, chato, vinilo, red, gorila.
algo poco agradable. Causas psicológicas.
Respuesta libre.
• portátil: objeto que se puede llevar de un lugar a otro. Ac-
tualmente un tipo de ordenador que se puede transportar 6. Intenta explicar el porqué de las siguientes etimologías
con facilidad. Por el contexto. populares: mondarina (por mandarina), destornillar-
• pluma: cada una de las piezas que cubren el cuerpo de las se (por desternillarse), andalias (por sandalias), alicóptero
aves. Pasó a significar instrumento para escribir con tin- (por helicóptero), esparatrapo (por esparadrapo), alqui-
ta, por fabricarse los primeros con estas piezas. Causas lino (por inquilino), lenteojos (por anteojos), doble dosis
históricas. (por sobredosis), doctor Rino (por otorrino).
• ventana: abertura en un muro de un edificio. Ahora, tam- Respuesta libre.
bién, espacio delimitado en la pantalla de un ordenador.
Causas históricas. EL RINCÓN DE LA NORMA
• afroamericano: causas sociales. Es un eufemismo para 1. Explica por qué colocamos la diéresis en pingüino o pira-
evitar la palabra negro, de connotaciones racistas, consi-
güismo y no en guarro o piragua.
derada un tabú.
La diéresis se pone cuando los grupos gue y gui deben pro-
• satélite: cuerpo celeste que gira alrededor de un planeta.
nunciarse distinguiendo la u y la i por separado, lo cual no
Por extensión, persona que depende de otra y que siempre
afecta nunca a los grupos con las vocales a y o (gua, guo). Por
anda tras ella. Causas psicológicas.
eso sí se pone en pingüino y piragüismo, pero no en guarro
• embarcar: subir a un barco o a un avión para realizar un y piragua.
viaje. También implicar a alguien en algo que puede ser
poco grato o costoso. Causas psicológicas. 2. Estas palabras designan especialidades médicas: odon-
• centro de salud: causas sociales. Eufemismo que pretende tología, ginecología, cardiología, urología. ¿Qué regla
evitar palabras como hospital o ambulatorio, que se con- ortográfica exige aquí usar la g? ¿Y en los numerales or-
sideran tabúes por su relación con la enfermedad. dinales: vigésimo, trigésimo, etc.?
• cabina: recinto pequeño para usos diversos. También, entre Odontología, ginecología, etc. se escriben con g porque la re-
otros muchos significados, lugar desde el que se proyectan gla dicta que sea así con las secuencias gia o gio, lleven tilde
las películas en el cine. Causas históricas. o no la lleven.
• techo: cubierta de un edificio. Se entiende también como También se escriben con g las palabras terminadas en -gé-
el puesto más alto logrado por alguien en un determinado simo, -gésima y -gesimal. De ahí que los numerales de esta
trabajo. Causas psicológicas. actividad deban escribirse con g.

Lengua castellana y Literatura. 3.º ESO. Solucionario 25


3 ÚLTIMA HORA

3. Resuelve en tu cuaderno estos revoltigramas: con mayor frecuencia, de la casa al contenedor amarillo. Los
hogares españoles reciclaron el 56,6 % de las 656.000 toneladas
QIEPAJUE:  _ Q _ _ _ _ J _
puestas en el mercado en 2013, una cantidad que convierte
ONOSGITACO:  C _ _ _ _ G _ _ S _ al país en el segundo de la Unión Europea que más recicla
XODEAREAG:  E _ _ G _ _ _ _ _ el material por habitante, según el informe que este martes
presentó Cicloplast.
QIEPAJUE  EQUIPAJE
Los españoles separaron 371.218 toneladas de plásticos el año
ONOSGITACO  CONTAGIOSO
pasado, un 3,7 % más que el año anterior y tres veces más que
XODEAREAG  EXAGERADO hace una década. Con este volumen se llenarían 28 estadios
4. Completa en tu cuaderno las palabras siguientes con g o del tamaño del Santiago Bernabéu.
j y forma con ellas diversas oraciones: Cada ciudadano recicló 7,7 kilogramos del material, una cifra
va_illa dili_ente pota_e pá_ina que se encuentra por encima de la media europea —de 7,1 ki-
logramos por habitante— y por delante de países como Reino
sumer_ir ru_ido analo_ía esp_ismo Unido, Italia, Bélgica y Finlandia. Alemania encabeza la lista
tatua_e an_inas extra_eron relo_ería con 12 kilogramos por habitante.
co_era in_erto hemorra_ia ori_en A pesar de los resultados, España «no puede dormirse», ma-
tizó Teresa Martínez, directora general de Cicloplast. Si bien
pilla_e farin_e _imnasia falan_e
el país «supera y duplica» la meta de reciclado establecida
ve_etal su_eto re_ente _eneral por la directiva europea en 2008, que se sitúa en el 22,5 %,
• vajilla • diligente • potaje • página Bruselas ya ha marcado un nuevo objetivo para 2025: un
• sumergir • rugido • analogía • espejismo 60 % de reciclado de plásticos y cero residuos reciclables en
los vertederos.
• tatuaje • anginas • extrajeron • relojería
El País, 30 de julio de 2014.
• cojera • injerto • hemorragia • origen
En la noticia podemos distinguir bien las tres partes princi-
• pillaje • faringe • gimnasia • falange
pales que la componen:
• vegetal • sujeto • regente • general
• El titular: Los hogares españoles, los segundos en reciclaje
5. Completa las siguientes palabras con la grafía correcta: de plástico en la Unión Europea.
her_í_ora, ama_ilidad, _i_liotecario, _íbora, absor_er, absol_ • La entradilla: Separaron el 56,6 % de las 656.000 toneladas
er, _icepresidenta, _iodegradable, _ete, e_idente, é_ano, puestas en el mercado en 2013, según Cicloplast.
disol_er, aperci_ir, contri_uir, mantu_imos, _ipartidista,
• El cuerpo: el resto de la noticia.
andu_o, tu_ieron, _enefactor, í_amos, estudia_a, _ioquími-
ca, decisi_o, bre_e. 2. ¿Cuáles de los factores de importancia (actualidad, proxi-
• herbívora • amabilidad • bibliotecario midad, magnitud e interés humano) están presentes en la
noticia anterior? Razónalo.
• víbora • absorber • absolver
El principal factor de importancia en este caso tal vez sea el
• vicepresidenta • biodegradable • vete
de la proximidad, ya que habla del porcentaje elevado y satis-
• evidente • ébano • disolver factorio obtenido por España en el reciclaje de plásticos. La
• apercibir • contribuir • mantuvimos noticia nos toca de cerca pues dice algo positivo sobre nuestro
• bipartidista • anduvo • tuvieron país. También es importante el interés humano, ya que mues-
tra la buena conciencia ecológica de un elevado porcentaje de
• benefactor • íbamos • estudiaba
españoles. Incluso podríamos decir que es actual, ya que en
• bioquímica • decisivo • breve los tiempos que corren se le da mucha importancia al reciclaje
6. Copia en tu cuaderno el siguiente dictado. Pon atención de residuos contaminantes.
a las grafías b/v: 3. ¿Crees que se respetan en esta noticia las características
a) Es obvio que van a disolver el gobierno a causa de la lingüísticas propias del género? ¿Por qué? Analiza cada
evasión del vicepresidente. una de ellas.
b) Si escribes con letra tan grande, no cabe todo el tema Respuesta libre.
de biología. 4. Busca información en Internet sobre el tema del recicla-
c) Me han atribuido una burla incisiva que yo no tuve a je y elabora un reportaje en el que incluyas estadísticas,
bien realizar. imágenes y otros datos de interés.
d) Elvira anduvo toda la víspera atareada preparando un Respuesta libre.
bizcocho nuevo. 5. ¿Te parece que es importante reciclar? Elabora una re-
Respuesta libre. dacción en la que manifiestes tu opinión.
Respuesta libre.
LA FACTORÍA DE TEXTOS
1. Lee la noticia e identifica las partes que la componen. ACTIVIDADES FINALES
Los hogares españoles, los segundos en reciclaje Repasa lo que has aprendido
de plástico en la Unión Europea 1. El siguiente texto es el primer capítulo de Platero y yo, de
Separaron el 56,6 % de las 656.000 toneladas puestas en Juan Ramón Jiménez. Léelo atentamente y responde des-
el mercado en 2013, según Cicloplast. pués a las actividades.
Botellas de agua y refrescos, envases de comida o detergen- Platero es pequeño, peludo, suave; tan blando por fuera, que
tes, bolsas, bandejas, tarrinas… Los plásticos van, cada vez se diría todo de algodón, que no lleva huesos. Solo los espejos

26 Lengua castellana y Literatura. 3.º ESO. Solucionario


ÚLTIMA HORA 3
de azabache de sus ojos son duros cual dos escarabajos de c) Se trata de un comparativo de igualdad, ya que muestra la
cristal negro. cualidad de la ternura de Platero asociada a la de un niño
Lo dejo suelto, y se va al prado, y acaricia tibiamente con y la equipara con este.
su hocico, rozándolas apenas, las florecillas rosas, celestes d) Se trata de un adjetivo explicativo, ya que va delante del
y gualdas… Lo llamo dulcemente: «¿Platero?», y viene a mí sustantivo y aporta una información que no es imprescin-
con un trotecillo alegre que parece que se ríe, en no sé qué dible para entender la oración. Sí es un epíteto, puesto que
cascabeleo ideal… la gota de miel, en sí misma, es cristalina, transparente.
Come cuanto le doy. Le gustan las naranjas mandarinas, las e) Adverbios: tan (adverbio de cantidad, cuantificador), solo
uvas moscateles, todas de ámbar; los higos morados, con su (adverbio de modo, complemento circunstancial de modo),
cristalina gotita de miel… tibiamente (adverbio de modo, complemento circunstan-
Es tierno y mimoso igual que un niño, que una niña…; pero cial de modo), apenas (adverbio de negación, complemen-
fuerte y seco por dentro, como de piedra. Cuando paso sobre to circunstancial de modo), dulcemente (adverbio de modo,
él, los domingos, por las últimas callejas del pueblo, los hom- complemento circunstancial de modo), dentro (adverbio de
bres del campo, vestidos de limpio y despaciosos, se quedan lugar, complemento circunstancial de lugar), cuando (ad-
mirándolo: verbio de tiempo, complemento circunstancial de tiempo).
—Tien’ asero… f) Todas es un pronombre indefinido, ya que está sustituyen-
Tiene acero. Acero y plata de luna, al mismo tiempo. do a las uvas moscateles que acaba de citar.
a) Busca los adjetivos que hay en el texto y clasifícalos 2. En el siguiente fragmento de la égloga I de Garcilaso de la
según sean de una o dos terminaciones en cuanto al Vega aparecen varios epítetos. Localízalos y explica qué
género. crees que aportan al contenido del texto.
Por ti el silencio de la selva umbrosa,
Una terminación Dos terminaciones por ti la esquividad y apartamiento
del solitario monte me agradaba;
por ti la verde hierba, el fresco viento,
el blanco lirio y colorada rosa
y dulce primavera deseaba.
b) Di qué funciones desempeñan los sintagmas adjetiva-
les siguientes: Selva umbrosa, solitario monte, verde hierba, fresco vien-
to, blanco lirio, colorada rosa, dulce primavera.
• Platero es pequeño
Los epítetos, marcados en negrita, repiten todos cualidades
• escarabajos de cristal negro
que ya están presentes en los sustantivos a los que acompa-
• lo dejo suelto ñan, por lo que su aportación al texto es el color, el ambiente,
• un trotecillo alegre lo sensitivo: todos ellos refuerzan el significado de los sustan-
c) ¿Qué grado del adjetivo está presente en la siguiente tivos e invitan al lector a disfrutar con los sentidos del paraje
que se describe.
frase: «Es tierno y mimoso igual que un niño»? Razona
tu respuesta. 3. Abusar de los adverbios terminados en -mente resulta poco
d) Por su posición y su significado, ¿qué tipo de adjetivo elegante y denota pobreza léxica. Sustituye los del siguien-
es cristalina en la expresión «con su cristalina gotita te texto por otros que no tengan esa terminación o por
de miel»? ¿Crees que podría ser un epíteto? ¿Por qué? expresiones equivalentes:
e) Localiza en el texto los adverbios presentes, di de qué Te diría que prácticamente recientemente hemos comprado
tipo son y analiza sus funciones. el coche que últimamente llevamos a casi todos los viajes,
porque en él se va perfectamente y divinamente, sobre todo
f) En el sintagma todas de ámbar, ¿qué tipo de palabra cuando ves a muchos conductores que caprichosamente te
es todas? Explica por qué. pasan velozmente y tú, tranquilamente, sigues a tu ritmo,
a) tan ricamente, sin necesidad de conducir locamente, que hay
Una terminación Dos terminaciones muchos que probablemente piensan que la carretera es mis-
mamente un circuito.
suave pequeño
rosas peludo
Te diría que apenas hace unas semanas hemos comprado el
coche que en los últimos días llevamos a casi todos los viajes,
celestes blando porque en él se va muy bien y muy a gusto, sobre todo cuando
gualdas duros ves a muchos conductores que por capricho te pasan a gran
alegre negro velocidad y tú, tan tranquilo, sigues a tu ritmo, tan campan-
ideal morados
te, sin necesidad de conducir a lo loco, que hay muchos que tal
vez piensan que la carretera es sin más un circuito.
fuerte cristalina
tierno
4. Lee la siguiente noticia y resuelve luego las actividades:
mimoso Una pitón de un metro lleva cuatro días suelta por
seco
Zaragoza
despaciosos La serpiente se escapó de un piso el lunes, cuando su
dueño salió a comprarle comida.
b) • Atributo. El Ayuntamiento emite un mensaje de tranquilidad por-
• Adyacente. que el animal no es peligroso.
• Complemento predicativo. El propietario ha presentado la correspondiente denuncia de
• Adyacente. extravío.

Lengua castellana y Literatura. 3.º ESO. Solucionario 27


3 ÚLTIMA HORA

Una pitón de un metro de largo, que al parecer se escapó de • Dónde: en Zaragoza, más concretamente en el domicilio
un piso de Zaragoza el pasado lunes cuando su dueño salió a de Cristian Gómez.
comprarle comida, anda suelta por la ciudad desde entonces, • Por qué: porque la ventana de la cocina estaba abierta
sin que por el momento se haya encontrado rastro del animal y la serpiente pudo salir por ella.
ni se conozca su paradero.
• Cómo: abandonando el terrario por un respiradero.
Cristian Gómez, de 32 años, ha informado a EFE de que sa-
lió de su casa un momento «para ir comprar un ratón a la Recuerda lo que ya sabías
serpiente» y, en ese intervalo de tiempo, «debió de salir por
un respiradero del terrario, con tan mala suerte que tenía la
5. En el texto de Platero y yo (actividad 1) aparecen los sus-
ventana de la cocina abierta y se escapó». tantivos trotecillo y florecillas. ¿Qué tipo de morfemas se
Después de dos horas buscándola por el piso, alertó a la Po-
han empleado en su formación? ¿Qué aportan desde el
licía Local y a los Bomberos de Zaragoza para decir que su punto de vista del significado?
pitón se había escapado. En los sustantivos trotecillo y florecillas se emplean morfe-
Hasta allí se desplazaron los efectivos de los Bomberos y de la mas apreciativos, más concretamente diminutivos, que desde
Policía Local, que estuvieron revisando terrazas del inmueble el punto de vista del significado, y en el contexto en que se si-
y algunos rincones del edificio, pero no han encontrado nada. túan, aportan un tono cariñoso y de delicadeza, más allá de
su significado de algo más pequeño.
Fuentes del Ayuntamiento de Zaragoza han emitido un men-
saje de tranquilidad a la población porque el animal no es 6. En la siguiente oración identifica los distintos sintagmas
peligroso. que la componen y señala sus partes: «Los espejos de aza-
www.elmundo.es, 1 de agosto de 2014. bache de sus ojos son duros».
a) ¿Cuáles de los factores principales de una noticia (actua- Los espejos de azabache de sus ojos son duros:
lidad, proximidad, magnitud e interés humano) ves pre- • Los espejos de azabache de sus ojos (SN: Det + N + SPrep
sentes en ella? Razona la respuesta y ten en cuenta el + SPrep).
medio de comunicación en el que se ha difundido la • son duros (SV: V + SAdj).
noticia. 7. Di cuál es el sujeto en el fragmento de la égloga I de Garci-
b) ¿Te parece que esta noticia tiene la suficiente relevan- laso de la actividad 2 y explica sus características.
cia para figurar en un periódico de tirada nacional? ¿Por El sujeto es la primera persona del singular, yo, tal y como
qué? deducimos de los verbos: agradaba y deseaba. Se trata de un
sujeto elíptico u omitido, ya que no está presente en el texto
c) Analiza en el texto las características lingüísticas (obje- pero se puede identificar con facilidad.
tividad, claridad, concisión y corrección).
8. Divide las siguientes palabras en monemas y explica de
d) ¿Te parece que el titular está bien planteado? Razona qué tipo son en función de la combinación de estos: pe-
tu respuesta. ludo, cascabeleo, despaciosos.
e) ¿Están presentes en la entradilla las respuestas a las Peludo, cascabeleo, despaciosos.
seis preguntas básicas que deben resolverse en ella?
• peludo: pel-udo (derivada).
Copia en tu cuaderno las preguntas y sus respuestas.
• cascabeleo: cascabel-eo (derivada).
a) La noticia relata un suceso menor que, a lo sumo, podría
tener el interés de la proximidad si el periódico se difun- • despaciosos: despac-iosos (derivada).
diera solo en Zaragoza, pero al ser una noticia en un medio 9. Analiza la expresión «¿Platero?». ¿Crees que es una ora-
nacional y digital, casi carece de relevancia en este sentido. ción? ¿Podrías asimilarla a alguna de las modalidades ora-
Tal vez podamos percibir el interés humano, ya que tanto el cionales que conoces? Ten presente el contexto.
dueño de la serpiente como la policía local, los bomberos y La expresión ¿Platero? no es en sentido estricto una oración,
el ayuntamiento, han actuado para proteger a la población. pero sí es una frase, ya que, aunque no tiene verbo, sí expresa
b) Respuesta libre. un significado completo, pues entendemos una pregunta en
c) Todas estas características están presentes en el texto. La la que se pueden haber eliminado algunas palabras, como por
objetividad no se puede poner en duda, ya que el autor de ejemplo: Platero, ¿vienes? En este sentido podríamos asimilar
la noticia se limita a informar sin hacer ningún comentario la expresión al grupo de las interrogativas.
sobre lo que cuenta. La claridad se percibe en que tras la
lectura del texto somos capaces de imaginar perfectamente MIRA A TU ALREDEDOR Y…
la situación y no nos queda ninguna duda sobre los hechos
… ve más allá
narrados. También está presente la concisión, pues la noti-
cia se cuenta con brevedad y con las palabras exactas, sin 1. Una de las funciones principales del uso de los adjetivos
dar rodeos innecesarios que dificulten la comprensión del con un fin artístico en literatura es la de servir de adorno,
suceso. Por último, la corrección se percibe en los párrafos embellecer aquello que se dice con palabras que pueden
del texto, bien estructurados, en los que la información va ser innecesarias (o no imprescindibles), pero que despier-
progresando y se expresa lo acaecido de forma cronológica. tan la sensibilidad del lector y sirven de recreo a los sen-
d) Respuesta libre. tidos. Lee el siguiente fragmento del Cántico espiritual
e) • Qué: una serpiente pitón se ha escapado de la casa de de san Juan de la Cruz y fíjate en los adjetivos que te
su amo. marcamos en negrita:
• A quién: le sucede el hecho a Cristian Gómez, el propie- Mi amado, las montañas,
tario de la serpiente pitón. los valles solitarios nemorosos,
• Cuándo: en el momento en que Cristian Gómez salió las ínsulas extrañas,
de su casa para comprar un ratón para la comida de la los ríos sonorosos,
serpiente. el silbo de los aires amorosos,

28 Lengua castellana y Literatura. 3.º ESO. Solucionario


ÚLTIMA HORA 3
la noche sosegada montañas; de estos mansos alcores y terreros, desde donde
en par de los levantes de la aurora, se divisa un camino que va en zigzag hasta un riachuelo. Las
la música callada, auras marinas no llegan hasta esos poblados pardos de casu-
la soledad sonora, chas deleznables, que tienen un bosquecillo de chopos junto
la cena que recrea y enamora. al ejido. Desde la ventanita de este sobrado, en lo alto de la
Nuestro lecho florido, casa, no se ve la extensión azul y vagarosa; se columbra allá
de cuevas de leones enlazado, en una colina una ermita con los cipreses rígidos, negros, a los
en púrpura teñido, lados, que destacan sobre el cielo límpido. A esta olmeda que
de paz edificado, se abre a la salida de la vieja ciudad no llega el rumor rítmico
de mil escudos de oro coronado. y ronco del oleaje; llega en el silencio de la mañana, en la paz
azul del mediodía, el cacareo metálico, largo, de un gallo, el
Como ves, todos ellos dan color al texto, pero no son golpear sobre el yunque de una herrería. Estos labriegos se-
absolutamente necesarios para su comprensión. Sí lo cos, de faces polvorientas, cetrinas, no contemplan el mar: ven
son para conmovernos, para hacernos disfrutar con su la llanada de las mieses, miran sin verla la largura monótona
sonoridad y con su significado. Algo parecido ocurre con de los surcos en los bancales. Estas viejecitas de luto, con sus
ciertos adornos que se emplean en la música y que tie- manos pajizas, sarmentosas, no encienden cuando llega el cre-
nen como principal intención decorarla. Uno de ellos es púsculo una luz ante la imagen de una Virgen que vela por los
el trino, que consiste en alternar de manera rápida dos que salen en las barcas; van por las callejas pinas y tortuosas
notas cercanas. Tiene una especial relevancia en la voz, a las novenas, miran al cielo en los días borrascosos y piden,
sobre todo si esta es aguda (soprano, tenor). Escucha el juntando sus manos, no que se aplaquen las olas, sino que las
nubes no despidan granizos asoladores.
aria de la Reina de la Noche, de la ópera La flauta mágica
de Wolfgang Amadeus Mozart (1756-1791), interpretada José M ARTÍNEZ RUIZ, A ZORÍN: Castilla.
por la soprano Graciela Armendáriz (teclea en YouTube a) Date un paseo por tu ciudad o tu pueblo y observa bien
«Graciela Armendariz» «Reina de la Noche»), así como la todo lo que te encuentres. Después, haz una redac-
canción Días de verano, del grupo Amaral (búscala con ción en la que mantengas un tono parecido al del texto
su título en YouTube o Spotify). Presta atención a la voz y precedente, poniendo atención al uso de los adjetivos.
escucha los trinos. Su finalidad es, como la de los adjeti- b) Busca en Internet imágenes de paisajes pintados por
vos, recrear nuestros sentidos. algunos pintores y observa detenidamente sus detalles.
Actividad sin respuesta. Ejercitar y recrear los sentidos. Elige una de estas imágenes y redacta una descripción
2. Con la misma finalidad decorativa que acabamos de de lo que ves en ella. Te sugerimos algunos paisajistas
comentar, los adjetivos se pueden emplear en la descrip- españoles que fueron contemporáneos de Azorín: Joa-
ción, y esta nos la encontramos tanto en la literatura como quín Sorolla, Ignacio de Zuloaga, Ricardo Arredondo,
en la pintura. Lee el siguiente fragmento de la novela Flor Aureliano de Beruete.
de mayo (1895), de Vicente Blasco Ibáñez (1867-1928), y c) ¿Por qué crees que en el texto se le da tanta impor-
compara lo que nos dicen los adjetivos con el cuadro de tancia al contraste entre el paisaje castellano y el mar?
Joaquín Sorolla (1863-1923) Sacando la barca (1916), que te ¿Te parece que el paisaje que nos rodea puede influir
adjuntamos a continuación del texto: en nuestro carácter y en nuestros estados de ánimo?
Algunas «parejas» habían de aguardar en seco hasta el día si- Razona tus respuestas.
guiente, y para tirar de ellas entraban olas adentro los bueyes
de la Comunidad de Pescadores, hermosos animales, rubios d) El texto de Azorín contiene una serie de sustantivos
y blancos, enormes como mastodontes, moviéndose con una que se refieren a elementos del paisaje. Cópialos en tu
pesada majestad y agitando su enorme papada con la altivez cuaderno y busca en un diccionario el significado de
de un patricio romano. los que no conozcas. Asocia luego estos sustantivos a
Estas yuntas, que hundían la arena bajo sus pezuñas y de un tu experiencia con el paisaje de tu tierra: ¿te parece
tirón arrastraban las barcas más grandes, guiábalas Chepa. que son todos aplicables o para describirla necesita-
Los bueyes descritos por Blasco Ibáñez tienen las mismas rías otros?, ¿cuáles?
características que los que ha dibujado Sorolla: hermosos, Si tu tierra es Castilla, valora la descripción que hace
rubios, enormes… También se mueven con la misma lentitud Azorín en función de tu propia experiencia: ¿crees que
que parecen mostrar los de la pintura. Y sobre todo, vemos es acertada?, ¿se parece la Castilla que él describe a la
cómo realizan la misma acción en ambos casos: sacar una
que tú conoces?
barca del mar.
Lo que en Blasco Ibáñez son adjetivos, en Sorolla son colores, e) Observa los adjetivos que emplea Azorín y di qué sen-
formas y luz solar. saciones producen. Redacta de nuevo la descripción
con adjetivos opuestos a los que usa él.
… encuentra la clave a) Respuesta libre.
1. El paisaje de tu tierra es una referencia de tu propia vida. Te b) Respuesta libre.
ofrecemos un fragmento de un texto de Azorín (1873-1967) c) Respuesta libre.
en el que describe Castilla por medio del contraste con las
d) Sustantivos que se refieren a elementos del paisaje: mar,
tierras que están junto al mar:
campiñas, barrancales, terrazgos, quiebras, alcores, ria-
No puede ver el mar la solitaria y melancólica Castilla. Está chuelo, poblados, casuchas, bosquecillo, ejido, colina, er-
muy lejos el mar de estas campiñas llanas, rasas, yermas, mita, cipreses, olmeda, llanada, surcos, bancales.
polvorientas; de estos barrancales pedregosos; de estos te-
rrazgos rojizos, en que los aluviones torrenciales han abierto Respuesta libre.
hondas mellas; de estas quiebras aceradas y abruptas de las e) Respuesta libre.

Lengua castellana y Literatura. 3.º ESO. Solucionario 29


4 LIBERTAD DE EXPRESIÓN

APERTURA DE UNIDAD Con las dos mujeres, además, le unía una relación más cerca-
na, ya que, según cuenta, el padre de estas era el jefe del suyo.
1. En el texto se habla de la imagen que tienen de España
5. Al leer el siguiente fragmento, ¿qué idea nos hacemos de
en algunos países extranjeros. Valora los argumentos que
los padres de Marisol y Sonsoles? Describe su persona-
maneja el periodista del que habla el autor para justificar
lidad a partir de lo que aquí se nos dice: «Sonsoles y Ma-
la pregunta de si «España había superado sus demonios
risol, nombres cuidadosamente escogidos en el santoral
antiguos, las pasiones y los enfrentamientos de otro tiem-
para distinguirlos de los que se repetían en las muchachas
po». ¿Crees que son suficientes para caracterizar a los
corrientes».
españoles o son solo tópicos? Razona tus respuestas.
Al leer esta oración tenemos la sensación de que los padres de
Respuesta libre.
las mediasmujeres eran personas que pretendían mostrarse
2. La película La vaquilla (1985), de Luis García Berlanga, na- diferentes a los demás, sin duda por algún complejo de supe-
rra un episodio que pudo ocurrir durante la Guerra Civil rioridad que les hacía creerse más importantes que el resto
española, en un tono en el que se mezcla la comedia con de los habitantes del pueblo. Posiblemente fueran personas
la crítica sociopolítica. En ella se nos presenta la lucha de superficiales, que daban más importancia a las apariencias
los dos bandos enfrentados, llena de contradicciones y que a la gente, sobre todo si esta procedía de un grupo social
con un claro simbolismo al final. Ved la película y después inferior al suyo.
organizad un debate sobre la realidad española. 6. Explica de manera razonada el porqué del mote de las dos
Respuesta libre. hermanas.
3. En grupos de cuatro, buscad información sobre momen- Según cuenta el propio narrador, una de ellas era fea pero
tos de la historia de España en los que pueda ser posible tenía un cuerpo bonito, mientras la otra, más baja de estatura,
era muy guapa, a pesar de que su cuerpo era poco agraciado.
hablar de enfrentamientos, pasiones extremas y fanatis-
Por eso Cuelloduro les puso el mote, porque juntando la mitad
mos. Cada grupo se ocupará de un momento histórico de cada una de las hermanas, se formaba una mujer hermosa
(la época de la Inquisición, el reinado de Fernando VII, (una tía buena, en palabras de Cuelloduro).
la Guerra Civil de 1936, la Transición, etc.). Después, un
7. «Marisol apretaba los párpados con tanta fuerza como
portavoz de cada grupo expondrá ante los demás las con-
si alguien le hubiera exprimido un limón encima de los
clusiones. Finalmente, se promoverá un debate en el que
ojos». ¿Qué valores expresivos tiene esta oración? ¿Qué
se valorarán los temas citados arriba e intentaréis sacar
crees que aporta a la descripción que está haciendo el
conclusiones.
narrador?
Respuesta libre.
La expresividad de la oración es muy grande, ya que si a al-
VIVE LA LECTURA guien le echaran zumo de limón en los ojos, el escozor que esto
le produciría le llevaría, tal vez, a cerrarlos con fuerza, para
Comprensión lectora aliviar la sensación desagradable. Almudena Grandes muestra
1. Explica el significado de la expresión «avanzando como con esta expresión el desagrado que le producía a Marisol oír
su mote. También define al personaje como una persona iras-
si pisaran huevos». ¿Crees que es una expresión culta o
cible y poco amiga de bromas, con lo que el lector se hace una
coloquial? ¿Por qué?
idea bastante aproximada de la personalidad de esta joven.
La expresión avanzando como si pisaran huevos significa que
caminaban muy despacio. Es una expresión de uso coloquial, Vocabulario
pues suele emplearse, por lo general, en la conversación infor-
8. En el texto hay varias palabras compuestas, búscalas y
mal. El tono cómico y la expresividad de la frase refuerzan su
utilización familiar.
di cuál es su composición. ¿Crees que de esa manera se
refuerza el significado por medio de este recurso? ¿Por
2. ¿Qué quiere decir la frase «escupía sus dos apellidos»? qué?
¿El verbo escupir está empleado en sentido recto o figu-
Palabras compuestas: mediamujer (mediasmujeres), Cuello-
rado? Razona la respuesta.
duro, culibajo.
Escupía sus dos apellidos significa que los decía como con
• Mediamujer: media + mujer (determinante + sustantivo).
rabia, con malas maneras, para dejar claro cuáles eran esos
dos apellidos. El verbo escupir, aquí, está usado en sentido • Cuelloduro: cuello + duro (sustantivo + adjetivo).
figurado, pues no se refiere a su significado recto de arrojar • culibajo: culo + bajo (sustantivo + adjetivo).
saliva por la boca, sino a lo que acabamos de decir arriba: ha-
El uso de la composición ofrece muchas posibilidades expresi-
blar como escupiendo, con desprecio, arrojando las palabras
vas, al reducir a una sola palabra lo que en su origen son dos.
como si fueran escupitajos.
De esta manera se crea una nueva palabra. Por ejemplo, con
3. ¿Por qué, según el narrador, nunca podría Marisol quitar- el mote del tabernero, de las palabras independientes cuello
se el mote que le había puesto Cuelloduro? y duro (que pueden emplearse juntas como complemento la
Según el narrador, nadie había conseguido nunca, en su pue- segunda de la primera, pero escritas de manera separada),
blo, liberarse de un mote bien puesto, como lo era el de las extraemos una sola que nos ofrece, de manera muy gráfica,
mediasmujeres, a su entender. la realidad que quiere representar.
4. ¿Qué relación tiene el narrador con los personajes de los 9. Busca en el diccionario la palabra estilete y explica luego
que habla? Explícalo razonadamente. qué quiere decir en el contexto en que la sitúa la autora:
Tanto con Cuelloduo como con las hermanas Rodríguez Peñal- «zapatos de tacón altísimo, estiletes pensados para las
va, la relación del narrador es principalmente la de ser vecinos lisas aceras de las ciudades».
del mismo pueblo, tal y como nos dice este (en el pueblo todo el Un estilete, según el diccionario de la RAE es un puñal de
mundo la conocía como Mediamujer, las calles de mi pueblo). hoja muy estrecha y aguda. La autora utiliza la palabra como

30 Lengua castellana y Literatura. 3.º ESO. Solucionario


LIBERTAD DE EXPRESIÓN 4
metáfora para referirse a los tacones de los zapatos de las 13. Busca información sobre la posguerra española y sobre
dos hermanas, que, de tan finos como eran, se asemejaban la vida en los pueblos, y compara tus hallazgos con lo que
mucho a estiletes. Aprovecha también para decir que este tipo se nos dice en el texto.
de tacones era excesivo para las calles mal empedradas de su
pueblo, y criticar así la afectada y artificial elegancia de las Respuesta libre.
mediasmujeres. 14. La costumbre de poner motes está muy enraizada en los
10. ¿Cómo entendemos el adjetivo afilado en relación con el pueblos de España. Pregunta a tus padres o a amigos que
mote («además de afilado, era certero»)? conozcan de cerca la vida en una pequeña población y
elabora una lista de los motes que allí se usan.
Afilado es sinónimo de agudo, y aquí tenemos que entender-
Respuesta libre.
lo en este sentido; es decir, el mote, en opinión del narrador,
estaba muy bien ideado y era muy ingenioso, perfecto casi.
ESTUDIO DE LA LENGUA
11. Busca y copia en tu cuaderno los adjetivos que aparecen
1. Identifica el sintagma verbal en las siguientes oraciones y
en el primer párrafo del texto y analiza su significado se-
a continuación separa el lexema y los morfemas del verbo:
gún su posición y según la intención del narrador.
a) Todas las tardes, Elvira y yo paseábamos por la orilla
• tardes tontas: por su posición, es un adjetivo especifica-
del mar.
tivo, que pretende diferenciar con claridad unas tardes de
otras. Tontas con el sentido de aburridas, monótonas. b) Encontró las llaves de su casa en el contenedor de
basura orgánica.
• las dos muy tiesas: también este adjetivo es especificativo,
por su posición posterior al nombre (en este caso el pronom- c) Los bomberos apagaron el fuego en menos de quince
bre dos, que sustituye a las hermanas). La autora marca con minutos.
este adjetivo la altivez y soberbia de las mediasmujeres. d) Nos gusta mucho la paella valenciana.
• a duras penas: la expresión es una locución adverbial, que e) ¿Recibiste la carta que te envié?
equivale a dificultosamente, por lo que el adjetivo forma El predicado se marca en negrita:
parte de dicha locución y no admite que lo analicemos por
a) Todas las tardes, Elvira y yo paseábamos por la ori-
separado.
lla del mar.
• tacón altísimo: una vez más un adjetivo especificativo, • paseábamos: pase-ába-mos.
situado detrás del nombre. En este caso es destacable,
b) Encontró las llaves de su casa en el contenedor de
igualmente, el uso del superlativo absoluto (-ísimo), con
basura orgánica.
el que la autora quiere ponderar la altura de los tacones
que llevaban Sonsoles y Marisol. • encontró: encontr-ó.
c) Los bomberos apagaron el fuego en menos de quince
• lisas aceras: aquí, en cambio, tenemos un adjetivo expli-
minutos.
cativo, colocado delante del sustantivo para matizar, de
pasada, una cualidad de este que no se considera impres- • apagaron: apag-aron.
cindible para la comprensión de lo que se quiere decir: solo d) Nos gusta mucho la paella valenciana.
con referirse a las aceras ya intuimos el suelo más regular • gusta: gust-a
de la ciudad con respecto al empedrado del pueblo. e) ¿Recibiste la carta que te envié?
• chinos redondos: adjetivo especificativo, que persigue de- • recibiste: recib-iste.
finir cómo eran esos chinos, nombre con el que se refiere la • envié: env-ié.
autora a las piedras de las calles de Fuensanta de Martos.
2. En las siguientes oraciones localiza las perífrasis y clasifí-
• rosa pálido: adjetivo especificativo que matiza la tonali- calas según su significado:
dad rosa de la rebeca.
a) Alfonso acaba de dejar Internet y ya está zapeando en
• azul celeste: exactamente lo mismo que en el caso anterior, el televisor.
variando el color y el matiz.
b) Iba a saltar desde el trampolín, pero ella se puso a
• rebeca de punto fino: también este es un adjetivo especi- gritar y me hizo bajar.
ficativo que pretende recalcar la labor primorosa con que
estaban tejidas las rebecas de las dos hermanas.
c) Dieron las tres y debí marcharme: ni se ponían a comer
ni me invitaban.
• perlas diminutas: como en el caso anterior, la pequeñez
expresada por el adjetivo especificativo aporta un refina-
d) Los cuatro amigos se echaron a descansar cuando tu-
miento a las mujeres, cuyas perlas no eran toscos y grandes vieron hechas todas sus tareas.
diamantes ostentosos, sino delicadas piedras diminutas. e) Andan diciendo que te vas a casar.
• barbilla alta: adjetivo especificativo con el que se nos quie- Las perífrasis aparecen marcadas en negrita:
re dar a entender, una vez más, la soberbia de las llamadas a) Alfonso acaba de dejar Internet y ya está zapeando en
mediasmujeres, que adoptaban esa postura de altivez ante el televisor.
los demás habitantes del pueblo. • acaba de dejar: aspectual perfectiva (acabar de +
infinitivo).
Investigación y redacción • está zapeando: aspectual durativa (estar + gerundio).
12. Fuensanta de Martos refleja la vida de cualquier pueblo b) Iba a saltar desde el trampolín, pero ella se puso a
durante la posguerra. A partir del texto, haz una redac- gritar y me hizo bajar.
ción en la que describas cómo crees tú que debía de ser • iba a saltar: aspectual incoativa (ir a + infinitivo).
el pueblo en aquel tiempo. • se puso a gritar: aspectual incoativa ( ponerse a +
Respuesta libre. infinitivo).

Lengua castellana y Literatura. 3.º ESO. Solucionario 31


4 LIBERTAD DE EXPRESIÓN

c) Dieron las tres y debí marcharme: ni se ponían a co- b) Alba me informó de su decisión de no acudir a la reu-
mer ni me invitaban. nión.
• debí marchar me: modal de obligación (deber + c) La carta fue firmada por todos los asistentes.
infinitivo).
d) Encontramos abiertas las ventanas de nuestra casa.
• se ponían a comer: aspectual incoativa (ponerse a +
infinitivo). e) Me habló de su relación con Marisa en la cafetería de
d) Los cuatro amigos se echaron a descansar cuando tuvie- la estación.
ron hechas todas sus tareas. f) Esteban abría las nueces con un martillo.
• tuvieron h ech a s: aspectual perfectiva (tener + g) Carolina es la amiga íntima de Lola.
participio).
a) • por el suelo: complemento circunstancial de lugar.
e) Andan diciendo que te vas a casar.
• todos los juguetes de Luisito: complemento directo.
• andan diciendo: aspectual durativa (andar + gerundio).
b) • me: complemento indirecto.
• te vas a casar: aspectual incoativa (ir a + infinitivo).
• de su decisión de no acudir a la reunión: complemento
3. Indica si las siguientes oraciones tienen complemento de régimen.
directo, complemento indirecto o ambos, y delimítalos.
c) • por todos los asistentes: complemento agente.
a) Implicamos a María en la organización de la fiesta. d) • abiertas: complemento predicativo.
b) Enrique contó a Silvia una historia de terror. • las ventanas de nuestra casa: complemento directo.
c) Desde el barco vimos a lo lejos las luces del puerto de e) • me: complemento indirecto.
Valencia.
• de su relación con Marisa: complemento de régimen.
d) En la piscina había muchas avispas alrededor de la
• en la cafetería de la estación: complemento circunstan-
ducha. cial de lugar.
e) Amelia habló a Joaquín de sus problemas. f) • las nueces: complemento directo.
a) A María: complemento directo. • con un martillo: complemento circunstancial de ins-
b) A Silvia: complemento indirecto. Una historia de terror: trumento.
complemento directo.
g) • la amiga íntima de Lola: atributo.
c) Las luces del puerto de Valencia: complemento directo.
6. Indica cuáles de las oraciones de la actividad anterior son
d) Muchas avispas: complemento directo. transitivas y explica por qué.
e) A Joaquín: complemento indirecto. Las oraciones transitivas son las que llevan complemento di-
4. Reconoce en las siguientes expresiones los casos de recto, por lo que, de las anteriores, pertenecen a esa categoría
laísmo, loísmo y leísmo que estén presentes y escríbelas las siguientes:
luego correctamente: • Tiraron por el suelo todos los juguetes de Luisito.
• Encontramos abiertas las ventanas de nuestra casa.
Corrige
• Esteban abría las nueces con un martillo.
a) A Sergio le he visto en la estación. 7. Transforma en pasivas las oraciones que acabas de selec-
b) Dalos la merienda a los niños. cionar en la actividad anterior. ¿Puedes poner en pasiva
c) La compré unas flores por su cumpleaños. el resto? ¿Por qué?
d) No las digas nada de lo que te conté. • Todos los juguetes de Luisito fueron tirados por el suelo.
e) Dilos que se callen de una vez. • Las ventanas de nuestra casa fueron encontradas abiertas
por nosotros.
f) A mis hijas las gustan mucho los caramelos.
• Las nueces eran abiertas por Esteban con un martillo.
a) A Sergio le he visto en la estación. El resto de las oraciones no se pueden poner en voz pasiva,
• leísmo: lo correcto es lo he visto. porque no tienen complemento directo, y este es imprescindi-
ble para realizar el proceso, pues es el que pasa a convertirse
b) Dalos la merienda a los niños.
en sujeto paciente.
• loísmo: lo correcto es dales la merienda.
8. Di razonadamente si las siguientes oraciones son reflexi-
c) La compré unas flores por su cumpleaños.
vas o recíprocas:
• laísmo: lo correcto es le compré unas flores.
a) El gato se rascaba una oreja con la pata derecha.
d) No las digas nada de lo que te conté.
b) Lucía y yo nos miramos a los ojos.
• laísmo: lo correcto es no les digas nada.
e) Dilos que se callen de una vez. c) Mis hermanos se pelean constantemente.
• loísmo: lo correcto es diles que se callen. a) Esta oración es reflexiva, ya que la acción realizada por
el sujeto (el gato) repercute sobre él mismo (rascarse).
f) A mis hijas las gustan mucho los caramelos.
b) Esta oración es recíproca, ya que se produce un intercam-
• laísmo: lo correcto es les gustan mucho.
bio de acciones entre los componentes del sujeto; en este
5. Analiza los complementos del verbo que hay en las si- caso Lucía y yo intercambiamos miradas.
guientes oraciones: c) Esta oración es recíproca, pues se produce también un in-
a) Tiraron por el suelo todos los juguetes de Luisito. tercambio: uno pelea contra el otro y viceversa.

32 Lengua castellana y Literatura. 3.º ESO. Solucionario


LIBERTAD DE EXPRESIÓN 4
9. Indica cuáles de las siguientes oraciones son transitivas EL TALLER DE LAS PALABRAS
y explica por qué:
1. Relaciona los verbos siguientes con las expresiones
a) Teníamos la cabeza llena de fantasías. que figuran más abajo: fijarse, desempeñar, declarar,
b) A Pilar le gusta mucho el arroz con leche. proporcionar.
c) Leí la increíble noticia en el periódico de mi barrio. a) dar información c) decir la verdad
d) Antonio se compró un libro muy antiguo en una librería b) tener un cargo d) tener una meta
de San Sebastián. a) dar información: proporcionar.
e) El tren llegó con media hora de retraso. b) tener un cargo: desempeñar.
f) Los jardines de Aranjuez son un lugar de gran belleza. c) decir la verdad: declarar.
g) Fuimos a Angola con un programa de intercambio. d) tener una meta: fijarse.
h) Ganarás el pan con el sudor de tu frente. 2. Señala los casos de deixis en el siguiente fragmento e in-
i) Explícame otra vez el teorema. dica si son anáforas o catáforas:
Kim meditó un momento. Tres años antes hubiera sacado todo
j) No entiendo a las personas pesimistas.
el provecho posible de su situación y hubiera continuado su
Las siguientes oraciones son transitivas porque tienen un com- camino como si tal cosa; pero el respeto que le manifestaba el
plemento directo (marcado en negrita): jat le probaba que era ya un hombre. Además, él también había
• Teníamos la cabeza llena de fantasías. tenido fiebres una o dos veces y las conocía lo bastante para
• Leí la increíble noticia en el periódico de mi barrio. reconocer la desnutrición cuando la veía.
• Antonio se compró un libro muy antiguo en una librería Rudyard K IPLING: Kim.
de San Sebastián. Aparecen tres casos de deixis textual y todas ellas son aná-
• Ganarás el pan con el sudor de tu frente. foras, ya que el elemento al que se refieren va delante: el res-
peto que (se refiere al respeto), las conocía (se refiere a las
• Explícame otra vez el teorema.
fiebres, citadas poco antes) y cuando la veía (se refiere a la
• No entiendo a las personas pesimistas. desnutrición).
10. Clasifica las siguientes oraciones en pasivas reflejas e im- 3. Sustituye por un sinónimo el término en negrita:
personales y explica por qué: a) Obtuvo el premio mediante muchos engaños.
a) Desde mi ventana se contempla un paisaje ma- b) Nuestros rivales eran equipos muy entrenados.
ravilloso.
c) Se demoró en el pago del alquiler.
b) No se ven bien las estrellas con este cielo tan nu-
blado. d) Los perros son animales leales.
c) En este país se lee poco. e) Siempre nos miraba con desdén.
d) Se siente, pero este año no habrá regalos para los ga- f) Tarzán creció en plena selva.
nadores del concurso. a) muchas mentiras

e) Se publicaron unos artículos muy interesantes en la b) oponentes, contrarios, enemigos


revista del instituto. c) se retrasó
f) Se come mucho en las cenas con amigos. d) fieles
e) desprecio
g) En equipo se trabaja de maravilla.
f) se crio, se desarrolló
h) Con este receptor no se escuchan bien algunas emi-
soras. 4. ¿Son sinónimos totales los verbos amar y querer? ¿Y gló-
bulos rojos y hematíes? ¿Por qué?
i) Se busca cocinero con experiencia.
Los verbos amar y querer no son sinónimos totales, porque,
a) Pasiva refleja, porque lleva un sujeto paciente: un paisaje
aunque pueden ser intercambiables perfectamente en algunos
maravilloso.
contextos (amo a Celia, quiero a Celia), no lo pueden ser en
b) Pasiva refleja, porque lleva un sujeto paciente: las otros (quiero un coche, *amo un coche).
estrellas.
Sin embargo, glóbulos rojos y hematíes sí son sinónimos tota-
c) Impersonal con se, ya que no existe ningún tipo de sujeto, les, pues son siempre intercambiables: hematíes es la forma
ni activo ni paciente. científica de referirse a los glóbulos rojos.
d) Impersonal con se, ya que no existe ningún tipo de sujeto, 5. Explica la diferencia de matiz (culto o familiar, más o me-
ni activo ni paciente.
nos intenso, etc.) entre estas parejas de sinónimos:
e) Pasiva refleja, porque lleva un sujeto paciente: unos artí-
culos muy interesantes.
prisión-chirona obedecer-acatar
f) Impersonal con se, ya que no existe ningún tipo de sujeto, ruido-estruendo difunto-finado
ni activo ni paciente. golpe-cachete finca-bien inmueble
g) Impersonal con se, ya que no existe ningún tipo de sujeto, • Prisión es más formal, mientras que chirona es coloquial.
ni activo ni paciente. • Obedecer es de uso común, también más coloquial, frente
h) Pasiva refleja, porque lleva un sujeto paciente: algunas a acatar, que parece más propio del lenguaje formal de las
emisoras. leyes.
i) Pasiva refleja, porque lleva un sujeto paciente: cocinero • Mientras que ruido es una palabra de uso común, estruen-
con experiencia. do parece tener un matiz más culto, tal vez literario.

Lengua castellana y Literatura. 3.º ESO. Solucionario 33


4 LIBERTAD DE EXPRESIÓN

• Difunto es una palabra de uso familiar, frente a finado que 3. Busca en un diccionario etimológico de dónde vienen
es, sin duda, una palabra más culta. las palabras hambre, harina, hermoso, hijo, hoja, hongo y
• Golpe muestra un uso menos coloquial que cachete. humo. ¿Explica su etimología el hecho de que actualmen-
• Si finca se asocia con un uso familiar, bien inmueble está te se escriban con h?
más en la línea de lo mercantil y de lo legal. • hambre: del latín vulgar famis.
6. Escribe los antónimos de las siguientes palabras e indica • harina: del latín farina.
de qué tipo de antonimia se trata: • hermoso: del latín formosus.
mortal alegre femenino nuera pequeño • hijo: del latín filius.
subir negro bueno sobrino fuera • hoja: del latín folia.
• mortal. Antónimo: inmortal. Complementario. • hongo: del latín fungus.
• alegre. Antónimo: triste. Gradual. • humo: del latín fumus.
• femenino. Antónimo: masculino. Complementario. El hecho de que todas estas palabras procedan del latín y co-
• nuera. Antónimo: suegra o suegro. Recíproco. menzaran con f- en esta lengua es la explicación de que en
español se escriban con h-, ya que al pasar del latín a esta
• pequeño. Antónimo: grande. Gradual.
última, la f- se transformó en h-.
• subir. Antónimo: bajar. Complementario.
4. Escribe dos palabras derivadas de hacer, de hombre y de
• negro. Antónimo: blanco. Gradual.
hierba. ¿Conservan la h?
• bueno. Antónimo: malo. Gradual.
• hacer: hacedero, hacedor.
• sobrino. Antónimo: tío o tía. Recíproco.
• hombre: hombría, hombruno.
• fuera. Antónimo: dentro. Complementario.
• hierba: herbolario, herbívoro.
7. Con los prefijos a-, des-, in- forma el antónimo de estas En estas palabras y en sus derivadas se mantiene la h- porque
palabras y escribe una oración con cada uno: proceden de palabras latinas que la tenían también.
coser digno orientada moral creíble 5. Escribe tres palabras que comiencen por cada uno de los
normal tejer hacer cauto real siguientes elementos compositivos: hecto-, helio-, hemo-,
• coser: descoser hemi-, hepta-, homo-, hiper-, hipo-.
• digno: indigno • hecto-: hectómetro, hectolitro, hectogramo.
• orientada: desorientada • helio-: heliocéntrico, heliotropo, helioscopio.
• moral: inmoral • hemo-: hemorragia, hemorroide, hemodiálisis.
• creíble: increíble • hemi-: hemiciclo, hemisferio, hemistiquio.
• normal: anormal • hepta-: heptaedro, heptasílabo, heptágono.
• tejer: destejer • homo-: homonimia, homosexualidad, homófono.
• hacer: deshacer • hiper-: hiperónimo, hipermercado, hipersensible.
• cauto: incauto • hipo-: hipocausto, hipogeo, hipodérmico.
• real: irreal 6. Escribe h donde corresponda:
Respuesta libre. __olgado __ormona __oval
__orfanato __eterogéneo __alde__uela
EL RINCÓN DE LA NORMA __omeopatía __invernar __emorragia
1. Escribe en tu cuaderno las siguientes oraciones que te __ermandad __oquedad __allazgo
dictará tu profesor: __ogareño __echado __umedecer
a) Las hienas son animales muy huidizos, en cuanto hue- • holgado • hormona • oval
len un olor extraño huyen. • orfanato • heterogéneo • aldehuela
b) En el hemiciclo se halló la solución al problema de la • homeopatía • invernar • hemorragia
habitabilidad humana. • hermandad • oquedad • hallazgo
c) Los huesos de los santos se hallan en un osario me- • hogareño • echado • humedecer
dieval.
7. Copia en tu cuaderno las siguientes oraciones que te dic-
d) ¡Hala!, ya me has echado encima la salsa de las habas. tará tu profesor:
e) Voy a hacer más habitable este hogar. a) Oyó que yendo hacia la plaza mayor de Uruguay había
f) Ha acudido a la consulta de un homeópata porque un gran bullicio.
tiene una hernia lumbar. b) En esa proyección subyace un problema apabu-
Respuesta libre. llante.
2. Escribe tres derivados de hueso, de huérfano, de huevo c) No creo que vaya esta tarde a arreglar la valla de los
y de hueco que no lleven h. caballos.
• hueso: osamenta, osario, óseo. d) No te escabullas a la hora de regar el bonsái.
• huérfano: orfanato, orfandad, orfelinato. e) No llegó un solo convoy, sino dos, e iban cargados de
• huevo: oval, ovoide, ovario. cigarrillos.
• hueco: oquedad, oquedal, oqueruela. Respuesta libre.

34 Lengua castellana y Literatura. 3.º ESO. Solucionario


LIBERTAD DE EXPRESIÓN 4
8. Completa con y o ll según corresponda: públicas, fundaciones, organismos, agencias patronales, me-
dios de comunicación públicos, etcétera, suman muchos mi-
bu__icio convo__ atribu__era __ate les de personas con sueldos a cargo de las arcas públicas. El
__ave cosi__a __emen co__ado problema es complicado, porque, paradójicamente, son ellos
mismos los que deben tomar la decisión de reducir ese número
__ermo mau__ar deca__ó pro__ecto
sobredimensionado.
Bomba__ le__es dis__unción pro__ector Necesitamos una nueva generación de políticos que incluyan
pi__o __ema ca__ena in__ección en el programa de su partido las reformas que se necesitan
hacer […]. Para ello se necesita voluntad de diálogo para con-
• bullicio • convoy • atribuyera • yate
seguir un amplio consenso.
• llave • cosilla • Yemen • collado
Andrés SANJURJO M ARTÍNEZ, Ferrol, A Coruña,
• yermo • maullar • decayó • proyecto en El País, 20 de julio de 2014.
• Bombay • leyes • disyunción • proyector a) El texto muestra el descontento de un ciudadano con el
• pillo • yema • cayena • inyección elevado número de políticos que hay en España. ¿Te pa-
rece que habrá mucha gente que comparta sus ideas?
LA FACTORÍA DE TEXTOS ¿Qué opinión te merece a ti lo que él plantea? Razona
tus respuestas.
1. A continuación te ofrecemos una columna de opinión.
Léela con atención y responde luego a las cuestiones b) Escribe tú una carta al director sobre un tema que te
que te planteamos: preocupe, manteniendo una extensión similar a la que
te ofrecemos.
Cualquiera que lleve hoy un periódico bajo el brazo no es que
esté mal informado, pero da la sensación de estar viviendo la a) y b) Respuesta libre.
realidad del día anterior. […] Aparte de eso, el periódico que
3. Algunos diarios digitales permiten la lectura en línea de
uno lleva bajo el brazo define ideológicamente al lector. […]
sus editoriales. Lee algunos de ellos y comenta luego los
Así sucedía también cuando en la República cada diario era
el estandarte de una bandería política, de la lucha de clases, aspectos más destacados de cada periódico: ideas políti-
incluso de un pensamiento religioso o anticlerical. Durante cas, preocupaciones sociales, visión de España, etc. Pue-
la larga ceniza de la posguerra el periódico llegaba al pueblo des consultar los editoriales de El Mundo (elmundo.es/
en el renqueante autobús de línea o en el correo ordinario, opinion.html) y de El País (elpais.com/elpais/opinion.
solo unos pocos ejemplares, que leía gente muy significada html).
[…]. Sobre un velador del café y en la barbería quedaba el Respuesta libre.
diario deportivo un poco grasiento después de haber pasa-
do de mano en mano. […] Al llegar la democracia la prensa
escrita se adaptó a la libertad y cada diario se acomodó de
ACTIVIDADES FINALES
nuevo a la manera de ser y de pensar de sus lectores. Pero con Repasa lo que has aprendido
la revolución digital hoy la prensa de papel siempre es la de
ayer y encima el periódico […] que el ciudadano compra en 1. El siguiente texto pertenece al capítulo XVII del libro pri-
el quiosco es un gesto ideológico que lo delata. No sucede así mero del Amadís de Gaula y describe un combate entre
con la tableta digital. Picoteando en el teclado del portátil con dos caballeros. Léelo y responde luego a las preguntas:
los dedos en el metro, en el tren, en una terraza al sol, nadie a Y metiendo mano a las espadas se dejaron a él ir muy brava-
tu lado puede saber si eres de derechas o de izquierdas. Leer mente. Amadís metió mano a su espada como aquel que era
el periódico de papel se va a convertir en el futuro en una ex- de gran corazón, y dejose a ellos ir muy sañudo por los haber
quisitez para estetas. Mientras todas las noticias en el digital quitado de su batalla y lo acometían tan malamente, e hirió al
son ya las de mañana, tampoco está tan mal ser un ciudadano uno de ellos por cima del yelmo de tal golpe, que le alcanzó en
elegantemente inactual. el hombro que las armas con la carne y huesos fue todo corta-
Manuel VICENT, en El País, 11 de marzo de 2012. do hasta descender la espada a los costados; así quedándole el
brazo colgado cayó del caballo abajo; y dejose ir a los dos que
a) Valora la actualidad de la columna. ¿Crees que el tema le herían bravamente y dio al uno por el yelmo tal golpe, que se
del que trata se ajusta a los tiempos en los que vivi- lo hizo saltar de la cabeza, y la espada descendió al pescuezo
mos? ¿A qué tipo de personas piensas que va dirigido y cortole todo lo más de él, y cayó el caballero. El otro, que
el artículo? Razona tus respuestas. esto vio, comenzó de huir contra donde viniera. Amadís, que
b) ¿Está presente en el artículo el criterio de subjetivi- lo vio en caballo corredor y que se alejaba, dejó de lo seguir
y tornó a Gandalín.
dad? Explica con tus palabras cuál es la opinión del
autor acerca del tema que plantea y cómo la percibi- Garci RODRÍGUEZ DE MONTALVO: Amadís de Gaula (1507)
mos al leer el texto. (texto modernizado).

c) Expresa ahora tu opinión y redacta un artículo en el que a) Busca en el texto los verbos en forma conjugada y
plantees lo que piensas sobre la prensa. analízalos morfológicamente. ¿Qué tiempos predo-
a), b) y c) Respuesta libre.
minan? ¿Crees que son adecuados para la narración?
¿Por qué?
2. Lee esta carta al director y contesta a las cuestiones:
b) Haz una lista con los verbos que indican acción y ana-
En España, según las últimas informaciones, tenemos so- liza qué aportan a la descripción del combate.
bredimensionado el número de políticos: entre diputados,
senadores, diputados autonómicos, diputados provinciales, c) ¿Qué formas no personales están presentes en el
miembros de los Gobiernos, concejales, asesores, así como texto? Cópialas en tu cuaderno y di de qué manera
cargos de designación política que trabajan en empresas contribuyen a hacer más intensa la acción.

Lengua castellana y Literatura. 3.º ESO. Solucionario 35


4 LIBERTAD DE EXPRESIÓN

d) ¿Qué crees que significa la expresión dejose ir que Todas estas formas no personales, unidas a las conjugadas
se usa varias veces en el texto? ¿Es una perífrasis? que hemos analizado arriba, ofrecen un enorme dinamis-
¿Por qué? mo a la acción que se está narrando, dando una impre-
sión de continuo movimiento y manteniendo la atención
a) Verbos: dejaron, metió, era, dejó, acometían, hirió, al- del lector ante lo trepidante de los hechos contados. Los
canzó, fue, cayó, herían, dio, hizo, descendió, cortó, vio, infinitivos muestran acciones que parecen estar siempre
comenzó, viniera, alejaba, tornó. en curso, mientras los gerundios indican claramente la
• dejaron: tercera persona del plural del pretérito perfec- duración de estas y los participios el resultado último de
to simple del verbo dejar. algunas de ellas.
• metió: tercera persona del singular del pretérito perfec- d) La expresión dejose ir significa que se dirigió voluntaria-
to simple del verbo meter. mente a un lugar concreto. Es una perífrasis, ya que indica
• era: tercera persona del singular del pretérito imperfec- una única acción y podríamos sustituirla por se dirigió.
to de indicativo del verbo ser. 2. Transforma en pasivas las siguientes oraciones siempre
• dejó: tercera persona del singular del pretérito perfecto que sea posible. En el caso de que alguna no se pueda
simple del verbo dejar. transformar, explica por qué:
• acometían: tercera persona del plural del pretérito im- a) Comían con ganas un bocadillo de chorizo.
perfecto de indicativo del verbo acometer.
b) Las ventanas del aula estaban abiertas.
• hirió: tercera persona del singular del pretérito perfec-
to simple del verbo herir. c) A Carmen le gustan mucho los helados de limón.
• alcanzó: tercera persona del singular del pretérito per- d) Ángel y Ana corrieron el maratón de Boston.
fecto simple del verbo alcanzar. e) Mantuvimos el fuego encendido toda la noche.
• fue: tercera persona del singular del pretérito perfecto a) Un bocadillo de chorizo era comido con ganas por ellos.
simple del verbo ir.
b) No puede ponerse en pasiva porque no tiene complemento
• cayó: tercera persona del singular del pretérito perfecto directo.
simple del verbo caer.
c) No puede ponerse en pasiva porque no tiene complemento
• herían: tercera persona del plural del pretérito imper- directo.
fecto de indicativo del verbo herir.
d) El maratón de Boston fue corrido por Ángel y Ana.
• dio: tercera persona del singular del pretérito perfecto
simple del verbo dar. e) El fuego fue mantenido encendido por nosotros toda la
noche.
• hizo: tercera persona del singular del pretérito perfecto
simple del verbo hacer. 3. Haz un análisis sintáctico de las oraciones de la actividad
• descendió: tercera persona del singular del pretérito
anterior.
perfecto simple del verbo descender. a) Comían con ganas un bocadillo de chorizo.
• cortó: tercera persona del singular del pretérito perfecto V CCM CD
simple del verbo cortar. b) Las ventanas del aula estaban abiertas.
• vio: tercera persona del singular del pretérito perfecto Sujeto V Atributo
simple del verbo ver. c) A Carmen le gustan mucho
• comenzó: tercera persona del singular del pretérito per- CI CI V CCC
fecto simple del verbo comenzar.
los helados de limón.
• viniera: tercera persona del singular del pretérito im-
perfecto de subjuntivo del verbo venir. Sujeto

• alejaba: tercera persona del singular del pretérito im- d) Ángel y Ana corrieron
perfecto de indicativo del verbo alejar. Sujeto V
• tornó: tercera persona del singular del pretérito perfec- el maratón de Boston.
to simple del verbo tornar. CD
En general predomina el pretérito perfecto simple, pero e) Mantuvimos el fuego encendido
también hay varios pretéritos imperfectos, tanto de indi-
V CD CPred
cativo como de subjuntivo. Estas formas verbales que ex-
presan el pasado son ideales para la narración de sucesos toda la noche.
(reales o ficticios), porque el narrador se encuentra en el CCT
presente y echa la vista atrás para contar lo que ya ha 4. Detecta y corrige los casos de laísmo, leísmo y loísmo
ocurrido.
presentes en las siguientes oraciones y explica por qué
b) Verbos que indican acción: acometían, hirió, alcanzó, son incorrectos. Atención, no todas las oraciones contie-
herían, dio, hizo, descendió, cortó, comenzó, viniera, nen errores.
alejaba, tornó. Corrige
Muchos de estos verbos indican movimiento, por lo que
se muestra el dinamismo del combate (descendió, comen- a) La dediqué un poema a mi novia.
zó, viniera, alejaba, tornó). Otros indican directamente b) No los des dinero, que son unos derrochones.
acciones bélicas y por ello contribuyen a dar viveza al
c) A Elvira la quiero mucho.
combate (acometían, hirió, alcanzó, herían, dio, cortó).
c) Formas no personales: metiendo, ir, haber quitado, cor-
d) Dilas que mañana vendremos temprano.
tado, descender, quedando, colgado, saltar, huir, seguir. e) Los vi alejarse en la oscuridad.

36 Lengua castellana y Literatura. 3.º ESO. Solucionario


LIBERTAD DE EXPRESIÓN 4

Corrige • curar: herir (herían). Recíproco.


• cobardemente: bravamente. Gradual.
f) Te dejo el disco, pero no me le destroces, que te
conozco. • ascender: descender. Complementario.

g) No la gusta que la mires de esa manera. 7. Lee el siguiente fragmento de un editorial de El País, del
h) Me le encontré en el parque y le dejé las cosas día 20 de agosto de 2014, y contesta luego a las cuestiones:
claras. Las protestas y disturbios que se han producido tras la muerte
de un joven negro por disparos de un policía en la localidad
i) La miro y no la reconozco.
estadounidense de Ferguson, en el estado de Misuri, han saca-
a) La dediqué un poema a mi novia. do a la luz de nuevo las tensiones raciales en Estados Unidos
que, aunque teóricamente se han resuelto en las leyes, siguen
• laísmo: utiliza el pronombre de complemento directo la
siendo una de las grandes asignaturas pendientes de la prin-
en lugar del de complemento indirecto, le.
cipal potencia mundial. […]
b) No los des dinero, que son unos derrochones.
Muy lejos queda ya 2008, cuando Obama se convirtió en el pri-
• loísmo: utiliza el pronombre de complemento directo los
mer presidente negro en la historia de EE. UU. y académicos,
en lugar del de complemento indirecto, les.
sociólogos y periodistas abusaron de la expresión América
c) A Elvira la quiero mucho. posracial como un reflejo de que el país norteamericano ha-
• Oración correcta. bía dejado atrás una era de discriminación y prejuicios. El mi-
d) Dilas que mañana vendremos temprano. crocosmos de Ferguson sirve, sin embargo, como ejemplo de
que hay una sospechosa desproporción en las actuaciones
• laísmo: utiliza el pronombre de complemento directo contra la comunidad negra. Mientras el 65 % de la localidad de
las en lugar del de complemento indirecto, le. poco más de 21.000 habitantes es negra, el 94 % de los policías
e) Los vi alejarse en la oscuridad. son blancos. El 84 % de los automóviles que estos agentes de-
• Oración correcta. tienen para solicitar la documentación están conducidos por
negros y el 92 % de los detenidos son negros. A estos datos
f) Te dejo el disco, pero no me le destroces, que te conozco.
hay que sumar un 21 % de familias que viven bajo el umbral
• leísmo: utiliza el pronombre de complemento indirecto de pobreza y una comunidad en la que no resulta excepcional
le en lugar del de complemento directo, lo. que se obstaculice la integración negra por un sentimiento
g) No la gusta que la mires de esa manera. discriminatorio que viene de muy lejos. Esa situación, con
• laísmo: utiliza el pronombre de complemento directo la variantes, se repite desigualmente a lo largo y ancho de la
en lugar del de complemento indirecto, le. El segundo geografía de EE. UU.
la está usado correctamente. a) ¿Cuál es la opinión del periódico sobre el tema que
h) Me le encontré en el parque y le dejé las cosas claras. trata el editorial? ¿Se percibe con claridad? ¿Por qué?
• leísmo: utiliza el pronombre de complemento indirecto b) ¿Te parece que la acumulación de datos es-tadísticos,
le en lugar del de complemento directo, lo. El segundo que sirve para dar objetividad, puede estar usándose
le está usado correctamente.
aquí, paradójicamente, para reforzar la opinión subje-
i) La miro y no la reconozco. tiva del diario? Razona tu respuesta.
• Oración correcta.
c) Tras leer el editorial, ¿te formas tú una opinión sobre el
5. Distingue si las siguientes oraciones son reflexivas, recí- problema del racismo en Estados Unidos? ¿Qué pien-
procas, pasivas reflejas o impersonales. Justifica tu res- sas al respecto? Puedes tener en cuenta tus conoci-
puesta en cada caso: mientos sobre el país a partir de películas o series de
a) Clara y Vicente se aman con pasión. televisión.
b) Se veía perfectamente el destrozo que hizo la tormenta a), b) y c) Respuesta libre.
en la huerta.
c) Se lavó los dientes después de comerse un plato de Recuerda lo que ya sabías
cochinillo asado. 8. Con los datos que te ofrece el editorial que acabas de
d) Los jugadores se cambiaron las camisetas al final del leer, elabora una noticia en la que estén presentes las
partido. características propias de este género, que aprendiste
a) Oración recíproca, ya que entre los sujetos hay un inter- en la unidad anterior.
cambio de acciones, en este caso se aman el uno al otro. Respuesta libre.
b) Pasiva refleja. Aparte de la presencia del pronombre se, 9. Busca en el texto del Amadís de Gaula de la actividad 1 los
la oración tiene un sujeto paciente: el destrozo que la tor- adjetivos y los adverbios que estén presentes y di luego
menta hizo en la huerta.
qué aportan a la descripción del combate que se narra.
c) Es una oración reflexiva, ya que la acción que realiza el
sujeto recae sobre él mismo. Adjetivos: gran, sañudo, corredor. Adverbios: bravamente,
malamente, así, abajo, donde.
d) Oración recíproca, ya que entre los sujetos hay un inter-
cambio de acciones, en este caso se cambian las camisetas El adjetivo sañudo aporta ferocidad al caballero y hace que
unos a otros. la acción narrada sea más intensa. Por su parte, los adverbios
indican circunstancias que redundan en esa misma fiereza
6. Busca en el texto de la actividad 1 antónimos de las siguien- del combate, como es el caso de bravamente (que aparece
tes palabras y di qué tipo de antonimia se da en cada caso: dos veces) o malamente. El resto de adjetivos y adverbios no
curar cobardemente ascender aportan nada especial al desarrollo del combate.

Lengua castellana y Literatura. 3.º ESO. Solucionario 37


4 LIBERTAD DE EXPRESIÓN

10. Analiza los sintagmas nominales presentes en el primer una redacción en la que narres la escena que has visto,
párrafo del editorial de la actividad 7 e indica los comple- empleando todos los verbos necesarios para recrear la
mentos del núcleo, cuando los haya. ¿Qué otros tipos de intensidad de la acción.
sintagmas están presentes en el fragmento? Respuesta libre.
Las protestas y disturbios que se han producido tras la
2. Observa la imagen que reproduce la escultura Apolo y
muerte de un joven negro por disparos de un policía en la
localidad estadounidense de Ferguson, en el estado de Misu- Dafne de Gian Lorenzo Bernini (1598-1680) y lee luego el
ri, han sacado a la luz de nuevo las tensiones raciales en soneto XIII de Garcilaso de la Vega (c. 1499-1536). Valora
Estados Unidos que, aunque teóricamente se han resuelto en la expresión de movimiento que producen los verbos en
las leyes, siguen siendo una de las grandes asignaturas el poema y compárala con el dinamismo de la escultura.
pendientes de la principal potencia mundial. Describe la escultura explicando de qué modo se plasman
• Los sintagmas nominales aparecen marcados en negrita: en ella los distintos movimientos recogidos en el poema.
las protestas y disturbios funciona como sujeto del verbo A Dafne ya los brazos le crecían
han sacado; las tensiones raciales, complemento directo y en luengos ramos vueltos se mostraban;
de han sacado; una de las grandes asignaturas pendien- en verdes hojas vi que se tornaban
tes, complemento directo de siguen siendo. los cabellos que el oro escurecían;
• En el fragmento hay también sintagmas preposicionales de áspera corteza se cubrían
(tras la muerte, de un joven negro, por disparos de la prin- los tiernos miembros que aun bullendo estaban;
cipal potencia mundial, etc.). Encontramos igualmente sin- los blancos pies en tierra se hincaban
tagmas verbales (han sacado a la luz..., se han resuelto en y en torcidas raíces se volvían.
las leyes..., siguen siendo...). Aquel que fue la causa de tal daño,
11. Investiga sobre el significado de los siguientes latinismos a fuerza de llorar crecer hacía
y anótalo en tu cuaderno: este árbol, que con lágrimas regaba.
a) in dubio pro reo e) in vino veritas ¡Oh miserable estado, oh mal tamaño,
que con llorarla crezca cada día
b) panem et circenses f) ex cathedra la causa y la razón por que lloraba!
c) delirium tremens g) corpore insepulto Garcilaso DE LA VEGA
d) alter ego h) ab intestato Respuesta libre.
a) in dubio pro reo: al haber dudas en un juicio se debe actuar
3. Observa estas tiras gráficas de El Roto y Forges y explica
en beneficio del reo.
qué quieren expresar sus autores. Valora el uso de la ironía:
b) panem et circenses: pan y circo. Se emplea para referirse
a la manera de entretener al pueblo para que no proteste Respuesta libre.
y se conforme con lo que tiene.
… encuentra la clave
c) delirium tremens: estado de delirio provocado por la in-
gesta excesiva de alcohol. 1. La prensa escrita comenzó su andadura en el siglo XVII,
d) alter ego: otro yo. Se emplea para referirse a alguien que pero se desarrolló, sobre todo, a partir de la segunda
ocupa a veces el lugar de uno o con quien este se siente mitad del XIX, con el auge de la industrialización. Te invita-
muy identificado. mos a que te des un paseo por la prensa histórica a través
e) in vino veritas: en el vino está la verdad. Se utiliza para de Internet. Busca en alguna de las siguientes páginas
referirse al estado que provoca el vino en quien lo bebe, algún periódico antiguo de tu provincia o de tu región y
que lo lleva a decir las verdades sin reservas. realiza las actividades que te proponemos después:
f) ex cathedra: desde la cátedra. Se emplea para aludir a • Biblioteca Virtual de Prensa Histórica: prensahistori-
alguien que habla o se expresa como si fuera el más sabio, ca.mcu.es
como si hablara desde la cátedra, como un catedrático.
• Hemeroteca digital de la Biblioteca Nacional: hemero-
g) corpore insepulto: cuerpo sin sepultar. Se usa para refe- tecadigital.bne.es
rirse a funerales u homenajes que se hacen a los difuntos
antes de que sean enterrados. a) Selecciona dos periódicos de épocas diferentes (por
h) ab intestato: sin haber hecho testamento. Es una fórmula
ejemplo, finales del siglo XIX y la década de los años
judicial que sirve para expresar que una determinada per- treinta del siglo XX) y elabora un esquema con las sec-
sona ha muerto sin hacer testamento. ciones que hay en cada uno. Compáralos entre sí y
después con un periódico actual.
MIRA A TU ALREDEDOR Y… b) Busca en esos mismos periódicos las secciones de opi-
… ve más allá nión y lee algún artículo. ¿Qué diferencias ves entre
este y los artículos de opinión actuales? Analízalas.
1. Ya sabemos que los verbos se utilizan, a menudo, para
narrar las acciones y pueden generar un gran dinamismo c) Una curiosidad de la prensa antigua son los anuncios
que hace la lectura más rápida e intensa. La misma fun- publicitarios. Léelos en varios periódicos y haz una rela-
ción que los verbos la realiza la cámara en el cine, cuando ción de productos anunciados. ¿Son los mismos que
actúa como narradora de escenas de acción. Ve un frag- ahora? ¿De qué manera tratan de llamar la atención
mento de la película En busca del arca perdida (1981), del lector? ¿Qué imagen nos dan de la sociedad de su
de Steven Spielberg. Teclea en YouTube «En busca del tiempo?
arca perdida» «El despotricador cinéfilo». Escribe luego a), b) y c) Respuesta libre.

38 Lengua castellana y Literatura. 3.º ESO. Solucionario


LIBERTAD DE EXPRESIÓN 4
2. Te ofrecemos a continuación el artículo 20 de la Cons- 5. Solo podrá acordarse el secuestro de publicaciones, gra-
titución española de 1978, que trata sobre la libertad de baciones y otros medios de infor mación en virtud de reso-
expresión. Léelo y contesta luego a las cuestiones que lución judicial.
figuran al final: a) Analiza el uso de las formas verbales conjugadas en
1. Se reconocen y protegen los derechos: el texto. ¿Qué tiempos predominan? ¿Qué personas
a) A expresar y difundir libremente los pensamientos, gramaticales? ¿Por qué crees que se usan estos y no
ideas y opiniones mediante la palabra, el escrito o cual- otros?
quier otro medio de reproducción. b) Fíjate ahora en las formas no personales y observa su
b) A la producción y creación literaria, artística, científica uso. ¿Cuáles aparecen? ¿Qué función desempeñan?
y técnica. c) Reflexiona sobre el contenido de este artículo  20.
c) A la libertad de cátedra. ¿Crees que contribuye a una mejor convivencia entre
d) A comunicar o recibir libremente información veraz por los ciudadanos? ¿Por qué?
cualquier medio de difusión. La ley regulará el derecho a) En el texto hay pocas formas verbales conjugadas, pero de
a la cláusula de conciencia y al secreto profesional en entre ellas predomina el uso del futuro imperfecto de indi-
el ejercicio de estas libertades. cativo (regulará, garantizará, podrá), que proyecta hacia
2. El ejercicio de estos derechos no puede restringirse me- ese tiempo las acciones que la ley tendrá que juzgar y anali-
diante ningún tipo de censura previa. zar. Hay algún presente de indicativo (reconocen, protegen),
3. La ley regulará la organización y el control parlamenta- que tiene la utilidad de indicar de forma neutra y duradera
rio de los medios de comunicación social dependientes del los preceptos contenidos en la ley. El uso exclusivo de la
Estado o de cualquier ente público y garantizará el acceso tercera persona garantiza una neutralidad y, por lo tanto,
a dichos medios de los grupos sociales y políticos signifi- una equidad que deben estar siempre presentes en la ley.
cativos, respetando el pluralismo de la sociedad y de las b) En cuanto a las formas no personales, la que predomina
diversas lenguas de España. es el infinitivo, que tiene un valor intemporal con una pro-
4. Estas libertades tienen su límite en el respeto a los dere- yección a un presente que podríamos llamar continuo, con
chos reconocidos en este Título, en los preceptos de las lo que parece garantizarse la pervivencia de los preceptos
leyes que lo desarrollen y, especialmente, en el derecho al contenidos en este artículo (expresar, difundir, comuni-
honor, a la intimidad, a la propia imagen y a la protección car, acordar...).
de la juventud y de la infancia. c) Respuesta libre.

Lengua castellana y Literatura. 3.º ESO. Solucionario 39


5 HÁBLAME DE TI

APERTURA DE UNIDAD e incomprensible. Esa es la impresión que a Ulises le provoca,


por ejemplo, el hecho de que los gorilas sepan hablar y expre-
1. Según Susie, la narradora, hay muchas personas que es- sen sentimientos humanos.
criben un diario. ¿Conoces a alguien que comparta esta
2. Dentro de la sociedad de los simios, ¿qué especie se com-
inquietud? ¿Por qué le dedican parte de su tiempo a
porta con mayor brutalidad? ¿Crees que existe algún mo-
esta actividad?
tivo que justifique dicha conducta? ¿Cuál?
Respuesta libre.
Dentro de la sociedad de los simios, son los gorilas, identifica-
2. De acuerdo con Susie, ¿qué se puede contar en un diario? dos como cazadores, quienes muestran un porte más aristo-
¿Es posible que ella consiga hacerse famosa con el relato crático y se comportan con mayor brutalidad. Dicha conducta
de sus historias? ¿Por qué? podría justificarse por los hábitos de su oficio, pero, sobre
todo, porque pertenecen a la especie con mayor fortaleza
Para Susie, en un diario tiene cabida todo lo que se quiera
física.
contar y le pase a uno por la cabeza. No podrá hacerse famo-
sa con su diario, porque, como solo quiere contar su verdad, 3. ¿Qué gestos utiliza el autor para animalizar a los seres hu-
prefiere mantenerlo en secreto. manos?
3. El último párrafo del texto plantea la cuestión de la pri- Los humanos no solo son cazados con trampas como los ani-
vacidad. ¿Consideras que la sociedad actual respeta la males, sino que su miedo a los simios es de tal calibre que el
privacidad del individuo? ¿Y los jóvenes? pánico les lleva a morder, a moverse frenéticamente, a patear
con sus cuatro extremidades, rechinar los dientes lanzando
Esta actividad y la siguiente están destinadas a la reflexión y
espuma por la boca y morder las cuerdas de la red que les
el debate, y deberían servir para alertar al alumnado para que
atrapa.
tomara conciencia de los peligros reales que existen en la socie-
dad actual de la información. Por eso, cabe poner el énfasis en 4. ¿En qué se diferencia Mérou de sus compañeros de jaula?
cómo el desarrollo de las nuevas tecnologías responde, en parte, A diferencia de los otros humanos, Mérou puede usar un len-
a la obtención de información privada que podrá ser utilizada guaje articulado, posibilidad que sorprende a los simios hasta
por empresas o por el Estado para sus propios fines. Asimismo, el punto de interpretarlo como una amenaza.
debería destacarse la necesidad de una conducta responsable
en las redes sociales para que los más jóvenes no pusieran sus
5. ¿Por qué podrías decir que la situación en que se halla
datos personales al alcance de cualquier persona o entidad. el narrador es desesperada? Imagínate que eres el pro-
pio Mérou. ¿De qué modo intentarías hacerte entender
4. Con la colaboración de algunos de tus compañeros, en- y doblegar la violenta hostilidad de los simios?
tablad un debate en el aula sobre la privacidad. Seguid
Aparte de estar desarmado y a merced de sus carceleros,
estos pasos:
el protagonista no sabe cómo hacer frente a una situación
a) En grupos de tres o cuatro, elaborad un guion en el que que le resulta increíble. Ahora le toca al alumno, imaginar el
respondáis a las siguientes cuestiones: modo en que el personaje podría afrontar el problema que se
le plantea.
• ¿Cómo afectan los medios de comunicación y las
redes sociales a la privacidad de cada uno? Identi- Vocabulario
ficad programas televisivos que atenten contra ese
derecho. 6. Escribe palabras que pertenezcan a la misma familia léxi-
ca de matanza, servidores, ojeadores y brutalidad.
• ¿Qué consecuencias tiene para el individuo que
• matanza: matar, matadero matador, matanza, matarife.
se hagan públicos sus secretos? ¿Conocéis algún
ejemplo? • servidores: servir, servicial, servicio, servidumbre, servil.
• ojeadores: ojeada, ojear, ojeo, ojo.
• ¿A qué mecanismos podemos recurrir para prote-
ger nuestra vida privada y para escapar a posibles • brutalidad: brutal, bruto, embrutecer.
manipulaciones? 7. Con la ayuda de un diccionario, define el término para-
b) Elegid un moderador que asigne los turnos de palabra doja. Luego, escribe tres oraciones con dicho sustantivo.
y regule las intervenciones. Luego, entablad el debate Podemos definir la paradoja como un contradicción aparente
respetando los turnos de palabra y las indicaciones entre dos cosas o ideas, pero también como una afirmación
del moderador. inverosímil que se presenta con apariencia de verdadera. Se-
rían sinónimos contradicción o absurdo.
Véase respuesta a actividad anterior.
8. ¿Qué distintos significados puede tener la palabra pista?
VIVE LA LECTURA Escribe una oración con cada uno de ellos.
• Rastro que deja cualquier criatura en el lugar por donde ha
Comprensión lectora pasado.
1. ¿Qué le sorprende al narrador de la civilización del pla- • Conjunto de señales que puede conducir a la averiguación
neta Soror? Uno de los tópicos literarios más antiguos es de algo.
el del «mundo al revés». Explica en qué consiste y, a con-
• Lugar habilitado para la práctica de deportes u otras
tinuación, señala cómo se plasma en el texto. actividades.
Al personaje-narrador le sorprende y aterroriza cómo se han
• Terreno acondicionado para el despegue y aterrizaje de
intercambiado los papeles y los simios someten a los humanos.
aviones.
Esta extraordinaria inversión se ajusta al tópico del «mundo
al revés», reconocible, cuando menos, desde el escritor griego • Camino o autopista.
Hesíodo y, según el cual, se han alterado completamente las • Cada uno de los espacios paralelos de una cinta magnética
leyes que rigen la realidad, hasta hacer del mundo algo ilógico donde se registran grabaciones independientes.

40 Lengua castellana y Literatura. 3.º ESO. Solucionario


HÁBLAME DE TI 5
9. Explica el significado de las expresiones: «tenían aire de h) Unos edificios me parecen muy grandes, otros me pa-
aristócratas» y «se debatían con desesperación». recen pequeños.
Tenían aire de aristócratas significa, en el texto, que los a) Nexo: o; disyuntiva excluyente.
gorilas se comportaban como aristócratas, tal era su poder. b) Nexo: o; disyuntiva excluyente.
Se debatían con desesperación significa que los humanos lu-
c) Nexo: esto es; explicativa.
chaban y forcejaban por escapar desesperadamente de una
situación muy adversa. d) Nexo: ya… ya; distributiva.
10. ¿Cuáles de los siguientes sustantivos podrían reem- e) Nexo: o sea; explicativa.
plazar al sustantivo tumulto en la expresión «Sin prestar f) Nexo: sino; adversativa.
atención alguna a este tumulto»? g) Nexo: ni; copulativa.
escándalo multitud motín h) Elementos correlativos: Unos… otros; distributiva.
jaleo gentío alboroto 2. Coloca el nexo correspondiente e indica qué relación
tranquilidad aglomeración desorden coordinante se da en estas oraciones compuestas:
En el contexto, podrían reemplazar al sustantivo tumulto los a) Unos coches se quedaron sin gasolina, ________ tenían lleno
sustantivos escándalo, motín, jaleo, alboroto y desorden. el depósito.
b) Dentro de mil años el cosmos se deshará, ________ llegará
Investigación y redacción
a su fin.
11. Ayudándote de alguna enciclopedia o de Internet, resume c) El niño se enfadó, ________ , perdió los papeles.
el desenlace de la inquietante aventura de Ulises Mérou
en el planeta Soror. d) No me dejaban salir, ________ escrutar tras la ventana.
Mérou consigue que una pareja de científicos simios le con- e) Los turistas llegaron todos empapados, ________ tres.
sidere como criatura inteligente procedente de otro planeta. f) Vino pronto ________ exhibió sus habilidades.
Parece adaptarse a su nueva situación y vive como un simio g) ¿Te sirvo croquetas ________ prefieres patatas?
más, hasta que deja embarazada a Nova y eso despierta los
recelos de las autoridades de Soror, que temen que los hijos del a) otros; distributiva.
protagonista sean tan inteligentes como él y pongan en peligro b) o sea, esto es, es decir; explicativa.
su dominio. Ayudado por Zyra y Cornellius, Ulises huye con c) o sea, esto es, es decir; explicativa.
Nova y su hijo recién nacido para embarcarse en una nave es- d) sino; adversativa.
pacial con dirección a la Tierra. A su llegada, descubre que el
planeta Soror no es otro que la misma Tierra, aunque, a causa e) excepto, salvo; adversativa.
de un cataclismo atómico, la era humana se vio reemplazada f) y; copulativa.
por la era de los simios. Así pues, la peripecia de Mérou es un g) o; disyuntiva excluyente.
viaje en el tiempo, dos mil años después de la gran tragedia. 3. Sustituye las palabras marcadas en negrita por una propo-
12. A partir del nombre del protagonista de la novela, es po- sición. ¿De qué tipo de subordinación hablamos en cada
sible establecer una curiosa relación con un célebre texto caso?
épico homérico. ¿Cuál? ¿Qué puntos en común tienen a) Nos gusta tu tranquilidad.
ambas historias?
b) Se desplazarán hasta tu casa.
El nombre del protagonista de la novela de Pierre Boulle nos
remite al Ulises de la Odisea de Homero. Además de la coinci- c) El gato extraviado no tenía dueño.
dencia onomástica, ambos personajes son viajeros que sufren d) Le aterraba la tormenta.
una serie de aventuras de carácter fantástico en lugares donde e) La muchacha se despidió a la madrugada.
imperan unas leyes distintas a las de la realidad conocida.
f) Os encontrasteis con el jugador lesionado.
13. La novela de Pierre Boulle ha sido adaptada al cine en
varias ocasiones. Investiga sobre el tema e indica las más g) Dime la hora.
famosas. Sin embargo, en tales películas se ofrece una Responder siguiendo los ejemplos siguientes:
versión bastante modificada de la historia compuesta por a) Nos gusta que seas una persona tranquila. Subordina-
el escritor francés. ¿Qué cambios sustanciales en el argu- da sustantiva.
mento adviertes entre el texto literario y sus adaptaciones b) Se desplazarán hasta el lugar en que vives. Subordina-
para la gran pantalla? da adjetiva.
Respuesta libre. c) El gato que andaba extraviado no tenía dueño. Subor-
dinada adjetiva.
ESTUDIO DE LA LENGUA d) Le aterraba que hubiera rayos. Subordinada sustantiva.
1. Subraya el nexo e indica el tipo de coordinación que se e) La muchacha se despidió cuando anocheció. Subordi-
establece en cada caso: nada sustantiva adverbial.
a) ¿La capital de Alemania es Bonn o es Berlín? f) Os encontrasteis con el jugador que estaba lesionado.
Subordinada adjetiva.
b) Devuélveme el dinero o no vengas más.
g) Dime qué hora es. Subordinada sustantiva.
c) Lamentaba su ausencia, esto es, lo echaba de menos.
4. Escribe cinco oraciones compuestas que contengan una
d) Ya me susurraba al oído, ya reía estrepitosamente.
proposición sustantiva cada una que realice la función soli-
e) Hay mucha cola, o sea, tardaré en pagar. citada: sujeto, complemento directo, atributo, complemento
f) No era ambidiestro, sino zurdo. de régimen, complemento del nombre.
g) No quiere a su madre, ni tampoco a sus amigos. Respuesta libre.

Lengua castellana y Literatura. 3.º ESO. Solucionario 41


5 HÁBLAME DE TI

5. Completa estas secuencias con el nexo oportuno: e) Con tal de que asistan: condicional.
a) No dijeron ______ iban a venir a la fiesta. f) Aunque era rico: concesiva.
b) Le preguntaron ______ estaba satisfecho. g) según les he dicho: de modo.
h) más calorías que todos los demás: comparativa de supe-
c) Mi padre ignora ______ he conseguido aprobar.
rioridad.
a) si; pero caben otras alternativas como cuántos, cuándo,
i) que extravió su botín: consecutiva ponderativa.
cómo.
j) cuando amaneció: de tiempo.
b) si; pero con cambio de significado cabe la opción por qué.
k) porque me has mentido: causal.
c) cómo
l) por tanto, no comeré más: consecutiva.
6. Subraya las proposiciones subordinadas, indicando cuáles
9. En este fragmento hay varias oraciones compuestas. Iden-
son sustantivas y cuáles adjetivas.
tifica el tipo de coordinadas y subordinadas que intro-
a) Tenía la esperanza de que ganaran el premio. ducen los nexos destacados y señala las proposiciones
b) Estoy junto a la mujer que extravió la cartera. respectivas:
c) La gente, que miraba el cuadro, sonreía. El mundo era armónico para Ramón en aquel momento, hasta
d) Es imposible que lo castiguen por eso. que en la esquina, la del Payaso Fofó con la avenida, descubrió
entre el público a las cuatro hembras de sus ojos, que le salu-
e) Quiero saber cuándo será tu cumpleaños.
daban enternecidas, con esa sonrisa que se dedica a los niños
f) Deseo que me compren un perro. cuando hacen una travesura perdonable. A Ramón se le heló
g) Te enojaste con el muchacho al que saludamos. la canción en los labios, pero venció la ligera incomodidad
h) Hallaron la cabaña donde el pirata dormía. interior para saludarlas con su mano-tijera, y volvió a su ver-
dadera naturaleza, la de huérfano de por vida, hijo póstumo,
a) de que ganaran el premio: subordinada sustantiva. niño eterno, aunque a principios de año fuera a cumplir ya
b) que extravió la cartera: subordinada adjetiva especificativa. dieciséis años. Su descenso al lado salvaje de la vida había
c) que miraba el cuadro: subordinada adjetiva explicativa. durado menos que un viaje en ascensor.
d) que lo castiguen por eso: subordinada sustantiva. Elvira L INDO: El otro barrio.
e) cuándo será tu cumpleaños: subordinada sustantiva. • que le saludaban enternecidas: subordinada adjetiva
explicativa.
f) que me compren un perro: subordinada sustantiva.
• que se dedica a los niños: subordinada adjetiva especi-
g) al que saludamos: subordinada adjetiva especificativa.
ficativa.
h) donde el pirata dormía: subordinada adjetiva especificativa.
• cuando hacen una travesura perdonable: subordinada
7. Localiza el antecedente de las subordinadas adjetivas de la adverbial de tiempo.
actividad anterior y trata de identificar la función sintáctica • pero venció la ligera incomodidad interior: coordinada
que desempeña el nexo relativo. adversativa.
b) antecedente: mujer; el nexo funciona como sujeto. • y volvió a su verdadera naturaleza, la de huérfano de por
c) antecedente: gente; el nexo funciona como sujeto. vida, hijo póstumo, niño eterno: coordinada copulativa.
g) antecedente: muchacho; el nexo funciona como comple- • aunque a principios de año fuera a cumplir ya dieciséis
mento directo. años: subordinada adverbial concesiva.
h) antecedente: cabaña; el nexo funciona como complemento • menos que un viaje en ascensor: subordinada adverbial
circunstancial de lugar. comparativa de inferioridad.
8. Subraya las subordinadas adverbiales e indica de qué tipo 10. Escribe una continuación para la historia de la actividad
es cada una: anterior en la que emplees al menos una proposición su-
a) Trajeron el cuadro para que lo colgaras en la pared. bordinada adverbial impropia de cada tipo.
b) Terminó el examen, por tanto, pudo salir al patio. Respuesta libre.
c) Los nómadas cabalgan hacia donde los llevan sus 11. ¿De qué tipo son las siguientes oraciones compuestas
camellos. por coordinación? Señala cuál es su nexo:
d) No le engañaron en absoluto, ya que los conocía muy a) Algunos vecinos no estaban de acuerdo, pero acepta-
bien. ron la decisión de la mayoría.
e) Con tal de que asistan, recibirán un magnífico regalo. b) Ni yo sé hacer el trabajo ni el sueldo me parece sufi-
f) Aunque era rico, siempre iba mal vestido. ciente.
g) Realizarán el examen según les he dicho. c) Todos los miércoles voy al gimnasio y al día siguiente
me siento fenomenal.
h) Ese plato tiene más calorías que todos los demás.
d) Ya los perros aullaban aterrados, ya parecían estar
i) El ladrón estaba tan rabioso que extravió su botín. conformes.
j) La expedición prosiguió su ruta cuando amaneció. e) La concursante hizo un doble salto, o sea, se jugó el
k) Estoy descontento porque me has mentido. todo por el todo.
l) Estoy satisfecho, por tanto, no comeré más. a) Nexo: pero; adversativa.
a) para que lo colgaras en la pared: final. b) Nexo: ni; copulativa.
b) por tanto, pudo salir al patio: consecutiva. c) Nexo: y; copulativa.
c) hacia donde los llevan sus camellos: de lugar. d) Nexo: ya… ya; distributiva.
d) ya que los conocía muy bien: causal. e) Nexo: o sea; explicativa.

42 Lengua castellana y Literatura. 3.º ESO. Solucionario


HÁBLAME DE TI 5
12. Lee el comienzo de un cuento de Carmela Greciet y ana- ralektryonoptekephalliokigklopeleiolagoiosiraiobaphetra-
liza sus oraciones. Sepáralas en simples y compuestas. ganopterygon.
Indica de qué tipo son estas últimas: • 163 letras: krungthepmahanakhonmonrattanakosin
Mi madre se entretuvo con el móvil en el momento en que MahintharayutthayaMahadilokphopNoppharatrat-
íbamos a entrar en el ascensor para bajar al parque y no pudo chathaniburiromudomratchaniwetmahasathanAmonphi-
evitar quedarse paralizada en el rellano cuando las puertas se manawatansathitSakkathattiyawitsanukamprasit,
cerraron conmigo dentro. nombre original de la ciudad tailandesa de Bangkok que
significa ciudad de ángeles, la gran ciudad, la ciudad de
Aparecí unos pisos más arriba, donde me esperaba una señora
joya eterna, la ciudad impenetrable del dios Indra, la mag-
a la que nunca antes había visto y que me gritaba enfadada
nífica capital del mundo dotada con nueve gemas precio-
en […] lengua ucraniana. Con el vértigo de mi primer viaje en
sas, la ciudad feliz, que abunda en un colosal Palacio Real
solitario, no tuve fuerzas para llevarle la contraria e hice lo
que se asemeja al domicilio divino donde reinan los dioses
que sus gestos, con el lenguaje universal de las madres, me
reencarnados, una ciudad brindada por Indra y construida
indicaban: «Entra pa casa».
por Vishnukam.
Se esbozan las proposiciones coordinadas y subordinadas:
• 130 letras, en sueco: nodöstersjökustartillerif lygspa-
• en que íbamos a entrar en el ascensor: subordinada adje- ningssimulatoranläggningsmaterielunderhallsuppfölj-
tiva especificativa. ningssy-stemdiskussionsinläggsförberedelsearbeten, con
• para bajar al parque: subordinada adverbial final. el significado de artillería de la costa norte del Báltico,
• y no pudo evitar…: coordinada copulativa. construcción de un simulador de vuelo, sistemas de moni-
torización y mantenimiento y preparación de pósteres de
• quedarse paralizada en el rellano: subordinada sustantiva
comunicación.
de complemento directo.
• cuando las puertas se cerraron conmigo dentro: subordi- • 85 letras: taumatawhakatangihangakoauauotamateatu-
nada adverbial de tiempo. ripukakapikimaungahoronukupokaiwhenuaki-tanatahu,
en maorí, para referirse a un topónimo: la cumbre donde
• donde me esperaba una señora…: subordinada adjetiva Tamatea, el hombre de grandes rodillas, el escalador de
explicativa. montañas, el que se traga la tierra cuando viaja, toca la
• a la que nunca antes había visto y que me gritaba enfada- flauta nasal a su amada.
da en […] lengua ucraniana: dos subordinadas adjetivas
2. Mediante un anagrama, obtén un adjetivo calificativo a
especificativas unidas mediante coordinada copulativa.
partir de la palabra argentino, y un sustantivo a partir del
• para llevarle la contraria: subordinada adverbial final.
adverbio armoniosamente.
• e hice…: coordinada copulativa.
argentino: tangerino; armoniosamente: enamoramientos.
• lo que sus gestos […] me indicaban: subordinada sustan-
tiva de complemento directo.
3. ¿Qué tienen de especial las frases Se van sus naves y No
di mi decoro, cedí mi don?
La única oración simple es «Entra pa casa».
Las frases Se van sus naves y No di mi decoro, cedí mi don
son palíndromos porque se leen igual hacia delante que hacia
EL TALLER DE LAS PALABRAS
atrás.
1. Investiga en Internet cuáles son las cinco palabras más 4. Lee atentamente el siguiente abecegrama de Francisco
largas del mundo. Briz Hidalgo. ¿Puedes deducir a través de él qué es un
• 1185 letras: acetylseryltyrosylserylisoleucylthreonylseryl- abecegrama? Defínelo en tu cuaderno.
prolylserylglutaminylphenylalanylvalylphenylalanylleu- Anoche brillaron cerca, chispeantes, dos estrellas fugaces;
cylserylserylvalyltryptophylalanylaspartylprolylisoleu- gravitaban hermosas iluminando juntas kilométricos luga-
cylglutamylleucylleucylasparaginylvalylcysteinylthreon- res; llevaban mágicos negros ñublos; originaban planetas
ylserylserylleucylglycylasparaginylglutaminylphenyla- que relucían surcando tenues universos…, vertiendo wolfra-
lanylglutaminylthreonylglutaminylglutaminylalanylar- mio, xenón y zafiros.
ginylthreonylthreonylglutaminylvalylglutaminylgluta-
Un abecegrama es un texto integrado por palabras que se
minylphenylalanylserylglutaminylvalyltryptophyllysyl-
disponen en orden alfabético, de modo que la primera empe-
prolylphenylalanylprolylglutaminylserylthreonylvalylar-
zará por a, la segunda por b y así sucesivamente hasta llegar
ginylphenylalanylprolylglycylaspartylvalyltyrosyllysyl-
a la palabra final que empezará con z.
valyltyrosylarginyltyrosylasparaginylalanylvalylleucylas-
partylprolylleucylisoleucylthreonylalanylleucylleucylgly- 5. Ahora inventa tú un abecegrama y escríbelo en tu cua-
cylthreonylphenylalanylaspartylthreonylarginylaspara- derno.
ginylarginylisoleucylisoleucylglutamylvalylglutamylaspa- Respuesta libre.
raginylglutaminylglutaminylserylprolylthreonylthreonyla-
6. En la literatura española de los siglos XVI y XVII los abece-
lanylglutamylthreonylleucylaspartylalanylthreonylargin-
ylarginylvalylaspartylaspartylalanylthreonylvalylalanyli- gramas recibieron el nombre abecé y fueron utilizados por
soleucylarginylserylalanylasparaginylisoleucylasparagin- escritores como Cervantes o Lope de Vega. En el Quijote
ylleucylvalylasparaginylglutamylleucylvalylarginylgly- aparece un abecé muy famoso, donde se explican las cua-
cylthreonylglycylleucyltyrosylasparaginylglutaminylas- lidades que debe tener un buen enamorado. Búscalo en
paraginylthreonylphenylalanylglutamylserylmethionylse- Internet y cópialo en tu cuaderno.
rylglycylleucylvalyltryptophylthreonylserylalanylprolyla- «Él es, según yo veo y a mí me parece, agradecido, bueno, ca-
lanylserine, en inglés para referirse a un término químico. ballero, dadivoso, enamorado, firme, gallardo, honrado, ilus-
• 182 letras, inventada por el escritor griego Aristófanes tre, leal, mozo, noble, honesto, principal, quantioso, rico y las
para referirse a una comida imaginaria: lopadotemacho- eses que dicen, y luego, tácito, verdadero. La x no le cuadra,
selachogaleokranioleipsanodrimhypotrimmatosilphiopa- porque es letra áspera; la y ya está dicha; la z, zelador de tu
raomelitokatakechymenokichlepikossyphophattoperiste- honra» (Quijote, Primera parte, cap. XXXIV).

Lengua castellana y Literatura. 3.º ESO. Solucionario 43


5 HÁBLAME DE TI

7. Lee el siguiente acertijo y resuélvelo mediante un ana- Algunos ejemplos de calambur:


grama: • Y lo es, y lo es. Quien no lo adivine tonto es: hilo.
Este enigma está basado • Si el Rey no muere, el Reino muere.
en los días de la semana, • Yo loco, loco, y ella loquita: Yo lo coloco y ella lo quita.
se trata de averiguar,
• ¡Ave!, César de Roma: A veces arde Roma.
mejor hoy que mañana,
la solución adecuada: • Alberto Carlos Bustos: Al ber tocar los bustos.
¿qué día de la semana • ¿Por qué lavo la rueda? ¿Por qué la bola rueda?
se oculta con anagrama
en una de estas palabras? EL RINCÓN DE LA NORMA
sábado: basado.
1. En estas oraciones, hay comas mal utilizadas. Explica en
8. Escribe en tu cuaderno cinco palíndromos. qué consiste el error en cada caso y corrígelo (presta es-
Ejemplos de palíndromos: allá, arañara, nadan, radar, rayar, pecial atención a los casos en que debas sustituir por punto
reconocer, seres, somos, etc. y coma):
9. Busca los famosos versos acrósticos que aparecen en La Corrige
Celestina y escribe en tu cuaderno la información que está
oculta en ellos. a) Mi hermana, tiene ya veinte años.
«El bachiller Fernando de Rojas acabó la comedia de Calysto b) Ha llovido toda la mañana, las calles están llenas
y Melybea y fue nascido en la puebla de Montalbán». de charcos.
10. Busca en el diccionario el significado de las siguientes pa- c) Hoy tenemos que comprar tela, hilo, alfileres, pre-
labras. ¿Son homónimas o polisémicas? parar los disfraces de la fiesta, redactar las invita-
a) cavo (verbo cavar) y cabo ciones, la lista de los que vendrán y sus teléfonos.
b) botar y votar d) Estimados amigos, os envío estas líneas para da-
ros una noticia inquietante.
c) vela (verbo velar) y vela
e) Se han agotado las entradas del concierto; de la
a) Homónimas.
función de teatro y de la exposición.
b) Homónimas.
c) Homónimas. a) Mi hermana tiene ya veinte años. No debe usarse la coma
11. Escribe una oración con cada una de las palabras de la entre el sujeto y el predicado.
actividad anterior. b) Ha llovido toda la mañana; las calles están llenas de
charcos. Se usa punto y coma porque el tono al final de la
Respuesta libre.
frase es descendente. Además, entre las dos proposiciones
12. Explica el significado de estas palabras e indica si se trata media una relación de causa-efecto subordinante.
de palabras homógrafas u homófonas: c) Hoy tenemos que comprar tela, hilo, alfileres; preparar
a) haya (sustantivo) y haya (verbo haber) los disfraces de la fiesta; redactar las invitaciones, la
b) a y ha lista de los que vendrán y sus teléfonos. Se usa el punto y
coma porque hay enumeraciones con comas en el interior
c) arroyo y arrollo (verbo arrollar) de los elementos enumerados.
d) basto y vasto d) Estimados amigos: os envío estas líneas para daros una
a) haya (sustantivo: árbol) y haya (verbo haber): homógrafas. noticia inquietante. En los encabezamientos de las cartas
b) a (preposición) y ha (verbo haber): homófonas. hay que usar dos puntos y no la coma.
c) arroyo (sustantivo: riachuelo) y arrollo (verbo arrollar): e) Se han agotado las entradas del concierto, de la función
homófonas. de teatro y de la exposición. No puede haber punto y coma
d) basto (sustantivo: palo de la baraja o adjetivo: grosero) y entre los términos enumerados.
vasto (adjetivo: extenso): homófonas. 2. Coloca en este fragmento de Eloísa está debajo de un
13. Construye una oración en la que utilices correctamente almendro, de Enrique Jardiel Poncela, los puntos, las ma-
los siguientes homófonos: yúsculas y los signos de interrogación y exclamación.
a) a ver y haber Corrige
b) así mismo y a sí mismo
PRÁXEDES.— se puede sí, porque no hay nadie que no hay
c) hasta y asta nadie bueno; hay alguien, pero como si no hubiera na-
d) agito y ajito die hola qué hay qué haces aquí perdiendo el tiempo,
no tú dirás que no, pero yo digo que sí qué ah bueno,
e) botar y votar
por eso… que por qué vengo porque me lo han man-
f) combine y convine dado quién la señora mayor.
Respuesta libre.
PRÁXEDES.— ¿Se puede? Sí, porque no hay nadie. ¿Qué no hay
14. El calambur es un recurso retórico que consiste en usar nadie? Bueno; hay alguien, pero como si no hubiera nadie.
palabras de fonética parecida, aunque con significado dis- ¡Hola! ¿Qué hay? ¿Qué haces aquí? Perdiendo el tiempo,
tinto (homófonos). Por ejemplo: oro parece, plata no es, ¿no? Tú dirás que no, pero yo digo que sí. ¿Qué? ¡Ah! Bueno,
¿qué es? Plátano. Anota tú otros ejemplos de calambures por eso… ¿Que por qué vengo? Porque me lo han mandado.
que conozcas o invéntate alguno. ¿Quién? La señora mayor.

44 Lengua castellana y Literatura. 3.º ESO. Solucionario


HÁBLAME DE TI 5
3. Incorpora los dos puntos siempre que sea necesario y ACTIVIDADES FINALES
explica por qué los has empleado:
Repasa lo que has aprendido
Corrige 1. Lee este apartado, extraído de un manual de formación
vial cuyo estudio es necesario para la obtención del per-
a) Eusebio me respondió «Eres muy atrevida». miso de conducción.
b) Sus preferencias eran la buena comida, la música Obligaciones del conductor que adelanta:
disco y la lectura. Antes de adelantar, el conductor debe comprobar que puede
c) Apenas lloró todavía le quedaban lágrimas. hacerlo sin ningún riesgo y advertirlo con suficiente ante-
d) Estos son los artículos rebajados bañadores, cha- lación. Asimismo, deberá comprobar que el carril que va a
lecos, pareos y gafas de sol. ocupar está libre y tener cuidado de no poner en peligro ni
entorpecer la marcha a los vehículos que se aproximen de
e) Apreciado cliente la nueva oferta telefónica que le frente ni a ningún otro usuario de la vía; en caso contrario,
proponemos le dejará sin habla. no se debe adelantar.
a) Eusebio me respondió: «Eres muy atrevida». Se reprodu- Además no se debe acercar demasiado al vehículo que va a
cen las palabras dichas por otra persona. ser adelantado ni quedarse demasiado lejos.
Durante el adelantamiento, el conductor que adelanta debe
b) Sus preferencias eran la buena comida, la música disco
circular a una velocidad notoriamente superior a la del ve-
y la lectura. No se incluyen si no hay elemento anticipador.
hículo que pretende adelantar y dejar, entre ambos, una sepa-
c) Apenas lloró: todavía le quedaban lágrimas. En proposi- ración lateral suficiente para realizar el adelantamiento con
ciones yuxtapuestas. seguridad.
d) Estos son los artículos rebajados: bañadores, chalecos, Después de adelantar, se debe poner el intermitente derecho
pareos y gafas de sol. En enumeraciones con un término y regresar al carril derecho de forma gradual y sin obligar al
anticipador (artículos rebajados). vehículo adelantado a modificar su trayectoria o velocidad.
e) Apreciado cliente: la nueva oferta telefónica que le pro- Manual Matfer
ponemos le dejará sin habla. En encabezamiento de carta a) El texto está basado en varios decretos legislativos so-
para dirigirse a un tú. bre tráfico y circulación de vehículos de motor. ¿Cómo
se pone de manifiesto su carácter prescriptivo? Anota
LA FACTORÍA DE TEXTOS en tu cuaderno las palabras que lo evidencien.
1. Una receta de cocina puede considerarse como un texto b) Subraya los conectores y expresiones que contribuyen
donde se dan unas instrucciones muy precisas. Invéntate a ordenar minuciosamente la exposición, e indica qué
un texto de este tipo para elaborar un plato exquisito. idea introduce cada uno de ellos.
Respuesta libre. c) ¿Qué semejanzas y diferencias encuentras entre este
2. Imagina que quieres pedirle a la directora de tu instituto texto y, por ejemplo, un escrito con instrucciones para
que os permitan utilizar el patio por la tarde, después del montar un mueble?
horario escolar. Escribe una instancia siguiendo la estruc- d) En el fragmento aparecen diversas oraciones compues-
tura expuesta más arriba. tas. Localiza en él los siguientes tipos de proposiciones:
Respuesta libre. • Una coordinada copulativa negativa.

3. El siguiente fragmento pertenece a Los sufrimientos del • Una subordinada adverbial de modo.
joven Werther, de J. W. Goethe, una novela decisiva en • Una subordinada adverbial final.
el surgimiento del Romanticismo alemán. Gran parte de a) El carácter prescriptivo del texto se concreta desde su mis-
la obra está integrada por una sucesión de cartas. Sin mo encabezamiento, donde se alude a las obligaciones o
embargo, en la que aquí se reproduce faltan algunos ele- normas a las que deberá atender el conductor. La exposi-
mentos típicos de la estructura clásica de las cartas per- ción sigue un orden claro: antes de adelantar, durante el
sonales. ¿Cuáles? Ayuda al protagonista a reescribir su adelantamiento y después de adelantar. Predomina, ade-
más, en el texto, la modalidad imperativa, materializada,
epístola para que se ajuste al esquema estudiado.
según se verá en actividades posteriores, en el empleo de
4 de diciembre las perífrasis verbales de obligación y el uso del se con
Te lo ruego… Mira, estoy acabado, ya no aguanto más. Hoy valor impersonal.
estaba sentado a su lado, estaba sentado; ella, al piano, tocan- b) Aparte de los conectores mencionados en el ejercicio ante-
do toda clase de melodías, y todas ellas ¡con qué expresión! rior que ordenan temporalmente los tres momentos de que
[…] A mí se me saltaban las lágrimas. Me incliné y cayó ante consta cualquier adelantamiento: antes, durante y después,
mi vista su anillo de desposada, se me saltaban las lágrimas. el texto emplea también conectores de contraste, como en
Y de repente le vino a la mente la vieja melodía, la melodía caso contrario, y de adición como asimismo y además,
celestial, así, de repente, y he aquí que por mi alma pasa un para contemplar todas las precauciones que debe tomar el
sentimiento de consuelo y un recuerdo del pasado, de los tiem- conductor a la hora de realizar el citado adelantamiento.
pos en que yo había oído esta canción. c) Ambos son textos prescriptivos con una finalidad prác-
En el fragmento reproducido faltan elementos típicos de la tica, pues a fin de obtener un determinado objetivo se
estructura clásica de la carta personal como el encabezamien- organizan mediante una enumeración ordenada de pasos
to, la introducción y la despedida. Son los aspectos que el que seguir. Coinciden también en el uso de la modalidad
alumnado deberá tener en cuenta para completar la epístola imperativa, siendo su principal diferencia el empleo de un
y, por tanto, la actividad. léxico específico que varía según el tema tratado.

Lengua castellana y Literatura. 3.º ESO. Solucionario 45


5 HÁBLAME DE TI

d) • No se debe acercar demasiado al vehículo […] ni que- • perteneció a


darse demasiado lejos. • sucedió
• Sin obligar al vehículo adelantado…
• alcanzó
• Para realizar el adelantamiento con seguridad.
d) • allí: adverbio de lugar, sustituye a en el pueblo.
2. Clive Staples Lewis fue un prestigioso profesor universi-
• que: pronombre relativo, sustituye a otros hombres.
tario que también alcanzó el éxito como escritor de no-
velas infantiles y juveniles. El texto que sigue pertenece • él: pronombre personal tónico, sustituye a Shasta.
a El caballo y el muchacho, uno de los siete libros que • le: pronombre personal átono, sustituye a Shasta.
integran la famosa serie de Las crónicas de Narnia. Léelo • aquella: determinante demostrativo, se refiere a lo que
y reescribe los tres primeros enunciados para convertirlos está al norte, lejos del hogar de Shasta.
en una carta personal.
Todas las palabras citadas son casos de deixis textual ana-
Shasta no sentía el menor interés por lo que estaba situa-
fórica, porque el elemento sustituido va antes del vocablo
do al sur de su hogar porque en una o dos ocasiones ha-
que lo sustituye.
bía estado en el pueblo con Arsheesh y sabía que allí no había
nada interesante. En el pueblo solo encontraba a otros hom- 4. Completa las siguientes oraciones con punto, dos puntos,
bres que eran iguales a su padre: hombres con largas túnicas coma, punto y coma, signos de interrogación o de excla-
sucias, zapatos de madera con las puntas vueltas hacia arriba, mación donde sea necesario:
turbantes en las cabezas y el rostro barbudo, que hablaban
entre sí muy despacio sobre cosas que parecían aburridas. Sin
a) Oye__ Sergio__ __estás seguro de lo que has dicho__
embargo, sí le atraía en gran medida todo lo que se encontraba b) Los ríos con los nombres más bonitos son tres__ el Lim-
al norte, porque nadie iba jamás en aquella dirección y a él tam- popo__ el Zambeze__ y el Amazonas__
poco le permitían hacerlo. Cuando estaba sentado en el exterior
remendando redes, y totalmente solo, a menudo dirigía ansio- c) Ayer me compré__ un bañador__ que ya me hacía falta__
sas miradas en aquella dirección. No se veía nada, a excepción unas sandalias __que las que tengo están muy usadas__
de una ladera cubierta de hierba que se alzaba hasta una loma y unas gafas de sol__
baja y, más allá, el cielo y tal vez unas cuantas aves en él. d) __Socorro__ auxilio__ me ahogo__
Respuesta libre. a) Oye, Sergio, ¿estás seguro de lo que has dicho?
3. Vuelve a leer el texto de C. S. Lewis y resuelve las siguien- b) Los ríos con los nombres más bonitos son tres: el Limpo-
tes tareas: po, el Zambeze y el Amazonas.
a) Indica la función sintáctica que desempeña el nexo en c) Ayer me compré un bañador, que ya me hacía falta; unas
la proposición subordinada adjetiva que eran iguales sandalias, que las que tengo muy usadas; y unas gafas
a su padre. de sol.
b) ¿Qué tipo de subordinación se da en la proposición d) ¡Socorro, auxilio, me ahogo!
que allí no había nada interesante? ¿Qué función
desempeña esta subordinada con respecto al verbo Recuerda lo que ya sabías
sabía? 5. El reglamento precedente de circulación nos permite re-
c) En eran iguales a su padre, el verbo ser posee su habi- pasar los tiempos verbales. Encuentra en el texto de la ac-
tual naturaleza atributiva. En cambio, en otros contextos tividad 1 una forma verbal en cada uno de estos tiempos:
se requiere de una mayor precisión léxica para recono-
• presente de indicativo
cer su valor predicativo. Sustituye en estas oraciones el
verbo ser por alguno de los siguientes: suceder, alcan- • futuro de indicativo
zar, tener lugar y pertenecer. • presente de subjuntivo
• El aterrizaje será en media hora. • está, pretende
• Ese coche fue de mi prima. • deberá
• Todo fue tan breve que ni siquiera nos besamos. • aproximen
• En el accidente la cifra de heridos fue de cien.
6. Conjuga en todas sus personas gramaticales los siguien-
d) Las palabras sombreadas en negrita en el texto de la acti- tes tiempos verbales:
vidad 2 desempeñan una función deíctica, puesto que se
refieren y sustituyen a otros vocablos o expresiones que a) El pretérito perfecto simple de indicativo del verbo
han aparecido previamente. ¿A qué categoría gramatical poner.
pertenece cada una de estas palabras? ¿Qué tipo de b) El pretérito imperfecto de subjuntivo del verbo saber.
deixis textual ejemplifican? c) El presente de indicativo del verbo dormir.
a) El relativo que funciona como sujeto en la proposición su-
bordinada adjetiva.
d) El pretérito pluscuamperfecto de indicativo del verbo
proveer.
b) … que allí no había nada interesante: subordinada sus-
tantiva en función de complemento directo. e) El presente de subjuntivo del verbo ir.
c) Junto con la sustitución del verbo ser por los mencionados, f) El presente de imperativo del verbo ir.
el alumno podrá realizar algunos mínimos cambios en las
oraciones para hacerlas gramaticales:
g) El condicional simple del verbo salir.
• tendrá lugar h) El futuro simple del verbo andar.

46 Lengua castellana y Literatura. 3.º ESO. Solucionario


HÁBLAME DE TI 5
a) puse, pusiste, puso, pusimos, pusisteis, pusieron. MIRA A TU ALREDEDOR Y…
b) supiera o supiese, supieras o supieses, supiera o supiese,
… ve más allá
supiéramos o supiésemos, supierais o supieseis, supie-
ran o supiesen. 1. Cuando se habla de famosos diarios personales, se nos
c) duermo, duermes, duerme, dormimos, dormís, duermen. viene de inmediato a la memoria la amarga experiencia
relatada en unos cuadernos por Ana Frank. Durante la
d) había proveído, habías proveído, había proveído, había-
Segunda Guerra Mundial, los ejércitos alemanes invadie-
mos proveído, habíais proveído, habían proveído.
ron Holanda. Ana pertenecía a una familia judía y estuvo
e) vaya, vayas, vaya, vayamos, vayáis, vayan. escondida con ella en una casa de Ámsterdam, entre los
f) ve (tú), id (vosotros). años 1942 y 1944, hasta que se descubrió su escondrijo y
g) saldría, saldrías, saldría, saldríamos, saldríais, saldrían. fue trasladada a un campo de concentración nazi. Desde
h) andaré, andarás, andará, andaremos, andaréis, andarán. su encierro escribió cosas como esta:
7. Enumera las perífrasis modales de obligación y posibili- Jueves, 19 de noviembre de 1942
dad que encuentres en el texto de la actividad 1, distin- A Kitty
guiendo en cada caso entre los verbos auxiliares y los No hay noche en que los coches militares verdes o grises no
auxiliados. recorran la ciudad; los alemanes llaman a todas las puer-
tas para dar caza a los judíos. Si los encuentran, embarcan
Perífrasis modal de inmediatamente a toda la familia; si no, llaman a la puerta
• puede (auxiliar) hacer (auxiliado)
posibilidad siguiente. Los que no se ocultan no escapan a su suerte. En
• debe (auxiliar) comprobar ocasiones, los alemanes se dedican a eso sistemáticamen-
(auxiliado) te, lista en mano, golpeando a las puertas tras las cuales,
• deberá (auxiliar) comprobar piensan, les aguarda un rico botín. A veces se les paga un
(auxiliado) rescate, a tanto por cabeza, como en los mercados de escla-
• debe (auxiliar) adelantar vos de antaño. Es demasiado trágico para que tú lo tomes a
Perífrasis modal de (auxiliado) broma. Por la noche, veo a menudo desfilar a esas caravanas
obligación • debe (auxiliar) acercar (auxiliado) de inocentes, con sus hijos llorando, arrastrados por algunos
• debe (auxiliar) circular (auxiliado) brutos que los azotan y los torturan hasta hacerlos caer. No
• (debe) (auxiliar) dejar (auxiliado) respetan a nadie, ni a los viejos, ni a las criaturas, ni a las
• debe (auxiliar) poner (auxiliado) mujeres embarazadas, ni a los enfermos: todos deben tomar
• (debe) (auxiliar) regresar parte en esa ronda de la muerte.
(auxiliado)
Ana F RANK: Diario.
a) Aunque ha habido estudiosos que han dudado sobre
8. Indica la función sintáctica que desempeñan los sintag- la autenticidad de este diario y han puesto en duda la
mas destacados: autoría de Ana, difícilmente puede discutirse el valor
• advertirlo con suficiente antelación informativo de su relato.
• [el carril] está libre • ¿Qué se conoce como el «holocausto judío»?
• entorpecer la marcha • ¿Por qué el régimen hitleriano persiguió a ese
• regresar al carril derecho de forma gradual pueblo?
• dejar una separación lateral suficiente • Su «antisemitismo» ¿era un caso de «racismo» o de
«xenofobia»?
• puede hacerlo sin ningún riesgo
• Con la ayuda de un diccionario, define los tres tér-
• entorpecer la marcha a los vehículos
minos entrecomillados.
• lo: complemento directo; con suficiente antelación: com-
b) ¿Conoces algún conflicto bélico internacional reciente
plemento circunstancial de modo.
que haya surgido por motivos raciales o religiosos?
• libre: atributo. ¿Cuáles?
• la marcha: complemento directo.
c) Investiga para qué servía este símbolo durante la dicta-
• de forma gradual: complemento circunstancial de modo. dura nazi y quiénes debían llevarlo. Averigua también
• una separación lateral suficiente: complemento directo. si existían otros símbolos para marcar a otras personas
• sin ningún riesgo: complemento circunstancial de modo. y cuáles eran.
• a los vehículos: complemento indirecto. d) En España también los judíos fueron víctimas de dra-
9. Explica el significado de los siguientes prefijos y escribe al máticas persecuciones y matanzas. ¿En qué época
menos dos palabras que los contengan: hecto-, hetero-, he- tuvo lugar tal fenómeno? Justifica tu respuesta con
mato-, helio-, hidra-, homo-. datos históricos.
• hecto-: cien; hectárea, hectogramo, hectolitro, hectómetro. e) ¿Qué semejanzas adviertes entre la narración de
• helio-: sol; heliocéntrico, helioterapia. Ana y una noticia periodística? Convierte el relato
de Ana en una noticia. No olvides que debes ser obje-
• hemato-: sangre; hematoma, hematosis, hematocrito.
tivo y veraz.
• hetero-: diferente, otro; heterodoxia, heterogéneo.
a) • El «holocausto judío» fue el terrible genocidio, durante
• hidro-: agua; hidroavión, hidroeléctrico, hidrosfera. la Segunda Guerra Mundial, de seis millones de judíos
• homo-: mismo, igual; homófono, homogéneo, homologar. por parte del gobierno nazi de Hitler.

Lengua castellana y Literatura. 3.º ESO. Solucionario 47


5 HÁBLAME DE TI

• El régimen hitleriano persiguió al pueblo hebreo am- 2. A los efectos de la presente Declaración, se entiende por
parándose en la creencia de que los alemanes pertene- «intolerancia y discriminación basadas en la religión o
cían a una raza superior, la aria, y, como los judíos eran las convicciones» toda distinción, exclusión, restricción
una etnia inferior, merecían morir por ello. Los nazis o preferencia fundada en la religión o en las convicciones
también consideraban inferiores a otras razas como los y cuyo fin o efecto sea la abolición o el menoscabo del
gitanos u otros pueblos eslavos como rusos y polacos. reconocimiento, el goce o el ejercicio en pie de igualdad
• Su «antisemitismo» es un caso de racismo. de los derechos humanos y las libertades fundamentales.
• Antisemitismo: actitud contraria y hostil frente a la raza a) ¿Con qué tipo de textos relacionarías el artículo? ¿Por
y cultura hebreas; racismo: aversión a otros grupos ét- qué?
nicos; xenofobia: odio u hostilidad a los extranjeros.
b) ¿Se respetan los principios enunciados en el escrito en
b) Los conflictos bélicos por cuestiones raciales siguen es- España? En el caso de no ser así, ¿qué medidas adop-
tando, desafortunadamente, a la orden del día. En muchas
tarías para evitar cualquier tipo de discriminación? Haz
ocasiones, a las diferencias raciales se le suman discre-
pancias religiosas y conflictos económicos. Tomando
tus propias propuestas.
como referencia algunos de los casos que se citan, podría a) El fragmento reproducido es relacionable con los regla-
animarse al alumnado a profundizar en tan trágicos epi- mentos. Se inscribiría dentro del ámbito jurídico, junto a
sodios: Rodhesia, apartheid sudafricano, conflicto de los los textos prescriptivos que se consideran como normas
Balcanes, genocidio de Ruanda. de obligado cumplimiento en los lugares donde ha sido
c) A los judíos se les marcaba con la insignia amarilla o es- aprobada la citada Declaración. De hecho, al igual que
trella de David sobre un fondo amarillo y con la palabra en los textos legales, el artículo va dividido, a su vez, en
Jude (judío). En los campos de concentración, los presos otros dos subapartados donde se precisa el alcance de los
eran clasificados también mediante una simbología que distintos casos contemplados por el órgano legislador.
se puede consultar en el siguiente enlace: wikipedia. b) Respuesta libre.
org/wiki/Sistema_de_marcado_en_los_campos_de_
2. Imagínate que te conviertes en un personaje de una serie
concentraci%C3%B3n_nazis
televisiva que relata los problemas de una comunidad
d) La hostilidad hacia los judíos en la península ibérica alcan- de vecinos.
zó, durante la Edad Media, periodos de auténtico terror.
A grandes rasgos, deben señalarse estos momentos. De pronto, pasas a ser el presidente de la finca y tienes
• En la época visigoda, la conversión del rey Recaredo al que resolver sendos problemas urgentes: las goteras del
catolicismo condujo a que ya en el siglo VII, durante el ático y el cambio de las puertas de los ascensores. Como
reinado de Sisebuto, se endureciesen las leyes antijudías. necesitas recaudar fondos, redacta una convocatoria para
• En el siglo XV, el antisemitismo se cebó contra los judíos citar a tus vecinos a una reunión inmediata. Recuerda
conversos. A raíz de las crisis económicas y políticas que acompañarla del orden del día correspondiente. Para ello,
tuvieron lugar entre 1449 y 1474, hubo revueltas contra quizá te resulte interesante buscar en Internet modelos
los conversos en varias ciudades, siendo la más cruenta de convocatorias.
la de 1449 en Toledo. Respuesta libre.
• En las Cortes de Toledo de 1480, se obligó a los judíos a
vivir en barrios separados o juderías.
3. En un partido de fútbol el árbitro no solo actúa como juez,
sino que, después de terminado el enfrentamiento, deja
• A través de la Inquisición, los Reyes Católicos decidieron
expulsar a los judíos de Andalucía en 1483. Los mismos
constancia de las principales incidencias en unas actas.
monarcas publicaron el decreto de expulsión de dicho Estas sirven, por ejemplo, para que los comités de disci-
pueblo a finales de abril de 1492, conquistada ya Granada. plina deportiva sancionen a los jugadores amonestados.
e) Lo dicho en su diario por Ana puede considerarse como Busca en Internet el acta arbitral del partido de vuelta de la
una información verosímil y de rabiosa actualidad en el Supercopa de España de 2014 para tomarla como modelo,
momento de la escritura (signifíquese el uso de los verbos ejerce tú también como árbitro y registra lo sucedido en
en presente), que pueden interesar a un público mayorita- un partido en el que tú mismo hayas participado o en uno
rio. Esas son las similitudes con una noticia periodística que hayas visto.
que deberían servir de acicate para realizar la actividad Respuesta libre.
propuesta a continuación.
4. Los anuncios publicitarios están plagados de eslóganes
… encuentra la clave que incorporan proposiciones subordinadas adverbia-
1. La Asamblea General de las Naciones Unidas proclamó el les. Con ellas se remarca el motivo, la finalidad y otras
25 de noviembre de 1981 la resolución 36/55, con la Decla- circunstancias que justifican la conveniencia de adquirir
ración sobre la eliminación de todas las formas de intole- determinado producto. Así, podemos encontrarnos con
rancia y discriminación fundadas en la religión o las convic- subordinadas del tipo «Porque yo lo valgo» (causal) o «Si
ciones. De ella entresacamos este artículo: bebes, no conduzcas» (condicional). Sirviéndote de Inter-
Artículo 2 net, elabora un mural o dossier donde aparezcan eslóganes
publicitarios que te permitan ejemplificar los distintos tipos
1. Nadie será objeto de discriminación por motivos de reli-
gión o convicciones por parte de ningún Estado, institu- de subordinación adverbial.
ción, grupo de personas o particulares. Respuesta libre.

48 Lengua castellana y Literatura. 3.º ESO. Solucionario


EN LA VARIEDAD ESTÁ EL GUSTO 6
APERTURA DE UNIDAD Las acotaciones nos dan información sobre los movimientos
de los personajes, lo que hacen y cómo reaccionan ante la
1. El texto que acabas de leer es el inicio de un entremés situación que se produce en el estanco.
muy divertido de los hermanos Álvarez Quintero. Busca
4. La escena supuestamente es dramática, ya que se trata
en Internet información sobre qué tipo de obra teatral es
de un atraco violento con un arma de fuego. Pero en ella
un entremés y sobre los autores de este.
se trasluce cierta comicidad. ¿Dónde puedes observarla?
El entremés es una pieza cómica breve, y fue uno de los géne-
La comicidad de la escena se trasluce sobre todo en el len-
ros más populares durante el Siglo de Oro. Generalmente se
guaje, con expresiones como se lanzan al lío, en un tris, un
intercalaba entre las distintas jornadas (actos) de la repre-
pistolón de aquí te espero, esto es un atraco, como en el cine,
sentación. Varios eran los tipos de entremeses, destacando
la mando al otro barrio, etc. Es especialmente patente en la
los que satirizan acerca de las costumbres sociales y los en-
manera de hablar de los atracadores.
tremeses cantados y bailados, mucho más creativos y fuera
de convenciones. 5. ¿Cuál de los dos atracadores crees que lleva las riendas
Los hermanos Álvarez Quintero (Serafín y Joaquín), se- de la situación, Tocho o Leandro? ¿Por qué?
villanos, escribieron la mayor parte de su producción li- En cierto modo Leandro, porque aunque sea Tocho el que ini-
teraria a finales del siglo XIX y principios del XX . Fueron cia el atraco, después Leandro le da órdenes, y su tono es más
ambos poetas, narradores, periodistas, pero, sobre todo cortante y agresivo.
comediógrafos. Son exponentes principales de un teatro po-
pular, costumbrista, basado en el sainete, el género chico Vocabulario
o la comedia.
6. El texto está lleno de expresiones coloquiales, propias de
2. En este fragmento aparecen dos recursos muy típicos del un registro informal de la lengua. Localízalas y escríbelas
teatro: la acotación y el monólogo. Señala qué partes co- en tu cuaderno. ¿Son apropiadas para un contexto comu-
rresponden a cada uno de ellos y qué información nos dan. nicativo formal?
Las acotaciones son las partes inicial y final, que están es- Se lanzan al lío, en un tris, un pistolón de aquí te espero, la
critas en cursiva. La primera nos informa de dónde se va a pasta o la mando al otro barrio, pa la compra, unos gritos
desarrollar la escena y nos describe cómo es Martirio, su que pa qué, se arranca por peteneras, a duras penas, la cosa
protagonista. La última nos indica la entrada en escena de se pone negra, le saco las tripas al aire a ventilarse, será
su novio, Julián. animal, podemos echar un tute si cuadra…

El monólogo es toda la intervención de Martirio, en la que esta Estas expresiones no serían adecuadas para un contexto co-
nos hace saber sus intenciones de reñir esa tarde con su novio. municativo formal.

3. ¿Te llama la atención la forma de expresarse de Martirio? 7. Dichas expresiones coloquiales nos dan una pista sobre el
¿Por qué? ¿Podemos deducir de qué zona de España es nivel social y cultural de los atracadores. ¿A qué clase social
este personaje gracias a la variante geográfica del caste- crees que pertenecen?
llano que utiliza? Los atracadores parecen ser de un nivel sociocultural bajo.
Deducimos que Martirio es andaluza, debido a su forma ca- 8. Aparecen también algunas expresiones propias de la jerga
racterística de hablar. juvenil, es decir, la manera de hablar propia de los jóvenes,
4. Ved ahora este entremés representado en teatro. Fíjate en como por ejemplo, decir la pasta para referirse al dinero.
la comicidad de la escena y en la manera de expresarse de Escribe en tu cuaderno otras expresiones que consideres
ambos personajes y anota los rasgos que te parezcan más propias del lenguaje que usáis los jóvenes.
llamativos. Para ver la escena, teclea en YouTube «María Respuesta libre.
Vidal y Paco Valladares» «Ganas de reñir». 9. ¿Encuentras en el texto algún vulgarismo o alguna inco-
Respuesta libre. rrección? Señala dónde y di cuál sería la forma correcta.
Pa en lugar de para, la pego un tiro en lugar de le pego un tiro.
VIVE LA LECTURA
10. En la primera acotación del fragmento se dice que Tocho
Comprensión lectora «se busca los duros» para, supuestamente, pagar el paque-
1. ¿Cómo reacciona la estanquera ante el intento de atraco? te de tabaco. ¿Recuerdas tú qué era un duro? ¿Sabrías
¿Se lo esperaban los atracadores? decir a cuánto equivale en céntimos de euro?
Un duro equivaldría a 0,03 céntimos de euro.
Reacciona resistiéndose, dando voces y pidiendo auxilio. Los
atracadores no se esperaban esta reacción.
Investigación y redacción
2. ¿Qué rasgos de la personalidad de los personajes pode-
11. En la biografía de José Luis Alonso de Santos, el autor de
mos deducir de sus intervenciones?
esta obra, se señalaba que fue durante unos años director
Los atracadores, pese a su papel de «malos», tienen mucha de la Compañía Nacional de Teatro Clásico. Busca infor-
gracia y sentido del humor, sobre todo Tocho. Leandro es algo mación en Internet sobre dicha compañía, por ejemplo,
más violento y agresivo. La abuela es muy expresiva y tiene procura averiguar cuáles han sido sus mayores éxitos o
mucho carácter, no se deja amedrentar por los atracadores
quién la dirige en la actualidad.
ni mucho menos.
La Compañía Nacional de Teatro Clásico es una unidad de
3. Fíjate en la información que nos dan las acotaciones. ¿Para producción del INAEM (Instituto Nacional de la Artes Escéni-
qué sirven? ¿Qué tipo de indicaciones da el autor a través cas y de la Música) que depende del Ministerio de Educación,
de ellas? Cultura y Deporte. Creada por Adolfo Marsillach en 1986, es

Lengua castellana y Literatura. 3.º ESO. Solucionario 49


6 EN LA VARIEDAD ESTÁ EL GUSTO

la institución de referencia en la recuperación, preservación, e) Variedad geográfica, se expresan de acuerdo a la zona de


producción y difusión del patrimonio teatral anterior al si- la que proceden.
glo XX, con especial atención al Siglo de Oro y a la prosodia f) Variedad social (pertenencia a un grupo) y también
del verso clásico. de situación (ya que se trata de una situación formal y
En sus casi treinta años de historia la Compañía ha llevado a académica).
cabo más de ochenta y cinco producciones que han viajado no g) Variedad de situación (ya hemos visto como ese mismo pro-
solo por todo el estado español, sino también por muchos lu- fesor se expresa de forma diferente en otras situaciones).
gares de Europa y América, afianzando con su presencia en
ellos la voluntad de difusión de nuestro patrimonio dramático. 2. Escribe una conversación en la cual dos jóvenes empleen
una variedad social propia de su edad (lenguaje juvenil) y,
La directora de escena Helena Pimenta dirige la Compañía
a continuación, la misma conversación entre dos adultos
desde septiembre de 2011.
empleando la lengua estándar. Cada una de ellas deberá
12. Inventa y escribe una pequeña escena teatral que conti- constar, como mínimo, de sesenta palabras.
núe la que acabas de leer. Debes conservar en escena a Respuesta libre.
Tocho, Leandro, la Abuela y Ángeles. Intenta mantener
el tono y la manera de hablar de los personajes. Deberá 3. ¿Alguien de tu entorno cercano emplea alguna variedad
constar, al menos, de cien palabras. geográfica o social de la lengua? Si es así, explica cuál es
dicha variedad y cuáles son sus peculiaridades más des-
Respuesta libre. tacadas.
13. Busca información en la red acerca de la adaptación al Respuesta libre.
cine de la obra teatral La estanquera de Vallecas (direc-
4. Lee los siguientes textos, identifica el dialecto del que se
ción, producción, actores principales…). A continuación
trata y señala sus rasgos más característicos:
ved la versión cinematográfica del fragmento que habéis
leído. Teclead en YouTube «La estanquera de Vallecas a) Y xunt a ello taba una mociquina tamién muy guapa; vistía
1987». Luego, comentad si os imaginabais así la escena. de blancu, quey dixo: «¿Qué te gusta más de tou?». El mozu
quedóse clisáu n’ello, yen vez ce icíi, «gústesme tú», dixoy:
La estanquera de Vallecas es un película de 1987 dirigida por «anquel anfiler dioru».
Eloy de la Iglesia, basada en la obra teatral homónima de José
Luis Alonso de Santos (1981). Fue producida por Ega Medios
b) L’ochetibo de o premio Pirene ye promober un millor
conoximiento de a reyalidá pirinenca en toz os suyos
Audiovisuales.
aspeutos. Pueden optar a o premio Pirene os treballos pe-
En el reparto tenemos a Emma Penella como doña Justa (la riodisticos (de prensa, arradio u telebisión) que seigan es-
Abuela), a José Luis Gómez como Leandro, a José Luis Manza- tatos publicatos u esparditos entre l’1 de chulio de 2009 e o
no como Tocho y a Maribel Verdú como Ángeles. 30 de chunio de 2010 e que faigan referenzia á cualsiquier
aspeuto rilazionato con os Pirineyos.
ESTUDIO DE LA LENGUA a) Se trata de un texto perteneciente al dialecto astur-leonés.
Lo sabemos por los diminutivos en -ina (mociquina), el
1. Explica razonadamente a qué tipo de variedades corres- cierre de vocales finales (blancu, tou, mozu, dioru), o la
pondería la lengua empleada en las siguientes situaciones: colocación de los pronombres (quedóse, gústesme).
a) Dos presos hablando con expresiones propias de la b) Texto perteneciente al dialecto navarro-aragonés, como
cárcel. podemos observar en la diptongación de vocales (seigan,
faigan), o el mantenimiento de la f- inicial latina (faigan).
b) Un grupo de andaluces empleando el dialecto propio
de su zona. 5. Investiga en la red cuáles fueron los primeros testimo-
nios escritos en lengua navarro-aragonesa, y de qué si-
c) Unos jóvenes en una fiesta expresándose de acuerdo glo datan, y escribe la información más relevante en tu
con su edad. cuaderno.
d) Un joven en una entrevista de trabajo utilizando un Parece que los primeros testimonios escritos del navarro-ara-
registro formal. gonés datan del siglo XI y proceden de las glosas emilianen-
e) Los actores de una telenovela venezolana. ses, anotaciones manuscritas en los márgenes de un códice
latino, compuestas en el monasterio riojano de San Millán de
f) Un profesor dando una conferencia a un grupo de la Cogolla.
alumnos universitarios.
6. Vuelve a leer el fragmento del entremés con el que se
g) El mismo profesor charlando distendidamente con sus abre la unidad, Ganas de reñir, y comenta los rasgos dia-
amigos. lectales del andaluz que observes.
a) Variedad social, se expresan así porque pertenecen a un Aparecen rasgos propios del andaluz como el seseo (grasia,
determinado grupo (en este caso, marginal). cabeza, durse, resibí), la confusión entre -r- y -l- (durse, sir-
b) Variedad geográfica, porque depende de la zona en la que ba), la pérdida de -d- intervocálica (pescao, to), pérdida de
se habla. consonantes finales (señó, reñí, tomá, é, durá, resibí, tené,
sorbé) o el yeísmo (ayí).
c) Variedad social, ya que se expresan de acuerdo a su edad
y a su grupo. 7. Copia este mapa de España en tu cuaderno. A continua-
ción, marca en él, con diferentes colores, las zonas donde
d) Variedad de situación, ya que depende de la situación co- se hablan los distintos dialectos del castellano.
municativa (ese mismo joven en otro contexto se expre-
saría de forma diferente). No te olvides de incluir, además, una leyenda en la que
expliques qué color corresponde a cada dialecto.

50 Lengua castellana y Literatura. 3.º ESO. Solucionario


EN LA VARIEDAD ESTÁ EL GUSTO 6
Mapa con los dialectos del castellano: con ellas, aprenderían a valorarlas y a la larga desaparecerían
ciertas actitudes heredadas».
Respuesta libre.
11. Investiga en Internet cuáles son las principales lenguas in-
dígenas que se hablan en América. Resume la información
Zona que consideres más importante en tu cuaderno.
de Castellano
influencia
del Zona
de
Las lenguas indígenas de América son aquellas habladas en
leonés
influencia
del el continente americano desde los primeros asentamientos
central aragonés
humanos hasta la llegada de los colonizadores europeos, afri-
canos y asiáticos.
Extremeño
Aunque muchas de ellas se han extinguido, aún se siguen ha-
blando más de mil lenguas indígenas, pertenecientes a diver-
Murciano sas familias.
Dialectos septentrionales
Andaluz del español Las principales lenguas indígenas son:
Variables del español

Canario
habladas en los territorios
del gallego, vasco y catalán • Quechua: de 9 a 14 millones de hablantes.
Seseo Dialectos meridionales • Guaraní: de 7 a 12 millones. Aunque algunos apuntan una
del español
Ceceo
cifra de entre 15 y 22 millones de hablantes.
• Aimara: de 2 a 3 millones.
• Nahua: de 1,3 a 5 millones.
8. Define en tu cuaderno los siguientes conceptos: lengua
estándar, diversidad lingüística, variedades de la lengua, • Maya: de 900.000 a 1,2 millones. Las lenguas de la familia
variedad geográfica o dialecto. maya son habladas por entre 2 y 7 millones de personas.

Lengua estándar: es la lengua común que todos los hablantes 12. Busca en el Diccionario de la lengua española (lema.rae.
de una lengua deben conocer para entenderse entre sí. Es la es) el significado de los siguientes términos que provie-
lengua del ámbito académico y de los medios de comunica- nen de lenguas indígenas, y anótalo en tu cuaderno junto
ción, y debe respetar una serie de reglas y normas. a la lengua de la que proceden:
Diversidad lingüística: son las diferencias en las maneras de choclo cacao chocolate
expresarse de los hablantes de una misma lengua. canoa alpaca vicuña
Variedades de la lengua: son aquellas diferencias ocasionadas cóndor tomate puma
por factores geográficos, sociales o de situación, y que origi-
nan la diversidad lingüística. guacamayo coyote cigarro
Variedad geográfica o dialecto: es la que depende de la zona • choclo (del quechua): mazorca tierna de maíz.
en la que se hable una determinada lengua. • canoa (del taíno): embarcación de remo muy estrecha, nor-
malmente de una pieza y sin diferencia de forma entre proa
9. Busca en Internet información sobre los guanches, los pri- y popa.
mitivos habitantes de las islas Canarias. Copia dicha infor-
• cóndor (del quechua): ave rapaz de poco más de un metro
mación en tu cuaderno y resúmesela a tus compañeros de
de longitud y tres de envergadura, y plumaje fuerte de color
forma oral.
negro azulado. Habita en los Andes y es la mayor de las
Los guanches eran los antiguos aborígenes que habitaban aves que vuelan.
en la isla de Tenerife, antes de la conquista de las islas por • guacamayo (del taíno): ave de América, especie de papa-
la Corona de Castilla en 1496. Por extensión este término se gayo, del tamaño de una gallina y fuertes colores.
ha aplicado también a los aborígenes del resto de las islas
Canarias. Poblaron las islas durante casi dos mil años hasta • cacao (del nahua): árbol de América cuyo fruto brota di-
que fueron sometidos por las tropas castellanas. Se dedicaban rectamente del tronco y las ramas y que se emplea como
fundamentalmente al pastoreo y a una agricultura básica de principal ingrediente del chocolate.
cereales. • alpaca (del aimara): mamífero rumiante, de la misma fa-
milia que la llama, propia de américa meridional y muy
10. Lee este fragmento de un texto publicado en la sección
apreciado por su pelo, que se emplea en la industria textil.
de cultura del ABC, que expone la opinión acerca de la
diversidad lingüística de una profesora del Instituto de • tomate (del nahua): fruto de la tomatera, que es una baya
casi roja, de superficie lisa y brillante, en cuya pulpa hay
Lengua, Literatura y Antropología del CSIC, Pilar García
numerosas semillas, algo aplastadas y amarillas.
Mouton. Después argumenta si estás o no de acuerdo
con sus afirmaciones. • coyote (del nahua): especie de lobo que se cría en México
y otros países de América, de color gris amarillento y del
García Mouton considera «una pena que se haya politizado tamaño de un perro mastín.
tanto todo lo relacionado con el legado lingüístico, que de-
• chocolate (del nahua): pasta hecha con cacao y azúcar mo-
bería considerarse patrimonio de todos, una riqueza cultu-
lidos, a los que generalmente se añade canela o vainilla.
ral». En su opinión, convendría seguir la recomendación de
la Unesco de escolarizar a los niños en su lengua materna • vicuña (del quechua): mamífero rumiante del tamaño del
y luego ir añadiendo otras a sus conocimientos. «Y también macho cabrío, pero con cuello más largo y erguido, cabeza
convendría que a los niños españoles se les enseñase desde más redonda y sin cuernos, orejas puntiagudas y derechas
pequeños que, además de la suya, hay otras lenguas en su y piernas muy largas. Vive salvaje en manadas en los Andes
entorno, y que aprendiesen rudimentos de esas otras lenguas del Perú y de Bolivia, y se caza para aprovechar su vellón,
a través de canciones, adivinanzas… Así se familiarizarían que es muy apreciado.

Lengua castellana y Literatura. 3.º ESO. Solucionario 51


6 EN LA VARIEDAD ESTÁ EL GUSTO

• puma (del quechua): felino americano de unos 180 cm de • mazo: mucho. (Jerga juvenil).
longitud, de color rojizo o leonado uniforme, que vive en • traqueotomía: abertura que se hace artificialmente en la
serranías y llanuras. tráquea para impedir en ciertos casos la sofocación de los
• cigarro (del maya): rollo de hojas de tabaco, que se enciende enfermos. (Jerga profesional, de la medicina).
por un extremo y se fuma por el opuesto. • trullo: cárcel, presidio. (Jerga marginal).
13. Corrige los vulgarismos que encuentres en las siguientes 15. Señala cuáles de estos enunciados te parecen propios
oraciones. ¿A qué nivel de la lengua pertenecerían? ¿Qué de un registro formal y cuáles de uno informal, argumen-
información nos dan acerca del grado de formación de los tando por qué:
hablantes que los emplean?
a) No se entera de la misa la media, está empanado.
Corrige b) Lamentamos profundamente el malentendido origina-
do por la falta de comunicación.
a) Amos, que no se ha enterao de na de lo que le he
c) ¿Me haría usted el favor de alcanzarme aquel libro de
contao.
la estantería?
b) Mi agüela hace unas almóndigas pa chuparse los
d) Te he dicho mil veces que no seas tan plasta.
dedos.
e) A causa de una mala gestión de los fondos de la em-
c) Ayer andamos ventidós kilómetros y estoy muy
presa, se han visto obligados a realizar reajustes de
cansao.
plantilla.
d) No me dijistes que la Marta y el Rubén se habían
arrejuntao. f) No tiene un pelo de tonta, está siempre a la que salta.
e) Me se ha estropeado el CD que me regalastes. g) Voy a darme un garbeo por ahí, estoy hasta el gorro
de estar metido en casa.
f) Joder, si te gusta tanto el vestido, cómpratele.
h) Le rogaría que nos confirmase la recepción de la do-
g) Se me ha infestado la herida y mi madre me ha
cumentación en un plazo razonable.
echado agua oxigenada.
a) Registro informal, ya que emplea términos coloquiales y
a) Vamos, que no se ha enterado de nada de lo que le he expresiones hechas.
contado. b) Registro formal. El léxico es más cuidado, la sintaxis más
b) Mi abuela hace unas albóndigas para chuparse los dedos. elaborada.

c) Ayer anduvimos veintidós kilómetros y estoy muy c) Registro formal, ya que emplea un léxico cuidado, y utiliza
cansado. el usted.
d) Registro informal, utiliza expresiones coloquiales como
d) No me dijiste que Marta y Rubén se habían ido a vivir
plasta.
juntos.
e) Registro formal, con un léxico cuidado, una sintaxis elabo-
e) Se me ha estropeado el CD que me regalaste.
rada, uso de eufemismos (reajustes de plantilla en lugar
f) Si te gusta tanto el vestido, cómpratelo. de despidos).
g) Se me ha infectado la herida y mi madre me ha echado f) Registro informal, ya que emplea frases hechas y coloquia-
agua oxigenada. les (un pelo de tonta, a la que salta).
Todas estas incorrecciones pertenecen al nivel vulgar de la g) Registro informal. Utiliza expresiones coloquiales (gar-
lengua y nos indican un nivel sociocultural bajo. beo, hasta el gorro).
14. Busca el significado de las siguientes palabras en el diccio- h) Registro formal. Emplea un léxico cuidado, una sintaxis
nario e indica si pertenecen a una jerga profesional, juvenil elaborada y hace uso de la forma de cortesía usted.
o marginal: molar (verbo), diurético, faca, pasma, cogorza, 16. Escribe un diálogo entre dos amigos que emplean un re-
sumario, colonoscopia, mazo, traqueotomía, trullo. gistro coloquial o informal. A continuación, escribe ese
• molar: gustar, resultar agradable o estupendo. (Jerga mismo diálogo entre dos interlocutores que no tienen
juvenil). confianza entre ellos, por lo que emplean un registro
• diurético: que tiene virtud para aumentar la excreción de
mucho más formal. Cada uno deberá constar, al menos,
orina. (Jerga profesional, de la medicina). de sesenta palabras.
Respuesta libre.
• faca: cuchillo corvo. (Jerga marginal).
• pasma: policía. (Jerga marginal y por extensión, jerga EL TALLER DE LAS PALABRAS
juvenil).
1. Escribe el significado denotativo de las siguientes palabras:
• cogorza: borrachera. (Jerga juvenil).
instituto, nieve, agosto, piscina, pueblo y ordenador. A con-
• sumario: conjunto de actuaciones encaminadas a preparar tinuación, anota en tu cuaderno qué significados connota-
el juicio criminal, haciendo constar la perpetración de los tivos tienen para ti esas mismas palabras.
delitos con las circunstancias que puedan influir en su cali-
Respuesta libre.
ficación, determinar la culpabilidad y prevenir el castigo de
los delincuentes. (Jerga profesional, del campo del derecho 2. Compara con un compañero las connotaciones que tie-
y la justicia). nen para vosotros los términos de la actividad 1. ¿Podéis
• colonoscopia: exploración del interior del colon mediante explicar las discrepancias?
un colonoscopio. (Jerga profesional, de la medicina). Respuesta libre.

52 Lengua castellana y Literatura. 3.º ESO. Solucionario


EN LA VARIEDAD ESTÁ EL GUSTO 6
3. En algunas culturas y religiones, las palabras tienen un • dar en el clavo: acertar. Has dado en el clavo con la solu-
determinado significado connotativo. Busca información ción a ese problema.
sobre cuál es el significado connotativo de vaca para los • estar a dos velas: estar sin dinero. A finales de mes estoy
hindúes, de cerdo para los musulmanes y de cordero para siempre a dos velas, no puedo permitirme salir a cenar.
los cristianos. • quedarse en blanco: no saber qué decir, no acordarse de
Para los hindúes las vacas son animales sagrados, que deben lo se iba a decir. Me quedé en blanco en el examen oral de
ser venerados y respetados. italiano.
Para los musulmanes el cerdo es un animal impuro y por lo • ser coser y cantar: ser muy sencillo. Esta tarea es coser y
tanto no consumen su carne. cantar, no tiene ninguna complicación.
Para los cristianos el cordero simboliza a Jesucristo (el Cor- • hincar los codos: estudiar. Estuvo hincando los codos toda
dero de Dios). la noche para el examen de álgebra.
• empinar el codo: beber alcohol. Aquel hombre ha empinado
4. Piensa en alguna palabra que esté muy marcada conno-
demasiado el codo y va dando tumbos.
tativamente en la sociedad en la que vives, y explica por
qué. • echar una mano: ayudar. Échame una mano con las ma-
letas, no puedo subirlas solo.
Respuesta libre.
• morder el polvo: ser derrotado. En el primer asalto el con-
5. Completa los refranes y explica su significado: trincante mordió el polvo.
a) A buenas horas… • dar el brazo a torcer: ceder. No voy a dar mi brazo a torcer
b) Se coge antes a un mentiroso… en ese asunto, sé que tengo razón.

c) A caballo regalado… • vivir del cuento: vivir sin trabajar, valiéndose de la ayuda
de otros. Juan no trabaja, vive del cuento.
d) A falta de pan…
• estar en las nubes: estar distraído. Mateo siempre está en
e) En boca cerrada… las nubes, no se entera de nada.
f) Quien a buen árbol se arrima… • tener mala pata: tener mala suerte. ¡Qué mala pata tengo!
g) Aunque la mona se vista de seda… Se me ha estropeado el coche otra vez.

a) A buenas horas, mangas verdes. (Se usa cuando alguien 8. Escribe en tu cuaderno tres refranes, tres proverbios y
llega demasiado tarde). tres frases hechas. A continuación inventa situaciones en
b) Se coge antes a un mentiroso que a un cojo. (Si mientes, las que podría ser apropiada su utilización.
tarde o temprano te acaban descubriendo). Respuesta libre.
c) A caballo regalado, no le mires el diente. (Si te regalan 9. Os sorprenderá descubrir la cantidad de refranes en es-
algo, acéptalo sin poner pegas). pañol que versan sobre un mismo tema. Dividid la clase en
d) A falta de pan, buenas son tortas. (Cuando no tenemos dos grupos, la mitad de vosotros se encargará de buscar
posibilidad de elegir, hay que conformarse con lo que se en Internet refranes que tengan como protagonistas a
tiene). los gatos; la otra mitad, refranes que traten sobre perros.
e) En boca cerrada no entran moscas. (A veces es mejor es- Una vez hecha la recopilación, ponedla en común ante el
tar callado para no decir alguna imprudencia). resto de compañeros. El profesor se encargará de llevar la
f) Quien a buen árbol se arrima, buena sombra le cobija. cuenta. ¿Qué grupo ha conseguido reunir más refranes?
(Si tienes buenas amistades, estas te ayudarán cuando lo Respuesta libre.
necesites).
g) Aunque la mona se vista de seda, mona se queda. (Por EL RINCÓN DE LA NORMA
mucho que se intente disimular con ropas o maquillajes, 1. Justifica el uso de rayas en el texto de Laura Gallego.
la fealdad no se puede esconder).
Las rayas en este texto sirven para marcar las intervenciones
6. Di si estás o no de acuerdo con los siguientes proverbios de los personajes, así como para incluir aclaraciones dentro
y justifica tu respuesta: del diálogo.
a) Quien bien te quiere te hará llorar. 2. Añade los paréntesis o las comas que consideres oportu-
b) La probabilidad de hacer el mal se encuentra cien ve- nos en el siguiente texto:
ces al día; la de hacer el bien, una vez al año. Corrige
c) El infierno está lleno de buenas intenciones y el cielo Tristán e Iseo es una leyenda del ciclo artúrico sobre el
de buenas obras. rey Arturo que relata los amores entre Iseo apodada la ru-
d) La palabra es plata y el silencio es oro. bia y el caballero Tristán de Leonís. «La rubia» es el apodo
que el autor utiliza para distinguirla de su homónima la de
e) Si lloras por no haber visto el sol, las lágrimas no te las blancas manos. La trama de raigambre posiblemente
dejarán ver las estrellas. vikinga la constituye el triángulo amoroso formado por
Respuesta libre. Tristán y ambas damas.
7. Explica el significado de las siguientes frases hechas, y Tristán e Iseo es una leyenda del ciclo artúrico (sobre el rey
después escribe en tu cuaderno una oración con cada Arturo), que relata los amores entre Iseo (apodada la rubia) y el
una de ellas: dar en el clavo, estar a dos velas, quedarse caballero Tristán de Leonís. La «rubia» es el apodo que el autor
en blanco, ser coser y cantar, hincar los codos, empinar el utiliza para distinguirla de su homónima (la de las blancas ma-
codo, echar una mano, morder el polvo, dar el brazo a tor- nos). La trama (de raigambre posiblemente vikinga) la constitu-
cer, vivir del cuento, estar en las nubes, tener mala pata. ye el triángulo amoroso formado por Tristán y ambas damas.

Lengua castellana y Literatura. 3.º ESO. Solucionario 53


6 EN LA VARIEDAD ESTÁ EL GUSTO

3. Escribe una breve autobiografía en la que uses paréntesis b) ¿Quién es el emisor de cada texto? ¿Se puede señalar
para ofrecer detalles o aclaraciones. en ellos la presencia de un narrador? Indica cuál es en
Respuesta libre. cada caso.
4. Escribe correctamente los siguientes compuestos (con c) En cuanto a la intención comunicativa de ambos emi-
o sin guion): sores, ¿es la misma? En caso contrario, ¿en qué se
a) obsesivo compulsivo diferencian?
b) cabeza rapada d) Observa el código de ambos textos y explica las dife-
c) telefonía móvil rencias que encuentres.
a) Es literario el primero de ellos, ya que posee una finalidad
d) físico químico
eminentemente estética, intenta crear belleza a través de
e) peruano ecuatoriano las palabras.
f) histórico artístico b) En el primero de ellos el emisor es Chejov. Además hay un
g) carril bici narrador omnisciente en tercera persona.
a) obsesivo-compulsivo En el segundo el emisor es el autor del texto (no sabemos
su nombre, aunque sí sabemos que el coordinador de la
b) cabeza rapada
obra es Luis Ribot). No hay narrador, ya que no se trata de
c) telefonía móvil un texto literario.
d) físico-químico c) La intención comunicativa del primero es crear belleza,
e) peruano-ecuatoriano deleitar con las palabras. En cambio, la del segundo es
f) histórico-artístico simplemente informativa y académica, se trata de dar a
conocer unos datos.
g) carril bici
d) El código en el primer caso es algo más elaborado, intenta
5. Escribe mayúscula donde sea necesario: producir extrañeza en el lector mediante el desvío de lo
habitual, se centra en la función estética. En el segundo,
Corrige
en cambio, pese a ser un código culto, no se pretende lla-
stendhal (1783-1842) es uno de los iniciadores del realis- mar la atención sobre cómo está escrito el mensaje, sino
mo francés, aunque su obra convive cronológicamente simplemente sobre lo que dice.
con algunas obras románticas, como es el caso del drama
2. El canal mediante el cual se transmite una obra literaria
hernani, de victor hugo, que se estrenó en 1830. Ese mismo
influye en gran medida en ella. Investiga cuáles son las
año se publicó una de las grandes obras de stendhal, rojo
y negro, crónica social de la francia de la restauración principales diferencias entre la literatura oral y la escrita,
borbónica, en la que el análisis psicológico de los perso- y prepara una presentación para exponer el tema a tus
najes es uno de los rasgos más interesantes. otro título compañeros.
importante de stendhal es la cartuja de parma (1839). El alumno puede hacer mención, por ejemplo, a que la litera-
tura oral tiene un carácter mucho más popular y tradicional.
Stendhal (1783-1842) es uno de los iniciadores del realismo Al irse transmitiendo de padres a hijos, está sujeta a un ma-
francés, aunque su obra convive cronológicamente con algu- yor número de cambios y variantes, y además posee diversas
nas obras románticas, como es el caso del drama Hernani, de fórmulas que facilitan su memorización. El género oral por
Victor Hugo, que se estrenó en 1830. Ese mismo año se publicó excelencia es el lírico, pero también el narrativo (épica, ro-
una de las grandes obras de Stendhal, Rojo y negro, crónica mances…). A menudo se trata de una literatura anónima, ya
social de la Francia de la Restauración borbónica, en la que que pertenece al pueblo.
el análisis psicológico de los personajes es uno de los rasgos
La literatura escrita tiene un carácter más culto, no está su-
más interesantes. Otro título importante de Stendhal es La
jeta a tantas variantes y suele estar firmada por un autor.
cartuja de Parma (1839).
El género más abundante en esta literatura es la narrativa
(novelas, cuentos…).
LA FACTORÍA DE TEXTOS
3. Haz una pequeña encuesta entre tus familiares y amigos
1. Lee con atención los siguientes textos y responde a las sobre el último libro que cada persona haya leído. Formu-
preguntas que tienes a continuación: la las siguientes preguntas:
Comieron. El estómago sentía un pequeño bienestar, las bocas
bostezaban, los ojos se angostaban por una dulce somnolen-
a) ¿Cuál es el último libro que has leído?
cia. El marido se puso a fumar un puro, se desperezó y se b) ¿Por qué lo has elegido?
tumbó en el sofá. La esposa se sentó en la cabecera y empezó
c) ¿Qué te ha aportado?
a tararear… Eran felices.
Anton CHEJOV: Siete cuentos (Cátedra). Investiga sobre los títulos que más te llamen la atención y
Hasta los años centrales del siglo XV siguieron prolongándose
averigua si son textos literarios o no.
los efectos de la terrible crisis de mediados del XIV, perfec- Respuesta libre.
tamente identificada con la Peste Negra. Fueron cien años 4. Lee el siguiente poema y responde las cuestiones que
de repetidas crisis epidémicas, de desaparición de núcleos
tienes a continuación:
rurales, de general ralentización de la actividad económica.
En fin a vuestras manos he venido,
Luis RIBOT (Coord.): Historia del mundo moderno.
do sé que he de morir tan apretado
a) ¿Cuál de los dos textos es literario? Justifica tu res- que aun aliviar con quejas mi cuidado
puesta. como remedio m’es ya defendido;

54 Lengua castellana y Literatura. 3.º ESO. Solucionario


EN LA VARIEDAD ESTÁ EL GUSTO 6
mi vida no sé en qué s’ha sostenido de la fortaleza de Le Muy, en Provenza, Garcilaso fue tras-
si no es en haber sido yo guardado ladado a Niza, donde murió.
para que solo en mí fuese probado Su escasa obra conservada, escrita entre 1526 y 1535, fue
cuánto corta una ’spada en un rendido. publicada póstumamente junto con la de Boscán, bajo el
Mis lágrimas han sido derramadas título de Las obras de Boscán con algunas de Garcilaso de
donde la sequedad y el aspereza la Vega (1543), libro que inauguró el Renacimiento literario
dieron mal fruto dellas, y mi suerte: en las letras hispánicas.
¡basten las que por vos tengo lloradas; Siempre se ha creído que muchas de sus composiciones
no os venguéis más de mí con mi flaqueza; reflejan la pasión de Garcilaso por la dama portuguesa
allá os vengad, señora, con mi muerte! Isabel Freyre, a quien el poeta conoció en la corte en 1526.
Garcilaso DE LA VEGA Sin embargo, recientes estudios ponen en duda esta teoría
a) ¿Quién es el emisor del texto? ¿Y los receptores? y defienden que la amada secreta de Garcilaso no era Isa-
bel, sino Beatriz de Sá, la esposa de su hermano.
b) ¿A quién se dirige el poeta cuando emplea la forma
Los cuarenta sonetos y las tres églogas que escribió se
vos? ¿Piensas que se trata de una persona en concre- mueven dentro del dilema entre la pasión y la razón que
to? ¿Quién? caracteriza la poesía petrarquista. Escribió también cinco
c) ¿Cuál es la intención comunicativa de Garcilaso al es- canciones, dos elegías, una elegía a Boscán y tres odas
cribir este poema? ¿Y la del yo poético que se expresa latinas, inspiradas en la poesía horaciana y virgiliana.
en él? ¿Coinciden ambas? f) El Renacimiento fue un movimiento cultural que se produ-
d) Fíjate bien en el lenguaje con el que está escrito el jo en Europa durante los siglos XV y XVI. Fue un período de
transición entre la Edad Media y el mundo moderno. Sus
poema, es decir, su código. ¿Qué te llama la atención
principales exponentes se hallan en el campo de las artes,
sobre él? Señala algún recurso poético empleado por aunque también se produjo una renovación en las ciencias.
Garcilaso y explica su finalidad. La ciudad de Florencia, en Italia, fue el lugar de nacimiento
e) Investiga sobre la biografía de Garcilaso de la Vega y y desarrollo de este movimiento.
sobre cómo pudo influir en su obra poética. El Renacimiento fue fruto de la difusión de las ideas del
f) Busca información sobre la época en la que vivió Gar- humanismo, que determinaron una nueva concepción
cilaso, el Renacimiento, y explica cuáles son las carac- del hombre y del mundo. El nombre «Renacimiento» se uti-
lizó porque este movimiento retomaba ciertos elementos
terísticas de este movimiento y cómo nos ayudan a
de la cultura clásica, griega y romana. Esta nueva etapa
entender el poema. planteó una nueva forma de ver el mundo y al ser humano,
g) ¿De qué tipo de composición poética se trata? Explica con nuevos enfoques en los campos de las artes, la política,
cómo está formada. la filosofía y las ciencias, sustituyendo el teocentrismo
medieval por cierto antropocentrismo.
a) El emisor del texto es Garcilaso de la Vega, y los recep-
tores son, en primer lugar, la mujer a la que se dirige, su Esta información nos ayuda a comprender mejor el poema
amada; y después el resto del lectores del poema. ya que observamos como ahora la temática es el hombre
y sus sentimientos (frente a la Edad Media donde todo
b) Con ese vos se está dirigiendo a la mujer que ama y que
giraba en torno a Dios y a la religión).
tanto le está haciendo sufrir.
g) Se trata de un soneto. Está formado por catorce versos
c) La intención de Garcilaso es sobre todo estética: quiere
endecasílabos, con rima consonante. Consta de dos cuar-
crear belleza con sus palabras. La intención del yo poéti-
tetos (rima ABBA) y dos tercetos (CDE, CDE).
co es expresar sus sentimientos amorosos, concretamente
quejarse a su amada del sufrimiento que le ha ocasionado
y que le está llevando a la muerte. Como siempre se ha con- ACTIVIDADES FINALES
siderado que la poesía de Garcilaso tiene una gran carga
Repasa lo que has aprendido
autobiográfica, esta intención coincidiría tanto en su autor
como en el yo poético. 1. Lee atentamente el siguiente diálogo y responde a las
d) Se trata de un código muy cuidado y elaborado, en el que
cuestiones que se te plantean a continuación:
predomina la función estética del lenguaje. Llama la aten- —Creo que tenemos pendiente una conversación, ¿no lo crees?
ción el uso del lenguaje figurado, con recursos como las —Sí, sí lo creo. Y creo que fue desde que te casaste con mi
hipérboles (sé que he de morir, con mi muerte) o metá- novio.
foras (cuánto corta una ’spada en un rendido). Lo que —Está bien, si lo quieres, empecemos por ahí. Tú tuviste un
pretende el poeta es llamar la atención sobre el lenguaje y novio indebidamente. No te correspondía tenerlo.
sorprender al lector.
—¿Según quién? ¿Según mamá o según tú?
e) Garcilaso de la Vega, (Toledo, 1501?-Niza, 1536) fue un
—Según la tradición de la familia, que tú rompiste.
poeta renacentista español. Perteneciente a una noble
familia castellana, participó ya desde muy joven en las —Y que voy a romper cuantas veces sea necesario, mien-
intrigas políticas de Castilla. En 1510 ingresó en la corte tras esa maldita tradición no me tome en cuenta. Yo tenía el
del emperador Carlos I y tomó parte en numerosas batallas mismo derecho a casarme que tú, y tú eras la que no tenía
militares y políticas. En 1530 Garcilaso se desplazó con derecho a meterse en medio de dos personas que se querían
Carlos I a Bolonia, donde este fue coronado. Permaneció profundamente.
allí un año, hasta que, debido a una cuestión personal man- —Pues ni tan profundamente. Ya ves cómo Pedro te cambió
tenida en secreto, fue desterrado a la isla de Schut, en el por mí a la menor oportunidad. Yo me casé con él, porque él
Danubio, y después a Nápoles, donde residió a partir de así lo quiso. Y si tuvieras tantito orgullo lo deberías de haber
entonces. Herido de muerte en combate, durante el asalto olvidado ya para siempre.

Lengua castellana y Literatura. 3.º ESO. Solucionario 55


6 EN LA VARIEDAD ESTÁ EL GUSTO

—Pues para tu información, se casó contigo solo por estar b) El verano: época más calurosa del año, que en el hemis-
cerca de mí. No te quería y tú lo sabías muy bien. ferio septentrional comprende los meses de junio, julio y
—Mira, mejor ya no hablemos del pasado, a mí no me impor- agosto. En el hemisferio austral corresponde a los meses
tan los motivos por los que Pedro se casó conmigo. Se casó y de diciembre, enero y febrero.
punto. Y yo no voy a permitir que ustedes dos se burlen de mí,
a) Se trata del significado connotativo, ya que no se trata de
¡óyelo bien! No estoy dispuesta a hacerlo.
algo objetivo y común a todos los hablantes, sino de un
Laura ESQUIVEL: Como agua para chocolate. significado muy personal para una persona en concreto,
a) ¿Consideras que el registro empleado por ambos per- que no tiene por qué ser compartido por otras.
sonajes es formal o informal? Busca ejemplos en el tex- b) Se trata de un significado denotativo, es el que recoge el
to para argumentar tu respuesta. diccionario y es común para todos los hablantes.
b) ¿Observas en el diálogo algún rasgo que te dé una 4. ¿Cuántos refranes conoces que hagan alusión a los me-
pista sobre la procedencia de los personajes? ¿Cuál? ses del año? Escribe en tu cuaderno todos los que sepas.
c) Continúa tú la conversación entre ambas hermanas. De- A continuación, pregunta a algún familiar si conoce alguno
bes mantener el mismo tono y registro. Escribe unas más, y cópialo también. Por último, busca otros en la red
setenta palabras. y escríbelos. ¿Cuántos has logrado reunir en total? ¿Has
conseguido al menos uno para cada mes del año?
a) Se trata de un registro informal o familiar, ya que emplea
expresiones bastante coloquiales (esa maldita tradición, Respuesta libre.
pues ni tan profundamente, tantito orgullo, mira, se casó 5. Escribe en tu cuaderno cinco palabras compuestas donde
y punto) e hipocorísticos como mamá.
sea necesario el uso de guion.
b) Expresiones como tantito orgullo o ustedes dos, nos llevan
Ejemplos: astur-leonés, navarro-aragonés, teórico-práctico,
a pensar que se trata de hablantes hispanoamericanos.
franco-alemán, histórico-crítico, lingüístico-literario…
c) Respuesta libre.
6. Explica el uso que se hace de la raya en el diálogo de la acti-
2. Di si las siguientes afirmaciones son verdaderas o falsas y, vidad 1.
en este último caso, escribe qué sería lo correcto:
La raya aquí se emplea para introducir las intervenciones de
a) El astur-leonés y el navarro-aragonés son dos dialec- los dos personajes que están dialogando.
tos procedentes del castellano medieval.
7. Recuerda cuáles son los principales usos de las mayúsculas,
b) Los dialectos meridionales de la Península son el an- escríbelos en tu cuaderno y añade un ejemplo para cada
daluz, el murciano, el extremeño y el canario. uno de ellos.
c) Algunos de los rasgos más característicos del dialecto Según la puntuación: al comienzo de un escrito, detrás de
andaluz son el seseo, el ceceo, el yeísmo, la pérdida de punto, tras puntos suspensivos, exclamaciones e interrogacio-
-d- intervocálica, la aspiración de -s final y la confusión nes que cierren un enunciado, tras dos puntos en el encabeza-
entre r y l. miento de una carta o comienzo de una cita textual.
d) Las variedades sociales se denominan también varie- Sin tener en cuenta la puntuación: los nombres propios de
dades diafásicas. persona, lugar, animal o divinidad; los apellidos y sobrenom-
bres; los nombres de calles y vías públicas; los nombres de
e) Las variedades de situación son las que emplea un mis- entidades o instituciones; la letra inicial de cualquier obra
mo hablante dependiendo del contexto comunicativo de creación artística; con los periodos históricos.
en el que se encuentre.
8. Explica en qué consiste un texto literario y cuáles son sus
f) Las frases hechas y los refranes son propios de un elementos desde el punto de vista de la comunicación.
registro informal de la lengua. Ejemplifícalo con una obra literaria que conozcas.
g) Una jerga es la forma de hablar característica de un Un texto literario es el que pretende crear belleza a través
grupo marginal. de las palabras, es decir, tiene una finalidad estética. Sus
a) Falso. En realidad proceden del latín medieval. elementos desde el punto de vista comunicativo son el emi-
b) Verdadero. sor (el autor), el receptor (los lectores), el mensaje (la propia
obra), el canal (oral o escrito), el código (lenguaje con el
c) Verdadero. que está escrito) y la situación (época, contexto en el que
d) Falso. Se denominan variedades diastráticas. se escribió).
e) Verdadero. 9. Lee el siguiente texto y a continuación responde a las
f) Verdadero. cuestiones que se te plantean:
g) Falso, también puede ser la de un grupo profesional, de Los goterones que caían de las altas hojas hacían un ruido
jóvenes… como de pisadas. Todo el bosque parecía estar lleno de gente
en marcha. Pero el Hombre aún tardó en llegar. Era menos alto
3. Explica si los siguientes enunciados corresponden al sig- que una aguijada y su traje roto y manchado de barro mos-
nificado denotativo o connotativo de la palabra definida y traba por más de un sitio la tierna carne infantil; agujereada
razona tu respuesta: en el centro, la boina dejaba asomar los cabellos; al tropezar
a) El verano: ese tiempo de relax, de levantarse tarde, de con los guijarros, las zuecas sonoras rimaban su caminata, y
no saber muy bien en qué emplear el día, de intermi- silbaba tan hórridamente un estribillo inventado por él, que
nables siestas y de noches al fresco con un granizado hasta los mismos cuervos le desaprobaban.
de limón. Wenceslao F ERNÁNDEZ F LÓREZ: El bosque animado.

56 Lengua castellana y Literatura. 3.º ESO. Solucionario


EN LA VARIEDAD ESTÁ EL GUSTO 6
a) ¿Crees que se trata de un texto literario? Justifica tu jardín del sol, piedra… El agujero que tenía en medio,
respuesta con ejemplos. se calentaba mucho al sol, estaba muy lisa (refiriéndose
a la piedra).
b) Observa el lenguaje con el que está escrito. ¿Te llama
la atención? ¿Por qué? c) Encontramos conectores aditivos o copulativos (y, tam-
bién, lo mismo que), causales (de tanto como, porque),
c) Busca en el texto ejemplos de personificación. ¿Qué temporales (hasta que, cuando), consecutivos (por eso)
efecto consigue con este recurso el autor? y locativos (alrededor de).
a) Sí, se trata de un texto literario, ya que hace un uso espe- 11. Escribe en tu cuaderno una página de un diario personal
cial del lenguaje (pretende provocar extrañeza y llamar (no tiene por qué ser real, puedes contar hechos ficticios),
la atención sobre el mismo). En él predomina la función
intentando que sea lo más literaria posible, es decir, que lo
estética (ruido como de pisadas, las zuecas sonoras
rimaban su caminata, hasta los mismos cuervos le
que escribas pueda producir un placer estético en quien
desaprobaban…). lo lea.
b) Sí, ya que difiere del lenguaje ordinario. Se emplean com- Respuesta libre.
paraciones (ruido como de pisadas, menos alto que una
aguijada) y personificaciones, que contribuyen a ese ex- MIRA A TU ALREDEDOR Y…
trañamiento del lector ante el lenguaje. … ve más allá
c) Las personificaciones (su traje mostraba…, la boina de-
1. Para entender la evolución del castellano y de los dialectos
jaba asomar…, las zuecas sonoras rimaban su camina-
peninsulares es fundamental conocer los acontecimientos
ta, hasta los mismos cuervos le desaprobaban) también
consiguen dicho efecto de extrañamiento, de salirse de históricos que tuvieron lugar en el pasado, especialmente
lo habitual. durante la Reconquista. En los siguientes mapas puedes
observar cómo cambió políticamente la Península durante
Recuerda lo que ya sabías la Reconquista. Fíjate en el retroceso de al-Ándalus y en
10. Lee el siguiente texto y resuelve las cuestiones plantea- el avance de los reinos cristianos hacia el sur. ¿Qué conse-
das a continuación: cuencias crees que tuvo para las lenguas romances de la
El jardín del sol tenía un almendro, junto a la valla, que echaba
Península la invasión árabe?
sus ramas hacia la calle. Los veranos se ponía una cigarra en Respuesta libre.
la corteza de este almendro y cantaba durante toda la siesta. 2. Busca en Internet información sobre los siguientes acon-
El aire se aplastaba sobre aquel canto y nadie podía ya mover- tecimientos y prepara una exposición oral para explicarla
se hasta que la cigarra no se callaba, de tanto como pesaban
en clase:
todas las cosas. Era la cigarra de los bochornos plomizos,
cuando se envenenan las sandías. a) La invasión musulmana en el año 711 d. C.
También había en el jardín del sol una rueda de molino, incli- b) Los primeros núcleos de resistencia en el norte de la
nada y hundida en la tierra. Alrededor de esta piedra nacía Península.
la hierba más fuerte, lo mismo que por el agujero que tenía
c) Cómo se organizó la Reconquista hacia el sur.
en medio. Se calentaba mucho al sol y por eso gustaba a las
lagartijas. En aquella piedra afilaba el maestro los cuchillos a) En el año 711, las tropas árabes y bereberes del noroeste
y las herramientas, porque estaba muy lisa. de África cruzaron el estrecho de Gibraltar, derrotaron
al ejército visigodo en la batalla de Guadalete y, en poco
Rafael SÁNCHEZ F ERLOSIO: Industrias
tiempo, se hicieron con el control de casi toda la penín-
y andanzas de Alfanhuí.
sula ibérica. El rápido avance de las tropas musulmanas
a) Observa las estructuras sintácticas que forman el tex- fue posible por la debilidad del Reino visigodo y la escasa
to. Busca en él ejemplos de oraciones coordinadas resistencia de su ejército, aunque también contó con la
copulativas, oraciones subordinadas adjetivas, y ora- ayuda de las poblaciones descontentas con dos siglos de
ciones subordinadas adverbiales de tiempo y causa. autoridad visigoda.
b) Fíjate ahora en la cohesión del texto. Señala todos b) En el norte de la Península siguen subsistiendo pueblos
los elementos de deixis que encuentres. que opondrán resistencia al nuevo invasor. Estos pequeños
núcleos resultarán cada vez más fortalecidos, ganando te-
c) Observa los conectores. Escríbelos en tu cuaderno rreno a costa de la España musulmana.
indicando de qué tipo son, y qué clase de relaciones
c) La situación cambió a partir del siglo XI, cuando los reinos
establecen entre las partes del texto. cristianos comenzaron a ganar terreno, en un largo proce-
a) Coordinadas copulativas: los veranos se ponía una ciga- so conocido como Reconquista. Durante los cinco largos
rra en la corteza de este almendro y cantaba durante toda siglos que duró este periodo, se alteraron épocas de lucha
la siesta; el aire se aplastaba sobre aquel canto y nadie y paz, de avance y retroceso.
podía ya moverse; se calentaba mucho al sol y por eso
gustaba a las lagartijas. 3. Lee el siguiente texto y responde a las preguntas que se
te plantean a continuación:
Subordinadas adjetivas: que echaba sus ramas hacia la
calle; inclinada y hundida en la tierra. La invasión musulmana del año 711 trastornó violentamente
el desarrollo lingüístico de los romances hispánicos. La Penín-
Subordinadas adverbiales de tiempo: hasta que la cigarra
sula quedó inundada por la última gran oleada expansiva del
no se callaba; cuando se envenenan las sandías.
islam. El latín se vio agobiado bajo el peso de otra lengua de
Subordinadas adverbiales de causa: porque estaba muy cultura, el árabe, órgano de una civilización completamente
lisa. extraña, que durante muchos siglos se mostró superior en múl-
b) Que echaba (se refiere al almendro); aquel canto (el de tiples aspectos a la civilización de Occidente. La lengua popu-
la cigarra); repetición de las palabras almendro, cigarra, lar románica, en la época de su más temprana evolución, se

Lengua castellana y Literatura. 3.º ESO. Solucionario 57


6 EN LA VARIEDAD ESTÁ EL GUSTO

vio, en gran parte de la Península, obligada a convivir vulgar Una de las interpretaciones que se le ha dado a este cuadrado
y diariamente con la extraña lengua oficial árabe. En esta con- mágico defiende que era en realidad una especie de anuncio
vivencia, el árabe, como lengua dominadora, llegó a debilitar o colocado en las casas de cristianos que ofrecían refugio a
a extinguir los dialectos románicos, y cuando progresivamente otros cristianos, quienes serían los únicos que podrían orde-
el árabe fue vencido, y los dialectos mozárabes fueron susti- nar las letras para obtener el significado real del cuadrado.
tuyéndose por los dialectos de los reconquistadores, esta sus- La gran cantidad de lugares donde se ha encontrado el cua-
titución suscita problemas muy varios, uno de los cuales, el drado es coherente con la interpretación cristiana; pero el
principal y más debatido, es el que se refiere al solar de las len- hallazgo de la inscripción grabada en una columna del gran
guas catalana, aragonesa, castellana, leonesa y portuguesa. gimnasio de Pompeya plantea dudas sobre este origen, pues,
Ramón MENÉNDEZ P IDAL , en Enciclopedia aunque existían comunidades cristianas en las ciudades ve-
Lingüística Hispánica. subianas antes del año 79, el Apocalipsis de san Juan, al que
harían referencia la «alfa» y la «omega» (la a y la o griegas)
a) Según Menéndez Pidal, ¿qué fue lo que motivó la pro- colocadas a los lados de la cruz, no se difunde en Italia hasta
funda transformación de las lenguas romances hispá- casi cien años después.
nicas? www.almacendeclasicas.blogspot.com.es
b) ¿Cómo influyó la lengua árabe en los incipientes dia- a) ¿Por qué se califica a este palíndromo como «mágico»
lectos románicos de la Península? y «misterioso»?
c) Una vez vencidos los árabes, ¿qué sucedió con los dia- b) ¿Por qué crees que los cristianos necesitarían colocar
lectos empleados por los reconquistadores? en sus casas un símbolo que únicamente entendieran
d) ¿Estás de acuerdo con Menéndez Pidal cuando afirma otros cristianos?
que «la fragmentación lingüística actual de la península c) ¿Qué es lo que puede poner en duda la interpretación
ibérica es, en lo fundamental y decisivo, resultado de cristiana del símbolo?
la Reconquista»? Argumenta tu respuesta. d) Busca información en Internet acerca de otros lugares
a) La invasión musulmana, que trajo consigo la entrada de donde se haya encontrado esta inscripción.
otra lengua de cultura, el árabe. e) Investiga otras posibles interpretaciones que se hayan
b) Llegó a debilitarlos o a extinguirlos. barajado sobre el significado de este misterioso palín-
dromo, copia en tu cuaderno la información más rele-
c) Que se empezaron a suscitar problemas, como el de la len-
gua catalana, aragonesa, castellana, leonesa y portuguesa.
vante y explícasela oralmente a tus compañeros. Pue-
des elaborar una presentación en Prezi o en PowerPoint
d) Respuesta libre. que te servirá de apoyo para la exposición oral.
4. El hecho de que el español de América comparta muchos a) Se le califica de mágico y misterioso porque no se ha podi-
de los rasgos propios de los dialectos meridionales de do llegar a una interpretación clara de lo que significa. Es
nuestro país obedece también a una razón histórica, rela- muy curioso que haya aparecido en tantos lugares diferen-
cionada con el origen de los colonizadores de las nuevas tes y que con sus letras se puedan formar esos mensajes,
tierras descubiertas. Investiga la causa de este fenómeno por lo que está rodeado de un halo de misterio.
y copia en tu cuaderno la información más relevante que b) Porque en esa época los cristianos eran perseguidos y ne-
halles sobre el tema. cesitaban mantener su religión en secreto.
Esto es debido a que una gran parte de los colonizadores de c) El haberse hallado esta inscripción en una columna de
las tierras recién descubiertas eran de origen andaluz y extre- Pompeya, datando de una fecha muy anterior a la difusión
meño. Por ello, llevaron consigo su particular variedad dialec- del apocalipsis de san Juan en Italia (las letras alfa y la
tal, y fue esta la que se extendió por las tierras colonizadas. omega de la cruz hacen referencia a dicho apocalipsis).
d) Se han encontrado en ruinas romanas de Cirencester (la
… encuentra la clave antigua Corinium), en Inglaterra. En Italia en la región
Abruzzo en la iglesia de San Pietro ad Oratorium, entre
1. En la unidad anterior estudiamos lo que era un palíndromo.
Bussi y Capestrano; en el castillo de Rochemaure, en Oppè-
Lee la siguiente información y realiza las actividades que de; en Siena (pared de la catedral); en la abadía de Colle-
tienes a continuación: pardo; en Santiago de Compostela, etc.
El cuadrado mágico SATOR AREPO e) Existen otras interpretaciones relacionadas con el esote-
Es un misterioso palíndromo cristiano escrito en latín. Se en- rismo y, en particular, con la alquimia. Cabe mencionar en
contró por primera vez inscrito en una columna de las ruinas este sentido la relación con los constructores de catedrales
de Pompeya y posteriormente ha sido hallado en numerosas mencionada por Louis Charpentier y, en el mismo sentido,
tumbas y templos cristianos. su vínculo con la teoría quiliásmica o milenarismo, hipó-
tesis que analiza el cuadrado sustituyendo las letras por
Está formado por cinco palabras de cinco letras que, en hori- números para conformar una matriz de cuya resolución
zontal, de arriba abajo y de abajo arriba, forman un palíndro- podría derivar en una suerte de calendario antiguo. Para
mo. Su significado no se conoce con exactitud. Una traducción Pedro Guirao, el cuadrado esconde el secreto hermético de
libre sería: «El sembrador dirige las ruedas del trabajo». la cuadratura del círculo. De igual forma, existen diversos
Con las veinticinco letras se forma el anagrama «ORO TE, PATER, desarrollos cabalísticos y matemáticos presentados por el
ORO TE, PATER, SANAS»(Te ruego Señor, te ruego Señor, cúrame). profesor Rafael de Cózar de la Universidad de Sevilla que,
partiendo de la configuración formal de un texto poético
Con veintiuna de las veinticinco letras en forma de cruz se
y en razón de sus exigencias métricas, desarrolla geome-
obtiene dos veces «Padre Nuestro».
trías herméticas con las que concluye que las razones de
Utilizando las cuatro letras restantes (A, A, O, O) se llega a «alfa su conformación parecen más estéticas que contener un
y omega, el principio y el fin». fundamento filosófico.

58 Lengua castellana y Literatura. 3.º ESO. Solucionario


AMAR, LUCHAR, REZAR… 7
APERTURA DE UNIDAD 4. Según Patronio, ¿hay alguna razón por la que merezca la
pena arriesgar la propia vida? ¿Cuál es?
1. ¿Qué dos personajes dialogan en este poema? ¿Coincide
lo que cuenta la muchacha acerca de dónde ha estado Sí, para defender su honra o por alguna cosa en la que esté
con lo que cree la madre? obligado.
Dialogan una madre y una hija. La hija miente a la madre di- 5. El propio autor de El conde Lucanor aparece en este y
ciendo que viene de la fuente y ha tardado porque los ciervos en todos los ejemplos del libro. ¿Dónde? ¿Qué es lo que
revolvían el agua, pero la madre sabe que en realidad viene hace?
de estar con su amado.
Aparece al final. Viendo don Juan que este cuento era bueno,
2. Fíjate en la forma en la que está escrito. ¿Te llama la lo hizo poner este libro y escribió unos versos que dicen así.
atención? ¿Por qué? Señala todas las repeticiones que
encuentres. Vocabulario
Al alumno le puede llamar la atención que la versión origi- 6. Busca en el diccionario el significado de los verbos preciar
nal está escrita en lengua galaico-portuguesa, así como que y preciarse. Escribe una frase con cada uno donde se ob-
está estructurado en forma de diálogo, y por supuesto, las serve claramente su significado.
abundantes repeticiones (dime hija, hija mía; ¿por qué tar-
daste…?; tardé, madre mía; porque los ciervos del monte…; • preciar: apreciar. Los que han estado en paro precian mu-
mientes, hija mía; nunca vi un ciervo que…) así como el cho su trabajo.
estribillo (amores tengo). • preciarse: gloriarse, jactarse y hacer vanidad de algo. Se
3. Estudiaremos que en la lírica popular o tradicional es muy precia constantemente de lo bien que hace las cosas.
frecuente el empleo de símbolos, que veladamente aluden 7. Escribe dos sinónimos de cada una de las siguientes pa-
a otros significados. Busca información sobre qué significa- labras: beneficio, riqueza, codicia, frivolidad, estimación.
ban los ciervos y las fuentes en dicha literatura. • beneficio: provecho, ganancia.
Las fuentes en la lírica popular simbolizaban el lugar de en-
• riqueza: fortuna, opulencia.
cuentro de los amantes, y los ciervos que bebían en el agua
de la fuente o del río eran un símbolo del encuentro sexual • codicia: avaricia, egoísmo.
entre los amantes. • frivolidad: trivialidad, banalidad.
4. La lírica galaico-portuguesa nos ha dejado poemas bellísi- • estimación: aprecio, estima.
mos. Escucha una cantiga de amor portuguesa tecleando
en YouTube «O que vos nunca cuidei a dizer». Pertenece 8. En el texto, Patronio menciona la honra. Busca el signifi-
al rey de Portugal don Dionís, que también fue un impor- cado de este término y cópialo en tu cuaderno. A conti-
tante trovador. ¿Qué sensaciones te evoca? Coméntalas nuación, investiga qué importancia tenía la honra para los
en voz alta. hombres y mujeres medievales.
Respuesta libre. La honra es la estima y respeto de la dignidad propia; buena
opinión y fama, adquirida por la virtud y el mérito.
VIVE LA LECTURA En la Edad Media la honra tenía muchísima importancia, por-
que perderla equivalía a estar muerto para la sociedad. Las
Comprensión lectora mujeres podían quedar deshonradas si mantenían relaciones
1. ¿Cuál es el problema que dice tener el conde? ¿Qué le sexuales fuera del matrimonio, y en este caso también eran
aconseja Patronio ante tal situación? deshonrados los varones de su familia.
El conde tiene miedo de que, si se queda en cierto sitio donde 9. ¿Crees que ha cambiado el concepto de honra en la ac-
puede ganar mucho dinero, ponga en riesgo su vida. Patronio tualidad? Razona tu respuesta.
le aconseja que nunca arriesgue su vida por enriquecerse; se
lo ejemplifica mediante un cuento. Respuesta libre.

2. Explica en qué partes se divide el texto, qué dos historias 10. Fíjate en la moraleja del cuento. ¿Conoces algún refrán po-
hay y cómo una historia se enmarca dentro de la otra. pular que signifique lo mismo? Escríbelo en tu cuaderno.
El texto se divide en las siguientes partes: La avaricia rompe el saco.
• El conde expone su problema a su consejero. 11. Busca en el diccionario el origen de la palabra majadero.
• Este le responde mediante un cuento o ejemplo. ¿Crees que el verbo del que procede puede tener algo
que ver con su significado? Explica por qué.
• A continuación le aconseja directamente lo que debe hacer.
La palabra majadero proviene del verbo majar (que significa
• Se explica que el conde sigue el consejo y le va bien, y cómo
machacar, y coloquialmente, molestar, importunar).
don Juan Manuel mandó escribirlo en su libro.
• Moraleja final en verso. Investigación y redacción
Hay dos historias, la del conde que le pide consejo a Patronio, 12. Busca información sobre otras tres importantes obras
y el cuento que le narra este sobre el hombre que se ahogó literarias escritas por don Juan Manuel. ¿De qué trata cada
en el río por no querer desprenderse de un saco de piedras una de ellas? ¿Tienen algo en común?
preciosas. Esta última está dentro de la primera, que le sirve
El alumno puede hablar, por ejemplo, de las siguientes obras:
de marco.
• El Libro del caballero y el escudero; narra la historia de
3. ¿Te parece que Patronio le da un buen consejo a Lucanor? un escudero mancebo aspirante a cortesano que ha de ir
¿Y la forma en que lo hace? Argumenta tus respuestas. a unas cortes convocadas por el rey y que recibe las más
Respuesta libre. diversas enseñanzas por parte de un ermitaño que ha sido

Lengua castellana y Literatura. 3.º ESO. Solucionario 59


7 AMAR, LUCHAR, REZAR…

caballero. Asiste a unas justas y vuelve a la ermita para • lenguas vernáculas: son las lenguas propias del país o lu-
recibir nuevas enseñanzas. El anciano excaballero muere gar de nacimiento de uno.
y su joven discípulo le sepulta. • escolástica: perteneciente o relativo a las escuelas medie-
• El Libro de los estados ofrece una visión de cómo debe vales o a quienes estudiaban en ellas.
ser una sociedad ideal en el siglo XIV, aunque es asimismo
3. Vuelve a leer la cantiga «Pela rebeira do rio». ¿Cuál es su
una narración de fin didáctico (educación de un príncipe)
inspirada en la leyenda de Barlaam y Josafat, forma cris-
temática? Señala en él todas las características de la lírica
tianizada en que fue transmitida a Occidente la leyenda de galaico-portuguesa que encuentres. Analiza también su
Buda. estructura.
• El Libro de la caza, tratado de cetrería que encierra dos Esta cantiga nos presenta a una muchacha que va cantando
aspectos novedosos: la inclusión de anécdotas personales y gozosa a la orilla del río, por el que se aproximan las barcas.
un capítulo con una descripción geográfica de los mejores En la lírica galaico-portuguesa, todo lo que tiene relación con
lugares para practicar la cetrería en el reino de Castilla. el mar, el agua, las barcas… guarda una estrecha conexión
Todos tienen en común que son libros didácticos, están pensa- con los sentimientos amorosos.
dos para enseñar algo a los príncipes de su época. Como es habitual en este tipo de lírica, la protagonista es una
13. Lee la siguiente moraleja de otro de los cuentos perte- doncella enamorada, feliz posiblemente porque va a encon-
necientes a El conde Lucanor. A partir de ella piensa una trarse con su amado, en armonía con la naturaleza.
situación que hubiera podido motivar dicho consejo y Formalmente destacan en el poema las repeticiones y el pa-
escribe el cuento en tu cuaderno, respetando la estruc- ralelismo (pela ribeira, cantando ia la dona), así como ese
tura: introducción donde alguien cuenta un problema que estribillo que se repite detrás de cada estrofa (venhan nas
barcas polo rio a sabor).
tiene y para el que pide consejo, respuesta en forma de
cuento, conclusión y esta moraleja final. Deberá constar, 4. Lee ahora la jarcha y el villancico. ¿Cuál es el tema de
al menos, de cien palabras. cada uno? Señala en cada uno de ellos todos los aspectos
Al que antes tu enemigo solía ser que te llamen la atención por lo que se refiere a su forma
ni en nada ni nunca le debes creer. y estructura.
Respuesta libre. En la jarcha nos encontramos una protagonista femenina, que
se lamenta a su amado (habib) por su sufrimiento amoroso.
14. Compara el cuento que has escrito con el de un compa- Está escrito en lengua mozárabe y es un poema muy breve,
ñero y comentad oralmente las diferencias y semejanzas. donde se observan repeticiones (tant’ amare).
Respuesta libre. En cuanto al villancico, es una composición tradicional en
castellano. En este caso su protagonista también es una mu-
LITERATURA chacha, que se dirige a su madre. El significado del poema
1. Busca datos sobre de la Reconquista de la península ibéri- es bastante enigmático, ya que tiene un contenido simbólico.
ca: quiénes la iniciaron, qué figuras históricas fueron más El vergel donde la muchacha va a coger rosas posiblemente
simbolice el encuentro sexual. Aquí las palabras morir y ma-
relevantes en ella, qué periodo de tiempo abarcó… Expón
tarme aluden al goce sexual.
tus averiguaciones oralmente a la clase.
Formalmente también es un poema breve basado en repeti-
La Reconquista abarcó un periodo de tiempo de siete siglos
ciones y paralelismos.
(del VIII al XV d. C.), en el cual los reinos cristianos del norte de
la Península que habían resistido la invasión árabe del año 711 5. Lee el siguiente fragmento (en versión modernizada) del
trataron de recuperar el territorio perdido y liberar España de Cantar de Mio Cid y responde a las preguntas:
la dominación islámica. Alargó entonces las manos el de la barba florida
Uno de los personajes más cruciales del inicio de la misma y a las niñas, sus dos hijas, en los brazos las cogía;
fue un noble asturiano, don Pelayo, que lideró la batalla de
Covadonga en el 722, considerada el inicio de la Reconquista. al corazón acercolas porque mucho las quería.

2. Busca el significado de los siguientes términos y explica Con lágrimas en los ojos muy fuertemente suspira:
qué importancia pudieron tener en el desarrollo de la —Oídme, doña Jimena, tan entera mujer mía;
cultura medieval: mozárabe, mudéjar, románico, gótico, como yo quiero a mi alma otro tanto a vos quería.
lenguas vernáculas, escolástica. Ya lo veis, nada más cabe que separarnos en vida.
• mozárabe: se dice del individuo de la población hispánica
a) ¿Qué rasgos de la personalidad de héroe se observan
que, consentida por el derecho islámico como tributaria,
vivió en la España musulmana hasta fines del siglo XI con- en él?
servando su religión cristiana e incluso su organización b) Busca algún epíteto épico que se use para referirse
eclesiástica y judicial. al Cid.
• mudéjar: se dice del musulmán a quien se permitía seguir c) ¿A cuál de los tres cantares crees que pertenece este
viviendo entre los vencedores cristianos sin mudar de reli- fragmento? Razona tu respuesta.
gión, a cambio de un tributo.
d) Analiza la métrica de este fragmento. ¿Qué tipo de
• románico: se dice de las lenguas derivadas del latín y de
sus correspondientes manifestaciones literarias y cultura-
rima aparece en él?
les. En arquitectura, se dice del estilo arquitectónico que a) Vemos aquí al Cid como un personaje muy humano, capaz
dominó en Europa durante los siglos XI, XII y parte del XIII. de emocionarse y llorar al despedirse de su mujer y sus
hijas.
• gótico: en arquitectura, se dice del arte que se desarrolla
en Europa desde el siglo XII hasta el Renacimiento. b) El de la barba florida.

60 Lengua castellana y Literatura. 3.º ESO. Solucionario


AMAR, LUCHAR, REZAR… 7
c) Al primero, ya que muestra la despedida del Cid cuando le Análisis de la forma y el contenido. El poema está escrito
obligan a marchar de Castilla. siguiendo la estrofa típica del mester de clerecía, la cuaderna
d) En cuanto a la métrica, se trata de versos de dieciséis sí- vía.
labas, separados por una cesura central. La rima es aso-
6. Cuenta los versos y señala cuál es su medida.
nante (i-a).
Son estrofas de cuatro versos, cada uno de ellos de catorce
6. El Sendebar y el Calila e Dimna son dos colecciones de sílabas.
cuentos de la época, que fueron traducidas del árabe al
castellano. Investiga cuál era la historia que servía de marco 7. Observa la rima. ¿Es asonante o consonante? ¿Cómo riman
a los cuentos en ambas obras y qué tienen en común con entre sí los versos?
El conde Lucanor. Los versos riman en consonante dentro de cada estrofa
En el Sendebar, el pretexto narrativo que enmarca los cuentos (AAAA) (BBBB), etc.
es la leyenda del hijo único de Alcos, rey de Judea, que rehúsa 8. En el texto abundan los sustantivos. Señálalos e indica a
los ofrecimientos amorosos de una de las mujeres del harén qué campos semánticos pertenecen.
de su progenitor. Este rechazo provoca que la cortesana lo
Términos relacionados con el hombre y sus necesidades car-
acuse falsamente de intentar violarla. El joven príncipe es
nales y físicas: hombre, sustentamiento, unión, hembra, hu-
sentenciado a muerte y se ve obligado a guardar silencio por
mano, pecador, mujeres, amor.
espacio de siete días. Para entretener la espera, los sabios
de la corte le narran cuentos que tienen carácter misógino; Relacionados con el principio de autoridad en el que basa sus
entre estos, su madrastra cuenta otros que tienen por objeto razonamientos: filósofo, sabio, razonar, verdad.
condenar al infante. El desenlace, sentenciado por el rey, es Relacionados con los seres vivos y la naturaleza: hombres,
la condena de su madrastra a morir en un «caldero seco» al aves, bestias, animal, cueva, natura, criaturas, naturaleza.
fuego, mientras que el príncipe se salva.
Relacionados con las consecuencias del amor: seso, mesura,
En Calila e Dimna, el marco son las conversaciones entre un locura, mal, fuego, cenizas, bien.
rey y un filósofo, que dan paso a cuentos o exempla protagoni-
zados y narrados por animales, entre ellos dos lobos llamados 9. Aunque no aparecen en el texto demasiados recursos
Calila e Dimna. poéticos, sí que se utiliza un símbolo muy llamativo para
referirse al pecado carnal. ¿Cuál es?
Ambos libros tienen en común con El conde Lucanor que son
ejemplarizantes, que tenían como objetivo la educación de los Se identifica el pecado carnal con el fuego.
príncipes. Conclusiones. Este fragmento es un buen ejemplo de la am-
7. Lee un cuento perteneciente a El conde Lucanor (búscalo bigüedad que caracteriza a la obra.
en la biblioteca o en Internet). Resume en tu cuaderno su 10. ¿Pretende Juan Ruiz advertirnos de los peligros del amor
argumento. Después, cuéntaselo oralmente a tus compañe- carnal o, por el contrario, invitarnos a gozar de él o al me-
ros. ¿Cuál es su moraleja? ¿Crees que el consejo que ofrece nos a probarlo?
Patronio es acertado?
El texto es una clara invitación a probar, al menos, el amor
Respuesta libre. carnal, y luego ya decidir y obrar en consecuencia.

COMENTARIO DE TEXTO 11. ¿En qué versos encontramos la conclusión de todo lo


planteado anteriormente?
Localización. Este fragmento del Libro de buen amor se ins-
Que probemos las cosas no siempre es lo peor, / el bien y el
cribe dentro de la corriente literaria del mester de clerecía mal sabed y escoged lo mejor.
del siglo XIV.
12. ¿Estás de acuerdo con la visión del amor que propone
1. ¿Qué características de dicha corriente se pueden apre-
el arcipreste?
ciar en él?
Respuesta libre.
Podemos inscribirlo dentro del mester de clerecía del siglo XIV
por el uso de la cuaderna vía.
EL JARDÍN DE LA LITERATURA
Tema y estructura. En el fragmento, el arcipreste nos habla
acerca del amor carnal. 1. Lee estos fragmentos de poesía medieval. A continuación,
resuelve las tareas.
2. ¿Cuál es su postura ante este tipo de amor? ¿Te parece
I
que lo defiende o que lo cuestiona?
Decidme, ay hermanitas,
El arcipreste nos invita a gozar del amor carnal.
¿cómo contener mi mal?
3. ¿En qué figura de la Antigüedad se apoya para defender Sin el amado no viviré:
sus argumentos? ¿Cómo se denomina dicho recurso? ¿adónde iré a buscarlo?
Defiende esta postura basándose en el argumento de autori- Mi corazón se me va de mí.
dad (citando a Aristóteles). Oh Dios, ¿acaso se me tornará?
¡Tan fuerte es mi dolor por el amado!
4. ¿Cómo se presenta a sí mismo el arcipreste con respecto Enfermo está, ¿cuándo sanará?
a las cuestiones amorosas? ¿Qué haré, madre?
Explica que él mismo es pecador y ha amado a muchas mu- Mi amado está a la puerta.
jeres.
II
5. ¿El poeta va de lo general a lo particular, o viceversa? Olas del mar de Vigo,
Va de lo general (los hombres y su necesidad de estar con mu- ¿visteis a mi amigo?
jeres) a lo particular (su propia experiencia amorosa). ¡Ay Dios! ¿Vendrá pronto?

Lengua castellana y Literatura. 3.º ESO. Solucionario 61


7 AMAR, LUCHAR, REZAR…

Olas del mar agitado, a) ¿Qué características del mester de clerecía se resaltan
¿visteis a mi amado? en el texto? ¿A qué se contrapone?
¡Ay Dios! ¿Vendrá pronto?
b) Comprueba que, efectivamente, la estrofa anterior
¿Visteis a mi amigo,
está escrita en cuaderna vía. Mide los versos y señala
aquel por quien yo suspiro?
su rima.
¡Ay Dios! ¿Vendrá pronto?
a) Que es un mester hermoso, sin pecado, ya que es de clere-
¿Visteis a mi amado,
cía. Utiliza la cuaderna vía lo cual denota una gran maes-
por quien tengo gran cuidado?
tría. Se contrapone al mester de juglaría.
¡Ay Dios! ¿Vendrá pronto?
b) Son cuatro versos de catorce sílabas, que riman entre sí
III en consonante (AAAA).
No me habléis, conde, 3. Fíjate en el estilo en el que está escrito el siguiente frag-
d’amor en la calle:
mento y en su sencillez.
cata que os dirá mal,
conde, la mi madre. a) ¿Por qué crees tú que el autor prefirió usar una forma
Mañana iré, conde, de expresión tan clara y sencilla?
a lavar al río: b) ¿Qué propósito podía tener la obra?
allá me tenéis, conde, V
a vuestro servicio.
ESCENA II
Cata que os dirá mal MELCHOR
conde, la mi madre. Señores, ¿a qué tierra queréis andar?
No me habléis, conde ¿Queréis ir conmigo al Criador honrar?
d’amor en la calle. ¿Lo habéis visto? Yo lo voy a adorar.
a) ¿Qué tienen en común los tres poemas? ¿En qué se GASPAR
Veamos por ventura si le podremos hallar.
diferencian?
Vayamos tras la estrella, veremos el lugar.
b) Señala quién es el yo poético en cada uno de los poe- MELCHOR
mas y a quién se dirige en cada caso. ¿Cómo comprobaremos si es hombre mortal,
c) ¿Sabrías decir qué composición poética es cada uno si es rey de la tierra, o si es Rey celestial?
de ellos? Argumenta tu respuesta. BALTASAR
¿Queréis saber bien cómo lo sabremos?
a) Los tres son composiciones poéticas breves, pertenecien- Oro, mirra e incienso a él ofreceremos.
tes a la lírica tradicional y popular. En los tres hay una Si fuere rey de tierra, el oro lo querrá,
muchacha que habla con diferentes interlocutores, y su si fuere hombre mortal, la mirra tomará,
temática es el amor. Se diferencian en que los dos prime- y si Rey celestial, estos dos dejará,
ros transmiten un sentimiento más desasosegado, de su- tomará el incienso que le pertenecerá.
frimiento amoroso por la pérdida o ausencia del amado, GASPAR Y MELCHOR
mientras que el tercero es más sereno, la muchacha está Andemos, pues, y así lo haremos,
concertando un encuentro amoroso con su amado. nuestros dones le ofreceremos.
b) En los tres poemas el yo poético es una muchacha, una Auto de los Reyes Magos
joven doncella. Los interlocutores difieren: en el primero
se dirige a sus hermanas y a su madre, en el segundo a un a) Su expresión es clara y sencilla para que sea más fácil-
elemento de la naturaleza (las olas del mar de Vigo) y en mente comprendido por el pueblo. Recordemos que en
el tercero a su amigo, el conde. esta época la mayoría de la población era analfabeta, y
los autos teatrales servían para que el pueblo conociera
c) El primero de ellos es una jarcha (muchacha enamora- los Evangelios y la vida de Cristo.
da que hace confidencias de amor a su madre y herma-
nas, sencillez y brevedad, abundantes interrogaciones y b) La obra tenía un fin didáctico y moralizante, enseñar la
exclamaciones). vida de Cristo y el modo de comportamiento cristiano.

El segundo es una cantiga, como vemos en la conexión de LA FACTORÍA DE TEXTOS


la muchacha con los elementos de la naturaleza, a los que
hace partícipes de sus sentimientos. Formalmente se basa 1. Vuelve a leer el cuento perteneciente a El conde Lucanor
en las repeticiones y los paralelismos. del apartado «Vive la lectura».
El último de ellos es un villancico, su tono es más sereno a) Analiza ahora en él todos los elementos de la narración:
y reposado. También aparece el sentimiento amoroso y la tipo de narrador, personajes, tiempo y espacio.
figura de la madre, esta vez no como confidente sino como
b) Observa cómo una historia se inserta dentro de otra.
ajena a los amores de la muchacha. Aparecen asimismo
repeticiones y paralelismos.
Distingue entre la historia marco y el cuento que sirve
como ejemplo.
2. Ahora lee este fragmento del Libro de Alexandre, una
a) y b) Vamos a distinguir entre la historia que sirve como
obra en verso de principios del siglo XIII, que narra la vida marco al cuento de Patronio, y dicho cuento.
de Alejandro Magno.
En la historia-marco, nos encontramos con un narrador om-
IV nisciente en tercera persona. Los personajes son el conde
Mester os traigo hermoso, no es de juglaría; Lucanor, su consejero Patronio (personajes principales) y el
mester es sin pecado, que es de clerecía; propio autor don Juan Manuel al que se menciona al final (per-
hablar curso rimado por la cuaderna vía sonaje secundario). El espacio no está definido, y tampoco el
a sílabas contadas, que es gran maestría. tiempo (Un día…).

62 Lengua castellana y Literatura. 3.º ESO. Solucionario


AMAR, LUCHAR, REZAR… 7
En cuanto al ejemplo que le cuenta Patronio a Lucanor para con moraleja es intemporal). El tiempo del discurso son
aconsejarle, el narrador es el propio Patronio, que es un na- unos días.
rrador omnisciente en tercera persona. El personaje principal d) El planteamiento sería cuando se nos presenta al ladrón,
es el hombre que quiere cruzar el río con la bolsa de piedras con sus vicios (robar) y sus virtudes (es devoto de la Vir-
preciosas, y el secundario el que le grita desde la orilla que la gen). Serían las dos primeras estrofas.
suelte o se ahogará. El espacio es un río, y el tiempo indefini-
El nudo sería cuando el ladrón finalmente es apresado y
do, ya que es un relato atemporal que vale para adoctrinar en
condenado a la horca, cómo la Virgen María lo socorre
cualquier época y situación.
y evita que muera, sujetándole por los pies con sus ma-
2. Tras leer el siguiente fragmento perteneciente a los Mila- nos, y cómo llegan sus parientes y descubren el milagro.
gros de Nuestra Señora, de Gonzalo de Berceo, responde (De la tercera a la sexta estrofa).
a las preguntas que se te plantean. Y finalmente el desenlace sería cómo le dejan vivir en paz,
Había un ladrón malo que prefería hurtar él cambia de vida y se reconduce y al fin muere cuando le
a ir a las iglesias o a puentes levantar; llega su hora. También se incluiría aquí la moraleja final
solía con lo hurtado su casa gobernar; de alabanza a la Virgen y sus virtudes. (Estrofas séptima
tomó costumbre mala que no podía dejar. y octava).
Entre todo lo malo tenía una bondad 3. Lee el cuento XXXIV de El conde Lucanor: «Lo que suce-
que al final le valió y le dio salvedad: dió a un ciego que llevaba a otro» (puedes encontrarlo en
creía en la Gloriosa de toda voluntad, una biblioteca o en la biblioteca virtual Miguel de Cervan-
y siempre saludaba hacia su majestad. tes), y a continuación responde a las siguientes preguntas:
Como aquel que mal anda en mal ha de caer, a) ¿Quién es el autor de este texto? ¿Y el narrador del
una vez con el hurto lo hubieron de prender; texto completo?
como ningún consejo lo pudo defender
juzgaron que en la horca lo debían poner. b) ¿Y el narrador del cuento dentro del cuento? ¿Hay un
narratario? ¿Quién es?
[…] La Madre gloriosa, tan ducha en acorrer,
la que suele a sus siervos en las cuitas valer, c) Explica cuál es el marco de ambas narraciones.
a este condenado quísolo proteger, d) ¿Qué personajes aparecen en la historia? ¿Cómo son?
recordose el servicio que solía hacer.
a) El autor del texto es don Juan Manuel. El narrador del
Puso bajo sus pies, donde estaba colgado,
texto completo es un narrador omnisciente en tercera
sus manos preciosísimas; túvolo levantado:
persona.
no se sintió por cosa ninguna embarazado,
ni estuvo más vicioso nunca, ni más pagado. b) El narrador del cuento dentro del cuento sería Patronio, el
consejero. Sí hay un narratario, que sería el propio conde
Al fin al tercer día vinieron los parientes,
Lucanor, que escucha el relato de Patronio.
vinieron los amigos y vecinos clementes;
venían por descolgarlo rascados y dolientes, c) El tiempo y el espacio en la narración-marco son bas-
pero estaba mejor de lo que creían las gentes. tante indeterminados. En cuanto al relato que le cuenta
Patronio a Lucanor, tiene lugar en una ciudad (indetermi-
[…] Lo dejaron en paz que siguiese su vía,
nada), por caminos y un barranco. El tiempo también es
porque no querían ir contra Santa María;
indeterminado.
su vida mejoró, se apartó de folía,
cuando cumplió su curso muriose de su día. d) En la historia-marco aparecen como principales Lucanor
y su consejero Patronio, y como secundario don Juan Ma-
A madre tan piadosa, de tal benignidad,
nuel, al que se menciona al final.
que en buenos como en malos ejerce su piedad,
debemos bendecirla de toda voluntad: En el cuento dentro del relato aparecen como protago-
aquel que la bendijo ganó gran heredad. nistas dos ciegos temerarios que acaban cayendo por un
barranco.
a) Este milagro se titula «El ladrón devoto». ¿Crees que
está bien elegido el título? ACTIVIDADES FINALES
b) Explica qué características se le atribuyen al ladrón y Repasa lo que has aprendido
cuáles a la Virgen. ¿Por qué esta intercede por aquel? 1. Lee el siguiente fragmento y después realiza las activida-
c) Analiza en este fragmento todos los elementos de la des que se te plantean:
narración: narrador, personajes, espacio y tiempo. De cómo el Amor visitó al Arcipreste
d) Divídelo en planteamiento, nudo y desenlace. y de la disputa que ambos sostuvieron:
a) Respuesta libre. Una noche sostuve combate peregrino:
b) El ladrón era malo, prefería robar a trabajar. Pero tenía pensaba yo en mi suerte, furioso (y no de vino)
una virtud: que era un gran devoto de la Virgen María. cuando un hombre algo hermoso, cortésmente a mí vino.
La Virgen aparece como piadosa, misericordiosa. Ejerce Le pregunté quién era; dijo: —Amor, tu vecino.
su piedad para con aquellos que la siguen y creen en ella. Con enojo muy grande le empecé a denostar;
Por esto intercede por el ladrón devoto. le dije: —Si amor eres, no puedes aquí estar,
c) El narrador es un narrador omnisciente en tercera perso- eres falso, embustero y ducho en engañar;
na. Los personajes principales son el ladrón y la Virgen salvar no puedes uno, puedes cien mil matar.
María. Como secundarios aparecen los parientes, vecinos Con engaños, lisonjas y sutiles mentiras
y amigos del ladrón. En cuanto al espacio, es indetermina- emponzoñas las lenguas, envenenas tus viras,
do, no sabemos dónde se desarrolla la acción. El tiempo de hiere a quien más te sirve tu flecha cuando tiras;
la historia también es indeterminado (al ser una historia separas de las damas a los hombres, por iras.

Lengua castellana y Literatura. 3.º ESO. Solucionario 63


7 AMAR, LUCHAR, REZAR…

Enloquecidos trae a muchos tu saber; Como diferencias podemos señalar, entre otras, que las jar-
les estorbas el sueño, el comer y el beber, chas son más breves y están escritas en lengua mozárabe.
haces a muchos hombres a tanto se atrever Las cantigas están escritas en lengua galaico-portuguesa y
por ti, que cuerpo y alma llegarán a perder. se basan en las repeticiones y en el paralelismo, así como en
No tienes regla fija, ni te portas con tiento, el procedimiento del leixa-pren. Además en ellas es muy im-
a voces arrebatas con ímpetu violento, portante la naturaleza y están dotadas de un gran simbolismo.
a veces, poco a poco, con maestrías ciento; Los villancicos, por su parte, están escritos en castellano y
en cuanto yo te digo tú sabes que no miento. poseen una mayor variedad temática, aparte del amor apa-
recen otro tipo de asuntos. Están formados por una glosa y
Eres tan enconado que al que hieres de golpe
un estribillo.
no sana medicina, emplasto ni jarope,
no hay hombre recio y fuerte que contigo se tope 3. Di si las siguientes afirmaciones son verdaderas o falsas.
que por diestro que sea no se haga blando y torpe. En caso de que sean falsas, explica qué sería lo correcto:
Arcipreste DE HITA: Libro de buen amor a) La sociedad medieval se basaba en las relaciones
(versión modernizada de María Brey Mariño, Castalia). feudales.
a) Busca en el diccionario el significado de estas palabras: b) En la lírica tradicional o popular un hombre se lamenta-
peregrino, denostar, ducho, lisonja, emponzoñar, viras, ba de la ausencia de su amada.
tiento, enconado, emplasto y jarope. c) Las jarchas estaban escritas en lengua mozárabe.
b) ¿Qué sabes acerca del Libro de buen amor, obra a la d) En los cantares épicos se narraban en prosa las haza-
que pertenece este fragmento? ñas de algún héroe.
c) Explica qué tipo de narrador es el que cuenta la e) El autor del Cantar de Mio Cid fue Per Abbat.
historia. f) Trovador y juglar significaba lo mismo en la Edad
d) ¿Qué le reprocha el protagonista al amor? Media.
e) ¿Cuáles son los personajes que aparecen? Señala g) El Libro de buen amor es una autobiografía amorosa
las características de cada uno de ellos. ficticia.
f) Analiza la métrica de los primeros ocho versos del a) Verdadero.
fragmento. ¿De qué tipo de estrofa se trata? b) Falso. Es una muchacha la que expresa sus sentimientos
a) • peregrino: extraño, especial, raro, pocas veces visto. amorosos.
• denostar: injuriar gravemente, infamar de palabra. c) Verdadero.
• ducho: experimentado, diestro. d) Falso. Se narraban dichas hazañas, pero en verso.
• lisonja: alabanza afectada para ganar la voluntad de e) Falso. Es un Cantar anónimo. Per Abbat fue tan solo el
alguien. copista.
• emponzoñar: envenenar. f) Falso. Los trovadores eran también autores de sus poemas,
y pertenecían al mundo de la corte. Los juglares general-
• viras (del verbo virar): cambiar de rumbo. mente eran retransmisores, no autores, y viajaban por
• tiento: cuidado, prudencia. pueblos y villas recitando y cantando los poemas.
• enconado: encarnizado, violento y muy porfiado. g) Verdadero.
• emplasto: preparado farmacéutico de uso tópico, sólido, 4. Investiga quién fue Rodrigo Díaz de Vivar, y escribe en
moldeable y adhesivo. tu cuaderno su biografía. Cuéntasela después a tus com-
• jarope: jarabe. pañeros.
b) Respuesta libre. El alumno puede recoger, por ejemplo, la siguiente información:
c) Se trata de un narrador en primera persona. Es el propio Rodrigo Díaz de Vivar, llamado El Cid Campeador (Vivar,
arcipreste que cuenta cómo una noche le visitó el Amor y actual España, h. 1043-Valencia, 1099). Caballero castellano,
la conversación que sostuvieron. fue educado junto al infante Sancho, hijo del rey Fernando I de
d) Que es falso y embustero, engaña a los hombres y los en- Castilla y León, quien, al acceder al trono castellano, lo nom-
venena, hiere a sus servidores y los vuelve locos, hacién- bró alférez real (1065). Como jefe de las tropas reales, Rodrigo
doles perder el hambre y el sueño. Por su culpa todos los participó en la guerra que enfrentó a Sancho II de Castilla
hombres pierden cuerpo y alma. con su hermano Alfonso VI de León. El destino, sin embargo,
quiso que Sancho II muriera en 1072, con lo que Alfonso VI se
e) Los personajes son el propio arcipreste (enojado y airado
convirtió en soberano de Castilla y León.
en contra del Amor) y el Amor (aquí aparece personificado
como un hombre hermoso y cortés). El nuevo monarca no solo no manifestó resentimiento hacia el
Campeador, sino que, consciente de la valía de sus servicios,
f) Se trata de estrofas en cuaderna vía: cuatro versos alejan-
lo honró concediéndole la mano de su sobrina, doña Jimena,
drinos con rima consonante AAAA, BBBB.
con quien se casó en julio de 1074. No obstante, unos años
2. Explica cuáles son las diferencias y las semejanzas princi- después, en 1081, una inoportuna expedición a tierras toleda-
pales entre las jarchas, las cantigas de amigo y los villan- nas sin el permiso real, provocó su destierro de Castilla y la
cicos. confiscación de todas sus posesiones.
Las principales semejanzas son que las tres son composicio- Acompañado de su mesnada, el Campeador ofreció sus ser-
nes poéticas pertenecientes a la lírica tradicional y popular, vicios primero a los condes Ramón Berenguer II y Berenguer
de carácter oral. Son breves y en ellas el sujeto lírico suele Ramón II de Barcelona, pero, al ser rechazado, decidió ayudar
ser una muchacha que le hace confesiones amorosas a algún a al-Muqtadir, rey de Zaragoza, en la lucha que mantenía con
confidente (generalmente su madre, hermanas o amigas). su hermano al-Mundir, rey de Lérida, Tortosa y Denia.

64 Lengua castellana y Literatura. 3.º ESO. Solucionario


AMAR, LUCHAR, REZAR… 7
Al servicio de al-Muqtadir, venció en Almenar a Berenguer Consiguen crear un efecto de mayor expresividad y desa-
Ramón II (1082) y cerca de Morella a al-Mundir y al soberano sosiego, nos transmite mejor los sentimientos apenados
aragonés (1084). Durante este período fue cuando recibió el de la joven.
sobrenombre de Cid, derivado del vocablo árabe sid, que sig- d) La primera de ellas es una composición muy breve, de ver-
nifica señor. (Fuente: biografiasyvidas.com). sos de arte menor (hexasílabos) y sin rima.
5. Investiga cómo se trabajaba en la Escuela de Traductores En la segunda, observamos que los versos son también de
de Toledo y expón la información que has recabado ante arte menor, en este caso octosílabos, y que sí aparece rima
la clase. asonante (-aabccbbaa).
Los métodos de traducción evolucionaron con el tiempo. En La última composición también es muy breve, son versos
un primer momento, un judío o cristiano conocedor del árabe de arte menor (de 5 y 6 sílabas), con rima asonante en
traducía la obra original al romance oralmente ante un ex- los pares.
perto conocedor del latín que, a continuación, iba redactando Este tipo de métrica es propia de la poesía tradicional y
en esta lengua lo que escuchaba. Más tarde, en la época de popular (brevedad, versos de arte menor, ausencia de rima
Alfonso X, los libros fueron traducidos por un único traduc- o rima asonante).
tor conocedor de varias lenguas, cuyo trabajo era revisado al
final por un enmendador. e) Observamos alguna personificación (mi desdicha lo de-
tiene), hipérboles (qué nascí tan desdichada, escribo en
(Fuente: www.uclm.es/escueladetraductores). el agua toda mi pena).
6. Lee los siguientes poemas pertenecientes a la lírica po- f) Respuesta libre.
pular y a continuación realiza las actividades propuestas:
I Recuerda lo que ya sabías
¡Oh madre, mi amigo 7. Lee el siguiente texto literario y realiza las actividades que
se va y no vuelve! se te plantean después.
Dime qué haré, madre,
si mi pena no afloja. Antes de partir hacia un largo viaje, el comerciante fue a des-
pedirse de su mujer.
II
—Nunca me has dado un regalo que esté a mi altura —dijo
Si la noche hace escura
ella.
y tan corto es el camino,
¿cómo no venís, amigo? —Mujer ingrata, todo lo que te he dado me costó años de tra-
La media noche es pasada bajo —respondió el hombre—. ¿Qué más te podría dar?
y el que me pena no viene: —Algo que sea tan bello como yo.
mi desdicha lo detiene, Durante dos años, la mujer esperó su regalo. Finalmente el
¡qué nascí tan desdichada! comerciante regresó.
Háceme vivir penada
y muéstraseme enemigo: —Conseguí encontrar algo tan bello como tú —dijo él—. Lloré
¿cómo no venís, amigo? ante tu ingratitud, pero decidí cumplir tu deseo. He pasado
todo este tiempo pensando qué regalo sería tan bello como
III tú, y acabé encontrándolo.
Al río me salgo Y le tendió a su mujer un pequeño espejo.
y en su ribera,
escribo en el agua Paulo COELHO: Maktub (Planeta DeAgostini).
toda mi pena. a) Analiza el relato desde el punto de vista de la comuni-
a) ¿Quién es el yo poético en cada una de las tres com- cación (emisor y receptor, mensaje, código, canal, situa-
posiciones? ¿A quién se dirige? ción e intención comunicativa).
b) Indica cuál es la estructura de esta narración: plantea-
b) ¿Qué sentimiento se muestra en todas ellas?
miento, nudo y desenlace.
c) En la lírica tradicional son muy frecuentes las oracio-
c) Señala todos los adverbios que aparezcan en el texto e
nes exclamativas e interrogativas. Busca algún ejem-
indica de qué tipo son. Haz lo mismo con determinantes
plo en estos textos. ¿Qué efecto producen?
y pronombres.
d) Observa el tipo de métrica empleada (medida de los
d) Analiza todos los verbos que aparezcan en este relato.
versos, rima). ¿Qué conclusiones puedes sacar?
e) Copia en tu cuaderno todos los sintagmas nominales
e) ¿Hay en estos poemas algún recurso literario? ¿Cuál?
y analízalos.
f) ¿Cuál de las tres composiciones crees que transmite f) Analiza morfológica y sintácticamente las siguientes
mejor la desazón que siente la mujer? Argumenta tu oraciones:
opinión.
• Durante dos años, la mujer esperó su regalo.
a) En todas ellas el yo poético es una muchacha enamorada.
En la primera se dirige a su madre, en la segunda a su • Y le tendió a su mujer un pequeño espejo.
amigo y en la tercera no hay un interlocutor determinado. a) El emisor es el narrador. El receptor el lector del cuento.
b) En todas el sentimiento expresado es de pena por la au- El mensaje es la propia historia que se narra del comer-
sencia del amado. ciante y su esposa, el código es el castellano, el canal
es escrito, la situación es que pertenece a un libro de
c) I ¡Oh madre, mi amigo se va y no vuelve! / Dime qué
pequeños relatos de Paulo Coelho y la intención comuni-
haré, madre…
cativa es entretener, enseñar y crear belleza a través de
II ¿Cómo no venís, amigo? / ¡Qué nascí tan desdichada! las palabras.

Lengua castellana y Literatura. 3.º ESO. Solucionario 65


7 AMAR, LUCHAR, REZAR…

b) El nudo sería cuando la mujer le dice a su marido que nun- • años: sustantivo.
ca le ha regalado nada tan bello como ella. El planteamien- • la: determinante artículo.
to cuando él se marcha de viaje y tras dos años buscando
• mujer: sustantivo.
ese regalo vuelve a casa. Y el desenlace cuando le da a su
esposa un regalo a su altura: un espejo. • esperó: verbo.
• su: determinante posesivo.
c) • Adverbios: antes (de tiempo), nunca (de tiempo), más
(de cantidad), finalmente (de tiempo). • regalo: sustantivo.
• Determinantes: un (indefinido), el (artículo), su (po- • y: conjunción.
sesivo), mi (posesivo), dos (numeral), la (artículo), tu • le: pronombre personal.
(posesivo), todo (indefinido), este (demostrativo), qué • tendió: verbo.
(interrogativo). • a: preposición.
• Pronombres: me (personal), que (relativo), ella (perso- • su: determinante posesivo.
nal), todo (indefinido), te (personal), qué (interrogati-
• mujer: sustantivo.
vo), algo (indefinido), yo (personal), tú (personal), él
(personal), lo (personal), le (personal). • un: determinante indefinido o numeral.
d) Partir (infinitivo), fue (pretérito perfecto simple de indi- • pequeño: adjetivo.
cativo, tercera persona de singular), despedirse (infiniti- • espejo: sustantivo.
vo), has dado (pretérito perfecto de indicativo, segunda Análisis sintáctico:
persona de singular), esté (presente de subjuntivo, terce-
ra persona de singular), dijo (pretérito perfecto simple de • Durante dos años la mujer
indicativo, tercera persona de singular), he dado (pretérito SPrep-CC SN-Suj
perfecto de indicativo, primera persona de singular), costó esperó su regalo.
(pretérito perfecto simple de indicativo, tercera persona
NV SN-CD
de singular), respondió (pretérito perfecto simple de indi-
cativo, tercera persona de singular), podría (condicional Sintagma nominal sujeto: la mujer (Det + Núcleo)
simple de indicativo, primera persona de singular), sea Sintagma verbal predicado: durante dos años esperó su
(presente de subjuntivo, tercera persona de singular), es- regalo.
peró (pretérito perfecto simple de indicativo, tercera per- • Y le tendió a su mujer
sona de singular), regresó (pretérito perfecto simple de
CI NV Sprep-CI
indicativo, tercera persona de singular), conseguí (pre-
térito perfecto simple de indicativo, primera persona de un pequeño espejo.
singular), encontrar (infinitivo), dijo (pretérito perfecto SN-CD
simple de indicativo, tercera persona de singular), lloré Sujeto omitido: él (tercera persona del singular).
(pretérito perfecto simple de indicativo, primera persona
Sintagma verbal predicado: y le tendió a su mujer un pe-
de singular), decidí (pretérito perfecto simple de indicati-
queño espejo. Y (conjunción), le (complemento indirecto);
vo, primera persona de singular), cumplir (infinitivo), he
un pequeño espejo (CD).
pasado (pretérito perfecto de indicativo, primera perso-
na de singular), pensando (gerundio), sería (condicional
simple de indicativo, tercera persona de singular), acabé MIRA A TU ALREDEDOR Y…
(pretérito perfecto simple de indicativo, primera persona … ve más allá
de singular), encontrándolo (gerundio), tendió (pretérito
perfecto simple de indicativo, tercera persona de singular). 1. Es imposible desligar la literatura medieval de la música,
ya que casi siempre la recitación de poemas iba acom-
e) • un largo viaje (Det + Adj + Núcleo)
pañada de instrumentos musicales. Si buscas «Old Music
• el comerciante (Det + Núcleo) Middle Ages» en YouTube, podrás disfrutar de unos minu-
• su mujer (Det + Núcleo) tos de música medieval, acompañada además de imáge-
• un regalo (Det + Núcleo) nes de cuadros y miniaturas de la época.
• mi altura (Det + Núcleo) a) ¿Qué te sugiere la música que has escuchado?
• mujer ingrata (Núcleo + Adj) b) Investiga cuáles son los principales instrumentos musi-
• años de trabajo (Núcleo + CN) cales de la Edad Media. ¿Se siguen tocando en la actua-
• el hombre (Det + Núcleo) lidad?
• dos años (Det + Núcleo) c) Escucha ahora al trovador occitano Peire Raimon de
Tolosa (busca su nombre en YouTube), mientras canta y
• la mujer (Det + Núcleo)
toca instrumentos musicales. ¿Crees que música y poe-
• su regalo (Det + Núcleo) sía forman una buena combinación? ¿Qué te transmite?
• tu ingratitud (Det + Núcleo) a) Respuesta libre.
• tu deseo (Det + Núcleo) b) En la Edad Media encontramos gran cantidad de instru-
• todo este tiempo (Det + Det + Núcleo) mentos, algunos provendrán de la Antigüedad y otros
• qué regalo (Det + Núcleo) vendrán de Oriente, introducidos a raíz de las cruzadas.
Vamos a citar algunos de los más importantes.
• un pequeño espejo (Det + Adj + Núcleo)
Cordófonos
f) Análisis morfológico:
• Laúd: instrumento de cuerda con clavijero hacia atrás y
• durante: preposición. caja de resonancia en forma abombada. Se tocaba con
• dos: determinante numeral. los dedos. Es de origen árabe.

66 Lengua castellana y Literatura. 3.º ESO. Solucionario


AMAR, LUCHAR, REZAR… 7
• Vihuela de arco o fídula: instrumento de arco. Será el b) Un tipo muy especial de miniaturas son los beatos. Bus-
prototipo del violín moderno. ca información sobre ellos. ¿Por qué se llaman así? ¿De
• Organistrum o zanfoña: es un instrumento de tres cuer- qué siglo datan? ¿Qué representaban?
das que se toca girando una manivela.
Aquí tienes algunas imágenes de uno de los más im-
• Arpa: de forma triangular y pequeño tamaño. Se toca portantes, el Beato de Liébana.
con las dos manos, una pulsa las cuerdas y la otra acor-
a) Se llamaban miniaturas porque se elaboraban con minio,
ta su longitud.
óxido de plomo de color rojo que se utilizaba como com-
Viento ponente fundamental para fabricar la tinta.
• Platerspiel: es una especie de gaita con forma de cuer- b) Se trata de códices manuscritos, compuestos entre los
no. Está hecho de madera y tiene seis agujeros. siglos X al XII. Se llaman así porque los más importantes
• Chirimía: especie de oboe de mayores dimensiones muy fueron creados por Beato de Liébana, un monje del monas-
usado en los desfiles y actos relevantes. terio de san Martín de Turieno (actualmente Toribio), en el
• Cornamusa: parecido a las gaitas actuales. valle de Liébana. Contienen comentarios al Apocalipsis de
san Juan, ilustrados con hermosas miniaturas.
• Flauta.
• Corneta. … encuentra la clave
Percusión 1. Aparte de la literatura, el arte también fue un instrumento
• Címbalos: compuesto por un arco de madera y una o clave para adoctrinar al pueblo durante la Edad Media. Lee
dos membranas tensas. Se percutía con la mano. el siguiente texto y resuelve las actividades que se te plan-
• Tejoletas: rudimentarias castañuelas hechas con trozos tean a continuación:
de teja que se entrechocan. Instruir a los fieles era el principal propósito de las imáge-
• Darbuka: tambor con forma de copa y una membrana nes. Pintura y escultura, normalmente coloreadas, fueron fór-
tensa en la parte superior. Normalmente está hecho de mulas primordiales para difundir el mensaje divino; aquello
cerámica y se golpea con las manos. que la literatura daba a conocer con palabras, la decoración
• Carrillón. del templo debía mostrarlo mediante imágenes que permitie-
ran visualizar los acontecimientos bíblicos, única forma, junto
(Fuente: musicaedadmedia.com).
con la predicación oral, de acceso al conocimiento que tenía
Algunos de ellos se siguen empleando en la actualidad en la inmensa mayoría de los devotos.
sus formas evolucionadas (arpa, flauta, corneta, carri-
El arte debía enseñar y emocionar mediante la sencillez, la
llón…). Otros solo se tocan en conciertos que recrean la
claridad y la expresividad de sus imágenes, las cuales hacían
música medieval.
visible al ser humano aquello que era invisible. […]
c) Respuesta libre.
Los motivos más representados fueron los que narraban histo-
2. La figura del Cid Campeador ha despertado gran inte- rias del Antiguo Testamento, los Evangelios o el Apocalipsis.
rés a lo largo de la historia, no solo en la literatura, sino Les siguen los que no pretenden contar, sino hacer visible un
también en el cine. En 1961 se produjo una gran película, universo simbólico: el que se quiere transmitir a través de las
dirigida por Anthony Mann y con Charlton Heston en el luchas entre hombres y animales, la representación de anima-
papel del Cid. les rampantes enfrentados, o imágenes del bestiario —arpías,
sirenas, basiliscos, grifos, quimeras—. Se trata de figuras car-
Busca en YouTube el fragmento en el cual Rodrigo Díaz de
gadas de un fuerte simbolismo que apunta a la dicotomía entre
Vivar se despide de su esposa, doña Jimena: teclea «El Cid el bien y el mal, al horror del pecado, a las visiones infernales
1961 (español) 12/18». ¿Te imaginabas así la ambientación y y demoniacas, etc.
a los personajes? ¿Por qué deben separarse? […] También se reproduce el mundo cotidiano, amable y cerca-
a) Después de despedirse, el Cid comparece ante el rey. no de las faenas agrícolas o los oficios, los rostros familiares y
¿Cómo se llama el monarca? ¿Sobre qué discuten? las costumbres, sucesos lúdicos, la fauna local o los animales
b) A continuación, el Campeador se dirige a un monas- domésticos.
terio. ¿Con quién se reencuentra allí? ¿Cómo se lla- VV. AA.: Historia del arte de la antigua Edad Media
maban las hijas del Cid en el Cantar? ¿Y en realidad? (Editorial Universitaria Ramón Areces).

a) El monarca es Alfonso VI (aunque no se menciona su nom- a) Resume el texto que acabas de leer.
bre) y discuten sobre la necesidad de tomar Valencia antes b) Di si las siguientes afirmaciones son verdaderas o falsas:
de enfrentarse a Ben Yusuf. Finalmente, se obliga a don
Rodrigo combatir en Sagrajas si quiere ser indultado. • La única forma que tenían los fieles de acceder al
mensaje divino era mediante la pintura y la escultura.
b) En el monasterio el Cid se reencuentra con su mujer y sus
hijas. Las hijas en el Cantar se llaman doña Elvira y doña • Las imágenes únicamente servían para narrar, en-
Sol, pero en realidad eran María y Cristina. señar y transmitir cultura.
3. Los códices y manuscritos medievales eran frecuentemente • Para cumplir con su propósito era importante que
decorados en sus márgenes con miniaturas: pequeños las imágenes fueran claras, sencillas y expresivas.
dibujos ornamentales de gran colorido, que representa- • Los temas más representados fueron las escenas de
ban escenas propias de la época. Aquí tienes como ejemplo lucha entre hombres y animales, y las faenas coti-
una miniatura de Le roman de la rose, un libro didáctico del dianas.
siglo XIII, y la letra capitular V de un cantoral gregoriano. • El arte medieval servía para simbolizar la lucha en-
a) Investiga por qué se llamaban miniaturas. ¿De dónde tre el bien y el mal, y para disuadir a los hombres de
procede el término? pecar.

Lengua castellana y Literatura. 3.º ESO. Solucionario 67


7 AMAR, LUCHAR, REZAR…

c) Investiga acerca de los principales animales fantásti- • basilisco: animal fabuloso, al cual se atribuía la propie-
cos del bestiario medieval: arpías, basiliscos, grifos y dad de matar con la vista.
quimeras. ¿Cómo eran y qué simbolizaban? • grifo: animal fabuloso, con la mitad superior del cuerpo
d) Observa estas imágenes y di si se corresponden con de un águila, y la mitad inferior de un león.
historias de la Biblia, del bestiario medieval o de fae- • quimera: monstruo imaginario que, según la fábula, vo-
nas agrícolas y cotidianas. ¿Qué es lo que representa mitaba llamas y tenía cabeza de león, vientre de cabra
cada una? y cola de dragón.
a) Respuesta libre.
Todos ellos eran seres fantásticos y monstruosos que sim-
b) • Falso, otra forma era la predicación oral. bolizaban el mal y el pecado.
• Falso, también debían emocionar.
d) La primera representa una faena cotidiana, agrícola y
• Verdadero.
ganadera.
• Falso, los motivos más representados fueron los
bíblicos. La segunda pertenece al bestiario medieval, en ella obser-
• Verdadero. vamos lo que parecen quimeras con cabeza de león.
c) • arpía: ave fabulosa, con rostro de mujer y cuerpo de En la tercera aparece una imagen bíblica, de la vida de
ave de rapiña. Jesucristo, concretamente es la huida a Egipto.

68 Lengua castellana y Literatura. 3.º ESO. Solucionario


MELIBEO SOY… 8
APERTURA DE UNIDAD rendir vasallaje durante un largo tiempo antes de recibir
de ella un galardón que mostrara su aceptación. ¿Respeta
1. En el poema dialogan dos personajes. ¿Quiénes son?
Calisto estas convenciones?
¿Crees que se trata de una composición perteneciente al
género lírico o al narrativo? Razona tu respuesta. No, Calisto es un enamorado impaciente que quiere recibir su
recompensa demasiado rápido; esto enoja a Melibea.
Dialogan un hombre y una mujer que han tenido relaciones
amorosas. Pertenece al género narrativo ya que a través del 5. Los sentimientos hacia Melibea que Calisto expresa a su
diálogo de los personajes conocemos su historia, y lo que les criado Sempronio son muy exagerados. ¿Qué es lo que
ha ocurrido a ambos. desea? ¿Por qué esto se opone a ser un buen cristiano?
2. ¿Quién es rosa fresca y por qué rechaza a su amigo? ¿Cuál Desea la muerte, y estos sentimientos son contrarios a la re-
dirías entonces que es el tema del poema? ligión cristiana.
Rosa fresca es la mujer que rechaza a su amigo porque se ha Vocabulario
enterado de que este está casado y tiene hijos en tierras de
León. El tema del poema sería, pues, el desengaño amoroso 6. La palabra acatamiento procede de catar («mirar»). ¿Qué
de esta mujer al enterarse de la verdad sobre su amante. significado tiene el verbo acatar en la actualidad? Escribe
3. Escucha al compositor y cantautor español Amancio Pra- una frase donde se ejemplifique.
da cantando este romance: teclea en YouTube «Amancio Acatar en la actualidad significa obedecer, aceptar con sumi-
Prada» «Romance de la rosa fresca». A continuación, es- sión una autoridad o normas. Ejemplo: Los cristianos deben
acatar los mandamientos de la Iglesia.
cribe en tu cuaderno qué sensaciones te ha generado la
audición. ¿Qué crees que le aporta la música al romance? 7. La palabra galardón es un germanismo que significa «re-
¿Piensas que en el siglo XV los romances se cantaban de compensa a los servicios prestados». El galardón tenía una
este modo? gran importancia en el amor cortés. Investiga por qué y en
Respuesta libre. qué consistía.
El galardón en el amor cortés era una prenda (por ejemplo,
4. Los romances son composiciones poéticas típicas del si-
un pañuelo de seda o una cinta) que la mujer entregaba a su
glo XV. Uno de sus temas más importantes es el amor, enamorado para mostrarle que aceptaba su vasallaje y lo re-
como en el que acabas de leer. Pero también son muy conocía como siervo (en el sentido amoroso) suyo.
frecuentes los romances que cuentan acontecimientos re-
lacionados con la Reconquista y las luchas entre cristianos 8. Las palabras inmérito, incomparablemente, indignamente,
y árabes. Escucha ahora el Romance de la mora cautiva, desventuradas, ilícito y desdichado tienen en común el
tecleando en YouTube «Amancio Prada» «Romance de la estar formadas con un prefijo que les aporta un deter-
hermana cautiva». ¿Cuál es su tema? ¿Qué personajes minado significado. ¿Cuál es? Construye una frase con
aparecen en él? Explica oralmente su argumento. cada una de ellas.
En todas ellas el prefijo significa negación, sin (sin mérito,
En este romance aparecen un caballero y una mora cautiva.
sin comparación, sin dignidad, sin ventura, que no es lícito,
El caballero se lleva a la mora de vuelta a tierras cristianas, y
sin dicha).
cuando van llegando, debido a los comentarios hace la joven
acerca de los lugares por los que pasa, se da cuenta de que Frases: respuesta libre.
en realidad es una hermana suya que se llevaron los moros 9. En el diálogo que mantiene Calisto con Melibea, este
cuando era niña. emplea un gran número de términos relacionados con
la religión y lo divino. Señálalos. ¿Por qué piensas tú que
VIVE LA LECTURA
se vale de este tipo de palabras? Piensa de nuevo en la
Comprensión lectora relación de la obra con el amor cortés.
1. Esta escena recoge el momento en el que Calisto y Meli- Emplea palabras y expresiones relacionadas con la religión
bea se ven por vez primera. ¿Con qué compara Calisto el como la grandeza de Dios, obras pías, tengo a Dios ofreci-
sentimiento que tiene al contemplar a Melibea? do, cuerpo glorificado, los gloriosos santos, visión divina, si
Dios me diese en el cielo la silla sobre sus santos.
Lo compara con contemplar una visión divina.
Recordemos que en el amor cortés la amada era como una
2. Melibea, al principio, parece darle esperanzas a Calisto. especie de dios, al que el enamorado debía rendir culto y
¿Dónde se manifiesta esto? devoción.
Cuando le dice que aún mayor galardón le dará ella, si per- 10. En la actualidad, decimos de alguien que es una celestina
severa.
cuando se dedica a intentar unir a dos enamorados. Este
3. Sin embargo, después, se muestra enojada con él. ¿Qué procedimiento, que consiste en poner el nombre apelati-
le dice? ¿A qué crees tú que se debe este cambio de ac- vo por el propio (o viceversa), se denomina antonomasia.
titud? Así, decimos, por ejemplo, que una persona que lucha por
Se enfada con él y le dice que la paga será tan fiera como sus ideales sin tener en cuenta la realidad es un quijote.
merece su atrevimiento, se escandaliza de que quiera hacerla Busca otros cinco ejemplos del mismo fenómeno. No tie-
perder su virtud y lo echa de su lado sin miramientos. Posi- nen por qué ser únicamente ejemplos literarios, también
blemente se enoje tanto porque Calisto está siendo demasiado pueden ser inventores cuyo nombre sirve para designar
osado en sus declaraciones, no está respetando las reglas del el invento creado por ellos.
amor cortés.
Por ejemplo, ser un nerón (despiadado y cruel), hacer de cice-
4. Quizá te ayude conocer cuáles eran las convenciones en rone (de guía o maestro), ser un hércules (hombre muy fuerte),
las que se basaba el amor cortés: la mujer era considerada ser un atila (destruir todo lo que se te pone por delante), ser
una especie de divinidad, a la que el enamorado debía un judas (un traidor).

Lengua castellana y Literatura. 3.º ESO. Solucionario 69


8 MELIBEO SOY…

Investigación y redacción la religión de sus súbditos, por lo cual en 1492 se procedió a


expulsar a los judíos. Ya en 1478 se había creado la Inquisi-
11. Muchos de los nombres de los personajes de La Celes- ción para perseguir a los cristianos nuevos que volvían a sus
tina no están escogidos al azar, sino que tienen algo que antiguas creencias. En 1504 fallecía la reina Isabel y aunque
ver con las características del personaje al que designan. dejaba como regente de la heredera al trono, Juana I, a su
Investiga qué significado tienen los nombres de los ena- marido Fernando el Católico, la nobleza castellana no lo apo-
morados Calisto y Melibea, del criado Sempronio y del yó por lo que este marchó a sus estados de Aragón. De este
padre de Melibea, Pleberio. modo quedaba encargado del gobierno de Castilla Felipe de
Austria, el Hermoso, esposo de la reina Juana I de Castilla,
Calisto significa el más bello, Melibea quiere decir dulce como
la Loca. Pero la muerte de Felipe en 1506 obligó a restituir
la miel, Sempronio viene de siempre (lo cual es irónico porque
a Fernando, llamado por el Cardenal Cisneros a Castilla en
se trata del criado que traiciona a su señor) y Pleberio hace
1507. Los últimos años de su reinado se caracterizaron por los
referencia a la plebe (también irónicamente, ya que Pleberio
enfrentamientos con Francia en terreno italiano. A la muerte
pertenece a la nobleza).
de Fernando el Católico heredó el trono su nieto Carlos I de
12. En 1996 Gerardo Vera llevó la obra al cine. El resultado fue España.
una película muy bien ambientada, que refleja con bastante (Fuente: www.artehistoria.com).
fidelidad la vida de la época. Ved en clase un resumen, al
2. Ve este fragmento de la serie televisiva Isabel, tecleando
que podéis acceder tecleando «mi película Celestina» en
en YouTube «Ya es reina Isabel», donde asistirás al momen-
un buscador. Después, anota en tu cuaderno qué cosas te
to de su coronación como reina de Castilla. Fíjate bien en
han llamado la atención en cuanto a la caracterización de
cómo está recreada la época histórica. Isabel cumplió un
los personajes, el vestuario, la ambientación, etc.
importantísimo papel para la historia de Castilla. ¿Crees
Respuesta libre. que era lo normal en la época, tratándose de una mujer?
Respuesta libre.
LITERATURA
3. Investiga y escribe en tu cuaderno cuál es el argumento
1. Prepara una presentación en Prezi o PowerPoint con al- de la novela sentimental Cárcel de amor.
gunos de los datos más relevantes del reinado de los Re- El autor, que aparece como un personaje en la obra, se ex-
yes Católicos. Recoge información sobre sus problemas travía en Sierra Morena, y se encuentra con un joven enca-
para acceder al trono, las luchas contra los musulmanes, denado; este joven es Leriano, hijo del duque Guersio, que
su apoyo a la empresa de Cristóbal Colón, su política de es llevado a la prisión del Amor por un monstruo llamado
alianzas matrimoniales a través de sus hijos o qué ocurrió Deseo. Siguiéndoles, el autor entra en un castillo en la mon-
con la sucesión al trono cuando ellos murieron. taña, donde Leriano le confiesa su pasión por Laureola, hija
del rey Gaulo de Macedonia, y le ruega que actúe como inter-
El alumno podrá recoger en su presentación, entre otros, los
mediario. El autor va en busca de Laureola y, tras hablarle
siguientes datos:
de Leriano y del dolor que padece, logra ablandar su corazón
Isabel, hija de Juan II de Castilla y de Isabel de Portugal, y y consigue que la muchacha le escriba una carta. Con la
Fernando, hijo de Juan II de Aragón y de Juana Enríquez, ayuda de una serie de personajes alegóricos (la Esperanza,
contrajeron matrimonio en Valladolid el 19 de octubre de la Satisfacción, la Tranquilidad, la Alegría…) libera de su
1469, entre fuertes oposiciones al mismo. Isabel heredaría cautiverio a Leriano, que se dirige a Macedonia para ver a
el trono de Castilla en 1474 después de la muerte de su her- Laureola, quien le recibe con agrado. Pero otro personaje
mano Enrique IV, autoproclamándose reina, ya que había un que también pretende a Laureola, el malvado Persio, hijo
conflicto sucesorio entre ella y Juana, hija de Enrique IV, de del señor de Gavia, difunde rumores que ponen en peligro
la que se decía era hija de Beltrán de la Cueva y no del rey. el honor de la doncella, de modo que Leriano lo desafía, ven-
Mientras tanto Fernando era nombrado heredero a la muerte ciéndole en dos ocasiones. La victoria de Leriano sobre Per-
de su hermano Carlos. En 1468 recibió el trono de Sicilia y sio no soluciona la situación, porque el rey Gaulo, padre de
a la muerte de su padre en 1479, el de la corona de Aragón. Laureola, ha dado crédito a los falsos testimonios y conde-
Participó en las luchas a favor de su esposa Isabel y a par- na a muerte a su hija. Leriano y sus partidarios asaltan la
tir de esta fecha se produjo la unión dinástica de Aragón y prisión donde Laureola aguarda la ejecución y la liberan,
Castilla y el comienzo del reinado conjunto. Este matrimonio enfrentándose a todo el ejército real en una cruel batalla. En
ha sido considerado como el punto de partida de la unidad la lucha, capturan a uno de los calumniadores, que admite
y de la grandeza de España. El primer objetivo de los nue- su culpa, con lo que el rey perdona a su hija. Laureola, preo-
vos monarcas fue el de restablecer la autoridad real, para cupada por defender su honor y no levantar más sospechas,
lo cual se sirvieron de una poderosa organización: la Santa rechaza definitivamente a Leriano. Este, echa en una copa
Hermandad. También constituyeron el Consejo Real que sus- las cartas de Laureola, que ha roto, se bebe su contenido
tituía a las Cortes y nombraron corregidores para controlar y se deja morir de inanición, mientras su madre entona un
las ciudades y vincularon la dirección de la Mesta al Consejo desesperado planto.
Real. De este modo quedaba controlada la política del reino,
aunque estas medidas pesaron más sobre el reino de Castilla
4. Lee el siguiente fragmento de las Danzas de la Muerte y
que sobre el de Aragón. La siguiente misión era concluir la realiza las actividades que se te plantean:
Reconquista en el reino nazarí de Granada, lo que consiguie- MUERTE.— ¡Oh, cuán sin acuerdo de mí, y sin temor
ron en 1492. La paz interior y la buena organización del reino yaces en vicios terrenos jatando1,
permitieron que las arcas reales se llenaran y con ellas se la gloria pasible de acá procurando,
acometieran nuevas empresas como el apoyo al almirante soberbia mostrando por ser gran señor;
genovés Cristóbal Colón, que descubriría América en 1492, en quien la humildad, según que a Pastor,
aportando riquezas para el reino y un fuerte expansionismo había de ser grande ejemplo al ganado!
exterior. El éxito de la guerra antimusulmana y la presión Y, pues fue al revés, irás muy priado2
de los confesores de la reina indujeron a los Reyes a unificar conmigo a do cuenta darás de tu error.

70 Lengua castellana y Literatura. 3.º ESO. Solucionario


MELIBEO SOY… 8
PAPA.— ¡Oh, muerte!, no vengas con tanto furor; 7. Relee el Romance de la rosa fresca (página 147). Señala
aplaca tu ira; ten más sufrimiento: en él las características típicas del género.
mira que es grande mi merescimiento
En el Romance de la rosa fresca aparecen también caracte-
de muy alta estima mi estado y valor;
rísticas como la concisión y el fragmentarismo (se nos pre-
no muestres conmigo tan grande rigor,
senta la historia en su mitad y no sabemos cómo concluye), la
que tengo en la tierra muy gran señorío.
expresividad, la sencillez, el uso de diálogos para dar mayor
MUERTE.— Muy poco te excusa tan gran desvarío
dramatismo al texto, así como algunas repeticiones, anáforas
el golpe mortal de mi pasador.
y paralelismos.
Sin más resistencia sabrás, sin mentir,
aunque tu estado a todos hoy sobre, 8. Lee la siguiente serranilla del marqués de Santillana y
muy breve serás igual con el pobre, realiza las actividades:
en solo este paso que llaman morir. La vaquera de Morana
Juan DE PEDRAZA En toda la su montana
de Trasmoz a Beratón
non vi tan gentil serrana.
1
jatando: jactando, presumiendo. 2
priado: presto, rápido. Partiendo de Conejares,
allá suso en la montaña,
cerca de la Travesaña,
a) ¿De qué acusa la Muerte al Papa? ¿Qué le responde camino de Trasobares,
este? ¿Le sirven de algo sus protestas? encontré moza lozana
poco más acá de Añón,
b) En cervantesvirtual.com podrás leer esta danza de riberas d’una fontana.
la Muerte completa. ¿Qué moraleja se desprende del […]
poema? Dije: «Dios te salve, hermana;
a) Le acusa de haberse dejado llevar por los placeres terre- aunque vengas d’Aragón,
nales, así como de soberbia. El Papa le responde que su d’esta serás castellana».
estado y valor son grandes, así como su señorío, pero no Respondiome: «Caballero,
le sirve de nada porque la Muerte no se apiada de él. non penséis que me tenedes,
ca primero probaredes
b) Que todos somos iguales ante la muerte, ricos y pobres,
este mi dardo pedrero;
poderosos y humildes, todos habremos de morir algún día.
ca después d’esta semana
5. Lee el siguiente romance e indica qué características pro- fago bodas con Antón,
pias del género observas en él. Luego, señala cuál es su vaquerizo de Morana».
tema y realiza el análisis métrico. Biblioteca Virtual MIGUEL DE CERVANTES
Que por mayo era por mayo, a) Busca en el diccionario el significado de las palabras
cuando hace la calor, lozana, fontana y guisa.
cuando los trigos encañan
y están los campos en flor,
b) Investiga dónde se encuentran los lugares menciona-
cuando canta la calandria dos en el poema: Trasmoz, Beratón, Conejares, Traso-
y responde el ruiseñor, bares, Añón y Morana.
cuando los enamorados c) ¿Qué es lo que pretende hacer el hombre con la serra-
van a servir al amor, na? ¿Qué le responde esta?
sino yo, triste, cuitado,
a) • lozana: fresca, garrida.
que vivo en esta prisión,
que ni sé cuándo es de día • fontana: fuente.
ni cuándo las noches son, • guisa: modo, manera o semejanza de algo.
sino por una avecilla b) • Trasmoz: provincia de Zaragoza.
que me cantaba al albor:
• Beratón: provincia de Soria.
matómela un ballestero;
¡dele Dios mal galardón! • Conejares: provincia de Soria.
• Trasobares: provincia de Zaragoza.
Observamos en el romance características propias de su
género, como la concisión y la fragmentación (no sabemos • Añón: provincia de Zaragoza.
nada del antes ni del después del prisionero), además es muy • Morana: provincia de Pontevedra.
expresivo (a ello contribuye el uso de la primera persona), y c) El hombre pretende llevársela a Castilla. Ella le responde
abundan en él las repeticiones y los paralelismos, que favore- que antes probará uno de sus dardos, ya que está compro-
cen su memorización. metida con un vaquerizo de Morana.
Su tema sería el contraste entre la primavera que llega al resto 9. Lee atentamente las anteriores coplas y responde:
del mundo, y el estado del prisionero, que vive encerrado y ha
perdido su única conexión con el exterior: un pajarillo que le a) ¿Cuál es el tema de cada una de ellas?
cantaba cada mañana y ha muerto en manos de un ballestero. b) Analiza métricamente la primera copla. ¿Cuáles son
Formalmente, está compuesto por versos octosílabos, que ri- los pies quebrados?
man en asonante en los pares, quedando sueltos los impares. c) La copla III se basa en una metáfora. ¿Cuál es el tér-
6. Teclea en YouTube «Paco Ibáñez» «Romance del prisione- mino real y cuál es el término imaginario?
ro» y escucha una versión cantada. d) Explica cómo se manifiesta el tópico del ubi sunt? en
Audición. la copla XVII.

Lengua castellana y Literatura. 3.º ESO. Solucionario 71


8 MELIBEO SOY…

a) La copla I nos habla de la rapidez con que se pasa la vida 4. Respecto a la copla VIII, ¿mantiene el mismo tono que la
y nos llega la muerte; la III de cómo todos somos iguales anterior? ¿En qué se diferencia?
ante la muerte; y la XVII desarrolla el tópico del ubi sunt?,
La copla VIII pretende hacernos reflexionar sobre cómo la be-
el autor se pregunta qué fue de las damas, los trovadores
lleza y la fuerza de la juventud desaparecen al llegar la vejez,
y la vida de la corte.
y lo hace mediante preguntas retóricas.
b) La copla está formada por dos sextillas, con la siguien-
te disposición: versos octosílabos y tetrasílabos, con
5. La copa XXV trata otro tema. ¿Qué cualidades del padre
estructura 8a8b4c, 8a8b4c, 8d8e4f, 8d8e4f. La rima es se destacan?
consonante. En la copla XXV Manrique elogia las virtudes de su padre, don
Los pies quebrados son los versos tetrasílabos, que son Rodrigo, destacando especialmente cómo este era amado por
más cortos que el resto. todos, virtuoso, valiente, famoso…

c) Está identificando nuestras vidas (término real) con ríos Análisis de la forma y el contenido. El poema está escrito en
(término imaginario), que van a dar a la mar (término ima- coplas de pie quebrado o manriqueñas.
ginario), que es el morir (término real). 6. Cuenta los versos y señala cuál es su medida.
d) El autor se va preguntando qué se hizo de las damas de la Se trata de coplas agrupadas en dos sextillas: conjunto de seis
corte, de sus amadores, de su trovar y danzar…, para que versos con la siguiente estructura 8a8b4c, 8a8b4c.
reflexionemos cómo todos ellos han muerto y ese mundo 7. Observa la rima. ¿Es asonante o consonante? ¿Cómo riman
de la corte ha desaparecido para siempre.
los versos?
10. Investiga acerca del festival celestinesco que se celebra Se trata de rima consonante abc, abc.
cada año en La Puebla de Montalbán y expón dicha infor-
8. En el texto abundan los sustantivos. Señálalos e indica a
mación ante tus compañeros.
qué campos semánticos pertenecen. Señala también los
El festival de La Celestina de La Puebla de Montalbán se em-
adjetivos que los complementan.
pezó a celebrar en el verano de 1999, con motivo del V centena-
rio de la publicación de la Comedia. Se celebra todos los años Sustantivos que hacen referencia a la vida como camino: mun-
en el mes de agosto, los dos últimos fines de semana de dicho do, camino, morada, tino, jornada.
mes. En él se realizan diversas actividades relacionadas con Aquellos que se refieren a las características de la juventud:
La Celestina y la vida medieval: mercado de época, actuacio- hermosura, frescura, tez, cara, color, blancura, mañas, li-
nes teatrales en las cuevas y, sobre todo, la representación gereza, fuerza, juventud.
de la obra que tiene lugar en la plaza mayor de la localidad, y Sustantivos relativos a la vejez: vejez, graveza, arrabal,
cuya escena final del suicidio de Melibea se realiza en la torre senectud.
de san Miguel. Todo el pueblo participa y se implica muchísi- Sustantivos que hacen referencia a su padre: abrigo, gente,
mo en esta celebración. Además los visitantes pueden acudir maestre, Rodrigo Manrique, hechos, mundo.
al museo celestinesco de La Puebla, donde están expuestos los
No abundan los adjetivos, pero sí aparecen algunos acom-
cuadros con los que el pueblano Teo Puebla ilustró la edición
pañando a ciertos sustantivos: buen tino, gentil frescura,
conmemorativa del centenario.
fuerza corporal, hechos grandes e claros.
COMENTARIO DE TEXTO 9. Anota los recursos literarios que aparezcan y explica el por-
qué de su uso.
Localización. Estas tres coplas pertenecen a diferentes blo-
ques temáticos. No abundan tampoco los recursos literarios, aunque sí ob-
servamos una metáfora en la copla V, que identifica nuestras
1. ¿A qué bloque pertenece cada una? Justifica tu respuesta. vidas con un camino, la duración de la vida con una jornada
Tanto la copla número V como la VIII pertenecen a la primera y el fin del camino con la muerte.
parte de la obra: reflexión general sobre la vida y la fugacidad En la copla VIII aparecen preguntas retóricas para hacer-
de las cosas terrenales. La copla XXV, en cambio, pertenece nos reflexionar acerca de cómo desaparece la juventud y la
a la tercera y última parte de la obra, en la que Manrique se belleza.
centra en la figura de su padre y lo elogia como ideal de caba-
llero y ejemplo del saber morir cristianamente.
Conclusiones. Estas coplas nos muestran claramente la fina-
lidad de la obra.
2. ¿En qué expresiones se refleja claramente la mentalidad
medieval? 10. ¿Cuál es esa finalidad?
La finalidad de la obra es doble: por un lado hacernos reflexio-
Se observa dicha mentalidad medieval en la religiosidad de la
nar sobre cómo se pasa de rápido la vida y de la necesidad de
aceptación de la muerte, al considerar, por ejemplo, que este
despreciar las cosas terrenales y prepararnos para la vida que
mundo es el camino para el otro, el de la vida eterna, o que
de verdad importa: la vida eterna. Por otro lado pretende re-
cuando morimos descansamos. Así mismo lo observamos en
saltar las virtudes del padre muerto y presentarlo como ideal
el desprecio que hace de las cosas terrenales (la belleza, la ju-
de caballero cristiano, así como un ejemplo en su aceptación
ventud, la fuerza). Todo eso no importa porque es perecedero,
de la muerte.
lo importante es preparar el alma para la vida eterna.
Tema y estructura. Las coplas V y VIII comparten el mismo 11. ¿Qué tipo de mentalidad se refleja en las coplas?
tema. La mentalidad es claramente medieval, y se observa por ejem-
plo en dicho desprecio por las cosas terrenales, en la importan-
3. ¿Cuál es el tema de la copla V? ¿En qué recurso literario cia que se le concede a la religión y a la preparación para la vida
se sustenta? eterna, en la defensa de ciertos valores y virtudes cristianas y
El tema de la copla V sería cómo nuestra vida es un camino, y caballerescas… Sin embargo, tiene un punto de modernidad al
al llegar a nuestra meta, morimos. Está basado en el recurso presentar una tercera vida (junto con la terrenal y la eterna),
de la metáfora, donde el término real es la vida y el imaginario la vida de la fama, el hecho de que nuestra fama perdurará tras
dicho camino que nos lleva a la muerte. nuestra muerte si nuestros actos han sido dignos de ello.

72 Lengua castellana y Literatura. 3.º ESO. Solucionario


MELIBEO SOY… 8
EL JARDÍN DE LA LITERATURA 4. En el acto X, Celestina consigue que finalmente Me-
libea confiese su amor por Calisto, gracias a sus grandes
1. Lee el siguiente fragmento del acto I de La Celestina, en
dotes dialécticas. Léelo y señala cómo va convenciendo
el que Calisto describe a Melibea. Copia en tu cuaderno
a Melibea hasta que esta lo reconoce.
los rasgos físicos con los que la describe y busca informa-
ción sobre cómo era el canon de belleza femenina de la CELESTINA.— Tu llaga es grande, tiene necesidad de áspera
cura. […] No concibas odio ni desamor, ni consientas a tu
época. ¿Responde a la descripción que hace Calisto de
lengua decir mal de persona tan virtuosa como Calisto,
Melibea? ¿Hay en el texto algún recurso poético? que si conocido fuese…
CALISTO.— Comienzo por los cabellos. ¿Vees tú las madejas del MELIBEA.— ¡Oh por Dios, que me matas! ¿Y no te tengo dicho
oro delgado que hilan en Arabia? Más lindos son, y no res- que no me alabes ese hombre ni le nombres en bueno ni
plandecen menos. Su longura hasta el postrero asiento de malo?
sus pies, después, crinados y atados con la delgada cuerda, CELESTINA.— Señora, este es otro y segundo punto: si tú con
como ella se los pone, no ha más menester para convertir tu mal sofrimiento no consientes, poco aprovechará mi
los hombres en piedras. […] venida; y si, como prometiste, lo sufres, tú quedarás sana
Los ojos verdes, rasgados; las pestañas, luengas; las cejas, y sin deuda, y Calisto sin queja y pagado. […]
delgadas y alzadas; la nariz, mediana; la boca, pequeña; MELIBEA.— Más agradable me sería que rasgases mis carnes
los dientes, menudos y blancos; los labios, colorados y gro- y sacases mi corazón, que no traer esas palabras aquí.
sezuelos; el torno del rostro, poco más luengo que redondo; CELESTINA.— Sin te romper las vestiduras se lanzó en tu pecho
el pecho, alto; la redondeza y forma de las pequeñas tetas, el amor; no rasgaré yo tus carnes para le curar.
¿quién te la podrá figurar? ¡Que se despereza el hombre MELIBEA.— ¿Cómo dices que llaman a este mi dolor, que así se
cuando las mira! La tez lisa, lustrosa; el cuero suyo escure- ha enseñoreado en lo mejor de mi cuerpo?
ce la nieve; la color, mezclada, cual ella la escogió para sí. CELESTINA.— Amor dulce.
Editorial A NAYA MELIBEA.— Eso me declara qué es, que en solo oírlo me alegro.
Cabellos como el oro, lindos y resplandecientes, largos y ata- CELESTINA.— Es un fuego escondido, una agradable llaga, un
dos en una coleta. Ojos verdes y rasgados, pestañas largas, sabroso veneno, una dulce amargura, una delectable do-
cejas delgadas y altas, nariz mediana, boca pequeña, dientes lencia, un alegre tormento, una dulce y fiera herida, una
pequeños y blancos, labios gruesos y colorados, rostro alar- blanda muerte.
gado, pecho redondo, tez lustrosa y blanca. Editorial A NAYA
Esta descripción sí corresponde con el ideal de belleza de la Primero le dice que su llaga necesita cura, y alaba a Calisto.
época, en el que las mujeres eran rubias, de largos cabellos, Cuando Melibea se enfada al oír nombrar a Calisto, Celesti-
bocas sonrosadas y piel muy clara, así como bien proporcio- na le asegura que para curarse necesita algo de sufrimien-
nadas de cuerpo. to. A continuación da nombre a lo que siente Melibea: amor,
para terminar describiéndolo con una serie de metáforas y
Aparecen algunos recursos poéticos, por ejemplo un símil
contradicciones.
cuando compara sus cabellos con las madejas de oro que hilan
en Arabia, o cuando dice que su piel oscurece la nieve. Son, 5. ¿De qué recurso poético se vale Celestina para describir
además, sendas exageraciones o hipérboles. el amor?
2. Lee ahora la descripción que hace Pármeno de Celestina y Se vale de dos recursos, por un lado la metáfora, ya que va
sus oficios. Este la conoce bien porque su madre fue com- identificando el amor con diferentes cosas: fuego escondido,
pañera suya y él mismo trabajó para ella de niño. Teniendo agradable llaga, sabroso veneno, etc. Por otro se basa en las
en cuenta cómo la describe, ¿crees que le tiene aprecio? antítesis o contradicciones: la llaga es agradable, el veneno
sabroso, la amargura dulce, la dolencia delectable, el tormento
¿Por qué?
alegre, la herida es dulce y fiera y la muerte es blanda.
PÁRMENO.— Si entre cien mujeres va y alguno dice: «¡Puta vie-
ja!», sin ningún empacho luego vuelve la cabeza y respon- LA FACTORÍA DE TEXTOS
de con alegre cara. En los convites, en las fiestas, en las
bodas, en las cofradías, en los mortuorios, en todos los 1. Lee los primeros versos de este romance, busca su conti-
ayuntamientos de gentes, con ella pasan tiempo. Si pasa nuación en Internet y responde a las cuestiones siguientes:
por los perros, aquello suena su ladrido; si está cerca las Yo me era mora Moraima,
aves, otra cosa no cantan; si cerca los ganados, balando morilla de un bel catar;
lo pregonan; si cerca las bestias, rebuznando dicen «¡Puta cristiano vino a mi puerta,
vieja!». […] cuitada, por me engañar.
Ella tenía seis oficios, conviene a saber: labrandera, per- a) Busca en el diccionario las palabras bel, catar, cuitada,
fumera, maestra de hacer afeites y de hacer virgos, alca- algarabía, almejía, brial.
hueta y un poquito hechicera. […] ¿Quién te podrá decir lo
que esta vieja hacía? Y todo era burla y mentira.
b) Copia en tu cuaderno los verbos que aparecen en el
poema. ¿Qué tiempos verbales son los más abundan-
Editorial A NAYA
tes? ¿Predominan los verbos de acción? Señálalos.
No parece demostrarle mucho aprecio, ya que insiste en cali-
c) Analiza los elementos de la narración presentes en
ficarla de puta, diciendo además que esto a ella no le importa.
Cuando da cuenta de todos sus oficios termina diciendo y todo
este romance: narrador, personajes, tiempo y espacio.
era burla y mentira. Con todo esto entendemos que Pármeno d) Señala su estructura: planteamiento, nudo y desen-
no se fía en absoluto de ella. lace. ¿Hay un final cerrado? ¿Qué crees que le sucede
3. Copia en tu cuaderno todos los oficios de Celestina. a la mora Moraima después de abrir las puertas?
Dice que es labrandera, perfumera, maestra de hacer aceites e) Indica qué elementos típicos de los romances se dan
y virgos, alcahueta y hechicera. en este. Analiza también su métrica.

Lengua castellana y Literatura. 3.º ESO. Solucionario 73


8 MELIBEO SOY…

a) • bel: bello (adjetivo en desuso). b) ¡Oh muerte, cruel enemiga, que ni perdonas los culpados ni
• catar: mirar. absuelves los inocentes! Tan traidora eres, que nadie para
contigo tiene defensa. Amenazas para la vejez y llevas en la
• cuitada: afligida, desventurada.
mocedad. A unos matas por malicia y a otros por envidia.
• algarabía: lengua árabe. Aunque tardas, nunca olvidas. Sin ley y sin orden te riges.
• almejía: túnica o manto árabe que usaban también los Más razón había para que conservases los veinte años
cristianos. del hijo mozo que para que dejases los sesenta de la vieja
• brial: vestido de seda o tela rica que usaban las mujeres. madre. ¿Por qué volviste el derecho al revés? Yo estaba
harta de ser viva y él en edad de vivir. Perdóname porque
b) Era, vino, engañar, hablome, sabe, ábrasme, guarde,
así te trato, que no eres mala del todo, porque si con tus
abriré, sé, serás, soy, dejo, viene, abres, verás, matar,
obras causas los dolores, con ellas mismas los consuelas
oí, comenceme, levantar, vistiérame, hallando, fuérame,
llevando a quien dejas con quien llevas, lo que si conmigo
abrila.
haces, mucho te seré obligada.
Predomina el pretérito perfecto simple de indicativo alter-
Diego DE SAN PEDRO: Cárcel de amor.
nado con el presente simple de indicativo.
• ¿A qué subgénero pertenece cada texto? Resume
Hay abundancia de verbos de acción, como vino, ábrasme,
abriré, viene, abres, matar, comenceme, levantar, vistié-
brevemente el tema de cada fragmento.
rame, fuérame y abrila. • ¿Quiénes protagonizan el primer texto? ¿Por qué
c) El narrador es la propia mora Moraima, la protagonista del luchan? Averigua el nombre de la amada de Amadís.
romance, que cuenta lo que le sucedió en primera persona. • En el segundo texto, ¿quién se lamenta por una pér-
Los personajes son dicha mora, y el cristiano que pretende dida? ¿A quién crees que ha perdido?
hacerse pasar por su tío, el moro Mazote. El tiempo no
• Ambos son fragmentos de una novela. En el caso del tex-
aparece señalado en el romance, pero podemos situarlo
to a), se trata de una novela de caballerías, y su tema sería
en la época medieval de la Reconquista y las luchas entre
la lucha entre dos caballeros. El texto b) pertenece a una no-
árabes y cristianos. En cuanto al espacio, se desarrolla en
vela sentimental, y en este fragmento una mujer increpa a la
la puerta de la casa de la mora Moraima.
muerte lamentándose porque se ha llevado a un ser querido.
d) El planteamiento sería cuando un supuesto árabe llama a
• El primer texto está protagonizado por el caballero Ama-
las puertas de Moraima pidiéndole que le deje entrar.
dís de Gaula, y su contrincante, el caballero Angriote.
El nudo, cuando Moraima duda y el hombre la convence Ambos están luchando para que el otro reconozca que su
diciéndole que es su tío y que ha matado a un cristiano y amada es más bella que la de su rival. La amada de Amadís
por eso está siendo perseguido. se llama Oriana.
El desenlace ocurre cuando Moraima es engañada, se vis- • La que se lamenta es la madre de Leriano, el protagonista
te y acude a abrir la puerta. de la novela Cárcel de amor. Ha perdido a su hijo, ya que
El final queda totalmente abierto a la imaginación del lec- este se ha suicidado por el desdén de su amada.
tor. Podemos suponer que el cristiano se llevará cautiva a
la mora Moraima. ACTIVIDADES FINALES
e) Aparecen las características propias de los romances, Repasa lo que has aprendido
como el fragmentarismo y el final abierto, la concisión,
1. ¿Qué importantes acontecimientos históricos tuvieron
el dramatismo y la expresividad gracias a los diálogos, la
capacidad sugestiva…
lugar en la península ibérica durante el siglo XV?
Entre otros, el alumno podrá hacer alusión al reinado de los
En cuanto a su métrica, vemos que está formado por ver-
Reyes Católicos, a la creación del Tribunal de la Inquisición,
sos octosílabos y que riman en asonante los pares, que-
a la conquista de Granada, que supondrá el final de la Recon-
dando sueltos los impares.
quista, a la expulsión de los judíos o al descubrimiento de
2. Lee los siguientes fragmentos y responde: América en el año 1492.
a) Amadís se lo otorgó, pues que le placía, y luego se fueron 2. Completa en tu cuaderno:
ambos y tomaron sendas lanzas, las que más les conten-
taron, y alongándose uno de otro se dejaron venir contra
a) Los autores más importantes de la poesía de Cancio-
sí e hiriéronse de las lanzas muy gravemente; y Angriote nero fueron…
fue en tierra y el caballo sobre él, y Amadís, que pasaba, b) Los temas principales de las Coplas a la muerte de su
tropezó en el caballo de Angriote y fue caer con él de padre, de Jorge Manrique, son…
la otra parte, y un trozo de la lanza que por el escudo c) Los dos subgéneros narrativos más importantes del
le había entrado —con la fuerza de la caída— entrole
siglo XV fueron…
por el arnés y por la carne, mas no mucho; él se levantó
muy ligero […] y poniendo mano a la espada, se dejó ir d) Las danzas de la Muerte eran poemas escenificados
contra Angriote, que le vio con su espada ir en la mano, en los que…
y Angriote le dijo: e) La autoría de La Celestina ha sido controvertida
—Caballero, yo os tengo por buen mancebo y ruégoos que, porque…
antes que más mal recibáis, otorguéis ser más hermosa mi f) El argumento de La Celestina es…
amiga que la vuestra.
a) … el marqués de Santillana, Juan de Mena y Jorge Manrique.
—Callad —dijo Amadís—, que tal mentira nunca será por
b) … la brevedad de la vida, el desprecio de los bienes terre-
mi boca otorgada.
nales, el poder igualitario de la muerte, la importancia de
Garci RODRÍGUEZ DE MONTALVO: Amadís de Gaula la vida eterna, la existencia de una tercera vida de fama,
(tomado de Aventuras de los libros de caballerías, el elogio a su padre don Rodrigo y la aceptación cristiana
Editorial Akal). de la muerte.

74 Lengua castellana y Literatura. 3.º ESO. Solucionario


MELIBEO SOY… 8
c) … las novelas de caballerías y las novelas sentimentales. Muy de prisa se calzaba,
d) … la Muerte se personificaba para ir sacando a bailar a más de prisa se vestía;
todos los miembros de la sociedad, representando así que ya se va para la calle
todos somos iguales ante ella y todos vamos a morir. en donde su amor vivía.
«¡Ábreme la puerta, blanca,
e) … porque el propio Fernando de Rojas nos dice en la Carta ábreme la puerta, niña!».
del autor a un su amigo, que él no fue el autor del acto I, «¿Cómo te podré yo abrir
sino que se lo encontró y decidió continuarlo. si la ocasión no es venida?
f) … que Calisto se enamora de Melibea y para conseguir Mi padre no fue al palacio,
su amor pide ayuda a una vieja alcahueta, Celestina. Di- mi madre no está dormida».
chos amores llevarán a un fin desgraciado tanto a los jó- «Si no me abres esta noche,
venes amantes, como a sus criados y a la propia Celestina. ya no me abrirás, querida;
3. Explica en tu cuaderno los siguientes conceptos: Cancio- la Muerte me está buscando:
junto a ti vida sería».
nero, petrarquismo, amor cortés, mito, epopeya y fábula.
«Vete bajo la ventana
• Cancionero: colecciones de poemas en estilo cortesano, donde labraba y cosía,
compuestos para ser cantados, de tema alegórico, satírico te echaré cordón de seda
y sobre todo amoroso. Destacan el de Palacio, el de Stúñiga para que subas arriba,
o el de Baena. y si el cordón no alcanzare
• petrarquismo: corriente italiana encabezada por Frances- mis trenza añadiría».
co Petrarca, en la que destaca el culto a la belleza femenina, La fina seda se rompe;
junto a una naturaleza bucólica y el predominio del amor la Muerte que allí venía:
cortés. «Vamos, el enamorado,
• amor cortés: convención literaria de la época medieval que que la hora ya es cumplida».
presenta a la amada como un ser superior, divino, a la
a) ¿Cuál es el argumento del romance? ¿Qué significado
que el enamorado deber rendir vasallaje y sumisión.
tiene?
• mito: historia protagonizada por dioses, héroes o mons-
truos que da explicación a ciertos fenómenos de la natura- b) ¿Qué características típicas de su género se observan
leza o del origen del mundo. Destaca la mitología griega y en él?
romana. c) Explica qué función cumplen los diálogos. ¿Y las repe-
• epopeya: narración muy extensa en verso que relata las ticiones?
hazañas de un héroe.
d) En el poema hay una cierta identificación entre amor
• fábula: narración en verso o en prosa, cuyos protagonis- y muerte. ¿Dónde se observa?
tas generalmente son animales, y que sirve para enseñar
mediante una moraleja final. e) Recuerda la clasificación de los romances en cuanto a
4. Resume mediante un esquema cuáles fueron los poetas su temática. ¿Dentro de qué tipo de romance podría-
cultos más importantes del siglo XV y señala sus obras prin- mos encuadrar este? Justifica tu respuesta.
cipales. f) Analiza la métrica del romance.
En el siglo XV destacan los siguientes autores cultos: g) Escucha una versión del romance tecleando en You-
• Íñigo López de Mendoza, marqués de Santillana, reconoci- Tube «Amancio Prada» «Romance del enamorado y
do por su poesía de cancionero, sus serranillas y sus sone- la muerte».
tos fechos al itálico modo.
a) Un hombre sueña que la muerte va a ir a buscarle, y para
• Juan de Mena, que destacó especialmente por su obra La- escapar de ella huye a casa de su amada. Pero precisamen-
berinto de Fortuna. te cuando intenta subir con una cuerda a su ventana, esta
• Jorge Manrique, que además de poesía amorosa de cancio- se rompe, y el hombre cae y muere.
nero compuso su obra cumbre Coplas a la muerte de su Su significado es que no podemos escapar de la muerte
padre. cuando nos llega nuestra hora.
5. Lee el siguiente romance y responde: b) El fragmentarismo (no sabemos nada del antes de los
Un sueño soñaba anoche personajes), el dramatismo (gracias a los diálogos y a la
soñito del alma mía, primera persona), las repeticiones y los paralelismos, su
soñaba con mis amores capacidad de sugestión…
que en mis brazos los tenía. c) Los diálogos contribuyen a dotar al poema de mayor ex-
Vi entrar señora tan blanca presividad y dramatismo. Las repeticiones tienen la fun-
muy más que la nieve fría. ción de favorecer la memorización del poema.
«¿Por dónde has entrado, amor? d) El joven está soñando con sus amores cuando ve entrar a
¿Cómo has entrado, mi vida? una señora blanca y fría, a la que confunde con su amada.
Las puertas están cerradas, Un poco más adelante a su enamorada también la califica
ventanas y celosías». de blanca.
«No soy el amor, amante: e) Estaría dentro de los romances novelescos y líricos, ya que
la Muerte que Dios te envía». su temática es amorosa, de misterio…
«¡Ay, Muerte tan rigurosa,
déjame vivir un día!». f) Está formado por versos octosílabos que riman en aso-
«Un día no puede ser, nancia en los pares (í-a), quedando sueltos los impares.
una hora tienes de vida». g) Audición.

Lengua castellana y Literatura. 3.º ESO. Solucionario 75


8 MELIBEO SOY…

6. En el último acto de La Celestina, Pleberio, el padre de d) Posiblemente, ya que todos los personajes que se han de-
Melibea, se lamenta amargamente por la trágica pérdida jado llevar por las pasiones y el loco amor han terminado
de su hija. Lee este fragmento y resuelve las tareas: desastradamente. Calisto, Melibea, Celestina, Pármeno y
Sempronio, todos han muerto por dejarse llevar o bien por
¡Oh mi hija y mi bien todo! Crueldad sería que viva yo so- el amor o bien por la codicia.
bre ti; más dignos eran mis sesenta años de la sepultura
que tus veinte. […] ¡Oh duro corazón de padre! ¿Cómo no e) Tienen varios elementos en común, ya que la madre de
te quiebras de dolor, que ya quedas sin tu amada herede- Leriano también se queja contra la muerte y la increpa
ra? ¿Para quién edifiqué torres? ¿Para quién adquirí honras? amargamente, diciendo que se rige sin ley y sin orden. Se
¿Para quién planté árboles? ¿Para quién fabriqué navíos? […] asemejan mucho en la parte en la que la madre de Leriano
¡Oh vida de congojas llena, de miserias acompañada! ¡Oh dice más razón había para que conservases los veinte
mundo, mundo! […] Yo pensaba en mi más tierna edad que años del hijo mozo que para que dejases los sesenta de
eran tus hechos regidos por alguna orden; agora, visto el pro la vieja madre, y Pleberio a su vez se queja diciendo: más
y la contra de tus bienandanzas, me pareces un laberinto de dignos eran mis sesenta años de la sepultura que tus
errores, un desierto espantable, una morada de fieras, juego veinte. Ambos se lamentan de seguir vivos tras las muer-
de hombres que andan en corro, laguna llena de cieno, región tes de sus hijos, aunque quizá el planto de Pleberio sea más
llena de espinas, monte alto, campo pedregoso, prado lleno emotivo en sus increpaciones al mundo y al amor.
de serpientes, huerto florido y sin fruto, fuente de cuidados,
río de lágrimas, mar de miserias, trabajo sin provecho, dulce
Recuerda lo que ya sabías
ponzoña, vana esperanza, falsa alegría, verdadero dolor. […] 7. Explica cuál sería el registro lingüístico empleado por Ple-
¡Oh amor, amor! ¡Que no pensé que tenías fuerza ni poder berio en el planto que has leído en la actividad 6 y usa ex-
de matar a tus sujetos! […] ¿En qué pararon tus sirvientes presiones que aparezcan en el texto a modo de ejemplo.
y ministros? […] ¡Oh mi hija despedazada! ¿Por qué no que- Se trataría de un registro lingüístico culto y formal, propio
siste que estorvase tu muerte? ¿Por qué no hobiste lás tima de su condición de noble. Pese a su dolor, Pleberio emplea
de tu querida y amada madre? ¿Por qué te mostraste tan expresiones bien elaboradas, un léxico rico y cuidado y una
cruel con tu viejo padre? ¿Por qué me dejaste cuando yo sintaxis compleja, por ejemplo cuando se lamenta diciendo:
te había de dejar? ¿Por qué me dejaste triste y solo in hac ¡oh duro corazón de padre! ¿Cómo no te quiebras de dolor,
lachrimarum valle? que ya quedas sin tu amada heredera? ¡Oh vida de congojas
Biblioteca Didáctica A NAYA llena, de miserias acompañada! ¿Por qué me dejaste triste
y solo in hac lachrimarum valle? (aquí incluso emplea una
a) Compara la manera de afrontar la muerte de Plebe- expresión en latín).
rio con la de don Rodrigo en las Coplas de Manrique.
8. Indica en qué tiempo están las formas verbales del roman-
¿Cuál es más cristiana? Razona tu respuesta.
ce de la actividad 5.
b) Pleberio se queja contra el mundo y contra el amor. Aparecen sobre todo verbos en pasado: pretérito imperfecto
¿Crees que esa rebeldía es ya muestra de la nueva sen- de indicativo (soñaba, tenía, se calzaba, se vestía, vivía, la-
sibilidad renacentista? ¿Por qué? braba, cosía, venía), pretérito perfecto simple de indicativo
(vi, fue) y pretérito perfecto compuesto de indicativo (has
c) ¿Cómo se describe el mundo? ¿De qué recursos lite-
entrado).
rarios se vale Rojas?
Pero también abundan los verbos en presente, especialmente
d) Como recuerda Pleberio, el amor loco ha hecho que casi en los diálogos: están soy, envía, puede, tienes, se va, es,
todos los personajes de la obra tengan un fin trágico. abres, está, rompe, es.
¿Crees que aquí se encuentra el propósito moralizador Aparecen además algunos imperativos (déjame, ábreme,
de la obra? Justifica tu respuesta. vete, vamos), futuros (abrirás, echaré) y condicionales (se-
e) Compara este planto con el de la madre de Leriano, en ría, añadiría).
el fragmento que leíste de Cárcel de amor. ¿Qué tienen 9. Analiza los elementos de la narración de dicho romance, y
en común? ¿Cuál te parece más emotivo? divídelo en planteamiento, nudo y desenlace.
a) Es mucho más cristiana la visión de la muerte que nos El narrador es un narrador en primera persona, ya que es el
presenta Manrique en las Coplas, ya que en ellas hay una propio enamorado el que cuenta lo que le sucede. Los perso-
aceptación total de la muerte como algo natural y necesa- najes son el enamorado, su amada y la muerte. El espacio es
rio. Sin embargo en el planto de Pleberio no hay tal acep- la casa del enamorado y después la de su amada, y el tiempo
tación, el padre se rebela contra Dios y contra la muerte es la noche anterior a la narración.
por haberse llevado a su hija antes de tiempo. En cuanto a la estructura de la narración, el planteamiento
b) Sí, ya que en la mentalidad medieval no cabía tal rebeldía, sería cuando el enamorado está soñando y se le aparece la
la muerte se aceptaba como un designio de la voluntad muerte, que le concede tan solo una hora más de vida. El nudo,
divina. Pleberio, en cambio, increpa a la vida, al mundo y cuando el enamorado, intentando escapar de la muerte, se va
al amor como culpables de su desdicha. Considera que en a casa de su amada y esta le propone que trepe por un cordel
el mundo no hay orden sino que está regido por el capricho hasta su ventana, y el desenlace, cuando finalmente el cordel
y la falta de lógica. Esto es más renacentista que medieval, se rompe y la muerte llega a buscar al enamorado.
ya que en la Edad Media Dios era el centro de todo y había 10. Explica las semejanzas y diferencias que encuentres entre
que aceptar su voluntad sin quejas. la lírica tradicional medieval (jarchas, cantigas y villancicos)
c) Rojas describe el mundo basándose en las metáforas. Así, y el Romancero del siglo XV.
lo identifica, por ejemplo, con un laberinto de errores, un La lírica tradicional medieval estaba formada por poemas
desierto espantable, una morada de fieras, un juego de muy breves, normalmente sin rima o con rima asonante, don-
hombres que andan en corro, etc. Todas ellas son imágenes de el yo poético generalmente era una muchacha que expresa-
del caos y del desorden más absoluto. ba sus sentimientos amorosos hacia su amigo. En numerosas

76 Lengua castellana y Literatura. 3.º ESO. Solucionario


MELIBEO SOY… 8
ocasiones se dirigía a algún confidente, generalmente su ma- Siglo XV
dre, sus hermanas o amigas… Era una poesía sencilla, fácil Lírica
de memorizar, y muy expresiva. Estaba escrita en las dife-
rentes lenguas romances de la Península (mozárabe, gallego- • Por un lado tenemos la lírica popular, cuyo mayor repre-
portugués o castellano). Su temática, por tanto, era sobre todo sentante es el romance. De temática variada (romances
el amor, acompañado en muchas ocasiones de alusiones a la históricos, fronterizos, moriscos, novelescos), se trata de
naturaleza. Formalmente destacan en ellos las repeticiones, poemas no estróficos de versos octosílabos con rima aso-
anáforas y paralelismos. nante en los pares. Se basan en la expresividad, la conci-
sión, el fragmentarismo, las repeticiones, etc.
En cuanto a los romances, son poemas más extensos, que si-
guen una métrica fija: versos octosílabos con rima asonante • Lírica culta y cortesana: compuesta en la corte por autores
en los pares, quedando sueltos los impares. Si bien sigue apa- cultos, se basa en los presupuestos del amor cortés. Desta-
reciendo en ellos la temática amorosa, ahora predominan otro can autores como Juan de Mena (Laberinto de Fortuna),
tipo de asuntos: romances históricos, fronterizos, moriscos, el marqués de Santillana (serranillas, Sonetos fechos al
novelescos… Se caracterizan por tener también una gran ex- itálico modo) y Jorge Manrique (Coplas a la muerte de su
presividad, gracias a los diálogos, las llamadas de atención al padre).
oyente, etc. Además son muy concisos y fragmentarios, con Narrativa
una gran capacidad sugestiva. Al igual que en la lírica tradi- Destacan dos subgéneros:
cional, abundan en ellos las repeticiones para favorecer su
memorización. Están escritos en lengua castellana. • La novela sentimental, como Cárcel de amor, de Diego de
San Pedro.
11. Recuerda lo que era un juglar y lo que era un trovador. ¿En
• La novela de caballerías, como el Amadís de Gaula, de
el siglo XV continúan existiendo dichos oficios? Justifica tu
Garci Rodríguez de Montalvo.
respuesta.
Teatro
Los juglares eran los encargados de difundir la literatura oral
por los pueblos y las villas. Llevaban, por tanto, una vida am- • Continuación de los autos medievales.
bulante, y la mayoría de las veces no eran autores, sino simple- • Aparece un nuevo subgénero: las danzas de la Muerte, don-
mente transmisores de dicha literatura, cuyo recitado solían de la muerte iba sacando a bailar a diferentes personajes de
acompañar de música. todas las clases sociales.
Los trovadores, en cambio, sí eran los creadores de sus poe- • La obra clave del teatro del siglo XV es La Celestina, de
mas. Su lírica se acompañaba también de música y el entorno Fernando de Rojas, que relata los amores desgraciados
en el que desarrollaban su arte era el mundo de la corte. Su de Calisto y Melibea bajo la intercesión de la alcahueta
clase social era, pues, más elevada que la de los juglares. Celestina.
En el siglo XV ambas prácticas comienzan a decaer. Con el fin
de la Edad Media va desapareciendo el oficio de juglaría, al MIRA A TU ALREDEDOR Y…
igual que sucede con los trovadores, cuya poesía será susti-
tuida por la poesía de Cancionero. … ve más allá
12. Haz un esquema donde recojas las obras más importantes 1. Las danzas de la Muerte se relacionan muy estrechamente
de la lírica, el teatro y la narrativa de la Edad Media, inclu- con el arte de la danza. En algunos pueblos, como Verges,
yendo también el siglo XV. Puedes organizarlo por siglos y en Girona, se celebra cada Jueves Santo un ritual de dan-
señalar en él características principales, autores y obras. zas de la Muerte.
Siglos XI-XIV a) Investiga desde cuándo se celebra, en qué consiste y
Lírica qué simbología emplea.
• Lírica tradicional y popular: jarchas, cantigas y villanci- b) Teclea en YouTube «danzas de la Muerte» y ve algún
cos. De autor anónimo, se caracterizan por su sencillez y vídeo sobre la fiesta de Verges. ¿Qué sensaciones te
brevedad. Su temática suele ser el amor en boca de una produce?
muchacha enamorada. Se basan en repeticiones, parale- a) Esta danza se conserva desde la Edad Media, cuando se
lismos y anáforas. Escritos en los principales dialectos de asociaba a las epidemias de peste negra. La danza tiene
la Península. lugar de noche y forma parte de una procesión, en la que
• Lírica culta: lírica trovadoresca, en la línea del amor cortés. diez esqueletos recorren las calles al ritmo de un timbal,
Narrativa iluminando la escena con antorchas. Los esqueletos van
danzando y saltando, y portan elementos simbólicos como
• Poemas épicos y caballerescos. Cuentan en verso las haza-
guadañas, banderas, cuencos con cenizas, o un reloj sin
ñas de un héroe. Destaca el Cantar de Mio Cid, anónimo,
agujas que nos recuerda que la muerte puede llegar en
donde se narran las heroicidades de Rodrigo Díaz de Vivar,
cualquier momento.
el Cid Campeador.
• Mester de clerecía: autores cultos, normalmente clérigos, b) Respuesta libre.
que escribían una poesía narrativa, de temática religiosa, 2. La relación entre literatura y cine siempre es muy inte-
en cuaderna vía. Destacan Gonzalo de Berceo (Milagros de resante, en el caso de La Celestina muy especialmente.
Nuestra Señora) y el arcipreste de Hita (Libro de buen amor). Lee este fragmento de la escena amorosa entre Calisto y
• Prosa de autores cultos, como don Juan Manuel (El conde Melibea en el huerto, justo antes de su muerte, y realiza
Lucanor). las actividades:
Teatro MELIBEA.— ¡Oh sabrosa traición! ¡Oh dulce sobresalto! ¿Es mi
• Muy escaso en esta época. La única obra conservada es el señor de mi alma? ¿Es él? No lo puedo creer. ¿Dónde esta-
Auto de los Reyes Magos, anónima, de estilo sencillo y tono bas, luciente sol? ¿Dónde me tenías tu claridad escondida?
ingenuo. ¿Hacía rato que escuchabas? […]

Lengua castellana y Literatura. 3.º ESO. Solucionario 77


8 MELIBEO SOY…

CALISTO.— Pues, señora y gloria mía, si mi vida quieres, no cese católica, por lo que aceptan en las Cortes de Toledo de 1480 el
tu suave canto. […] confinamiento de los judíos en barrios apartados. El siguiente
MELIBEA.— ¿Qué quieres que cante, amor mío? […] Y pues tú, paso fue su definitiva expulsión.
señor, eres el dechado de cortesía y buena crianza, ¿cómo El 31 de marzo de 1492 Fernando e Isabel publicaron por me-
mandas a mi lengua hablar y no a tus manos que estén que- dio de las Chancillerías de Aragón y Castilla dos edictos muy
das? […] Deja estar estas mis ropas en su lugar, y si quieres similares por los que obligaban a sus súbditos de religión judía
ver si es el hábito de encima de seda o de paño, ¿para qué a abandonar sus reinos si antes del 31 de julio de ese año no se
me tocas en la camisa? Pues cierto que es de lienzo. Hol- habían bautizado. Asimismo se informaba a los cristianos de
guemos y burlemos de otros mil modos que yo te mostraré; que si ayudaban a algún hebreo a desobedecer las órdenes
no me destroces ni maltrates como sueles. ¿Qué provecho serían castigados con la pérdida de sus bienes. A los semitas
te trae dañar mis vestiduras? que decidieran no convertirse al cristianismo el edicto les per-
CALISTO.— El que quiere comer el ave quita primero las plumas. mitía sacar de la península sus bienes muebles, salvo caballos
[…] y mulas, metales preciosos y moneda española. La decisión
MELIBEA.— ¿Señor mío, quieres que mande a Lucrecia traer de la expulsión de los judíos no debe atribuirse únicamente
alguna colación? a los monarcas sino que la influencia e incluso presión de la
CALISTO.— No hay otra colación para mí sino tener tu cuerpo Inquisición jugaron un papel decisivo. Esta medida fue presen-
y belleza en mi poder. Comer y beber, donde quiera se da tada como necesaria pues era de vital importancia separar a
por dinero, en cada tiempo se puede haber, y cualquiera lo los judeoconversos de sus antiguos correligionarios para que
puede alcanzar; pero lo no vendible, lo que en toda la tierra su cristianización fuera verdadera y efectiva. Con el cumpli-
no hay igual que en este huerto, ¿cómo mandas que se me miento de este edicto concluyó la desintegración de la que fue
pase ningún momento que no goce? durante varios siglos la mayor comunidad judía de Europa.
LUCRECIA.— (Ya me duele a mí la cabeza de escuchar, y no a Parece que la cifra final de los hebreos que se negaron a cam-
ellos de hablar, ni los brazos de retozar, ni las bocas de be- biar de religión y que, por consiguiente, se vieron obligados
sar. ¡Andar! Ya callan; a tres me parece que va la vencida). a abandonar la península, fue de entre 50 000 y 70 000 perso-
CALISTO.— Jamás querría, señora, que amaneciese, según la nas. Durante el verano de 1492 se produjo dentro de los reinos
gloria y descanso que mi sentido recibe de la noble con- de Fernando e Isabel un importante movimiento forzoso de
versación de tus delicados miembros. población.
MELIBEA.— Señor, yo soy la que gozo, yo la que gano; tú, señor,
el que me haces con tu visitación incomparable merced. El destino de los expulsados fue diverso. Algunos optaron
por no abandonar la península y establecerse en los vecinos
Biblioteca Didáctica A NAYA
reinos de Portugal (donde el 3 % de la población era de religión
a) ¿Qué opinas del comportamiento de Calisto? ¿Se está judía) y Navarra, en los que su presencia era tolerada, aunque
conduciendo como un verdadero enamorado dentro por poco tiempo, pues en ambos lugares se seguirán los pasos
de las convenciones del amor cortés? dados por Isabel y Fernando. Otros destinos fueron Italia, el
Imperio Otomano o el norte de África.
b) Melibea ha perdido algo muy importante para una mujer
de su época: la honra. ¿Crees que eso puede explicar su Laura PÁRAMO DE VEGA: La historia
de las tres culturas.
suicidio tras la muerte de Calisto?
c) Ved este fragmento de la adaptación al cine, tecleando a) ¿Por qué comienzan a tomarse medidas represoras
en YouTube «La Celestina escena huerto». ¿Qué te contra los judíos? ¿Cuál fue la primera de ellas?
parece la adaptación? ¿Reflejan Calisto y Melibea el b) ¿Cuál era la condición imprescindible para permane-
mismo amor que en el texto? Razona tus respuestas. cer en España a partir de 1492?
a) El alumno puede señalar que Calisto no está respetan- c) ¿Quiénes eran los semitas? Investiga en la red por qué
do las convenciones del amor cortés, ya que este era un recibían este nombre.
amor de tipo platónico, en cambio Calisto tiene mucha pri-
sa por consumar carnalmente su amor con Melibea. d) ¿Podían llevarse los judíos sus posesiones cuando se
marcharan de España?
b) Es una posible explicación, ya que al haber quedado des-
honrada (y siendo dicha deshonra conocida por varios ha- e) ¿Cuál fue la principal razón alegada para llevar a cabo
bitantes de la ciudad), pierde su honor y el de su familia, la expulsión? ¿Quién la defendía?
y no le sería ya posible encontrar marido; seguramente su f) ¿Cuál fue el destino de los judíos que se marcharon?
única salida fuera ingresar en un convento.
g) Investiga la relación entre este acontecimiento y la ex-
c) Respuestas libres.
pansión y pervivencia de la lengua sefardí en diversos
… encuentra la clave lugares del mundo.
1. Durante el reinado de los Reyes Católicos se produjo uno h) En la actualidad, ¿en qué países del mundo se dan
de los acontecimientos decisivos de la historia de España: problemas similares de intolerancia religiosa o choque
la expulsión de los judíos. Lee el siguiente texto sobre el entre diferentes creencias? Investígalo y exponlo ante
tema y responde a las cuestiones que se plantean: tus compañeros.
La expulsión de los judíos de España a) Porque los Reyes Católicos opinan que el contacto con los
judíos es perjudicial para la fe católica. La primera medi-
El reinado de los Reyes Católicos fue decisivo tanto para los da fue tomada en 1480 cuando se decidió confinarlos en
judíos como para los conversos. Sabedores de su importancia barrios apartados.
económica para su monarquía, el matrimonio real mostró ac-
titudes protectoras hacia la minoría hebrea. Sin embargo, los b) Bautizarse.
monarcas tendrán la opinión de que el contacto entre judíos c) Los semitas son los judíos. Se les llamaba así porque eran
y cristianos (ya sean nuevos o viejos) es perjudicial para la fe los descendientes de Sem, primer hijo de Noé.

78 Lengua castellana y Literatura. 3.º ESO. Solucionario


MELIBEO SOY… 8
d) Podían llevarse sus muebles, pero no sus caballos ni mulas el castellano que se hablaba aquí en dicha época. Esta
o metales preciosos y moneda española. lengua se ha conservado durante siglos sin sufrir apenas
e) Decían que era necesario separar a los conversos de los modificaciones, por lo que hoy en día podemos encontrar
judíos, para que su cristianización fuera auténtica y defi- comunidades judías diseminadas por todo el mundo que
nitiva. Defendió la expulsión el tribunal de la Inquisición. aún hablan el castellano sefardí del siglo XV.
f) Algunos se quedaron un tiempo en Portugal y Navarra, otros h) Respuesta libre. Los alumnos pueden hablar, por ejem-
marcharon a Italia, al norte de África o al Imperio otomano. plo, del problema israelí-palestino, o de la persecución de
g) El hecho de que los judíos se vieran obligados a abando- cristianos por parte de radicales musulmanes en algunos
nar la Península hizo que se llevaran consigo su lengua, países árabes.

Lengua castellana y Literatura. 3.º ESO. Solucionario 79


9 COGED DE VUESTRA ALEGRE PRIMAVERA…

APERTURA DE UNIDAD 5. En la tradición del amor cortés, el yo poético se ve a sí


mismo como un vasallo de la amada, mujer idealizada y
1. Rafael Alberti (1902-1999) es uno de nuestros escritores
elevada, a la que el enamorado somete su voluntad. ¿Está
contemporáneos más sobresalientes. En la composición
presente esta tradición en el soneto de Garcilaso que aca-
anterior, muestra su admiración hacia el poeta renacen-
bamos de leer? ¿Por qué?
tista Garcilaso de la Vega, resaltando sobre todo dos as-
pectos: ¿cuáles son? Sí. El poeta le dedica toda su vida, la convierte en fin de su
existencia.
Su condición de caballero sobre todo. Pero también habla de
su poesía (¡Qué dulce oírle!). Vocabulario
2. De Garcilaso dice Alberti que «buen caballero era»… Pero 6. Observa las siguientes palabras y expresiones del tex-
¿sabes en qué consistía ser un caballero en el Renacimien- to: gesto (verso 1), me guardo (verso 4), puesto (verso 5).
to? Investiga sobre este particular y anota las cualidades ¿Qué sentido tienen en el soneto? Compara sus signifi-
que, según el código de conducta de la época, debía tener cados con los que les damos en la actualidad.
un hombre para ser considerado un buen caballero. Gesto: rostro; en la actualidad se utiliza con el sentido de mueca
Patriotismo y lealtad hacia su rey. Valentía y cortesía. Debía o ademán. Me guardo: me escondo; en la actualidad el verbo
dominar el ejercicio de la guerra pero también debía tener una guardar no suele utilizarse con sentido reflexivo y casi siempre
formación pulida en las doctrinas humanísticas. se emplea con el sentido de recoger. Puesto: dedicado; en la
3. En el Centro Virtual Cervantes (cvc.cervantes.es) puedes actualidad se emplea a veces con un sentido semejante en pe-
rífrasis verbal (me he puesto a trabajar) pero no es sinónimo;
encontrar este poema junto a otros destinados a homena-
fuera de perífrasis, el verbo poner se utiliza casi siempre para
jear a Garcilaso. Teclea en su buscador «Homenaje poético referirnos a colocar o dejar algo en una posición determinada.
a Garcilaso». Visita la página, escoge uno de los poemas
que encontrarás ahí, cópialo en tu cuaderno y recítalo en 7. ¿Con qué sentido se utiliza la palabra hábito en el verso 11?
clase para todos tus compañeros. Se emplea con doble sentido: costumbre y ropa de una orden
religiosa.
Respuesta libre.
8. En el último verso, el yo poético dice «por vos muero».
4. Realiza una línea del tiempo con todos los autores que has
¿Qué sentido tiene aquí el verbo morir?
encontrado en la página de la actividad anterior y sitúa
en ella a todos los poetas que aparecen en el homenaje. Amar desesperadamente algo inalcanzable y sufrir por ello.
Es el sentido habitual en el amor cortés.
Respuesta libre.
Investigación y redacción
VIVE LA LECTURA
9. Como has visto en la biografía de Garcilaso de la Vega,
Comprensión lectora este poeta estuvo muy enamorado de una mujer llamada
1. En el primer cuarteto, el poeta dice que su amada es la Beatriz de Sá. Investiga sobre esta historia y escribe un
verdadera autora de sus versos, puesto que ella los ins- resumen.
pira. Explica esta afirmación teniendo en cuenta el con- Respuesta libre.
tenido de dicha estrofa. 10. Garcilaso fue un gran admirador del poeta italiano Fran-
El poeta entiende que escribir es leer los versos que ella le cesco Petrarca, quien abrió una corriente (el petrarquismo)
inspira. No obstante, esa tarea la realiza en absoluta soledad que seguirían muchos escritores europeos a partir del siglo
y ocultamiento. XV. Busca la información necesaria sobre Petrarca para con-
2. El yo poético manifiesta su absoluta incondicionalidad ha- testar a las siguientes preguntas:
cia la amada. Localiza las frases o expresiones del poema a) ¿En qué época vivió y desarrolló su obra?
en las que se manifieste dicho sentimiento. b) ¿Cuál es la temática del Cancionero?
Especialmente en los dos tercetos. Yo no nací sino para que- c) ¿Quién era Laura?
reros […] por vos nací, por vos tengo la vida, por vos he de
morir, y por vos muero. a) Nació en 1304 y murió en 1374. Sería uno de los iniciadores
del Renacimiento italiano, que se desarrolló antes que el
3. Además, el afecto está expresado como si se tratase de español.
una religión: la «religión del amor», esa es la fe que profe- b) En el Cancionero Petrarca reflexiona sobre el sentimiento
sa el amante. Justifica esta afirmación con citas del texto. amoroso, y presenta a la amada como un ser idealizado
Aunque no cabe en mí cuando en vos veo, de tanto bien lo aunque distante, que desdeña al poeta amante.
que no entiendo creo. c) Laura es la amada idealizada de los poemas de Petrarca.
4. El motivo temático de la «religión del amor» estaba ya pre-
sente en la centuria anterior. ¿Recuerdas la declaración LITERATURA
«Melibeo soy, en Melibea creo y a Melibea adoro»? ¿Qué 1. Investiga y responde:
personaje hacía esta manifestación? ¿En qué obra? ¿Cómo a) ¿En qué año se expulsó a los judíos? ¿Qué otros he-
relacionas esas palabras con los versos de Garcilaso? chos ocurrieron ese año?
Calisto en La Celestina. El amante entiende que la amada es b) ¿Qué corriente cultural asentó con su obra Erasmo de
el objeto de su vida y en ella reside todo lo que anhela. Las
Róterdam? ¿En qué consistía?
cualidades de la amada son tales que incluso están por encima
de su comprensión, pero el poeta se declara incondicional de c) ¿Qué novedades introducía en el cristianismo la Re-
su fe (su amor). forma protestante de Martín Lutero?

80 Lengua castellana y Literatura. 3.º ESO. Solucionario


COGED DE VUESTRA ALEGRE PRIMAVERA… 9
d) ¿Durante qué años se extiende el reinado de Carlos I? 4. Investiga sobre la película El club de los poetas muertos,
¿Y el de Felipe II? dirigida en 1989 por Tom Schulman y protagonizada por
e) ¿Cuándo se inició la Contrarreforma? ¿Con qué acon- Robin Williams. Escribe un resumen de su argumento en
tecimiento histórico? tu cuaderno y, a continuación, señala qué tópico literario
importantísimo se trata en ella.
f) El Renacimiento italiano se desarrolló antes que el
La película narra la historia de un profesor de literatura que
español. ¿En qué época fue? ¿Qué figuras literarias
ejerce en un colegio masculino de estrictas normas. Los mé-
y artísticas destacaron en ese periodo? todos poco ortodoxos del profesor no encajan en las tradicio-
g) Averigua qué era el Índice de libros prohibidos y si al- nes de la institución pero ayudan a sus alumnos a descubrir
guna de las obras del periodo que ahora estudias figu- su propio potencial creador y la importancia de exprimir lo
raban en él. mejor de la vida. El tópico literario que aborda esta película
es el Carpe diem.
a) En 1492. Unificación territorial por parte de los Reyes
Católicos. Descubrimiento de América. Publicación de la 5. Investiga sobre el mito de Orfeo y Eurídice y resume su
Gramática de Nebrija. contenido. A la luz del mito, ¿cómo interpretas el verso 8
b) Humanismo. Corriente de pensamiento que considera al del soneto que acabas de leer?
ser humano como medida de todas las cosas (antropocen- La ninfa Eurídice era la esposa del músico Orfeo. Eurídice
trismo) y capaz de desarrollar todo su potencial, tanto el murió por una mordedura de serpiente y Orfeo, desconsolado,
físico como el espiritual, por encima de sus limitaciones, va a buscarla al Hades. Su canto es tan bello y conmovedor
incluso por encima de su propia condición social. que convence a Caronte para que le deje pasar y a Perséfone
c) Buscaban la renovación de ciertos hábitos desarrollados y a Hades para que le permitan llevarse a su amada. Ellos
por la Iglesia católica, con la finalidad de devolver el cris- se lo conceden con la única condición de que no se vuelva a
tianismo a sus valores primitivos. Cuestionaban el domi- mirarla hasta que salgan nuevamente a la luz del sol. Orfeo
nio del Papa sobre toda la cristiandad. cumple esta condición hasta que están a punto de abando-
nar el Hades, pero en ese momento tiene la sospecha de que
d) Carlos I: 1516-1556. Felipe II: 1556-1598. Perséfone puede haberle engañado, así que mira hacia atrás
e) Con el Concilio de Trento, que se desarrolló entre 1545 para ver si es realmente Eurídice quien le sigue. Así la pierde
y 1563. definitivamente. En el verso 8 del soneto la cita de los reinos
del espanto es una referencia al Hades.
f) Desde finales del XIV y durante el siglo XV. Como inicia-
dor literario podemos citar a Petrarca (1304-1374). Es 6. ¿Qué elementos característicos de la temática renacentis-
muy importante la figura del filósofo político Maquiavelo ta podrías comentar en este poema? Señala si encuentras
(1469-1527). En el arte destacan Miguel Ángel (1475-1564) epítetos y explica su valor.
y Leonardo da Vinci (1452-1519).
El tratamiento del amor desde la perspectiva petrarquista (la
g) Era una lista publicada por la Inquisición española que amada esquiva) y la alusión a motivos de la mitología clási-
incluía aquellos libros cuya lectura estaba prohibida en el ca. El mito clásico sirve al poeta de ejemplo para tratar el
territorio nacional. Su primera edición es de 1551, aunque asunto del poema. Como epítetos pueden citarse fieros, fríos,
luego se fue corrigiendo en ediciones sucesivas. En 1564, y trabajosa.
el Papa promulgaría otro Índice (Index librorum prohi-
7. Haz un resumen del contenido del soneto XIII y, aten-
bitorum) que afectaría a toda la cristiandad. De las obras
estudiadas en esta unidad, pueden citarse como lecturas
diendo a este, diferencia las partes en que podrías dividir
prohibidas la obra de Erasmo de Róterdam y el Lazarillo este texto.
de Tormes. Los brazos de Dafne se transformaban en ramas, sus cabellos
en hojas, sus piernas en corteza y sus pies en raíces. Apolo
2. Fray Luis de León fue un gran defensor de la lengua cas-
llora y riega con sus lágrimas el árbol, de este modo su pro-
tellana. Fíjate en esta afirmación suya y explícala después: pio llanto alimenta la razón de su dolor. Estructuración en
Assí que no piensen porque veen romance, que es poca estima partes: los dos cuartetos describen el momento de la trans-
lo que se dize; mas al revés, viendo lo que se dize, juzguen que formación; el primer terceto se refiere al llanto de Apolo; el
puede ser de mucha estima lo que se escrive en romance, y segundo terceto contiene la reflexión final del yo poético, que
no desprecien por la lengua las cosas, sino por ellas estimen aclara la relación que existe entre el sentimiento expresado y
la lengua. el mito que le ha servido de ejemplo para expresarlo.
En el siglo XVI aún existía el prejuicio de que los temas impor- 8. ¿De qué se lamenta el poeta al final del soneto XIII? ¿Hay
tantes y elevados se trataban en latín (por escrito) y que el algún hecho paradójico en el poema? Explícalo.
romance era solo para los asuntos menores. Fray Luis desea
Se lamenta, en efecto, de una gran contradicción: el llanto
desmontar este prejuicio y demostrar que la lengua romance
aumenta el motivo que lo provoca, es decir, mientras mayor
es vehículo perfecto para abordar toda clase de temas.
es el llanto, mayores son las razones para llorar.
3. Investiga sobre las figuras de Juan de Valdés y Juan Bos- 9. Redacta una descripción de la mujer a la que se dirige el
cán. Explica qué importancia tuvieron para la lengua y la yo poético en el soneto XXIII.
literatura españolas del Renacimiento.
Es una mujer joven, de largos cabellos rubios, de piel blanca
Juan de Valdés fue uno de los más importantes prosistas del y mejillas sonrosadas.
Renacimiento español. Destaca por su obra Diálogo de la len-
gua, en la que reflexiona sobre nuestro idioma, depura sus 10. Localiza y explica las metáforas utilizadas para describir
normas y presenta un ideal de estilo sobrio y cuidado. Juan a la dama de dicho soneto.
Boscán fue un poeta renacentista, amigo de Garcilaso de la Los colores de su rostro se identifican con la rosa y la azuce-
Vega, que tuvo una importancia decisiva en la introducción de na. La hermosa cumbre es la cabeza de la dama, que quedará
las formas métricas italianas en la poesía española de la época. cubierta de nieve, es decir, de canas.

Lengua castellana y Literatura. 3.º ESO. Solucionario 81


9 COGED DE VUESTRA ALEGRE PRIMAVERA…

11. Siguiendo con el mismo texto, ¿qué simboliza la primave- San Juan de la Cruz (1542-1591). Nació en Fontiveros (Ávila).
ra? Justo después, se alude al invierno, que también tiene Ingresó en los carmelitas y emprendió junto a santa Teresa
un tratamiento simbólico. ¿Cuál es? ¿En qué elementos de Jesús la reforma de esa orden. Esta tarea le supuso varios
problemas: estuvo encarcelado durante ocho meses en Toledo
del texto podrías justificar dicha alusión al invierno?
y además fue perseguido por varios de sus propios compañe-
La juventud. El invierno simboliza la vejez. Se alude al invier- ros, sin embargo llegó a ser prior de varios conventos. Las
no cuando se habla de la cumbre cubierta de nieve o del viento obras que le dan su importancia en el ámbito de la literatura
helado que marchitará la rosa. son sus poemas Cántico espiritual, Llama de amor viva y
12. Investiga sobre los orígenes del tópico carpe diem. ¿Qué Noche oscura del alma.
poetas lo habían utilizado en épocas anteriores? 18. ¿Cuál es la aspiración del poeta en el fragmento de la
Esta expresión procede del poeta Horacio que la había utiliza- Oda a la vida retirada? Cita los versos en que se ponga
do en una oda. Cuatro siglos después el poeta Ausonio acude de manifiesto.
al mismo concepto mediante el tópico collige, virgo, rosas. Liberarse de las cosas mundanas (la gloria y el dinero) que
13. Haz el análisis métrico de la estancia en la que Nemoroso arrebatan la verdadera felicidad. Un no rompido sueño […]
llora la muerte de su amada Elisa. no quiero ver el ceño / vanamente severo / de a quien la san-
gre ensalza o el dinero.
11A, 11B, 11C, 11B, 11A, 11C, 7c, 7d, 7d, 11E, 11E, 11F, 7e, 11F.
19. Analiza la métrica de las estrofas de la Noche oscura del
14. Busca en un diccionario la palabra nemoroso y relaciona alma que acabas de leer. ¿Qué tipo de composición se
su significado con el hecho de que el pastor se llame así. utiliza?
Perteneciente al bosque. Cubierto de bosques. El nombre del La lira.
personaje se relaciona con el espacio en que se ambienta la
égloga. 20. La poesía religiosa utiliza temas propios del Renacimiento
para hablar de asuntos religiosos. Explica esta afirmación
15. Investiga y elabora un breve resumen con el argumento basándote en los dos fragmentos que acabas de leer.
de las églogas de Garcilaso.
El fragmento de la Oda a la vida retirada de Fray Luis nos
En la Égloga I nos encontramos con dos pastores que lloran presenta un espacio idílico de la naturaleza donde el poeta
penas de amor: Salicio lamenta el distanciamiento de Galatea. pueda alejarse de las cosas mundanas que le impiden ser feliz.
Nemoroso llora la muerte de Elisa. Este poema puede interpretarse como una búsqueda de la paz
En la Égloga II se dan cita dos temas: el sufrimiento por amor espiritual. El fragmento de la Noche oscura del alma presenta
y la alabanza de la vida heroica y militar. Albanio se lamenta el motivo amoroso, el encuentro de los amantes en medio de la
de los desdenes de su amada Camila, quien llega a hacerle noche es la máxima aspiración de la amada. La interpretación
enloquecer de dolor con su rechazo. Presa de la neurosis, Sali- religiosa de este poema nos remite a la búsqueda del alma, que
cio y Nemoroso se ven obligados a reducirle. Nemoroso narra ansía encontrar a Dios y unirse a él.
entonces una historia épica sobre la carrera del duque de Alba. 21. ¿Qué rasgos de la novela picaresca puedes comentar en
La dureza de la vida militar se presenta como el mejor antídoto el fragmento?
contra las locuras del amor herido.
El relato en primera persona, con narrador personaje (pro-
En la Égloga III, cuatro ninfas del Tajo bordan tapices que tagonista). El realismo. La baja extracción social de los
representan escenas de tragedias amorosas. Tres de ellas se personajes. El aprendizaje del protagonista: la lección de la
refieren a historias de la mitología clásica, pero la última es desconfianza para saber salir adelante en un mundo hostil.
nuevamente la historia de Elisa y Nemoroso.
22. ¿Cuál es la enseñanza que Lázaro aprende con ese epi-
16. Una de las canciones de Garcilaso es la Oda a la flor de sodio?
Gnido. En ella, el poeta utilizó una composición métrica A no confiar en nadie, pues está solo en el mundo.
que acabó conociéndose como lira, debido a que en la
23. ¿Qué rasgos diferencian al escudero de los dos amos que
primera estrofa Garcilaso hablaba de ese instrumento
Lázaro había tenido anteriormente? ¿Qué comparación
musical. Haz el análisis métrico:
hace el protagonista entre ellos?
Si de mi baja lira
Es bondadoso y educado, y trata bien a Lázaro. No le da de
tanto pudiese el son, que en un momento
comer porque no tiene nada que darle, pero Lázaro ve la di-
aplacase la ira
ferencia que hay entre él y los dos amos anteriores, quienes
del animoso viento
sí tenían, pero no le daban nada por avaricia y mezquindad.
y la furia del mar y el movimiento…
24. ¿Cuál es el único defecto que Lázaro encuentra en el
7a, 11B, 7a, 7b, 11B.
escudero? ¿Qué opinión le merece este personaje al pro-
17. Investiga sobre las figuras de fray Luis de León y san Juan tagonista?
de la Cruz y resume sus biografías. En el caso de fray Su presunción y el orgullo con que se comportaba, pese a es-
Luis, haz una breve relación de las obras que escribió y el tar muerto de hambre. Pese a ello, Lázaro le tiene lástima y
género al que pertenecen. aprecio y afirma que le gustaría servir a alguien como él antes
Fray Luis de León (1527-1591) nació en Belmonte (Cuenca). que a cualquier otro.
Pertenecía a una familia con antepasados judíos. Ingresó en 25. Un motivo temático muy importante en la novela picares-
los agustinos. Estudió en Salamanca. Durante cinco años
ca es el contraste entre la apariencia y la realidad. Obser-
estuvo preso por la Inquisición por su defensa de la versión
va cómo se trata este tema en los episodios del escudero
original de la Biblia. También fue acusado de traducir el Can-
tar de los cantares al castellano. Obra poética: Oda a la vida y del buldero. ¿Qué intenta aparentar cada uno de ellos?
retirada, Noche serena, A Francisco de Salinas, Profecía del ¿Por qué?
Tajo. Obras en prosa: La perfecta casada, De los nombres de El escudero pretende aparentar riqueza y esconder la nece-
Cristo, Exposición del libro de Job. sidad que está pasando por puro orgullo y presunción. El

82 Lengua castellana y Literatura. 3.º ESO. Solucionario


COGED DE VUESTRA ALEGRE PRIMAVERA… 9
buldero era un estafador que intentaba agradar a sus víctimas Al descubrir la llave que Lázaro tenía en la boca y ver que era
para luego poder engañarlas. la del arcón.
26. ¿De qué recursos se valía el buldero para engañar a sus 4. Atendiendo al desarrollo del episodio, ¿qué partes dife-
víctimas? renciarías en el texto?
Hacía regalos a los clérigos para ponerlos a su favor y logran En el primer párrafo se narra la mala fortuna de Lázaro, al co-
que convenciesen a los feligreses a tomar la bula. También era locársele la llave en la boca y provocar el sonido que llamaría
muy observador y así veía con quiénes podía fingir cultura la atención del clérigo.
y formación, y con quiénes debía ser prudente para que no
En los párrafos segundo y tercero se narra cómo el clérigo
sospechasen su falta de conocimientos.
cree haber dado con la culebra y se dispone a matarla, para
27. En el fragmento del Tratado VII hay ciertas frases que descubrir al final que es Lázaro quien está allí.
aluden a la prosperidad que ha alcanzado Lázaro. Loca- En los dos últimos párrafos, el clérigo expulsa a Lázaro de
lízalas y anótalas. su casa.
Tengo en mi señor arcipreste todo favor y ayuda. Y siempre Análisis de la forma y el contenido. Las novelas picarescas
en el año le da, en veces, al pie de un carga de trigo: por
tienen un narrador autobiográfico. Además, como todo texto
las Pascuas, su carne; y cuando el par de los bodigos, las
calzas viejas que deja […]. Los domingos y las fiestas casi
narrativo, insertan otros tipos de discurso, como el diálogo
todas las comíamos en su casa. o la descripción.
28. Observa la intervención del arcipreste al final del último 5. ¿En qué elementos del texto puede apreciarse el tipo de
texto. La expresión «lo que le toca a una persona» solía narrador?
emplearse en el Siglo de Oro para referirse a la honra, lo En el uso de la primera persona del singular y los verbos en
que más debía importar a alguien. Sin embargo, el arci- pasado.
preste la usa con otro sentido. ¿Qué es, según él, lo que 6. Señala los momentos en que se introducen el diálogo y la
debe importar a Lázaro? descripción, y comenta sus rasgos.
Su provecho, la comodidad que ahora existe en su vida, la Podemos hablar de descripción en el primer párrafo, cuando
posición social que ha alcanzado. el narrador se refiere al sonido de su respiración al rozar la
29. A partir de los fragmentos del Lazarillo que has leído, llave que estaba en su boca. Obsérvense las estructuras mo-
caracteriza al personaje: ¿Es inteligente? ¿Es compasivo dales: de tal manera y postura que el aire que yo durmiendo
o cruel? ¿Qué diferencias hay entre el Lázaro niño y el echaba salía por lo hueco de la llave; de tal manera que el
adulto? Justifica tus respuestas. sobresaltado de mi amo lo oyó. El diálogo se introduce al final
del texto, en la intervención del clérigo. No obstante, también
Es inteligente, pues aprende rápido y se las arregla para so- antes se ha empleado este tipo de discurso para exponer los
brevivir. Es compasivo, como queda bien demostrado en el pensamientos del cura: Debió de decir el cruel cazador: El ra-
episodio del escudero, a quien Lázaro mantiene, mendiga tón y culebra que me daban guerra y me comían mi hacienda
para él aunque para ello él mismo deba quedarse con hambre. he hallado. Se utiliza el estilo directo. Obsérvense también los
El Lázaro niño era inocente, pero perdió esa inocencia con el verba dicendi: debió de decir, díjome.
paso de los años y las vivencias. Se convierte en un hombre
escéptico que conoce bien cuáles son las diferencias entre la Conclusiones. El pícaro es un personaje que aprende a so-
apariencia y la realidad de la vida, y decide vivir lo más feliz brevivir con su astucia.
y cómodamente que pueda con ese aprendizaje. 7. ¿En qué momentos del texto se alude a ese aprendizaje?
COMENTARIO DE TEXTO Al final, con la alusión a su pasado como mozo de ciego.

Localización. El segundo amo de Lázaro es un clérigo ava- EL JARDÍN DE LA LITERATURA


ro que esconde bollos de pan en un arca, bajo llave. Lázaro,
1. La obra de santa Teresa de Jesús (1515-1582) tuvo una gran
hambriento, se hace con una copia de esta. El clérigo cree al
importancia en la literatura mística de la segunda mitad
principio que los ratones se comen su pan y, después, que se
del siglo XVI. Lee este poema suyo y contesta después a
trata de una culebra.
las preguntas:
1. ¿Qué nos da a entender la mención al ciego que se hace
Vivo sin vivir en mí,
al final? y tan alta vida espero,
Que los mozos de ciego (como lo ha sido el propio Lázaro) que muero porque no muero.
tienen una vida tan dura que aprenden muchos recursos para
Vivo ya fuera de mí,
sobrevivir, no siempre lícitos.
después que muero de amor;
2. Señala las características de la novela picaresca que ob- porque vivo en el Señor,
serves en este fragmento. que me quiso para sí:
El narrador personaje (protagonista) que cuenta los hechos cuando el corazón le di
en primera persona, de manera autobiográfica. El relato de puso en él este letrero,
los ardides que el personaje se ve obligado a poner en práctica que muero porque no muero.
para poder salir adelante, ardides que tienen que ver con su Esta divina prisión,
aprendizaje de niño hambriento en un medio hostil. del amor en que yo vivo,
ha hecho a Dios mi cautivo,
Tema y estructura. Lázaro se las ingenia para saciar su ham-
y libre mi corazón;
bre, pero el clérigo le acaba descubriendo.
y causa en mí tal pasión
3. ¿Cómo comprende el avaro que es Lázaro quien ha esta- ver a Dios mi prisionero,
do comiendo el pan? que muero porque no muero.

Lengua castellana y Literatura. 3.º ESO. Solucionario 83


9 COGED DE VUESTRA ALEGRE PRIMAVERA…

¡Ay, qué larga es esta vida! fuese un enemigo poderoso y bello, que somete al
¡Qué duros estos destierros, amante. Localiza en el texto dos momentos en que se
esta cárcel, estos hierros aprecia ese enfoque.
en que el alma está metida!
Solo esperar la salida b) También es habitual en estas corrientes poéticas el
me causa dolor tan fiero, tópico de los ojos de la amada, que enamoran inmedia-
que muero porque no muero. tamente al poeta. Explica la presencia de este tópico
¡Ay, qué vida tan amarga en este soneto.
do no se goza el Señor! c) El yo poético habla de una «dulce flecha», del mal «que
Porque si es dulce el amor, sufro no cansado» y afirma que «en ciego engaño de es-
no lo es la esperanza larga: peranza muero». ¿Cómo interpretas estas expresiones?
quíteme Dios esta carga,
más pesada que el acero, a) Versos 1 y 2: unos bellos ojos que hirieron con dulce fle-
que muero porque no muero. cha un corazón cuitado. Verso 12: Yo veo ya perderse los
despojos.
Solo con la confianza
vivo de que he de morir, b) Se presenta ya desde el principio: el amante dice haber
porque muriendo el vivir caído rendido y herido con la mera contemplación de
me asegura mi esperanza; esos ojos que lo hirieron (primer cuarteto). Después, esos
muerte do el vivir se alcanza, ojos están ya presentes en todo el texto, como promesas
no te tardes, que te espero, que encienden el deseo del amante pero que nunca llegan
que muero porque no muero. a cumplirse.
a) Recuerda lo que has estudiado sobre los poetas as- c) La dulce f lecha nos remite al tópico del dios que lan-
céticos y místicos, y delimita el tema central de este za flechas de amor provocando heridas definitivas en
texto. los amantes. Es un arma que hiere, pero es dulce, por-
que es de amor. Que sufro no cansado es una expresión
b) ¿Qué sentido tienen los versos «vivo sin vivir en mí» y que alude a la obstinación del amante por aquello que le
«que muero porque no muero»? ¿Qué recurso estilís- provoca dolor. Es un tópico en la literatura que también
tico se emplea en ellos? encontramos al hablar de la dulce flecha. En el mismo
c) ¿Qué elementos propios del tema amoroso se em- sentido podemos interpretar la frase en ciego engaño de
plean en este poema? esperanza muero, es decir, se presenta una vez más la
contradicción del sentimiento amoroso. En este caso,
d) ¿Cómo interpretas las alusiones a la prisión que se dan la obstinación del amante se centra en una esperanza a
en todo el texto? la que se aferra aun sabiéndola vana.
a) El yo poético expresa su dolor por no poder alcanzar al
amado (Dios). LA FACTORÍA DE TEXTOS
b) La paradoja. Con estas expresiones se presenta el dolor por 1. ¿Cómo es el espacio que describe fray Luis? Reescribe su
el amor inalcanzable y el afán persistente de alcanzarlo. descripción empleando sinónimos, expresiones de nuestro
c) Se emplea el léxico propio del tema amoroso (muero de idioma en la actualidad y, en definitiva, tus propias palabras.
amor, el corazón le di, prisión del amor, pasión, no te Eso sí: mantén el orden de los elementos tal y cómo ha or-
tardes, que te espero). El planteamiento es el mismo del ganizado el poeta la información sobre el espacio descrito.
amor cortés: el ser amado es inalcanzable y la espera del
amor provoca dolor en el yo poético. Espacio idílico de la naturaleza. Locus amoenus.
d) La ausencia del amado hace sentir la vida como un lugar 2. ¿Cuáles son las categorías gramaticales más presentes en
de dolor y de angustia, de asfixia. los textos descriptivos? Justifica tu respuesta basándote
2. También tuvo una gran relevancia en el Renacimiento la en el fragmento de la Oda a la vida retirada.
obra del poeta sevillano Fernando de Herrera (1534-1597). Los adjetivos calificativos. Bella, cierto, codiciosa, airosa,
El siguiente soneto es suyo, y aborda el tema amoroso. pura, sosegada, diversas, manso.
Yo vi unos bellos ojos, que hirieron 3. Piensa en cuál sería para ti un lugar idílico, donde te en-
con dulce flecha un corazón cuitado; contrarías sereno y feliz. Visualiza ese espacio, dibújalo y, a
y que, para encender nuevo cuidado, continuación, descríbelo en un texto de entre diez y quince
su fuerza toda contra mí pusieron. líneas.
Yo vi que muchas veces prometieron Respuesta libre.
remedio al mal, que sufro no cansado,
y que cuando esperé vello acabado, 4. Lee atentamente el fragmento del Lazarillo de Tormes y,
poco mis esperanzas me valieron. a continuación, divídelo en partes. ¿Qué contenido tiene
Yo veo que s’esconden ya mis ojos cada una?
y crece mi dolor, y llevo ausente En el primer párrafo se habla de su capacidad para fingir de-
en el rendido pecho el golpe fiero. voción y humildad. En el segundo, de sus habilidades cura-
Yo veo ya perderse los despojos tivas para sacar el dinero a la gente. En el último párrafo se
y la membrana de mi bien presente alude a su condición mezquina y avarienta.
y en ciego engaño de esperanza muero. 5. Piensa ahora en alguna persona con la que hayas tenido
a) En la tradición petrarquista y del amor cortés, el sen- experiencias desagradables en el pasado, como le ocurrió
timiento amoroso solía expresarse con metáforas re- a Lázaro con el ciego. Descríbela física y psicológicamente.
lacionadas con la lucha o la guerra, como si la amada Respuesta libre.

84 Lengua castellana y Literatura. 3.º ESO. Solucionario


COGED DE VUESTRA ALEGRE PRIMAVERA… 9
ACTIVIDADES FINALES b) Atendiendo al contenido, diferencia los elementos
propios del amor cortés y los que se refieren a la na-
Repasa lo que has aprendido
turaleza idílica.
1. Define los siguientes conceptos y pon ejemplos: carpe
c) El yo poético hace una invitación o propuesta a su ama-
diem, égloga, estancia, poesía mística, novela picaresca,
da: ¿cuál es? ¿qué argumentos le da para que acepte?
texto descriptivo.
a) 11A, 11B, 11C, 11B, 11A, 11C, 7c, 7d, 7d, 11E, 11E, 11F, 7e, 11F.
Carpe diem: tópico literario heredado de la poesía clásica que
consiste en la invitación al goce de la belleza y la juventud b) El amor cortés se refleja en la actitud esquiva de la amada,
presentes. Ejemplo: soneto XXIII de Garcilaso. que huye del amante desesperado y dolorido, y en la pro-
pia expresión del sufrimiento de él. La naturaleza idílica
Égloga: composición poética de tema amoroso, ambientada en
está presente en la descripción del lugar donde antaño los
un espacio idílico de la naturaleza y protagonizada por ninfas
amantes se encontraron y que ahora se llena de recuer-
y pastores. Ejemplo, las tres que escribió Garcilaso.
dos tristes para el amante, por la ausencia y esquividad
Poesía mística: corriente poética de tema religioso que aborda de ella. Es un espacio que corresponde al tópico del lo-
el tema del ansia de unión con Dios, utilizando para ello los cus amoenus (un prado lleno de verdura, un´espesura,
tópicos y motivos temáticos de la lírica amorosa. Ejemplo: un´agua clara…).
Noche oscura del alma, de san Juan de la Cruz.
c) La propuesta es que ella vuelva a ese lugar que una vez le
Novela picaresca: subgénero narrativo que se desarrolló en fue tan querido, y le asegura que si él es la razón por la que
España a partir de la publicación del Lazarillo de Tormes. Es se niega a venir, él se alejará.
un relato en forma autobiográfica y epistolar por parte de un
personaje de baja extracción social, que tiene diversas expe- 4. Como sabes, los poetas religiosos del Renacimiento utiliza-
riencias, amos y oficios, y va aprendiendo a desenvolverse en ban los tópicos amorosos para expresar su sentir religioso.
un medio hostil, con recursos no siempre lícitos ni honestos. Explica esta afirmación a partir del siguiente fragmento del
Este personaje es un pícaro. Cántico espiritual de san Juan de la Cruz:
Texto descriptivo: tipo de discurso que expone ordenadamen- Allí me dio su pecho;
te la información relativa a los rasgos y cualidades de objetos, allí me enseñó ciencia muy sabrosa,
personas, espacios, etc. Su finalidad es pintarlos con palabras y yo le di de hecho
para que puedan ser visualizados o comprendidos por el lec- a mí, sin dejar cosa;
tor. Ejemplo: descripción del locus amoenus en la Oda a la allí le prometí de ser su esposa.
vida retirada de Fray Luis de León. Mi alma se ha empleado,
2. En esta unidad has leído un famoso soneto de Garcilaso y todo mi caudal, en su servicio;
que comenzaba así: «En tanto que de rosa y azucena…». ya no guardo ganado,
ni ya tengo otro oficio,
¿Qué elementos propios de la descripción encuentras en
que ya solo en amar es mi ejercicio.
ese texto?
Pues ya si en el ejido1
Presencia de sustantivos que nombran el objeto descrito (co- de hoy más no fuere vista ni hallada,
lor, gesto, mirar, cabello, cuello, cumbre, rosa, etc.); abun- diréis que me he perdido;
dancia de adjetivos calificativos (ardiente, honesto, clara, que andando enamorada,
presto, alegre, dulce, airado, hermosa, helado, ligera) o, en me hice perdidiza, y fui ganada.
su lugar, de elementos que complementan al nombre (del oro);
presencia de verbos que expresan acciones (mueve, esparce, De flores y esmeraldas,
desordena, cubra, etc.) Comparaciones y metáforas (de rosa en las frescas mañanas escogidas,
y azucena se muestra la color, por «rosa y blanco»; vena del haremos las guirnaldas,
oro, por «rubio»; primavera por «juventud»; cubra de nieve la en tu amor florecidas
hermosa cumbre, por «llene canas la cabeza»; marchitará la y en un cabello mío entretejidas.
rosa el viento helado por «se ajará la belleza con la llegada
de la vejez», etcétera). 1
ejido: pasto.
3. En el siguiente fragmento de la Égloga I de Garcilaso de la
Vega, el pastor Salicio lamenta el desprecio de su amada. La unión del alma con Dios se expresa como si se tratase
Léelo atentamente y realiza después las actividades: de un encuentro amoroso entre amantes: le prometí de ser
su esposa (verso 5); que ya solo en amar es mi ejercicio
Mas ya que a socorrerme aquí no vienes,
(verso 10); que andando enamorada (verso 14); haremos las
no dejes el lugar que tanto amaste,
guirnaldas, en tu amor florecidas y en un cabello mío en-
que bien podrás venir de mí segura.
tretejidas (versos 10-20). La idea de esa unión es una idea
Yo dejaré el lugar do me dejaste;
feliz, llena de promesas e ilusiones, como las del sentimiento
ven si por solo aquesto te detienes.
amoroso.
Ves aquí un prado lleno de verdura,
ves aquí un’ espesura, 5. ¿Encuentras alguna relación entre el contenido del frag-
ves aquí un’ agua clara, mento anterior y el soneto de Garcilaso que has leído en
en otro tiempo cara, esta unidad, «Escrito está en mi alma vuestro gesto»? ¿En
a quien de ti con lágrimas me quejo; qué partes?
quizá aquí hallarás, pues yo m’alejo,
La declaración de incondicionalidad y la devoción que se di-
al que todo mi bien quitar me puede,
rige al amado están presentes en ambas composiciones. En
que pues el bien le dejo,
el fragmento de san Juan, obsérvese especialmente la segun-
no es mucho que’l lugar también le quede.
da lira (mi alma se ha empleado, y todo mi caudal, en su
a) Haz el análisis métrico del fragmento. servicio...).

Lengua castellana y Literatura. 3.º ESO. Solucionario 85


9 COGED DE VUESTRA ALEGRE PRIMAVERA…

6. Haz el análisis métrico del fragmento del Cántico espiri- b) Las diferencias se refieren a las que existen entre aquellos
tual que acabas de leer. ¿Qué tipo de estrofa se emplea? que heredaron sus bienes y los que han tenido que salir
adelante en la vida por sus propios medios. Lázaro está
Todas las estrofas son liras. Esquema métrico: 7a11B7a7b11B.
en el segundo grupo.
7. Investiga en alguna enciclopedia y averigua cuál es el c) En las últimas líneas: Cuánto más hicieron los que, sién-
argumento de la Diana, de Jorge de Montemayor. A con- doles contraria [la Fortuna] con fuerza y maña remando
tinuación, haz un breve resumen de la trama en tu cua- salieron a buen puerto.
derno y explica qué rasgos propios de la novela pastoril
puedes señalar. Recuerda lo que ya sabías
Los siete libros de la Diana, de Jorge de Montemayor, es una 10. ¿Qué representativo autor español había escrito sonetos
obra que se articula en torno a un argumento lleno de conflic- antes que Garcilaso, en el siglo XV?
tos sentimentales y triángulos amorosos: la pastora Diana es
El marqués de Santillana.
famosa por su hermosura. Es amada por los pastores Sireno
y Sylvano, pero ella solo corresponde al primero. Sin embar- 11. ¿Cuál es la estructura métrica del soneto?
go este tuvo que ausentarse durante largo tiempo del reino y Catorce versos endecasílabos agrupados en dos cuartetos y
ella se casó con otro, Delio, lo cual desencadena el conflicto dos tercetos.
en la trama, que además se complica con la introducción de
otros personajes. Elemento propio de la literatura pastoril es 12. La novela picaresca, por el hecho de tener un antihéroe
el espacio bucólico, pues la novela está ambientada junto al río como protagonista, se opone a otros géneros narrativos
Esla y protagonizada por pastores idealizados cuyas historias que se habían desarrollado durante el siglo XV. ¿Cuáles?
son amorosas. ¿En qué consistían esos relatos?
8. Elabora un esquema con la vida de Lázaro de Tormes en Los géneros que se habían desarrollado eran sobre todo los
el que recojas su origen, los amos que tuvo y la situación relatos sentimentales y los caballerescos. Los sentimenta-
final que consiguió alcanzar y desde la que escribe. les narraban historias amorosas basadas a menudo en conflic-
tos y triángulos, y sus personajes estaban siempre tipificados
Nacimiento de Lázaro junto al río Tormes, en familia de baja
e idealizados. Los caballerescos narraban las aventuras de
extracción social.
caballeros andantes, heroicos, idealizados e intachables, y
Amos (Tratados I-V): ciego, clérigo avaro, escudero pobre, protagonistas de historias igualmente idealizadas y, en oca-
fraile de la Merced y buldero estafador. siones, con elementos fantásticos.
Oficios (Tratados VI y VII): repartidor de agua, ayudante de 13. En el siguiente fragmento, localiza los adjetivos califica-
alguacil y pregonero de vinos. tivos y, cuando proceda, clasifícalos en especificativos y
Situación final: marido de la criada de un arcipreste, posición explicativos:
social buena.
Corrientes aguas, puras, cristalinas,
9. El siguiente fragmento pertenece al prólogo del Lazarillo árboles que os estáis mirando en ellas,
de Tormes. El protagonista se dirige al caballero a quien verde prado de frescas sombras lleno,
se dispone a contar su historia. Léelo y contesta después aves que aquí sembráis vuestras querellas,
a las preguntas: hiedra que por los árboles caminas,
torciendo el paso por su verde seno:
Suplico a Vuestra Merced reciba el pobre servicio de mano de
yo me vi tan ajeno
quien lo hiciera más rico si su poder y deseos se conformaran.
del grave mal que siento,
Y pues Vuestra Merced escribe se les escriba y relate el caso
que de puro contento
muy por extenso, paresciome no tomalle por el medio, sino
con vuestra soledad me recreaba,
del principio, porque se tenga entera noticia de mi persona; y
donde con dulce sueño reposaba,
también porque consideren los que heredaron nobles estados
o con el pensamiento discurría
cuán poco se les debe, pues Fortuna fue con ellos parcial, y
por donde no hallaba
cuánto más hicieron los que, siéndoles contraria, con fuerza
sino memorias llenas de alegría.
y maña remando salieron a buen puerto.
Garcilaso DE LA VEGA: Égloga I.
a) Repasa lo que has estudiado sobre el argumento del
Lazarillo y explica el contenido de este fragmento. Corrientes, puras, cristalinas, verde (verso 3), frescas, lleno,
verde (verso 6), grave, puro, dulce. Especificativos: puras,
b) Lázaro justifica su trayectoria por la existencia de dife- cristalinas, lleno. Explicativos: corrientes, verde (en los dos
rencias sociales. ¿Cuáles son esas diferencias, según casos), frescas, grave, puro, dulce.
sus palabras? ¿En qué grupo se sitúa él?
14. ¿Recuerdas lo que es un epíteto? ¿Hay ejemplos en el texto
c) La novela picaresca narra el modo en que un persona- anterior?
je sale adelante pese a las dificultades de su entorno. Adjetivo calificativo explicativo, con valor retórico y expre-
¿En qué parte del fragmento se alude a esta idea? sivo. En ese texto pueden citarse como ejemplos de epítetos
a) Lázaro atiende la demanda que le ha hecho cierto caballe- los dos usos de verde, así como grave o puro.
ro (cuyo nombre no se desvela) para que explique el caso
15. ¿A qué sustantivos complementaban los adjetivos que
que motiva la narración. Este caso no se desvela de mo-
mento, pero al final de la novela sabremos a qué se refiere:
has señalado en el fragmento de la actividad 13? ¿Qué
hay rumores sobre la reputación de Lázaro, pues al parecer otros complementos del nombre localizas en ese texto?
su mujer tiene relaciones con el arcipreste que los protege Corrientes, puras y cristalinas se refieren a agua. Verde (ver-
a ambos. Lázaro dice que para atender esa demanda, le so 3) y lleno, a prado. Frescas, a sombras. Verde (verso 6), a
es necesario contar toda su historia, pues solo así será seno. Grave se refiere a mal. Puro se refiere a contento. Dulce
comprendido. se refiere a sueño.

86 Lengua castellana y Literatura. 3.º ESO. Solucionario


COGED DE VUESTRA ALEGRE PRIMAVERA… 9
16. Señala ahora los verbos de ese fragmento y di qué com- b) Redacta una descripción de Venus. ¿Responde al
plementos los acompañan. canon de belleza de la época?
Estáis mirando: está complementado por el circunstancial c) Explica el parecido físico que hay entre la Venus de
de lugar en ellas. Botticelli y la dama a la que se refería Garcilaso en el
Sembráis: está complementado por el objeto directo vuestras poema «En tanto que de rosa y azucena».
querellas y por el circunstancial aquí. d) En el cuadro La primavera, aparecen varios persona-
Caminas: está complementado por el circunstancial por los jes de la mitología clásica: Mercurio, las tres Gracias,
árboles y por la subordinada adverbial construida con gerun- Venus, Cupido, Flora y Céfiro. ¿Quiénes son? Explí-
dio torciendo el paso por su verde seno. calo por escrito.
Torciendo: se refiere a la oración anterior (hiedra que cami- a) Venus nació de la espuma del mar, que a su vez había
nas torciendo el paso). Está complementado por el objeto di- surgido de los órganos arrancados a Urano. Emerge en
recto el paso y por el circunstancial por su verde seno. el interior de una concha y es empujada por los vientos
Vi: está complementado por el objeto directo (pronombre re- céfiros hasta la playa, donde la aguardan las Estaciones
flexivo) me y por el complemento predicativo tan ajeno. (las Horas), quienes la visten, la adornan, la cuidan y la
Siento: está complementado por el pronombre relativo en fun- conducen al Olimpo. En el cuadro de Botticelli vemos a Ve-
ción de objeto directo, que (referido al grave mal). nus emergiendo del mar en la concha. A un lado los vientos
céfiros soplan sobre ella para impulsarla hacia la playa.
Recreaba: está complementado por el circunstancial con vues-
Allí la aguarda la primavera dispuesta para cubrirla con
tra soledad.
su manto.
Reposaba: está complementado por el circunstancial con dulce
b) Sí responde a dicho canon. Es rubia, de piel blanca y son-
sueño.
rosada, y ojos claros. Responde al tópico de la donna
Discurría: está complementado por el circunstancial con el angelicata.
pensamiento y por la subordinada adverbial por donde no
c) Ambas responden al tópico de la donna angelicata, por
hallaba sino memorias llenas de alegría.
la blancura sonrosada de su piel y el color dorado de sus
Hallaba: está complementado por el objeto directo memorias cabellos.
llenas de alegría y por el circunstancial no.
d) Mercurio era el mensajero de los dioses. Las tres Gracias
17. Di qué modalidades oracionales se dan en este poema: representan el placer, el amor y la belleza, respectivamen-
Ojos claros, serenos, te. Venus, diosa del amor, la belleza y la fertilidad, presi-
si de un dulce mirar sois alabados, de la escena y sobre ella vuela su hijo, Cupido. Céfiro (el
¿por qué, si me miráis, miráis airados? viento) había raptado por amor a Clori, y de su unión había
Si cuanto más piadosos, nacido Flora, que simboliza la primavera.
más bellos parecéis a aquel que os mira, 2. Otro artista italiano que trabajó motivos mitológicos fue
no me miréis con ira, el escultor Benvenuto Cellini. Observa esta escultura suya,
porque no parezcáis menos hermosos.
titulada Perseo, que se alza en la plaza de la Signoria de Flo-
¡Ay, tormentos rabiosos!
rencia. Busca información sobre Perseo y explica la escena
Ojos claros, serenos,
ya que así me miráis, miradme al menos. que se reproduce en dicha escultura.
Gutierre DE CETINA Se reproduce la escena en que Perseo ha vencido a Medusa,
cortándole la cabeza. Recordemos que Perseo logró derrotarla
Modalidad apelativa interrogativa: ¿Por qué, si me miráis, sin mirarla directamente, pues su mirada era letal. Luchó con
miráis airados?. ella orientándose por su reflejo en el espejo de su escudo.
Modalidad apelativa exhortativa: no me miréis con ira; mi-
3. El tema del amor ha recorrido siglos de poesía, a través
radme al menos.
de la literatura y la música. El siguiente fragmento corres-
Modalidad expresiva exclamativa: ¡Ay, tormentos rabiosos! ponde a una bella canción del álbum La danza de la ciudad,
18. «Ya que así me miráis, miradme al menos»: ¿cuál es el sujeto del dúo Cómplices.
de la oración? ¿Es una oración compuesta? ¿Por coordina- Lee la letra y relaciona su contenido con el soneto de
ción o por subordinación? ¿De qué tipo? Garcilaso «Escrito está en mi alma vuestro gesto».
El sujeto es gramatical, omitido, segunda persona del plural, y Es por ti que veo ríos
se refiere a los ojos, que aparecen como vocativo. La oración donde solo hay asfalto
es compuesta por subordinación, pues la estructura ya que así Es por ti que hay océanos
me miráis es una subordinada adverbial causal. donde solo había charcos
Es por ti que soy un duende
MIRA A TU ALREDEDOR Y… cómplice del viento
… ve más allá que se escapa de madrugada
para colarse por tu ventana
1. El Renacimiento fue una corriente que floreció en el arte y y decirte:
en la escultura, especialmente en Italia. Contempla estos Tus labios son de seda,
cuadros del pintor Sandro Botticelli, y contesta a las pre- tus dientes del color de la luna llena,
guntas: tu risa la sangre que corre por mis venas,
a) El nacimiento de Venus representa un motivo de la tus besos la tinta de mis versos,
mitología clásica. Investiga: ¿cómo nació esta diosa que siempre te cuentan.
según el mito? Explica cómo se recoge esta historia La amada justifica el modo de vivir del amante. Obsérvese
en el cuadro de Botticelli. cómo se reproduce el paralelismo sintáctico que recogía la

Lengua castellana y Literatura. 3.º ESO. Solucionario 87


9 COGED DE VUESTRA ALEGRE PRIMAVERA…

incondicionalidad del enamorado en el soneto de Garcilaso: arzobispo de Sevilla. Fue presidente de la chancillería de
por vos nací, por vos tengo la vida, por vos he de morir, por Valladolid e inquisidor general a partir de 1547. Se empeñó
vos muero, y aquí se dice: es por ti que veo ríos […] es por ti en la confección del Índice de libros prohibidos y en la
que hay océanos […] es por ti que soy un duende. Por otra represión de los focos protestantes de Valladolid y Sevilla.
parte, en el mencionado soneto de Garcilaso se presentaba a Carlos V: Carlos I de España y V de Alemania, reinó entre
la amada como inspiradora de los versos del poeta (cuanto yo 1516 y 1556.
escribir de vos deseo, vos sola lo escribisteis, yo lo leo) y este
Felipe II: hijo de Carlos I, reinó tras la abdicación de su
mismo motivo temático aparece en la canción de Cómplices:
padre, entre 1556 y 1598.
tus besos la tinta de mis versos, que siempre te cuentan.
Don Carlos de Seso, corregidor de la ciudad de Toro y fun-
… encuentra la clave dador del grupo protestante de Valladolid.
1. El escritor español Miguel Delibes (1920-2010) es el autor b) En el año 1558, Carlos I escribe una carta a su hija Juana.
de una de las novelas históricas más documentadas e inte- En este momento el rey es ya Felipe II, y Juana gobier-
na como regente en su ausencia. En dicha carta, Carlos I
resantes de la narrativa española contemporánea: El hereje,
recomienda rigor y represión inflexible contra el protes-
ambientada en el siglo XVI, durante el reinado de Felipe II,
tantismo para evitar que ocurriese lo mismo que había
en la época de los conflictos religiosos. En el siguiente sucedido en Alemania. Esto se relaciona con el proceso
fragmento se recogen los temores de un grupo de pre- de la Contrarreforma, iniciado en la segunda mitad del
sos luteranos que han sido descubiertos por la Inquisición siglo XVI por la Iglesia católica para frenar el avance de
y son conducidos ante el tribunal del Santo Oficio. Léelo los luteranos.
atentamente y contesta después a las preguntas: c) Respuesta libre.
Don Carlos de Seso conocía la carta del inquisidor Valdés a 2. Hay ciertos temas que recorren los siglos sin perder su
Carlos V, retirado en Yuste, en la que rogaba que se atajase
interés y su vigencia. Cambian solo las formas de expre-
tan gran mal y que los culpados fueran punidos y castigados
con el mayor rigor sin excepción de ninguna clase. Seso inter-
sión. Compruébalo en el siguiente poema del poeta Fran-
pretaba esto en el sentido de que se preparaba un escarmiento cisco Brines, y responde a las preguntas después de leerlo:
ejemplar, sin precedentes en España. El corregidor de Toro dis- Collige, virgo, rosas
ponía de una gran habilidad para hacer amigos y hablaba con Estás ya con quien quieres. Ríete y goza. Ama;
unos y otros sin distinción, tanto con los oficiales como con los Y enciéndete en la noche que ahora empieza,
soldados y, si se terciaba, con los familiares de la Inquisición. y entre tantos amigos (y conmigo)
Estaba al día de todo. Sabía todo. Temía tanto a Felipe II como abre los grandes ojos a la vida
a Carlos V, y tenía el convencimiento de que antes de 1558 los con la avidez preciosa de tus años.
castigos habrían sido más leves, pero hoy Pablo IV no ceja- La noche, larga, ha de acabar al alba,
ba, decía. En los descansos de la tarde les informaba de estos y vendrán escuadrones de espías con la luz;
asuntos, de la carta del inquisidor Valdés al Emperador, de las se borrarán los astros, y también el recuerdo;
de este a su hija, la gobernadora en ausencia de su hermano, y la alegría acabará en su nada.
y a Felipe II, pidiendo prisa, rigor y recio castigo. Muchos no
Mas aunque así suceda, enciéndete en la noche,
saldremos de esta, decía y llegó a tramar un plan para fugarse
pues detrás del olvido puede que ella renazca,
pero no encontró ocasión de llevarlo a cabo.
y la recobres pura, y aumentada en belleza,
a) ¿Qué personajes históricos aparecen en el fragmento? si en ella, por azar, que ya será elección,
¿Qué sabes sobre ellos? sellas la vida en lo mejor que tuvo,
b) En el texto se habla acerca del año 1558. Investiga cuando la noche humana se acabe ya del todo,
y venga esa otra luz, rencorosa y extraña,
sobre esa fecha e indica por qué el corregidor de Toro
que antes que tú conozcas, yo ya habré conocido.
piensa que antes de ese año los castigos habrían sido
más leves. ¿Cómo relacionas esto con lo que has estu- a) ¿A qué tema estudiado en esta unidad alude el título
diado al principio de la unidad sobre la sociedad y la del poema? ¿Qué significa?
cultura españolas del siglo XVI? b) Escribe un texto literario con el siguiente tema: las
c) Vivimos hoy en una sociedad en la que existe la liber- cosas más bellas de la vida son efímeras. Puedes esco-
tad religiosa. Desde esta perspectiva del siglo XXI, ger entre el verso o la prosa poética.
¿cómo entiendes las luchas y persecuciones religio- a) El título significa «coge, doncella, las rosas». Se alude al
sas del siglo XVI? Redacta un escrito argumentado tema del disfrute de la juventud, la belleza y el momento
sobre estas ideas de unas quince líneas de extensión. presente, representado habitualmente con el tópico del
a) Inquisidor Valdés: Fernando de Valdés (1487-1568), pre- Carpe diem.
lado contrarreformista, obispo de Oviedo y Sigüenza y b) Respuesta libre.

88 Lengua castellana y Literatura. 3.º ESO. Solucionario


AQUELLOS LOCOS TAN INGENIOSOS 10
APERTURA DE UNIDAD resulta chocante que alguien que no existe posea tantos ape-
llidos. De algún modo, la palabra sustituye en él a la materia.
1. ¿A qué tipo de texto corresponde este fragmento? Jus-
4. ¿Qué actitud manifiesta el emperador Carlomagno hacia
tifica tu respuesta.
sus caballeros? Justifica tu respuesta con algún ejemplo
Corresponde a un texto descriptivo. El narrador se interesa
del texto.
en perfilar la apariencia externa de Monipodio y representa
su aspecto físico y su indumentaria con gran abundancia de El emperador no parece guardarles el debido respeto a sus
adjetivos. caballeros. Las palabras del narrador que complementan la
dificultad del personaje a la hora de pronunciar el larguísimo
2. Al poco de llegar a Sevilla, los muchachos Rincón y Cor- nombre de Agilulfo: Si tuviera que acordarme del nombre de
tado se integran en la cofradía de maleantes cuyo jefe todos ¡estaría avisado!, evidencian el distanciamiento, casi
es Monipodio. ¿Qué impresión te causan los rasgos con indiferencia, de Carlomagno hacia los hombres que pelean
que es caracterizado dicho personaje? Por su parte, ¿qué por él.
sienten hacia él todos los miembros de aquella «virtuosa 5. ¿Qué significa el pasaje: «cuanto más duraba la guerra me-
compañía»? nos respeto por la limpieza veía en los paladines»?
La apariencia de Monipodio es, sin duda, aterradora e inspira El pasaje en cuestión puede resultar altamente polisémico.
un profundo respeto. No obstante, sus cofrades sienten hacia Conforme avanza la guerra, los paladines, cansados y heridos,
él una extraña «devoción», pues parecen verlo con buenos ojos se preocupan menos por su limpieza. Pero, además, también
y se le muestran sumisos al recibirlo con reverencia. puede pensar que la guerra embrutece a los contendientes
3. ¿Te parece que el retrato que ofrece Cervantes del señor moralmente, de forma que el narrador estaría aludiendo a una
Monipodio es verosímil o es demasiado exagerado? ¿Qué limpieza de espíritu.
rasgos horrendos destaca el narrador en él? ¿Cómo lo
hace? Vocabulario
Al recurrir a la hipérbole, el narrador transforma a Monipodio 6. En este fragmento se mencionan varias piezas de la arma-
en una criatura tan desmesurada como poco verosímil. Su dura del caballero medieval. Con la ayuda del diccionario,
pecho y sus manos están cubiertas por el bello; su estatura es describe lo que es el quijote.
tan descomunal como la talla de unos pies llenos de juanetes. El quijote es una pieza de la armadura que cubría el muslo
Para exageran, se ha echado mano, especialmente, de la adje- del caballero.
tivación en grado superlativo: muy negra (barba), el hombre
7. Aparte de ser un objeto usado por los caballeros para
más basto y el más feo que se haya visto jamás en el mundo.
proteger la cabeza, ¿qué otra cosa puede ser una celada?
4. Desde la más remota antigüedad, los textos literarios Busca la palabra en el diccionario.
siempre han identificado a los personajes más perversos
Una celada puede ser también una «pieza de la ballesta», aun-
con una fealdad física, como dando a entender que son que, en general, se emplea el sustantivo como sinónimo de
incompatibles la hermosura y la maldad moral. «trampa o engaño», y también con el significado de «embos-
Haz un poco de memoria y enumera los personajes del cada de gente armada en un lugar oculto para asaltar por
mundo de la literatura, el cine o el cómic que cumplan con sorpresa al adversario».
la tendencia aludida. 8. ¿Por qué Carlomagno emplea la segunda persona del
A continuación, intenta citar algún personaje que incum- plural (alzáis, mostráis, etc.) para dirigirse al caballero
pla dicha costumbre. Agilulfo?
Respuesta libre. En el texto se remeda el estilo arcaico de las crónicas me-
dievales, donde se usaba el vos, como fórmula de respeto, en
VIVE LA LECTURA lugar del tú.

Comprensión lectora 9. ¿Mediante qué verbos representa el narrador el efecto


acústico de las palabras del protagonista?
1. ¿Cuál es el rasgo más sobresaliente del caballero Agilul-
La voz de Agilulfo llegaba metálica, es decir, «vibraba» y «re-
fo? ¿Dirías que es un personaje fantástico o una invención
tumbaba» dentro del yelmo cerrado.
absurda? ¿Por qué?
El rasgo más sobresaliente de Agilulfo es que carece de exis-
10. ¿Con qué palabras podrías reemplazar en el texto el adje-
tencia corporal. Con tan increíble insuficiencia, más que un tivo férrea y el sustantivo escalofrío? Enumera cinco térmi-
personaje fantástico, se antoja una invención absurda forjada nos intercambiables para cada uno de ellos.
así con un propósito entre alegórico y burlesco. férrea: metálica, dura, fuerte, resistente, sólida; escalofrío:
2. A pesar de su curiosa carencia, ¿qué le permite identifi- estremecimiento, sacudida, espasmo, temblor, impresión.
carse con un caballero real? ¿Cuáles son sus objetivos? Investigación y redacción
La fuerza de voluntad es la que identifica a Agilulfo, al igual
que le ocurría a don Quijote con un caballero real. Sus objeti-
11. Averigua qué otras dos novelas completan la trilogía de
vos son los de servir con las armas la causa de la fe cristiana, Calvino Nuestros antepasados.
de ahí lo de santa causa. a) ¿En qué consiste la originalidad de sus respectivos
3. ¿Qué intención crees que tenía el autor al bautizar a su argumentos y en qué época sitúan su acción?
protagonista con un nombre con tantos apellidos? b) ¿Existe, pues, alguna relación entre el título genérico
El empleo de numerosos apellidos tiene una doble finalidad de la trilogía y la localización temporal de las novelas
paródica. Por un lado, podría considerarse como una burla que la integran?
hacia la ancestral costumbre aristocrática de enlazar más a) Junto con El caballero inexistente, completan la trilogía: El
apellidos que acrediten la longevidad de un linaje. Por otro, vizconde demediado y El barón rampante. El primer relato

Lengua castellana y Literatura. 3.º ESO. Solucionario 89


10 AQUELLOS LOCOS TAN INGENIOSOS

transcurre en el siglo XVII. El vizconde Medardo de Terralba y dramas históricos (Julio César, Ricardo III), aunque son
es partido en dos por una bala de cañón mientras participa más famosas sus tragedias (Romeo y Julieta, Otelo, Macbeth,
en una batalla contra los turcos. Los médicos consiguen El rey Lear y Hamlet).
salvarle la vida, pero el personaje se escinde en dos: una El italiano Torquato Tasso (1544-1595) escribió el libro Diá-
mitad es buena y la otra es perversa. La segunda novela se logos, siguiendo la moda de los diálogos platónicos, así como
sitúa en el siglo XVIII y relata cómo Cosme Piovasco, barón dos mil poemas líricos (Rime) y los poemas Mondo creato
de Ombrosa, se subió a un árbol, huyendo de la orden que le (basado en la Biblia) y Aminta, oscilando entre lo dramáti-
imponían sus familiares de comer un plato de caracoles, y co y lo pastoril. No obstante, su obra más reconocida fue el
ante la amenaza de recibir un castigo cuando descendiese poema épico la Jerusalén liberada (Jerusalén conquistada,
del árbol, decidió permanecer allí arriba durante años. en su versión definitiva), donde relataba las hazañas de los
b) De acuerdo con el título genérico de la trilogía, Calvino cristianos durante los últimos tiempos de la Primera Cruzada
hace tres calas en épocas distintas, y pretéritas, de nuestra hasta liberar el Santo Sepulcro.
historia para reflexionar sobre la excepcionalidad de la Finalmente, en Portugal nos hallamos con Luis Vaz de Ca-
condición humana. mões (¿1524?-1580). Escribió poemas líricos, adaptándose a
12. Para Agilulfo, la fuerza de voluntad y la fe son virtudes la moda petrarquista. Pero fue Os Lusiadas su obra más ce-
suficientes para desempeñar una misión que, en su caso, lebrada. Se trataba de un poema épico en diez cantos, cuyo
es imposible: el ejercicio militar. ¿También tú consideras modelo era la Eneida de Virgilio y donde hacía un repaso de
que tales impulsos son suficientes para conseguir alcan- los episodios más gloriosos de la historia de su país, a partir
del motivo central del viaje de Vasco de Gama hasta las Indias
zar cualquier meta? ¿Por qué?
Orientales.
Respuesta libre.
4. En los capítulos XXXVII y XXXVIII de la primera parte del
13. Cuéntales a tus compañeros alguna situación comprome- Quijote, el protagonista enlaza su famoso discurso sobre
tida de la que tú mismo hayas logrado salir con la ayuda las armas y las letras, donde dice cosas como estas: «Quí-
de la fuerza de voluntad. Luego, convierte dicha historia tenseme delante los que dijeren que las letras hacen ven-
en un breve relato, de unas veinte líneas. Por último, ex- taja a las armas, que les diré […] que no saben lo que dicen»
plica qué diferencias sustanciales existen entre tu versión o «No hay ninguno más pobre [que el soldado] en la misma
oral y tu relato escrito. pobreza, porque está atenido a la miseria de su paga, que
Respuesta libre. viene o tarde o nunca». ¿Qué relación adviertes entre estas
afirmaciones y algún aspecto de la biografía cervantina?
LITERATURA
Al decir que No hay ninguno más pobre [que el soldado],
1. En la unidad anterior has investigado sobre el Índice de Cervantes estaba reflejando su propia experiencia personal.
libros prohibidos, publicado por la Inquisición en 1551. Si bien siempre se sintió orgulloso de su participación en la
Explica por qué podía ser peligroso leer los libros en él batalla de Lepanto, estaba muy dolido por el hecho de que su
incluidos. sacrificio no fuera recompensado por sus superiores, de forma
que, siendo soldado, quedó manco, sufrió cautiverio en Argel
Los libros que figuraban en tal Índice resultaban peligrosos y nunca pudo escapar a la miseria.
porque desde el punto de vista de la Iglesia católica podían
difundir ideas heréticas. Téngase en cuenta que, con el avance 5. ¿Qué visión nos ofrece Cervantes de los poetas en estos
de la Reforma protestante en el siglo XVI, la Iglesia y también tercetos de su Viaje al Parnaso? Justifica tu respuesta con
la monarquía intentaron controlar lo que se imprimía en las ejemplos.
prensas, porque con su invención la imprenta se transformó Pasa, raro inventor, pasa adelante
en un cauce idóneo para difundir todo tipo de ideas. con tu sutil designio, y presta ayuda
2. Felipe II ordenó la construcción del Monasterio de San a Apolo, que la tuya es importante,
Lorenzo de El Escorial, donde tuvo su residencia. Ave- antes que el escuadrón vulgar acuda
rigua qué motivos animaron al monarca a levantar tan de más de veinte mil sietemesinos
singular edificio. poetas que de serlo están en duda.
Llenas van ya las sendas y caminos
El enorme edificio (compuesto por un palacio, una basílica,
desta canalla inútil contra el monte,
un panteón para dar sepultura a los monarcas, un monasterio
que aun de estar a su sombra no son dignos.
y una biblioteca), fue mandado construir, sobre todo, porque
el rey Felipe II deseaba conmemorar la victoria de sus tropas Aunque diversos estudiosos señalan que la visión que Cer-
contra los franceses en la batalla de San Quintín, que tuvo vantes expresa de los poetas contemporáneos es convencio-
lugar el día de San Lorenzo, el 10 de agosto de 1557. Aparte nal, en sus versos se advierte cierto rechazo hacia ellos, que
de darle a su familia un lugar donde ser enterrada, también pudo deberse a su resentimiento por no ser valorada su propia
quería darle gracias a Dios, creando un centro que encarnase obra por los demás. Así, en su alegórico viaje, los poetas se
el nuevo espíritu religioso surgido tras el Concilio de Trento. presentan como escuadrón vulgar […] de más de veinte mil
sietemesinos y como canalla inútil e indigna.
3. Más arriba se ha relacionado la figura de Cervantes con
la de escritores contemporáneos como Shakespeare, Tas- 6. En el entremés El viejo celoso, Cervantes teatralizó un
so y Camões. ¿En qué géneros literarios brillaron dichos motivo ya desarrollado en varios cuentos del Decamerón
autores? Indica los títulos más representativos de la pro- de Boccaccio. ¿De qué tema se trata? Investiga e intenta
ducción de cada uno de ellos con una breve referencia descubrir en qué novela ejemplar el autor se ocupó del
al asunto tratado. mismo asunto y qué diferencias hay entre el argumento
William Shakespeare (1564-1616) fue, sin duda, la figura cum- de una obra y otra.
bre del teatro isabelino inglés e incluso de la dramaturgia uni- El asunto tratado en el entremés es el del marido viejo y ce-
versal. Escribió comedias (El sueño de una noche de verano) loso que se casa con una mujer joven y hace todo lo posible

90 Lengua castellana y Literatura. 3.º ESO. Solucionario


AQUELLOS LOCOS TAN INGENIOSOS 10
para que la esposa no conozca otro varón. A partir de aquí, verdugos que nos tienen lástima y las mozas que nos socorren
siempre aparece un galán joven que usará de su picardía cuando acabamos en la trena2. ¡Qué sería de nosotros sin nues-
para acceder al lugar donde está la esposa y engañar al ma- tros bienhechores…! Son tantos los favores que les debemos
rido. Astucia e infidelidad son motivos que Cervantes vol- que, cada año, celebramos su adversario3 con la mayor popa4
vió a manejar en la novela ejemplar El celoso extremeño, y soledad5.
cuyo argumento debe contrastar el alumno con el del citado
entremés. 1
guros: alguaciles. 2 trena: cárcel. 3 adversario: deformación de
7. En El licenciado Vidriera, tras visitar diversas ciudades «aniversario». 4 popa: deformación de «pompa». 5 soledad: defor-
europeas y haber completado sus estudios en Salamanca, mación de «solemnidad».
Tomás Rodaja es pretendido por una dama que le engaña
de este modo para conseguir su amor: a) ¿Qué aspectos de este pasaje permiten situar el re-
Comió en tal mal punto Tomás el membrillo, que al momento lato en el grupo de las novelas realistas? Justifica tu
comenzó a herir de pie y de mano como si tuviera alferecía1, respuesta con ejemplos extraídos del texto.
y sin volver en sí estuvo muchas horas, al cabo de las cuales
b) Explica cómo queda caracterizado el personaje de
volvió como atontado, y dijo con lengua turbada y tartamuda
que un membrillo que había comido le había muerto, y declaró
Monipodio a través de su forma de hablar.
quién se le había dado. La justicia, que tuvo noticia del caso, c) ¿Te resulta cómico el fragmento? Explica por qué.
fue a buscar la malhechora; pero ya ella, viendo el mal suceso, a) En este pasaje, así como en el conjunto del relato, se ignora
se había puesto en cobro, y no pareció jamás. por completo la temática amorosa. En cambio, cobra pro-
Seis meses estuvo en la cama Tomás, en los cuales se secó y tagonismo la crítica social a partir de la complicidad que
se puso, como suele decirse, en los huesos, y mostraba tener desvela Monipodio entre su cofradía de malhechores y una
turbados todos los sentidos; y aunque le hicieron los remedios justicia que es corrupta y favorece a los delincuentes: Son
posibles, solo le sanaron la enfermedad del cuerpo, pero no tantos los favores que les debemos.
de lo del entendimiento; porque quedó sano, y loco de la más b) Monipodio queda ridiculizado como orador y líder al co-
extraña locura que entre las locuras hasta entonces se había meter notables incorrecciones lingüísticas. Prueba de su
visto. Imaginose el desdichado que era todo hecho de vidrio, incultura es que confunde términos como aniversario,
y con esta imaginación, cuando alguno se llegaba a él, daba pompa y solemnidad.
terribles voces, pidiendo y suplicando con palabras y razones
c) La complicidad del fragmento deriva de la inversión de
concertadas que no se le acercasen, porque le quebrarían; que
papeles (según el tópico del «mundo al revés») entre los de-
real y verdaderamente él no era como los otros hombres: que
lincuentes y los miembros de la justicia, transformados
todo era de vidrio, de pies a cabeza.
aquí en bienhechores. No es habitual que unos pícaros re-
zaran por aquellos que, supuestamente, deberían ser sus
1
alferecía: enfermedad similar a la epilepsia, que provoca des- perseguidores.
mayos y convulsiones. 9. Averigua cuál es el final de la segunda parte del Quijo-
te. ¿Cómo puedes relacionar esto de algún modo con la
a) ¿Qué sorprendentes efectos tienen en el personaje aparición de la falsa continuación escrita por Fernández
las artes perversas de la malhechora? Asimismo, ¿qué de Avellaneda?
explicación verosímil podría dársele a un suceso tan Al final de la segunda parte, don Quijote regresa a su casa
inaudito? derrotado y, poco después, enferma y recupera el juicio an-
tes de morir cristianamente. Si su muerte puede ser interpre-
b) Aparte de la extraña enfermedad que aflige al licen- tada como un suceso que apuntala aún más la verosimilitud de
ciado, ¿de qué otro modo deslumbra el personaje a la historia, no es menos cierto que con este desenlace Cervan-
todo el mundo? Busca información al respecto. tes cerraba las puertas a la posibilidad de que otros autores
a) Al ingerir el extraño membrillo, Tomás Rodaja tiene con- volvieran a continuar su relato como ya lo hizo el enigmático
vulsiones y luego sufre un desmayo. Pese a recuperar la Alonso Fernández de Avellaneda.
conciencia, permanece seis meses convaleciente. Pero lo 10. En el capítulo IX de la primera parte del Quijote, el narra-
que es peor, creyéndose ser de cristal y temiendo que su dor nos refiere el casual descubrimiento de unos papeles
cuerpo se resquebraje al menor contacto. Si a nivel ficticio,
que le servirán de gran utilidad para su tarea literaria.
puede llegar a pensarse en un caso de brujería, de modo
más verosímil cabe pensar en una misteriosa suerte de
¿En qué persona está narrado el siguiente fragmento?
envenenamiento. ¿Es omnisciente el narrador principal? ¿Por qué?
b) Asimismo, resultará sorprendente lo que ocurre con el li- Estando yo un día en el Alcaná de Toledo, llegó un mucha-
cenciado Vidriera tan pronto sale a la calle. Su aparente cho a vender unos cartapacios y papeles viejos a un sedero;
locura no le impide responder a las preguntas que le formu- y como yo soy aficionado a leer aunque sean los papeles
la la gente con singular lucidez, convirtiéndose sus juicios rotos de las calles, llevado de esta mi natural inclinación
en un instrumento de crítica social. En su doble condición tomé un cartapacio de los que el muchacho vendía y vile con
de loco-cuerdo, Tomás Rodaja es un personaje muy similar caracteres, que conocí ser arábigos. Y puesto que aunque
a don Quijote. los conocía no los sabía leer, anduve mirando si parecía por
allí algún morisco aljamiado1 que los leyese, y no fue muy
8. En Rinconete y Cortadillo, apenas ingresan los protago- dificultoso hallar intérprete semejante […]. En fin, la suerte
nistas en una cofradía de delincuentes, Monipodio les me deparó uno, que, diciéndole mi deseo y poniéndole el libro
recomienda lo siguiente: en las manos, le abrió por medio, y, leyendo un poco en él,
Y os pido asimismo que recéis por los bienhechores de nuestra se comenzó a reír. […].
cofradía, que son los guros1 que nos alertan de que la justi- Le di priesa que leyese el principio, y haciéndolo ansí, vol-
cia nos persigue y los jueces que hacen la vista gorda y los viendo de improviso el arábigo en castellano, dijo que

Lengua castellana y Literatura. 3.º ESO. Solucionario 91


10 AQUELLOS LOCOS TAN INGENIOSOS

decía: Historia de don Quijote de la Mancha, escrita por Cide no solo es más prudente, sino que rehúye a los peligros (co-
Hamete Benengeli, historiador arábigo. menzó a llorar y bien podemos torcer el camino y desviarnos
del peligro). Es, por tanto, menos idealista y más práctico, e
incluso cobarde.
1
morisco aljamiado: morisco que habla castellano.
13. ¿Qué tema de los mencionados reconoces en el siguiente
pasaje (Parte I, cap. XXI)? ¿En qué discrepan la opinión
El fragmento está narrado en primera persona: Estando yo. de don Quijote y la de su escudero?
No se trata, sin embargo, de un narrador-personaje, sino de un
narrador que habla de su propia tarea creativa y cuenta cómo De allí a poco, descubrió don Quijote un hombre a caballo, que
descubrió unos papeles en árabe que mandó traducir al cas- traía en la cabeza una cosa que relumbraba como si fuera de
tellano. De ese modo, al dar con una nueva fuente, aparte de oro, y aún él apenas le hubo visto, cuando se volvió a Sancho
conferirle a su relato un mayor verismo historiográfico, de- y le dijo:
muestra que no es un narrador omnisciente, ya que contrasta —[…] Si no me engaño, hacia nosotros viene uno que trae en
fuentes ajenas. su cabeza puesto el yelmo de Mambrino […].
11. A partir del motivo del manuscrito encontrado, Cervantes —Mire vuestra merced bien lo que dice, y mejor lo que hace
deja ver su tendencia a jugar con los tópicos heredados. —dijo Sancho— […]. Lo que yo veo […] no es sino un hombre
En el relato de una de las aventuras más cómicas de la pri- sobre un asno pardo, como el mío, que trae sobre sobre la
mera parte, se nos desvela la insólita identidad del arriero cabeza una cosa que relumbra.
que pretende mantener una relación amorosa con Mari- En el pasaje cobra vida el tema del perspectivismo, del con-
tornes, la fea criada de una venta. De él se dice: flicto entre los sentidos y las apariencias. Una misma realidad
puede ser interpretada de formas distintas. Mientras, a lo le-
Era uno de los ricos arrieros de Arévalo, según lo dice el autor
jos, Sancho solo divisa un hombre sobre un asno llevando en
desta historia [Cide Hamete], que deste arriero hace particular
la cabeza una cosa que relumbra, la imaginación libresca de
mención porque le conocía muy bien, y aun quieren decir que
don Quijote dicta que aquel individuo va montado sobre un
era algo pariente suyo.
caballo y lleva sobre la cabeza el yelmo de Mambrino.
a) Teniendo en cuenta este pasaje, ¿cómo podemos pre- 14. Gustavo Martín Garzo empieza el relato de Dulcinea y el
sumir que conoció el historiador arábigo los sucesos Caballero Dormido del modo siguiente:
ocurridos en la venta que él relatará sin haber estado
presente en dicho lugar? Ya nadie recuerda el caballero. Hace años sus aventuras es-
taban en boca de todos, y eran motivo de regocijo en ventas y
b) ¿Qué consigue Cervantes con esta estrategia? mercados, pero hoy nadie pregunta por él […]. Ahora que soy
a) A partir de este pasaje, puede presumirse que el historia- vieja, cuando veo a los niños del pueblo correr y jugar por las
dor árabe conoció unos sucesos en los que no había estado calles, me pregunto cómo podrán enfrentarse a la tristeza del
presente a partir de la versión de los mismos que pudo mundo sin la ayuda de alguien como él. También pienso en lo
facilitarle su pariente el arriero. afortunada que fui, pues entre todas las mujeres del mundo
me eligió a mí para transformarme en su dama. Dulcinea del
b) Con tal estrategia, Cervantes no solo juega con la comple- Toboso, ¿hay un nombre más gracioso y delicado que ese?
jidad de las voces narrativas, sino que le otorga a una de Suele decirse que Dulcinea no era de este mundo sino una figu-
sus fuentes principales una corporeidad física. De algún ra fantástica que el caballero engendró en su entendimiento,
modo venía a decirnos que el tal Cide Hamete podía ser pintándola con todas las gracias y perfecciones que acertó a
tan real, y además contemporáneo, como el arriero de la imaginar, pero esto no es cierto y basta para desmentirlo el
venta. que mi nombre y mi linaje aparezcan escritos sin veladura en
12. ¿Con qué rasgos describirías a los personajes que in- el libro que recorrería el mundo llevando noticias de su única
tervienen en este diálogo del capítulo XX de la primera y maravillosa vida.
parte? a) ¿Quién es el narrador en el fragmento citado? ¿Qué
—Sancho amigo, has de saber que yo nací por querer del cielo intenta demostrar sobre sí mismo?
en esta nuestra edad de hierro para resucitar en ella la de oro, b) ¿Cuál es su actitud hacia don Quijote? ¿Con qué tér-
o la dorada, como suele llamarse. Yo soy aquel para quien
minos la expresa?
están guardados los peligros, las grandes hazañas, los vale-
rosos hechos […]. Yo soy, digo otra vez, quien ha […] de poner a) El narrador es en este fragmento el personaje de la mis-
en olvido los Platires, los Tablantes, Olivantes y Tirantes, los ma Dulcinea del Toboso. Intenta demostrar que no es una
Febos y Belianises […]. figura fantástica.
Cuando Sancho oyó las palabras de su amo, comenzó a llorar b) Dulcinea evoca con nostalgia al caballero, hacia el que
con la mayor ternura del mundo y a decille: se siente agradecida (lo afortunada que fui) por haberla
convertido en su dama. Además, lamenta tanto la desa-
—Señor, yo no sé por qué quiere vuestra merced acometer
parición de don Quijote como que nadie se acuerde de él.
esta tan temerosa aventura. Ahora es de noche, aquí no nos ve
Para ella, el caballero fue una criatura única, capaz de
nadie: bien podemos torcer el camino y desviarnos del peligro,
enfrentarse en solitario a la tristeza del mundo.
aunque no bebamos en tres días.
15. Localiza y lee la pieza teatral «Sancho Panza en la ínsu-
Don Quijote se nos aparece como un ser idealista y soñador,
al sentirse destinado a resucitar la edad de oro. Es valien-
la», perteneciente al Retablo jovial, de Alejandro Casona.
te y no teme las posibles adversidades que puedan asaltarle Resúmela indicando cuáles son los valores que el autor
en su camino: Yo soy aquel para quien están guardados los destaca del famoso escudero.
peligros. En su vasto conocimiento de los más famosos ca- El resumen deberá dejar constancia de la reivindicación del
balleros literarios, demuestra ser, asimismo, un gran lector sentido común de Sancho para impartir, de forma instintiva,
de este tipo de literatura. Por su parte, su escudero Sancho justicia.

92 Lengua castellana y Literatura. 3.º ESO. Solucionario


AQUELLOS LOCOS TAN INGENIOSOS 10
COMENTARIO DE TEXTO La existencia de esa dorada edad a la que se refiere don Qui-
jote no solo es indemostrable, sino que parece un imposible.
Localización. Este fragmento está extraído del discurso de ¿Cómo, por ejemplo, podían saciar su hambre los antiguos con
la Edad de Oro que enlaza don Quijote, al ser acogido por los frutos que les daba la naturaleza y sin pelearse? ¿Quizá
unos cabreros, en el capítulo XI de la primera parte. A través entonces el hombre carecía de cualquier vicio o defecto? La
de su parlamento, el personaje rescata un tópico literario de creencia del personaje en tal utopía viene a destacar su exa-
procedencia clásica. cerbado idealismo.
1. ¿Qué relación existe entre el tópico de la Edad de Oro y EL JARDÍN DE LA LITERATURA
el mito de las Edades del Hombre?
Para el escritor griego Hesíodo, había cinco edades del hom- 1. En este fragmento (Parte I, capítulo II), el narrador del
bre. La Edad de Oro fue la primera de tales edades. En la Quijote reproduce las primeras palabras del protagonista
época imperial romana, autores como Virgilio redujeron a dos de la novela al abandonar su casa para convertirse en
las edades del mundo: la de Oro y la de Hierro. caballero andante:
Tema y estructura. Según lo dicho por los antiguos: Yendo, pues, caminando nuestro flamante aventurero, iba ha-
2. ¿Qué dos épocas se contrastan? ¿Cuáles son sus respec- blando consigo mismo y diciendo:
tivas características? ¿Cómo valorarías la evolución del —¿Quién duda sino que en los venideros tiempos, cuando sal-
mundo? ga a luz la verdadera historia de mis famosos hechos, que el
sabio que los escribiere no ponga, cuando llegue a contar esta
El discurso de don Quijote se ajusta a lo dicho por Virgilio
mi primera salida tan de mañana, desta manera?: «Apenas
y distingue dos épocas: una primera, dorada, remota y ex-
había el rubicundo Apolo tendido por la faz de la ancha y es-
cepcional; y otra, presente y detestable. En aquella, como to-
paciosa tierra las doradas hebras de sus hermosos cabellos,
dos los bienes terrenales eran comunes, a nadie le faltaba de
y apenas los pequeños y pintados pajarillos con sus harpadas
nada y no había conflictos de interés. En esta, el desarrollo
lenguas habían saludado con dulce y meliflua armonía la veni-
de la malicia, del amor por el oro y de otros vicios ha preci-
da de la rosada aurora, que, dejando la blanda cama del celoso
sado de la institución de la caballería andante para imponer
marido, por las puertas y balcones del manchego horizonte
la justicia en el mundo. Del contraste entre ambas etapas se
a los mortales se mostraba, cuando el famoso caballero don
desprende que el mundo ha experimentado una evolución ne-
Quijote de la Mancha, dejando las ociosas plumas, subió so-
gativa y descendente.
bre su famoso caballo Rocinante y comenzó a caminar por el
Análisis de la forma y el contenido. La evocación de tiem- antiguo y conocido campo de Montiel».
pos pretéritos se concreta mediante un armonioso ritmo
a) Según don Quijote, ¿quién recogerá por escrito sus
sintáctico.
aventuras? En cambio, ¿por qué crees que es el per-
3. Selecciona dos periodos enumerativos y analiza su estruc- sonaje quien se preocupa por aquello que debe decir
tura. ¿Qué se repite en ellos? Asimismo, ¿encuentras sin- en su libro?
tagmas en los que aparezcan dos palabras de una misma
b) Posiblemente, la forma de hablar de don Quijote te
categoría gramatical?
resultará un poco extraña, pues imita el estilo de los
La modulación sintáctica se evidencia mediante periodos libros de caballerías. Con ayuda de Internet o de alguna
paralelísticos que poseen, además, una función enfática.
enciclopedia, explica brevemente cómo era el estilo de
Adviértase la insistencia en la partícula negativa y los tres
complementos directos enumerados en No había la fraude,
dichas obras.
el engaño ni la malicia. Algo similar ocurre con la reitera- a) Don Quijote dice que sus aventuras las plasmará en un
ción, mediante coordinación copulativa, de dos subordinadas libro un sabio o mago, tal y como ocurría en los libros de
sustantivas de complemento directo: no había qué juzgar ni caballerías. Se preocupa tanto por lo que debe decir dicha
quién fuese juzgado. Asimismo, el personaje se muestra re- obra porque está obsesionado con la idea de superar a las
tórico al duplicar los adjetivos calificativos en sintagmas del figuras literarias que pretende imitar y desea transformar
tipo dulce y sazonado fruto y fértil y espacioso seno. su existencia en una auténtica obra de arte.
4. ¿Qué referencia del texto a un episodio de la mitología b) El estilo de los libros de caballerías era, en ocasiones, anti-
clásica ilustra la importancia de esta materia en el Rena- cuado. Otras veces, era enrevesado y complejo, con abun-
cimiento? dancia de oraciones subordinadas. Además, los autores
crearon un lenguaje bastante tipificado en cuanto al léxico
El texto se refiere al mito del laberinto de Creta, construido
y las estructuras sintácticas empleadas. De esto último es
por Dédalo y donde estaba encerrado el Minotauro al que lo-
reflejo el tópico del amanecer mitológico al que recurre
gró derrotar Teseo.
don Quijote, reproduciendo su misma retórica.
5. ¿Consideras la expresión «nuestra primera madre» una
2. En un magnífico estudio sobre el Quijote, el escritor mexi-
metáfora? ¿Por qué? Localiza alguna hipérbole.
cano Carlos Fuentes destacaba que el protagonista halla-
La expresión nuestra primera madre es el término imagina- ba en la lectura un asidero ante el desconcierto y la duda
rio con que se identifica el término real «la tierra». Puesto que
que asaltaban al hombre de la época:
ambos términos se equiparan podemos hablar de metáfora.
Por otra parte, en su intento de destacar las bondades de la La sinonimia de la lectura, la locura, la verdad y la vida en
mítica edad de oro, el caballero exagera aquel mundo paradi- don Quijote son de una evidencia llamativa cuando pide a los
síaco: Todo era paz entonces, todo amistad, todo concordia. mercaderes que se encuentra en el camino que confiesen la
belleza de Dulcinea sin haberla visto nunca, pues «lo impor-
Conclusiones tante es que sin haberla visto lo creyeres, confesares, jurares
6. ¿Con qué argumentos afirmarías que el discurso de don y defendieres». Ese lo es un acto de fe. Las fabulosas aventu-
Quijote tiene un valor utópico? ¿Qué rasgo del personaje ras de don Quijote son impulsadas por un propósito avasa-
destaca? llante: lo leído y lo vivido deben coincidir de nuevo, sin las

Lengua castellana y Literatura. 3.º ESO. Solucionario 93


10 AQUELLOS LOCOS TAN INGENIOSOS

dudas y oscilaciones entre la fe y la razón introducidas en el velarte 5, calzas de velludo 6 para las fiestas con sus pantu-
Renacimiento. flos7 de lo mismo, los días de entre semana se honraba con su
Cervantes o la crítica de la lectura vellorí8 de lo más fino. Tenía en su casa una ama que pasaba
de los cuarenta, y una sobrina que no llegaba a los veinte, y
a) ¿También a ti te merecen los libros la misma confianza un mozo de campo y plaza9, que así ensillaba el rocín como
que depositaba en ellos don Quijote? ¿Por qué? tomaba la podadera. Frisaba10 la edad de nuestro hidalgo con
b) ¿Qué podría ocurrir si el mundo de tus lecturas no se los cincuenta años, era de complexión recia, seco de carnes,
ajustase a la realidad? enjuto11 de rostro; gran madrugador y amigo de la caza. Quie-
a) y b) Respuesta libre. ren decir que tenía el sobrenombre de Quijada o Quesada (que
en esto hay alguna diferencia en los autores que de este caso
3. En La Galatea, Cervantes reproduce los tópicos de la escriben), aunque por conjeturas verosímiles se deja enten-
literatura pastoril, incorporando con frecuencia poesías der que se llama Quijana; pero esto importa poco a nuestro
y parlamentos con gran lirismo: cuento; basta que en la narración de él no se salga un punto
de la verdad.
El lastimado pastor, creyendo que ninguno le oía, soltó la voz
a semejantes razones: Quijote, Parte I, capítulo I.

—¡Amor, […] ya que tanto bien me hiciste, no quieras mos-


trarte agora, haciéndome el mal en que me amenazas, que es 1
astillero: estante para dejar la lanza. 2 adarga: escudo de
más mudable tu condición que la de la variable Fortuna. Mira, cuero. 3 duelos y quebrantos: huevos con tocino. 4 palomino:
señor, cuán obediente he estado a tus leyes, cuán pronto a pichón. 5 sayo de velarte: traje de paño. 6 velludo: terciope-
seguir tus mandamientos, y cuán sujeta he tenido mi voluntad lo. 7 pantuflos: prenda que cubría los zapatos. 8 vellorí: tela de
a la tuya. paño entrefino. 9 de campo y plaza: que servía para todo. 10 fri-
¡Oh verdes prados, que con su vista os alegrábades! ¡Oh flores saba: se aproximaba a. 11 enjuto: delgado.
olorosas, que de sus pies tocadas, de mayor fragancia érades
llenas! ¡Oh plantas, oh árboles desta deleitosa selva!, haced a) ¿Qué tipo de descripción nos ofrece el narrador? Jus-
todos, en la mejor forma que pudiéredes, aunque a vuestra
tifica tu respuesta.
naturaleza no se conceda, algún género de sentimiento que
mueva al cielo a concederme lo que le suplico! b) Según la imagen que se nos proporciona del protago-
a) ¿A qué famosas composiciones líricas de Garcilaso de nista de la historia, ¿en qué se podría distinguir inme-
la Vega te recuerdan los lamentos del pastor? diatamente de los caballeros andantes medievales o
de cualquier héroe del cine de aventuras actual?
b) ¿A quiénes les traslada el pastor sus penas en este
caso? ¿Encuentras alguna personificación? c) ¿Cuál dirías que es su situación económica? Justifica
tu respuesta con ejemplos.
a) Los lamentos del pastor cervantino recuerdan aquellos
otros de Salicio y Nemoroso en las églogas garcilasianas. d) Por su parte, ¿es acaso el narrador el primero que
b) El personaje, después de declararse vasallo del Amor y
escribe sobre el hidalgo? En todo caso, ¿cómo espera
quejarse, le confía sus penas a los prados, las flores, plan- que sea su relato?
tas y árboles, elementos todos de una naturaleza bucóli- e) Como en la mayoría de las descripciones, el narrador
ca, para que se conmuevan de su desdicha. Aparte de la recurre a palabras o sintagmas con valor especificati-
tópica personificación del Amor, también los elementos vo. Anota en tu cuaderno al menos cinco adjetivos del
naturales están personificados, en tanto que se transfor- texto, así como otros recursos que emplee el narrador
man en los confidentes del pastor. para proporcionar detalles sobre la realidad descrita.
LA FACTORÍA DE TEXTOS f) Los ejemplos que acabas de enumerar, ¿poseen un
carácter más objetivo o más subjetivo?
1. Escribe en diez líneas un retrato completo de tu mejor
a) El narrador nos ofrece un retrato del hidalgo manche-
compañero de clase.
go. Se da este tipo de descripción, porque, junto a los
Respuesta libre. rasgos físicos del personaje (prosopografía), se alude
2. Aplica los rasgos característicos de la descripción científi- también a sus costumbres y aficiones, por ejemplo, la
co-técnica para informar a tus compañeros de las virtudes caza (etopeya).
de un instrumento imaginario que acabas de inventar. b) La imagen del personaje resulta mucho menos atractiva
Respuesta libre. de la que se puede esperar en un héroe. Su complexión
recia, seco de carnes, enjuto de rostro y su edad madu-
ACTIVIDADES FINALES ra, además de otorgarle una apariencia poco agraciada,
ponen en duda su capacidad para soportar el peso de las
Repasa lo que has aprendido armas y para acometer empresas donde es fundamental
1. A continuación, te invitamos a leer el pasaje con que co- el uso de la fuerza.
mienza la primera parte del Quijote. Luego, resuelve las c) Como hidalgo que es, el personaje pertenece al escalón
cuestiones que se te plantean. más bajo de la nobleza. Por eso puede sobrellevar una
existencia modesta. De ahí que el narrador insista en la
En un lugar de la Mancha, de cuyo nombre no quiero acordar-
rutina de su menú semanal o en la poca variedad de su
me, no ha mucho tiempo que vivía un hidalgo de los de lanza
armario.
en astillero1, adarga2 antigua, rocín flaco y galgo corredor.
Una olla de algo más vaca que carnero, salpicón las más no- d) Supuestamente, el narrador no es el primero que ha escrito
ches, duelos y quebrantos3 los sábados, lentejas los viernes, sobre las peripecias del hidalgo, pues menciona otros au-
algún palomino4 de añadidura los domingos, consumían las tores que de este caso escriben. En todo caso, espera que
tres partes de su hacienda. El resto de ella concluían sayo de su relato sea verosímil y, sobre todo, se ciña a la verdad.

94 Lengua castellana y Literatura. 3.º ESO. Solucionario


AQUELLOS LOCOS TAN INGENIOSOS 10
e) En el fragmento aparecen adjetivos como antigua, flaco, las uñas y los dientes, por cuyo dolor don Quijote comenzó a
corredor, recia, seco, enjuto. Además, abundan los sin- dar los mayores gritos que pudo.
tagmas preposicionales en función de complemento del Quijote, Parte II, capítulo XLVI.
nombre: de los de lanza en astillero, de velarte, de vellu-
do; pero también sintagmas en función de complemento
del adjetivo: de carnes, de rostro. Asimismo, se reconocen
1
a plomo: verticalmente. 2
mayar: maullar.
proposiciones subordinadas adjetivas especificativas: que
pasaba de los cuarenta, que no llegaba a los veinte, que a) ¿Cómo reacciona don Quijote ante la gatuna inven-
predominan sobre las adjetivas explicativas como de cuyo
ción? ¿En qué sentido se reafirma su locura?
nombre no quiero acordarme.
b) Por su parte, ¿te parece cruel la burla de los duques?
f) En general, los ejemplos enumerados poseen un carácter
¿Por qué?
más objetivo.
c) ¿A qué obligaciones debían atender los nobles en la
2. De entre los temas del Quijote señalados en la unidad, literatura medieval y en los libros de caballerías para
identifica el que aparece en el siguiente pasaje. Justifica alcanzar la fama? ¿Por qué la conducta de los duques
tu respuesta. resulta indigna?
Viva la memoria de Amadís, y sea imitado de Don Quijote de la
d) Analiza los siguientes sintagmas nominales:
Mancha en todo lo que pudiere; del cual se dirá lo que del otro
se dijo, que si no acabó grandes cosas murió por acometerlas; • casi toda la gente del castillo
y si yo no soy desechado, ni desdeñado de mi Dulcinea, básta- • un gran saco de gatos
me, como ya he dicho, estar ausente della. Ea pues, manos a
la obra, venid a mi memoria, cosas de Amadís, y enseñadme • los mayores gritos que pudo
por donde tengo de comenzar a imitaros. e) ¿Qué tiempo verbal predomina en las narraciones?
Quijote, Parte I, capítulo XXVI. ¿Con qué otros tiempos alterna en este texto? Fíjate,
por ejemplo, en la distinta función de las formas fue,
El fragmento desarrolla el motivo de la imitación, estrategia
ardían o soy.
fundamental para el personaje, a partir de la cual cree que
será reconocido por la posteridad. f) Según su modalidad, ¿qué tipo de oraciones predomi-
nan en el texto? ¿Hay alguna frase imperativa o exhor-
3. Elabora una descripción técnica de un molino de viento tativa? ¿Cuál?
y una descripción literaria en la que lo compares con un
gigante. g) Indica la función sintáctica de los términos marcados
en negrita:
Respuesta libre.
• don Quijote quedó pasmado
Recuerda lo que ya sabías • fue tan grande el ruido
4. En la segunda parte del Quijote son varios los persona- • la gente del castillo estaba suspensa
jes que intentan alimentar la locura del protagonista para • les tiró muchas cuchilladas
burlarse de él. Así ocurre con la siguiente invención de
• le asió de las narices con las uñas
los duques:
h) Localiza en el texto al menos dos oraciones compues-
Aquí llegaba don Quijote de su canto, a quien estaban escu-
chando el duque y la duquesa, Altisidora y casi toda la gente
tas e indica de qué tipo son.
del castillo, cuando de improviso, desde encima de un corre- a) En primera instancia, don Quijote queda pasmado e in-
dor que sobre la reja de don Quijote a plomo1 caía, descolgaron cluso atemorizado. Pero, de inmediato, se acerca a la reja
un cordel donde venían más de cien cencerros asidos, y luego de su habitación. Su locura se ve reafirmada porque con-
tras ellos derramaron un gran saco de gatos, que asimismo sidera que el episodio es una invención de los malvados
traían cencerros menores atados a las colas. Fue tan grande encantadores que le persiguen.
el ruido de los cencerros y el mayar2 de los gatos, que aunque b) Respuesta libre.
los duques habían sido inventores de la burla, todavía les so-
c) En la literatura señalada, los nobles se definían como be-
bresaltó, y, temeroso, don Quijote quedó pasmado. Y quiso la
llatores. Esto es, tenían a su cargo el mantenimiento de la
suerte que dos o tres gatos se entraron por la reja de su es-
justicia, la integridad de sus señores y la defensa de la fe
tancia, y dando de una parte a otra parecía que una región de
cristiana. Encargados de preservar el orden establecido,
diablos andaba en ella: apagaron las velas que en el aposento
luchaban contra cualquier agresión con las armas, puesto
ardían y andaban buscando por do escaparse. El descolgar
que eran los únicos que podían permitirse su posesión y,
y subir del cordel de los grandes cencerros no cesaba; la ma-
además, estaban entrenados para su manejo. Por su parte,
yor parte de la gente del castillo, que no sabía la verdad del
los duques se comportan de forma indigna porque se apro-
caso, estaba suspensa y admirada.
vechan de su poder para divertirse a costa de don Quijote.
Levantose don Quijote en pie y, poniendo mano a la espada,
d) • casi toda la gente del castillo: Det + Det + Det + N + CN
comenzó a tirar estocadas por la reja y a decir a grandes voces:
(enlace + término-SN [Det + N]).
—¡Afuera, malignos encantadores! ¡Afuera, canalla hechice-
resca, que yo soy don Quijote de la Mancha, contra quien no • un gran saco de gatos: Det + Ady + N + CN (enlace +
valen ni tienen fuerza vuestras malas intenciones! término-SN [N]).

Y volviéndose a los gatos que andaban por el aposento les • los mayores gritos que pudo: Det + Ady + N + Sub. adj.
tiró muchas cuchilladas. Ellos acudieron a la reja y por allí se especificativa-Ady.
salieron, aunque uno, viéndose tan acosado de las cuchilladas e) En las narraciones predominan los verbos en pretérito per-
de don Quijote, le saltó al rostro y le asió de las narices con fecto simple de indicativo. En el texto, alterna con el pre-

Lengua castellana y Literatura. 3.º ESO. Solucionario 95


10 AQUELLOS LOCOS TAN INGENIOSOS

térito imperfecto de indicativo (ardían) y el presente de 2. Durante sus aventuras, don Quijote recorrió diversos
indicativo (soy). escenarios de la geografía manchega. Así, en la obra se
f) La modalidad oracional predominante es la enunciativa. mencionan poblaciones como El Toboso, Argamasilla,
Cuando irrumpe el estilo directo, sin embargo, observamos Puerto Lápice o Tembleque, y lugares como la comarca de
la presencia de la modalidad imperativa en ¡Afuera, canalla Campo de Montiel, la Cueva de Montesinos o las Lagunas
hechiceresca, que yo soy don Quijote…! de Ruidera. Elabora un mapa donde aparezcan situados
g) • pasmado: predicativo. dichos espacios.
• el ruido: sujeto. Respuesta libre.
• suspensa: atributo.
3. Don Quijote imaginaba que, a través de sus hazañas, podría
• les: complemento indirecto. alcanzar el amor de la «sin par Dulcinea del Toboso», moza
• con las uñas: complemento circunstancial de instru- labradora por la que, en un tiempo, estuvo interesado y
mento. cuya fantasía convertirá en una especie de diosa. Busca en
h) • Subordinada adverbial de tiempo: cuando de improviso el capítulo XXV de la primera parte la descripción que rea-
[…] descolgaron un cordel... liza de ella Sancho Panza y resúmela, distinguiendo entre
• Subordinada sustantiva de complemento directo: que dos sus rasgos físicos y morales. ¿En qué sentido te parece
o tres gatos se entraron por la reja de su estancia… acertada la imagen de Dulcinea de la Figura 10.11?
• Subordinada adjetiva especificativa: que andaban por Según la opinión de Sancho, Dulcinea es una mujer fuerte,
el aposento… robusta y con una voz que se oye a lo lejos. Si sus rasgos
físicos le dan un aspecto hombruno, sus cualidades morales
MIRA A TU ALREDEDOR Y…
(etopeya) se destacan con acento equívoco. Su fortaleza de
… ve más allá carácter es capaz de sacar de cualquier aprieto a un caballero,
pero su actitud poco melindrosa y su tendencia a burlar y a
1. En el capítulo I de la primera parte del Quijote, se nos
mostrarse donosa con el otro sexo hacen pensar también en
dice que, llevado por su deseo de ser caballero andante, una predisposición innata a mantener relaciones sexuales. Se
el hidalgo Alonso Quijano limpió: trata de una descripción totalmente opuesta a la que sublimó,
Unas armas, que habían sido de sus bisabuelos, que tomadas por ejemplo, la lírica garcilasista de mujer de cabellos dora-
de orín1 y llenas de moho, luengos2 siglos había que estaban dos, cutis muy blanco y labios rojos como el clavel. En todo
puestas y olvidadas en un rincón. caso, coincide con la imagen reproducida en su apariencia rús-
tica y en su desvío de las líneas estilizadas de la fémina ideal.
1
tomadas de orín: oxidadas. 2
luengos: largos. 4. Indudablemente, don Quijote es uno de los personajes
literarios cuyo recuerdo ha perdurado más en el tiempo.
Para cumplir con su misión, don Alonso necesitaba unos Por ejemplo, Rubén Darío le dedicó su «Letanía de nues-
instrumentos militares. Pero su relación con las armas ya tro señor don Quijote», mientras que León Felipe lo eligió
está planteada en la elección de su nombre caballeresco. como protagonista del poema «Vencidos», del que pue-
¿Qué pieza de la armadura medieval era un quijote? ¿Para des encontrar en YouTube una versión musical realizada
qué servía? Enumera otras cuatro piezas de una armadura por Joan Manuel Serrat. Lee ambos poemas y escucha la
y explica su función. canción. ¿Qué sentimientos despierta en los respectivos
El quijote era una pieza de la armadura que servía para pro- emisores el caballero manchego? ¿Qué aspectos se des-
teger el muslo de los caballeros. Para otros elementos de la tacan de su figura?
armadura, véase la siguiente ilustración: Respuesta libre.
Celada
5. Asimismo, famosos pintores han hallado motivo de ins-
Visera piración en el personaje cervantino, e incluso alguno de
Babera ellos llegó a ilustrar con sus dibujos y pinturas ediciones
Gola cuidadas de las dos partes de la historia. En el portal digi-
Hombrera tal Quijote Banco de Imágenes (www.qbi2005.com),
Ristre descubrirás un rico tesoro iconográfico con ilustracio-
Peto
Guardabrazo nes y grabados sobre los personajes y las aventuras de la
Codal obra. A partir de la consulta de esta página te propone-
mos dos actividades alternativas. Tras la selección de las
Avanbrazo imágenes oportunas:
a) Elabora un mural temático.
b) Realiza un breve cómic en soporte digital, incorpo-
Quijote
rando diálogos sobre las imágenes elegidas.
Respuesta libre.
Rodillera 6. Los libros cervantinos también han hallado su lugar en
el cine. Apoyándote en la información suministrada por
la red, elabora un listado de versiones cinematográficas
Greba
Espuela (distinguiéndolas según el público al que van dirigidas),
a partir de las cuales podrás demostrar su proyección
internacional.
Escarpe
Respuesta libre.

96 Lengua castellana y Literatura. 3.º ESO. Solucionario


AQUELLOS LOCOS TAN INGENIOSOS 10
… encuentra la clave c) Según afirma Mario, de Numancia «debe ser eterna la
1. Lee los siguientes tercetos, extraídos de la jornada IV de memoria». La fama se antoja un motivo recurrente en
La Numancia. En ellos, el soldado romano Mario elogia la vida y obra de Cervantes. ¿Cómo pretendió conse-
a los habitantes de la ciudad sitiada por el modo en que guirla don Miguel? ¿Y don Quijote? Pese a los infor-
han puesto fin a su tenaz resistencia contra las tropas tunios con que tuvieron que bregar el uno y el otro,
invasoras: ¿por qué dirías que alcanzaron su objetivo?
En balde, ilustre general prudente, d) En la actualidad, ¿perdura la aspiración del heroísmo?
han sido nuestras fuerzas ocupadas, ¿A través de qué actividades, oficios o empresas pue-
en balde te has mostrado diligente1, des contribuir a mejorar la sociedad que te rodea?
pues en humo y en viento son tornadas Escribe una redacción de unas cien palabras en la que,
las ciertas esperanzas de victoria, después de señalar cuáles son los principales proble-
de tu industria2 contino aseguradas. mas que azotan al mundo, aportes posibles soluciones.
Del lamentable fin y triste historia a) Respuesta libre.
de la ciudad invicta de Numancia b) También los saguntinos optaron por el suicidio colectivo
merece ser eterna la memoria: después de resistir ocho meses de asedio. En su caso, los
sacado han de su pérdida ganancia; romanos no eran sus enemigos, sino que, teóricamente,
quitado te han el triunfo de las manos, eran sus aliados, aunque renunciaron a acudir en su ayuda.
muriendo con magnánima3 constancia. Siendo diferente la nacionalidad de los sitiadores, romanos
Nuestros disignios han salido vanos, en Numancia y cartagineses en Sagunto, quienes ofrecen
pues ha podido más su honroso intento su vida en sacrificio de su reputación lo hacen impidien-
que toda la potencia de romanos; do que sus rivales puedan exhibirlos como muestra de su
triunfo militar.
el fatigado pueblo en fin violento
acaba la miseria de su vida, c) Cervantes buscó la fama distinguiéndose como soldado
dando triste remate al largo cuento. y como escritor. Mientras tanto, don Quijote quiso obte-
nerla convirtiéndose en caballero andante. Tanto el uno
Numancia está en un lago convertida
como el otro lograron su objetivo, curiosamente, después
de roja sangre, y de mil cuerpos llena,
de su muerte. Cervantes se vio eclipsado en vida por la fi-
de quien fue su rigor4 proprio homicida;
gura, por ejemplo, de Lope de Vega. Por su parte, su ficticia
de la pesada y sin igual cadena criatura, que legalmente no podía ser considerada como
dura de esclavitud se han escapado auténtico caballero porque su investidura fue un comple-
con presta5 audacia, de temor ajena. to escarnio, terminaría siendo percibido como un verda-
En medio de la plaza levantado dero caballero andante, pese a los fracasos que tuvo que
está un ardiente fuego temeroso, sortear.
de sus cuerpos y haciendas sustentado. d) Respuesta libre.
Prensas Universitarias de Zaragoza 2. Con la colaboración de tus compañeros de aula, enumera
las posibles deficiencias que haya en tu colegio o en tu
1
diligente: resuelto y laborioso. 2 industria: habilidad. 3
mag- barrio. Por ejemplo, piensa en personas que pueden tener
nánima: noble. 4 rigor: severidad. 5 presta: rápida. dificultades para integrarse socialmente u otras que care-
cen de los recursos económicos necesarios para llevar
una vida digna.
a) En opinión del personaje, la población numantina ha
a) ¿De qué modo podríais actuar como grupo para ayu-
vencido al enemigo optando por un suicidio colectivo.
darlas a salir de su situación?
¿Te parece una interpretación lógica? ¿Por qué?
b) ¿Qué tipo de recompensa obtendríais en caso de
b) En el siglo III a. C., también la próspera ciudad de
alcanzar el objetivo propuesto?
Sagunto resistió heroicamente contra las tropas del
general Aníbal. Busca información sobre este episodio c) ¿Sería la vuestra una conducta heroica? Justifica tu
histórico e indica qué semejanzas existieron entre este respuesta.
asedio y el de Numancia. a), b) y c) Respuesta libre.

Lengua castellana y Literatura. 3.º ESO. Solucionario 97


11 ENTRE PÍCAROS Y POETAS

APERTURA DE UNIDAD serlo. Quienes no podían hacer ostentación de esos lujos no


eran nada en esa corte de las apariencias.
1. Fíjate en las condiciones de todo lo que rodea a la comida
del protagonista y haz una tabla en la que recojas cada 5. ¿Por qué le parece asombroso a Carlos I el recibimiento
elemento y las características negativas que se le adjudi- que le hacen?
can en el texto. Sin duda llamaría su atención el espectáculo de carrozas y no-
bles en procesión recorriendo las principales calles de Madrid,
Elemento Características negativas como demostración de los lujos y de las grandezas del imperio
español, en contraste con la rigidez de costumbres que luego le
banquillo cojo muestran Olivares y los consejeros al impedirle que se acerque
a saludar a la infanta.
[mantel] barredero de horno
6. ¿Cuál es el papel del pueblo en la celebración que se des-
salero suelo de cántaro
cribe en el texto? ¿Piensas que se nos muestra una división
agua tiesto de gallinas clasista de la sociedad de la época? ¿Por qué?
hogaza negra El pueblo tiene el papel de espectador, de mero comparsa en
medio de los grandes de España, de los que son «algo», a quie-
tortilla emplastro nes se limita a mirar y tal vez a admirar. El narrador nos lo
presenta como testigo de las celebraciones fastuosas de la
2. ¿Por qué crees que el narrador-protagonista termina di- Corte.
ciendo que tuvo todo aquello «por buena suerte»? Razona 7. ¿Qué personajes de los que se mencionan en el texto son
tu respuesta. los que van a contraer matrimonio?
Respuesta libre. Van a contraer matrimonio el príncipe de Gales, Carlos I, y
3. Recuerda las características de la novela picaresca y ex- la infanta de España, doña María, hermana del rey Felipe IV.
plica cuáles de ellas están presentes en este fragmento.
Respuesta libre.
Vocabulario
4. ¿Cómo crees que sería la venta en la que el protagonis- 8. Explica con tus palabras en qué consistía «hacer la rúa».
ta estuvo comiendo? Haz una redacción de unas quince Busca en el diccionario la palabra rúa y asocia su signifi-
líneas en la que imagines esta venta y utilices sobre todo cado a la expresión anterior.
la descripción. Respuesta libre.
Respuesta libre. 9. Explica el significado de la palabra granado en la expre-
sión «lo más granado de la Corte». Puedes utilizar un dic-
VIVE LA LECTURA cionario e informarte sobre la procedencia de la palabra.
Comprensión lectora Lo más granado de la Corte hace referencia a las personas
principales. Granado, en su segunda acepción según el DRAE,
1. ¿Qué actividades hacían las gentes en el Prado de San
significa notable, señalado, principal o ilustre. Viene de gra-
Jerónimo?
nar, que significa producir granos algunas plantas, es decir,
El Prado de San Jerónimo era lugar de paseo, en el que las crear su futuro fruto.
gentes concertaban citas y practicaban el galanteo, se encon-
traban los enamorados y se reunía la sociedad de la época. 10. Busca en el primer párrafo los nombres que se utilizan
para hacer referencia a los distintos tipos de vías públi-
2. ¿Quién es Carlos I? ¿Cuál es su papel en el texto que aca- cas, como calle, y, con la ayuda del diccionario, copia las
bas de leer? definiciones en tu cuaderno.
Carlos I era el príncipe de Gales, pretendiente de la infanta Nombres que designan vías públicas: rúa, carrera, calle, vía.
doña María, la hermana de Felipe IV. Llega a Madrid para
conocer a su futura esposa, y al rey español le sirve de pre- • rúa: calle de un pueblo o camino para carruajes.
texto su llegada para organizar un desfile y una fiesta para • carrera: camino real o carretera.
recibirlo, aunque dentro del más estricto protocolo de la corte
• calle: vía pública en una ciudad.
de los Austrias. Esto último se demuestra, sobre todo, cuando
Carlos I pide saludar a su prometida y el narrador nos hace • vía: camino por donde se transita.
ver que eso era impensable según la recta moral de Felipe IV. 11. ¿Con qué nombre conocemos a las palabras que tienen
3. Según el texto, ¿qué normas regían el comportamiento entre sí una relación como la que se observa en las de la
de la corte española en época de Felipe IV? actividad anterior?
Los límites y el decoro del protocolo de la corte española era Este tipo de palabras son sinónimos, pues existe una identidad
muy rígidos según el texto, ya que todas las actividades que o similitud en sus significados.
debía realizar la familia real estaban establecidas de antema- 12. En el texto se nos dice que «el pueblo ofició como testi-
no, de acuerdo con unos hábitos que regían cada uno de sus go de aquella exhibición caballeresca»; ¿qué significa la
movimientos.
palabra ofició?
4. ¿Qué crees que quiere decir el narrador con «todo aquel Oficiar es una palabra que está relacionada con las celebra-
que era algo en la Corte»? Razona tu respuesta. ciones religiosas y significa ayudar en una misa o en otra
Cuando el narrador emplea la expresión todo aquel que era solemnidad litúrgica. Aquí está usada con sentido figurado y
algo en la Corte se refiere a todas aquellas personas que te- viene a dar a entender que el pueblo contribuyó a esa exhibi-
nían importancia o relevancia, ya fueran nobles, damas, ca- ción caballeresca con su presencia y su admiración ante las
balleros o cualquier otro personaje principal o que aparentara grandezas de la Corte.

98 Lengua castellana y Literatura. 3.º ESO. Solucionario


ENTRE PÍCAROS Y POETAS 11
13. La palabra corte aparece en el texto escrita tanto con 4. Lee la siguiente octava real de la Fábula de Polifemo y
mayúscula como con minúscula inicial. ¿Por qué crees que Galatea y contesta a las preguntas:
sucede esto? Infórmate sobre los distintos significados Un monte era de miembros eminente
de esta palabra y construye luego una oración con cada este (que de Neptuno hijo fiero,
uno de ellos. de un ojo ilustra el orbe de su frente,
Como otros nombres que designan entidades o instituciones émulo casi del mayor lucero)
del estado, la palabra corte se escribe con mayúscula cuando cíclope, a quien pino más valiente,
hace referencia a esa institución y no lleva ningún complemen- bastón le obedecía tan ligero,
to: la Corte, frente a la corte española. Puede tener un valor y al grave peso junco tan delgado,
distintivo, para diferenciar la institución de otros significados que un día era bastón y otro cayado.
que se asocian a la misma palabra. Algunos de esos otros a) ¿Qué rasgos destaca Góngora en la descripción del
significados son: filo de un cuchillo, acción de cortar, herida, gigante mitológico? ¿Cuántos ojos tiene?
corral donde se recoge el ganado, etc.
b) ¿En qué distingues su estilo culterano? Localiza e iden-
Respuesta libre. tifica tres recursos literarios usados por el autor.
c) Reescribe tú el fragmento de la forma más natural po-
Investigación y redacción
sible.
14. Infórmate sobre los espacios reales que se citan en el d) Investiga sobre el ser mitológico Polifemo e indica cuá-
texto y que se sitúan en Madrid: les de sus características aparecen en el fragmento.
a) Busca información sobre ellos y descubre si siguen exis- a) Entre los rasgos sobresalientes en la descripción del gi-
tiendo, han cambiado de nombre o han desaparecido. gante, vemos que el autor lo identifica con un monte y dice
que utiliza como bastón un fuerte pino. De él nos dice que
b) Localiza estos espacios en un plano de Madrid e inten-
solo tiene un ojo.
ta trazar el itinerario que se cita en el texto.
b) El culteranismo lo vamos a identificar en el uso de deter-
a) y b) Respuesta libre. minados recursos literarios como los siguientes:
15. Infórmate sobre quiénes fueron Lerma y Olivares y so- • hipérbaton (cambio del orden lógico de la frase): un
bre cuál fue su papel en la España del siglo XVII. ¿Crees monte era de miembros eminente, en lugar de «era
que fue beneficioso o perjudicial para el país? Razona tu un eminente monte de miembros».
respuesta. • metáfora: en la misma identificación del gigante con un
Respuesta libre. monte.
• uso de una sintaxis compleja, como en cíclope, a quien
pino más valiente, / bastón le obedecía tan ligero,
LITERATURA donde, además del desorden de los términos (hipérba-
ton), vemos la ausencia de palabras; tal vez faltaría un
1. Busca información y define los siguientes tópicos litera- como comparativo entre los dos versos, que facilitaría
rios, acompañándolos de una imagen o de un texto ilus- la comprensión del símil que se produce entre el pino y
trativo: «pañales y mortaja», vanitas vanitatum y «el mun- el bastón.
do al revés». c) Respuesta libre.
• «Pañales y mortaja» hace referencia a la brevedad de la d) Respuesta libre.
vida, a la fugacidad del tiempo, que junta casi en un instan-
te el momento del comienzo de la vida (los pañales) con la
5. Escucha el soneto de Quevedo que comienza con el verso
muerte (la mortaja, que es la ropa con la que se viste a un «Cómo de entre mis manos te resbalas» (lo puedes encon-
difunto antes de enterrarlo). trar en YouTube). ¿En qué bloque temático de la poesía
quevedesca incluirías este soneto y por qué? ¿A quién o
• Vanitas vanitatum: vanidad de vanidades; se define así,
quiénes se dirige el poeta en tono de reproche?
en el Barroco, a la existencia humana, como algo vacío y
sin sentido, como consecuencia del pesimismo existencial El soneto se incluye dentro de la poesía filosófica y moral de
que domina a esta época: nada parece tener sentido ante Quevedo, ya que el autor medita sobre la brevedad y la fragi-
la certeza de la muerte. lidad de la existencia humana.
• «El mundo al revés» es otro tópico barroco que se centra en El reproche del poeta va dirigido a la muerte, que se acerca
la idea del caos, del desorden, de que nada es lo que parece a traición con sus mudos pasos, y a la vida, que pasa rápi-
y todo es cambiante. damente, y se resbala entre las manos de poeta, según él
mismo dice.
Imágenes y textos: respuesta libre.
6. Las disputas entre Quevedo y Góngora dieron lugar a poe-
2. Comenta la posible relación del cuadro de Bartolomé mas satíricos intercambiados entre ambos. Así comienza un
Murillo (Figura 11.2) con las circunstancias históricas de soneto de Quevedo: «¿Qué captas, noturnal, en tus can-
la época. ciones, / Góngora bobo, con crepusculallas, / si cuando
Respuesta libre. anhelas más garcivolallas, / las reptilizas más y subterpo-
nes?». ¿De qué manera se burla Quevedo en estos versos?
3. Buena prueba del interés barroco hacia la literatura es la
¿Crees que parodia la poesía de Góngora? ¿Por qué?
existencia en el Madrid de la época de los mentideros.
Busca información sobre ellos y trata de localizar los más La burla de Quevedo hacia Góngora, en estos versos, está
basada en la imitación del lenguaje recargado y comple-
importantes que existieron en dicha ciudad.
jo del poeta cordobés, al que se dirige citándolo y tachándolo
Respuesta libre. de bobo.

Lengua castellana y Literatura. 3.º ESO. Solucionario 99


11 ENTRE PÍCAROS Y POETAS

Se trata de una evidente parodia, ya que vemos cómo imita


Autor Título Protagonista
burlescamente el hipérbaton (las reptilizas más y subter-
pones), utiliza neologismos de su invención que recuerdan el Lazarillo de Tormes
Anónimo Masculino
lenguaje excesivamente culto de Góngora (garcivolallas, sub- (1554)
terpones, crepusculallas...) y hace referencia a la oscuridad Segunda parte de
de los versos de su rival, difíciles de entender, con palabras Anónimo Lazarillo de Tormes Masculino
que sugieren la noche: noturnal, crepusculallas. (1555)

7. Uno de los males del amor es la ausencia, la separación de Mateo Alemán


Guzmán de Alfarache
Masculino
los enamorados. Fíjate en los versos iniciales de un soneto (1599 y 1604)
de Lope de Vega: «Ir y quedarse y con quedar partirse, Mateo Luján de Guzmán de Alfarache
Masculino
/ partir sin alma y ir con alma ajena». Analiza la confusión Sayavedra (apócrifo, 1602)
que producen las antítesis y relaciónala con el estado de Gregorio González El Guitón Onofre (1604) Masculino
ánimo de los enamorados que se han tenido que separar
Francisco de Úbeda La pícara Justina (1605) Femenino
por un tiempo.
Francisco de
La antítesis provoca en estas palabras de Lope de Vega una Quevedo
El Buscón (c. 1606) Masculino
confusión que contradice lo que realmente se quiere decir. Es
un juego de contrarios que, mezclados, parecen hacer imposi- Alonso Jerónimo La ingeniosa Elena
Femenino
ble aquello que pretenden comunicar. De esta forma se refle- de Salas Barbadillo (1612-1614)
jan las incertidumbres del amor distante, de la separación de Vida del escudero Marcos
Vicente Espinel Masculino
los enamorados que, aunque se van, realmente se quedan: se de Obregón (1618)
van físicamente, pero se quedan en espíritu junto a la persona
Segunda parte de
amada, de ahí lo de partir sin alma y ir con alma ajena, pues Juan de Luna Lazarillo de Tormes Masculino
se produce un intercambio según el cual el que se va deja su (1620)
alma con la amada, mientras esta envía la suya con el amado.
En definitiva, un ser y no ser, un estar y no estar: la tristeza Juan Cortés de Lazarillo de Manzanares
Masculino
Tolosa (1620)
y la melancolía de los enamorados que se tienen que separar
contra su voluntad. Jerónimo de Alcalá Alonso, mozo de muchos
Masculino
Yáñez amos (1624)
8. El siguiente fragmento del Buscón nos muestra las activi-
dades delictivas del padre del protagonista: La niña de los embustes,
Alonso de Castillo
Teresa de Manzanares Femenino
Solórzano
a) ¿Cuál era la profesión del padre del protagonista? (1632)
¿Qué otra ejercía al mismo tiempo? Alonso de Castillo Aventuras del bachiller
Masculino
b) ¿Cómo crees que puede influir la actividad del padre Solórzano Trapaza (1637)
en el comportamiento posterior del protagonista? Ra- Antonio Henríquez Vida de don Gregorio
Masculino
zona tu respuesta. Gómez Guadaña (1644)

[Mi madre] Padeció grandes trabajos recién casada, y aun Vida y hechos de
después, porque malas lenguas daban en decir que mi padre Anónimo Estebanillo González Masculino
metía el dos de bastos para sacar el as de oros1. Probósele que (1646)
a todos los que hacía la barba a navaja2, mientras les daba con
agua, levantándoles la cara para el lavatorio, un mi herma- 10. Con la ayuda de una enciclopedia, descubre el argumen-
nico de siete años les sacaba muy a salvo los tuétanos de las to de El Criticón y explica en qué consiste su carácter
faldriqueras3. Murió el angelico de unos azotes que le dieron alegórico.
en la cárcel. Sintiolo mucho mi padre, por ser tal que robaba Respuesta libre.
a todos las voluntades4.
11. En las obras de María de Zayas, la mujer muestra una
Francisco DE QUEVEDO: Vida del Buscón.
libertad de acción impropia de su época. Señala qué com-
portamientos y actuaciones crees que le estarían vetados
1
metía el dos de bastos para sacar el as de oros: quiere decir a la mujer en aquella época.
que con dos dedos (el dos de bastos) le sacaba las monedas de los Respuesta libre.
bolsillos (el as de oros). 2 hacía la barba a navaja: afeitaba. 3 los
tuétanos de las faldriqueras: lo más escondido de los bolsillos, COMENTARIO DE TEXTO
hasta la última moneda. 4 robaba a todos las voluntades: juega
con el doble sentido de que todos se encariñaban de él y de que Localización. Este poema pertenece a los sonetos de juven-
él les quitaba lo que no querían dar voluntariamente. tud (hacia 1582) de Luis de Góngora. En él sigue de cerca la
tradición petrarquista que tanta importancia tuvo en la lírica
a) El padre del protagonista era barbero, pero al mismo tiem- del Renacimiento.
po ejercía el oficio de ladrón, ya que aprovechaba los des- 1. ¿Qué imagen ideal de la mujer se ofrecía en dicha tra-
cuidos de sus clientes para robarles el dinero. dición?
b) Respuesta libre. La figura femenina es retratada, en la tradición petrarquis-
ta, como un ser inalcanzable e idealizado, cúmulo de todas
9. Además de las novelas picarescas citadas, se escribie-
las perfecciones y de toda la belleza. También se nos presen-
ron varias obras más de este género. Elabora una rela- ta como una mujer desdeñosa y distante, que hace sufrir al
ción completa de títulos y autores distinguiendo si tienen enamorado con su comportamiento arbitrario. El poeta se
un protagonista masculino o femenino. convierte en un admirador de la dama y siente su amor tan

100 Lengua castellana y Literatura. 3.º ESO. Solucionario


ENTRE PÍCAROS Y POETAS 11
profundamente que se conformará con ser odiado por ella con En las primeras estrofas, los adjetivos están dispuestos for-
tal de estar en sus pensamientos. mando un quiasmo, ya que en cada verso aparecen, al princi-
2. Realiza un análisis métrico del poema y justifica después pio, delante del sustantivo, y al final, detrás, dando una clara
sensación de orden y armonía.
por qué es un soneto.
Precisamente esta ordenación de los adjetivos contribuye a
Es un soneto porque está compuesto por catorce versos en-
la creación de algunos paralelismos, como por ejemplo entre
decasílabos de rima consonante, divididos en dos cuartetos
los versos 5 y 6: pequeña puerta de coral preciado / claras
y dos tercetos, según la siguiente estructura métrica: ABBA
lumbreras de mirar seguro (adjetivo + nombre + de + nombre
ABBA CDE CDE.
+ adjetivo).
Tema y estructura. Como Góngora emplea un esquema me-
En la expresión ornan de luz, coronan de belleza podemos
tafórico sin mencionar el elemento real que compara con un apreciar un asíndeton, provocado por la ausencia del nexo
término figurado, quizá te resulte difícil descubrir la identidad copulativo (y). También hay otro paralelismo.
del personaje superior del que nos habla. Primero, debes ave-
riguar qué elementos describe: Conclusiones. Góngora fue el máximo exponente del culte-
ranismo.
3. ¿Qué versos abarca la descripción? ¿Qué partes del cuer-
po humano enumera y en qué orden? ¿Con qué elemen- 7. ¿Qué aspectos de este poema te permiten reconocer
tos imaginarios los identifica? dicho estilo literario?
La descripción abarca casi todo el poema, concretamente los El culteranismo se reconoce en este soneto principalmente
once primeros versos (los dos cuartetos y el primer terceto). por el uso continuado de metáforas puras que, con un sentido
alegórico, identifican el cuerpo de una mujer hermosa con un
Las partes del cuerpo que enumera, por orden, y la imagen templo (por ejemplo, pequeña puerta de coral preciado, en
con que las identifica son las siguientes: lugar de boca, por el color rojo del coral asimilado a los labios,
• el cuerpo y las piernas: un templo (gentil muro y bello que se abren como una puerta). También llama la atención el
cimiento). empleo del hipérbaton (De pura honestidad templo sagrado,
• la boca: pequeña puerta de coral preciado. donde se coloca el complemento del nombre delante del nom-
bre al que complementa).
• los ojos, verdes y luminosos, identificados con las claras
lumbreras. 8. ¿Qué crees que le aporta al texto la identificación de la
• la cabeza y los cabellos rubios: (soberbio techo y cimbrias mujer con un templo? ¿Piensas que tiene algo que ver con
de oro). la idea de la mujer que ofrece el petrarquismo? Razona
4. En cambio, ¿qué sentimientos le despierta al poeta su tus respuestas.
«ídolo bello»? ¿Dónde los expone? Respuesta libre.
Los sentimientos que despierta en el poeta ese ídolo bello apa-
recen reflejados sobre todo en el último terceto (versos 12-14).
EL JARDÍN DE LA LITERATURA
Para él, esta mujer es como una divinidad a la que hay que 1. Te mostramos a continuación un soneto satírico de Que-
adorar y rezar, a la que hay que suplicar, como si de un dios vedo:
se tratara, para que tenga piedad de uno.
Érase un hombre a una nariz pegado,
Análisis de la forma y el contenido. La correspondencia en- érase una nariz superlativa,
tre el contenido y la forma del poema es total. érase una alquitara medio viva,
5. ¿Qué significado poseen los adjetivos usados por el poe- érase un peje espada mal barbado;
ta en su descripción? Por su parte, ¿qué pretende decirle era un reloj de sol mal encarado,
a su destinatario? Fíjate en el sentido de los verbos adoro, érase un elefante bocarriba,
suspira, canta y reza. érase una nariz sayón y escriba,
un Ovidio Nasón mal narigado.
La mayor parte de los adjetivos que emplea Góngora en la des-
cripción son epítetos, ya que expresan cualidades que ya están Érase el espolón de una galera,
presentes en el sustantivo al que acompañan: pura honesti- érase una pirámide de Egipto,
dad, templo sagrado, blanco nácar, alabastro duro, claras las doce tribus de narices era;
lumbreras, claro sol... Con ellos se refuerzan los encantos y érase un naricísimo infinito,
maravillas de todas esas realidades con las que identifica la frisón archinariz, caratulera,
belleza física de la mujer a la que se dirige. sabañón garrafal, morado y frito.
Lo que pretende decirle a su destinatario es que admira pro- a) Haz una relación de todas las realidades con las que
fundamente su belleza, que todo su cuerpo es un templo en el Quevedo identifica a la nariz. ¿Sabes qué figura retó-
que rezar a la divinidad que ella representa y disfrutar de sus rica emplea?
divinos encantos. De ahí el uso de los verbos adoro, suspira,
canta y reza, con los que demuestra muy claramente la devo- b) Busca en el diccionario las palabras que no entiendas.
ción que siente hacia esa mujer perfecta: todos estos verbos c) Quevedo utiliza el recurso de la hipérbole (exagera-
se pueden relacionar con el culto a Dios y contribuyen, por ción). ¿Cómo crees que contribuye a la burla?
tanto, a la divinización de la figura femenina.
d) ¿Te parece que esta descripción es una caricatura?
6. La distribución de las palabras reproduce la misma ar- Razona tu respuesta.
monía y simetría de un templo. ¿Dónde aparecen los a) Palabras con las que se identifica la nariz: alquitara, peje
adjetivos en las dos primeras estrofas: antes o después espada, reloj de sol, elefante, espolón de una galera, pirá-
del término modificado? ¿Encuentras algún paralelismo? mide de Egipto. En todos estos casos, Quevedo está uti-
¿Qué recurso literario aparece en «ornan de luz, coronan lizando la metáfora, ya que se produce una identificación
de belleza»? de la nariz con una serie de realidades que, en su opinión,

Lengua castellana y Literatura. 3.º ESO. Solucionario 101


11 ENTRE PÍCAROS Y POETAS

se asemejan a ella. También utiliza la hipérbole, figura LA FACTORÍA DE TEXTOS


retórica basada en la exageración, porque es evidente que
los objetos o seres con los que identifica la nariz tienen un 1. Lee el siguiente texto de carácter expositivo y realiza lue-
tamaño desmesurado. go las actividades que te proponemos:
b), c) y d) Respuesta libre. Durante siglos, la danza tuvo por encima de todo una signi-
ficación ritual y religiosa. La adoración y propiciación de los
2. Hemos hablado mucho de los sonetos. Ahora podrás leer dioses, las rogativas de fertilidad, buen tiempo, etc., se expre-
uno en el que Lope de Vega nos da la receta de cómo es- saban con frecuencia mediante formas tradicionales o impro-
cribirlos: visadas de danza y de melodía. En la época de los griegos y
Un soneto me manda hacer Violante, romanos, la danza conoció una evolución lenta, convirtiéndo-
que en mi vida me he visto en tanto aprieto; se de rito en arte consciente. Sin embargo, ya sea sagrada o
catorce versos dicen que es soneto: profana, arte o no arte, hay algo fundamentalmente erótico en
burla burlando van los tres delante. la danza que no parecía agradar a la Iglesia. Durante la Edad
Media se desaprobaba cualquier tipo de música de danza e
Yo pensé que no hallara consonante
incluso la danza en general. A pesar de ello, la danza conservó
y estoy a la mitad de otro cuarteto,
su popularidad, hasta llegar finalmente a su renacimiento en
mas si me veo en el primer terceto,
las Cortes europeas del siglo XVI.
no hay cosa en los cuartetos que me espante.
Ottó K ÁROLYI: Introducción a la música
Por el primer terceto voy entrando,
(Alianza Editorial).
y parece que entré con pie derecho,
pues fin con este verso le voy dando. a) Explica de forma razonada cómo están presentes en
Ya estoy en el segundo, y aun sospecho el texto las tres cualidades que debe tener una ex-
que voy los trece versos acabando: posición (claridad, concisión y organización de ideas).
contad si son catorce, y está hecho. b) Analiza la estructura del texto en función de la orga-
a) Según nos dice Lope, ¿cuál es la estructura de un nización interna de las ideas, ¿de qué manera está
soneto? estructurada la información?
b) ¿Cómo tiene que ser la rima de un soneto, asonante o c) Di razonadamente cómo se emplea en el texto la fun-
consonante? Copia la parte en que el poeta nos informa ción representativa y por qué es un texto objetivo.
de esto. ¿Crees que, además, puede tener algún elemento
subjetivo? ¿Por qué?
a) Lope dice que un soneto son catorce versos y nos hace ver
que se compone de dos cuartetos y de dos tercetos. a) El texto se ajusta a las tres características señaladas. A
la claridad, porque no utiliza expresiones ni ideas enre-
b) La rima de un soneto tiene que ser consonante, y así lo
vesadas, y expone de manera sencilla aquello que quiere
expresa Lope de Vega cuando dice al principio del segundo
comunicar, sin dobles sentidos, como cuando dice: En la
cuarteto: Yo pensé que no hallara consonante.
época de los griegos y romanos, la danza conoció una
3. Lee ahora este fragmento de El Criticón y contesta luego evolución lenta, convirtiéndose de rito en arte consciente.
las preguntas: A la concisión, puesto que no hay digresiones de ningún
tipo y el autor se limita a exponer y explicar aquello que
—¿Cómo es esto? Viviendo entre las fieras, no me previniste
se había propuesto, sin salirse del tema. A la organización
de algún riesgo, ¿y ahora con tanta exageración me cautelas?
de ideas, porque va exponiendo ordenadamente los pasos
¿No era mayor el peligro entre los tigres, y no temíamos, y
que se han producido en el nacimiento y desarrollo de la
ahora de los hombres tiemblas?
danza, cronológicamente, desde los tiempos prehistóricos
—Sí —respondió con un gran suspiro Critilo—, que si los hom- en que tenía un carácter ritual, hasta el siglo XVI en que ya
bres no son fieras es porque son más fieros, que de su crueldad se ha convertido en arte, pasando por el mundo clásico y
aprendieron muchas veces ellas. Nunca mayor peligro hemos por la Edad Media.
tenido que ahora que estamos entre ellos. Y es tanta verdad
b) La estructura del texto está basada, como acabamos de
esta, que hubo rey que temió y resguardó un favorecido suyo
señalar, en la exposición cronológica del desarrollo de la
de sus cortesanos (¡qué hiciera de los villanos!) más que de los
danza: los ritos que la hicieron posible en tiempos remotos,
hambrientos leones de un lago; y así, selló con su real anillo la
su conversión progresiva en arte durante el periodo gre-
leonera para asegurarle de los hombres cuando le dejaba entre
corromano, su desaprobación por parte de la Iglesia en los
las hambrientas fieras. ¡Mira tú cuáles serán estos! Verlos has,
siglos medievales y su asimilación a las cortes europeas
experimentarlos has, y dirásmelo algún día.
durante el siglo XVI, como arte y como actividad para el
Baltasar GRACIÁN: El Criticón (texto modernizado). disfrute de los sentidos.
a) ¿Qué concepción del ser humano nos ofrece Gracián c) Respuesta libre.
por boca de Critilo? ¿Qué opinas tú de lo que dice? 2. Redacta un texto expositivo sobre las principales formas
b) Con lo que has estudiado sobre esta época, razona por del baile en el siglo XX (tango, rock and roll, hip hop, etc.),
qué el fragmento que acabas de leer se puede insertar en el que tengas en cuenta lo que has estudiado en esta
en el Barroco. página. Documéntate sobre el tema antes de hacer el
a) El ser humano se nos presenta como una fiera, como un texto.
ser despiadado al que hay que temer mucho más que los Respuesta libre.
auténticos animales. Nos hace ver que es más fiero un 3. Este texto argumentativo está extraído de los Sueños de
hombre que un león.
Quevedo. Léelo y contesta a las preguntas:
Respuesta libre.
Es cosa averiguada, así lo siente Metrodoro Chío y otros mu-
b) Respuesta libre. chos, que no se sabe nada y que todos son ignorantes. Y aun

102 Lengua castellana y Literatura. 3.º ESO. Solucionario


ENTRE PÍCAROS Y POETAS 11
esto no se sabe de cierto: que, a saberse, ya se supiera algo; a) Investiga sobre el rapto de Europa por Júpiter. ¿En
sospéchase. Dícelo así el doctísimo Francisco Sánchez, médi- qué animal se transforma el dios para engañarla?
co y filósofo, en su libro cuyo título es Nihil scitur: No se sabe
nada. En el mundo hay algunos que no saben nada y estudian b) Una constelación de estrellas tiene el nombre de este
para saber, y estos tienen buenos deseos y vano ejercicio: por- animal. Busca información sobre ella y sobre el tiempo
que, al cabo, solo les sirve el estudio de conocer cómo toda la en que el sol la ilumina, y relaciona lo que averigües
verdad la quedan ignorando. Otros hay que no saben nada y con los versos.
no estudian, porque piensan que lo saben todo. Son de estos
muchos irremediables. A estos se les ha de envidiar el ocio y la
c) Con los resultados de tus investigaciones y teniendo
satisfacción y llorarles el seso. Otros hay que no saben nada, en cuenta todo el fragmento, explica el significado de
y dicen que no saben nada porque piensan que saben algo de la metáfora pura que contiene el último verso.
verdad, pues lo es que no saben nada, y a estos se les había a), b) y c) Respuesta libre.
de castigar la hipocresía con creerles la confesión. Otros hay,
y en estos, que son los peores, entro yo, que no saben nada 2. En los siguientes sonetos se dan definiciones del amor.
ni quieren saber nada ni creen que se sepa nada, y dicen de Léelos y resuelve las actividades que se plantean a con-
todos que no saben nada y todos dicen de ellos lo mismo y tinuación:
nadie miente. Soneto amoroso definiendo el amor
Francisco DE QUEVEDO: «El mundo por de dentro»,
Es hielo abrasador, es fuego helado,
en Sueños y discursos.
es herida que duele y no se siente,
a) Analiza la estructura del texto y razona a cuál de los es un soñado bien, un mal presente,
tres tipos expuestos arriba pertenece. es un breve descanso muy cansado.
b) ¿Qué predomina en el texto, la objetividad o la subje- Es un descuido que nos da cuidado,
tividad? ¿Crees que el autor intenta implicar al lector? un cobarde con nombre de valiente,
Razona tu respuesta. un andar solitario entre la gente,
un amar solamente ser amado.
c) Busca en el fragmento las principales características
de los textos argumentativos y haz luego una tabla Es una libertad encarcelada
donde recojas cada uno de ellos con algún ejemplo que dura hasta el postrero parasismo1,
al lado. enfermedad que crece si es curada.

a) La estructura del texto es deductiva, ya que parte de una Este es el niño amor, este es su abismo:
tesis principal (Es cosa averiguada [...] que no se sabe ¡mirad cuál amistad tendrá con nada
nada y que todos son ignorantes), para luego desarrollar el que en todo es contrario de sí mismo!
esta idea por medio de argumentos varios que, en este Francisco DE QUEVEDO
caso, expresan las diferentes maneras de no saber nada
que considera el autor. 1
parasismo: paroxismo, momento extremo de una enfermedad,
b) Respuesta libre. agonía.
c) No hay un marcado uso de verbos de opinión, pero sí apa-
rece a menudo el verbo ser, empleado para señalar verda- Soneto 126
des que parecen indiscutibles en casi todas las ocasiones
Desmayarse, atreverse, estar furioso,
(es cosa averiguada, son los peores, etc.)
áspero, tierno, liberal, esquivo,
alentado, mortal, difunto, vivo,
Uso de verbos de opinión dícelo, dicen leal, traidor, cobarde y animoso;
Argumentos de autoridad «así lo siente Metrodoro Chío» no hallar fuera del bien centro y reposo,
Obras científicas Nihil scitur, de Francisco mostrarse alegre, triste, humilde, altivo,
Sánchez enojado, valiente, fugitivo,
satisfecho, ofendido, receloso;
4. Elabora tú un texto argumentativo en el que hagas una huir el rostro al claro desengaño,
reflexión sobre la introducción de palabras extranjeras en beber veneno por licor suave,
nuestra lengua. Valora de manera razonada si esta inclu- olvidar el provecho, amar el daño:
sión de vocablos te parece positiva o negativa. creer que el cielo en un infierno cabe,
Respuesta libre. dar la vida y el alma a un desengaño;
esto es amor, quien lo probó lo sabe.
ACTIVIDADES FINALES Lope DE VEGA
Repasa lo que has aprendido a) Compara los dos poemas y explica qué características
1. Los siguientes versos son el comienzo de la «Soledad pri- comunes tienen.
mera» de Luis de Góngora. En ellos se sitúa el tiempo de b) En los dos sonetos se emplea abundantemente la an-
la acción que se va a contar: la primavera. títesis. Haz una relación de todas las que veas en cada
Era del año la estación florida uno de ellos y explica qué sensaciones producen en
en que el mentido robador de Europa el lector.
(media luna las armas de su frente,
y el Sol todos los rayos de su pelo), c) Los dos autores manejan la enumeración, pero de forma
luciente honor del cielo, diferente. Analiza este fenómeno en los dos poemas y di
en campos de zafiro pace estrellas. qué aporta al contenido de estos.

Lengua castellana y Literatura. 3.º ESO. Solucionario 103


11 ENTRE PÍCAROS Y POETAS

d) Relaciona las sensaciones contradictorias que mane- y más, si pudiese, que todos. No sé si salí con ello, pero yo
jan los dos autores con la mentalidad barroca. ¿Te pa- aseguro a v. m. que hice todas las diligencias2 posibles.
rece que definen bien la época? ¿Por qué? Lo primero, yo puse pena de la vida a todos los cochinos que
se entrasen en casa y a los pollos de la ama que del corral pa-
a) Una característica común a los dos sonetos es la enumera-
sasen a mi aposento. Sucedió que un día entraron dos puercos
ción, que en el caso de Quevedo se basa en una sucesión de
del mejor garbo 3 que vi en mi vida. Yo estaba jugando con
metáforas y en el de Lope en un despliegue de sensaciones
los otros criados, y oílos gruñir, y dije al uno:
contradictorias que se sustentan en la antítesis. También
en Quevedo se percibe esta última figura, asociada a las —Vaya y vea quién gruñe en nuestra casa.
imágenes metafóricas que contiene el poema. Fue, y dijo que dos marranos. Yo que lo oí, me enojé tanto que
Otro rasgo común muy importante es el de postergar has- salí allá diciendo que era mucha bellaquería y atrevimiento
ta el último terceto el nombre de aquello de lo que se nos venir a gruñir a casa ajena. Y diciendo esto, envásole4 a cada
habla: el amor. En Quevedo vemos que lo desvela en el uno a puerta cerrada la espada por los pechos, y luego los
verso 12: este es el niño amor; en Lope de Vega, en el 14: acogotamos5. Porque no se oyese el ruido que hacían, todos a
esto es amor. la par dábamos grandísimos gritos como que cantábamos y
así expiraron6 en nuestras manos.
b) Antítesis en el poema de Quevedo:
hielo abrasador, fuego helado; herida que duele y no se
siente; un soñado bien, un mal presente; un breve des- 1
bellaco: perverso. 2 diligencias: trámites. 3 garbo: elegancia
canso muy cansado; un descuido que nos da cuidado; un en sus movimientos. 4 envásole: aquí, le metí. 5 acogotamos:
cobarde con nombre de valiente; andar solitario entre la sujetamos. 6 expiraron: murieron.
gente; una libertad encarcelada; enfermedad que crece
si es curada.
Antítesis en el poema de Lope de Vega: a) ¿Cómo se pone de manifiesto la desvergüenza del
Desmayarse, atreverse; áspero, tierno; liberal, esquivo;
protagonista?
alentado, mortal; difunto, vivo; leal, traidor; cobarde y b) ¿Quién actúa como narrador en el texto? ¿A quién se
animoso; alegre, triste; humilde, altivo; valiente, fugi- dirige? ¿Existe, pues, alguna semejanza con el Lazari-
tivo; satisfecho, ofendido; beber veneno por licor suave; llo de Tormes? ¿Por qué?
olvidar el provecho, amar el daño; creer que el cielo en
un infierno cabe.
c) En cambio, ¿qué diferencias sustanciales se ponen de
relieve en la descripción de pícaros como Pablos y en
En ambos casos crean en el lector una sensación de de- la de los protagonistas de los libros de caballerías tan
samparo y desasosiego, como si toda la vida se sustenta-
admirados por don Quijote?
ra en ese sufrimiento placentero que resulta ser el amor
según nos lo pintan estos dos poetas: algo que nos gusta d) Analiza los siguientes sintagmas nominales:
y a la vez nos inquieta. • todas las diligencias posibles
c) Como dijimos antes, estas antítesis se basan en la metá-
• dos puercos del mejor garbo
fora si hablamos de Quevedo, y en la enumeración si lo
hacemos de Lope. Con las metáforas se nos muestra un uni- e) ¿Cuál es el tiempo verbal que se suele usar en las narra-
verso de imágenes que se asocian a la idea del amor (hielo ciones? ¿Qué tiempo predomina en este texto? Fíjate,
abrasador, fuego helado, libertad encarcelada, etc.); con por ejemplo, en las formas puse, sucedió, entraron o
la enumeración se nos presentan multitud de sensaciones enojé.
y estados de ánimo que también representan con firmeza
lo que es estar enamorado.
f) La oración Vaya y vea quién gruñe en nuestra casa es
imperativa. Sin embargo, las formas verbales vaya y
d) Respuesta libre. vea no están en modo imperativo. ¿Por qué? ¿Cuál es
3. Busca información sobre los reyes del siglo XVII y elabora el tiempo y el modo de dichas formas?
un cuadro cronológico en el que recojas los principales g) Indica la función sintáctica de los términos marcados
acontecimientos históricos del siglo. en negrita:
Respuesta libre
• dije al uno
4. Infórmate sobre la obra poética de Lope de Vega y com- • yo puse pena de la vida a todos los cochinos
pleta la información que te hemos ofrecido en esta uni-
dad. Haz luego una tabla en la que clasifiques sus obras • envásole la espada por los pechos
en verso según sean líricas o épicas. • y luego los acogotamos
Respuesta libre. • no se oyese el ruido que hacían
• yo estaba jugando con los otros criados
Recuerda lo que ya sabías
• yo aseguro a v. m.
5. Como es habitual en la novela picaresca, el protagonista
de Historia de la vida del Buscón llamado don Pablos de a) El protagonista utiliza el engaño y la falsedad para conse-
Quevedo comete múltiples tropelías. Ahora puedes leer guir sus propósitos. En este caso, hacerse con los cerdos
para luego comérselos él. Para ello hace ver que está po-
lo que le ocurrió, estando en Alcalá de Henares al servicio
niendo orden en la casa en la que vive como huésped. Por
de don Diego Coronel. Después realiza las tareas que se otro lado, en lugar de intentar hacer el bien y enderezar
te plantean. su vida, decide irse por el camino más fácil, que es el de
«Haz como vieres» dice el refrán, y dice bien. De puro consi- hacerse bellaco, siguiendo el refrán «haz como vieres»,
derar en él, vine a resolverme de ser bellaco1 con los bellacos, como él mismo nos dice.

104 Lengua castellana y Literatura. 3.º ESO. Solucionario


ENTRE PÍCAROS Y POETAS 11
b) El narrador es la primera persona, que, en este caso, se Guarnición1 tosca de este escollo duro
identifica con el protagonista de la novela, quien se dirige troncos robustos son, a cuya greña2
a una tercera persona ausente, a quien llama vuestra mer- menos luz debe, menos aire puro
ced (v.m.). En estos dos aspectos, el Buscón sigue de cerca la caverna profunda, que a la peña;
al Lazarillo, que también está escrito en primera persona caliginoso3 lecho, el seno oscuro
y también se dirige a vuestra merced. ser de la negra noche nos lo enseña
c) La descripción de pícaros se centra en las características infame turba4 de nocturnas aves,
más propias de su condición, cercana a la delincuencia, gimiendo tristes y volando graves.
y, por tanto, habrá un realismo muy marcado en el que se
destacarán estos rasgos: bellaquería, mentiras, trampas, 1
guarnición: adorno. 2 greña: cabellos revueltos, de sor-
doble moral, etc. Por el contrario, los caballeros andantes denados. 3 caliginoso: oscuro, tenebroso. 4 turba: multitud
suelen ser descritos siempre con una tendencia a la ideali- desordenada.
zación, destacando sus cualidades caballerescas: el honor,
la valentía, las finezas como enamorados, la cortesía, etc.
d) todas las diligencias posibles a) ¿Qué elementos se emplean para dar la sensación de
Det Det N Adj recargamiento en cada caso?
dos puercos del mejor garbo b) En ambos casos se describe un lugar físico. Haz tú
Det N Prep Adj N una redacción en la que describas con tus palabras la
imagen del Transparente.
+ Det
a) En el altar podemos ver cómo hay una gran cantidad de
e) En las narraciones predominan los tiempos pasados, sobre
adornos y figuras de ángeles en medio de un recargamien-
todo el pretérito imperfecto de indicativo y el pretérito
to que deja pocos huecos sin decoración. Las columnas
perfecto simple. En el texto se da esta sucesión de tiempos
retorcidas que sustentan la parte alta y que delimitan el
y se mezclan también otros como el pretérito imperfecto
altar (llamadas salomónicas) contribuyen, con su extra-
de subjuntivo o el presente de indicativo, pero predomina
ña forma, a dar ese aspecto abigarrado al conjunto. Los
el pretérito perfecto simple, como demuestran los ejemplos
mármoles y los dorados completan el monumento barroco.
extraídos: puse, sucedió, entraron, enojé.
f) El imperativo es un modo que se expresa a menudo por Por su parte, los versos de Góngora se retuercen también
medio del presente de subjuntivo, como es el caso que por medio del hipérbaton: caliginoso lecho, el seno oscuro
aquí se da. En general suele ser la segunda persona del / ser de la negra noche nos lo enseña...; guarnición tosca
imperativo la que tiene una forma especial diferenciada de este escollo duro / troncos robustos son. Para recargar
de otras formas verbales (ve tú), mientras que la tercera se más la forma, el poeta introduce palabras de poco uso
suele expresar, como hemos dicho, por medio del presente y cultismos, como caliginoso. También llama la atención
de subjuntivo (vaya él). Como en este caso se emplea el el refuerzo de la oscuridad que se presenta en el verso
tratamiento de respeto (vuestra merced), aparece ligado infame turba de nocturnas aves, donde lo tenebroso se
a la tercera persona, como ocurriría hoy con usted: «Vaya subraya con los acentos rítmicos del verso, que recaen
usted a tal sitio». sobre la letra u en las sílabas que marcamos en negrita.

g) • dije al uno b) Respuesta libre.


Complemento indirecto. 2. La pintura de Juan de Valdés Leal (1622-1690) refleja de
• yo puse pena de la vida a todos los cochinos manera visual el pesimismo y el desengaño barrocos:
Complemento directo. a) La inscripción que se lee bajo el ataúd del primer plano
• envásole la espada por los pechos dice: Finis gloriae mundi («fin de las glorias del mundo»).
Complemento indirecto (le); complemento circunstan- Fíjate en los elementos que componen el cuadro y
cial de lugar (por los pechos). explica según ellos el sentido de la frase.
• y luego los acogotamos b) ¿Cómo contribuyen las tonalidades oscuras del cua-
Complemento circunstancial de tiempo (luego); com- dro a reflejar el pesimismo de la época?
plemento directo (los). c) La técnica del contraste, en pintura, se llama claros-
• no se oyese el ruido que hacían curo; infórmate acerca de este concepto y explícalo en
Sujeto. relación con la forma de pensar del Barroco.
• yo estaba jugando con los otros criados d) En los ataúdes hay un obispo y un hombre de aspecto
Complemento circunstancial de compañía. rústico, tal vez un pastor; ¿qué significado tiene la pre-
• yo aseguro a v. m. sencia de dos personas de estamentos sociales tan
Complemento indirecto. dispares? ¿Qué idea de la muerte nos transmite?
e) La balanza que hay en el centro pesa los atributos de los
MIRA A TU ALREDEDOR Y… dos personajes, sobre la inscripción «Ni más, ni menos».
… ve más allá Relaciona esto con la cuestión anterior.
1. El arte barroco se caracteriza por el recargamiento y el a) Llama la atención, sobre todo, la figura del obispo, en pri-
retorcimiento de las formas. Un ejemplo puede ser el altar mer plano, ataviado con todas sus galas terrenales: la mi-
del Transparente de la catedral de Toledo: tra, el báculo, sus vestimentas lujosas… Todo ello para
dar cobertura a un cadáver putrefacto cuya faz es ya una
Observa la relación entre la imagen que te ofrecemos calavera descarnada. En esto quedan las glorias del mun-
y los siguientes versos de Góngora, en los que el poeta do, como dice la inscripción que hay en primer plano, bajo
describe la cueva que habita el cíclope Polifemo: los ataúdes.

Lengua castellana y Literatura. 3.º ESO. Solucionario 105


11 ENTRE PÍCAROS Y POETAS

b) La oscuridad reinante en el cuadro es la de la sepultura, belleza. De nuevo la mentalidad pesimista del barroco, el tó-
la de un osario en el que reposan varios muertos. Esta pico del tempus fugit, del vanitas vanitatum.
negritud hace más terrible, si cabe, la triste escena que 2. En el siguiente texto se cuenta una de las bromas que
nos muestra el cuadro. La ausencia de luz remarca el pesi-
solían hacerse durante el carnaval:
mismo barroco, que se concentra en las tumbas del fondo:
todo es tristeza, todo es pesar, todo es muerte. Dos mujeres que están en una reja de un cuarto bajo, con un
instrumento de disparar agua, por las troneras1 de una celo-
c) Respuesta libre. sía2, a un hombre vestido de negro que, descuidado, arrimado
d) El pintor ha elegido a un obispo y a un rústico pastor para a ella pasaba, le dan una rociada por el rostro que le turban3
hacernos ver la terrible realidad de la muerte y su poder los ojos y le desaderezan4 la valona5. El hombre prosigue su
igualador, de tan clara procedencia medieval. Nadie es camino sin volver la cara al lugar de su ofensa. Pasa por la
más que nadie a los ojos de la muerte; para ambos se necedad del uso con silencio, no sé yo si con paciencia. […]
acaban con ella las glorias del mundo, sean estas cuales Esta agua ofensiva, que se arroja las Carnestolendas 6, ni se
sean. mezcla con la prudencia ni con la virtud. Con la prudencia no,
porque es error descubierto hacer a otro ni aun el menor daño.
e) La balanza que una mano misteriosa sostiene sobre los
No con la virtud, porque no puede dejar de ser malo causar
dos muertos, y en la que se pesan los atributos del pastor
enojos vengativos.
a la izquierda (un carnero, un perro...), y los del obispo a la
derecha (un libro, un corazón con el monograma de Cristo Juan DE Z ABALETA: El día de fiesta por la tarde.
[JHS], etc.), es una alegoría de esa igualdad de todos ante
la muerte: tanto vale haber sido un obispo o haber sido 1
troneras: abertura pequeña en una muralla para disparar con
un pastor. Por eso se refuerza la imagen con la terrible
los cañones. 2 celosía: enrejado de madera. 3 turban: inquietan,
inscripción: Ni más, ni menos.
incomodan. 4 desaderezar: descolocar. 5 valona: cuello grande
de una prenda de vestir. 6 Carnestolendas: Carnaval.
… encuentra la clave
1. Lee este fragmento del poema «A la rosa» de Francisco
de Rioja (1583-1659): ¿Qué piensas tú de esta forma de divertirse? ¿Te parece
que se siguen dando este tipo de bromas en la actuali-
Pura, encendida rosa,
dad? Razona tus respuestas.
émula de la llama
que sale con el día, Respuesta libre.
¿cómo naces tan llena de alegría 3. Lee ahora el comienzo de una letrilla de Luis de Góngora
si sabes que la edad que te da el cielo en la que se elogia la sencillez y la libertad de vivir según
es apenas un breve y veloz vuelo,
uno quiera, sin hacer caso del qué dirán:
y ni valdrán las puntas de tu rama
ni tu púrpura hermosa Ándeme yo caliente
a detener un punto y ríase la gente.
la ejecución del hado presurosa? Traten otros del gobierno
del mundo y sus monarquías,
En la poesía barroca se convirtieron en un tópico las com-
mientras gobiernan mis días
posiciones dedicadas a determinadas flores. Fue una ten- mantequillas y pan tierno;
dencia que puede relacionarse con la gran importancia y las mañanas de invierno
que adquirió en la época la pintura de bodegones como naranjada y aguardiente,
este de Antonio de Pereda (1611-1678). y ríase la gente.
¿Qué relación crees que existe entre los versos de Rioja, a) ¿Qué te parece esta forma de pensar? ¿Crees que es
el bodegón de Pereda, el tiempo y la belleza? una invitación a vivir? ¿Por qué?
Tanto en el poema de Rioja dedicado a una rosa como en el b) ¿Qué sensaciones provoca la vida que elige el poeta?
bodegón de Pereda, se nos muestran realidades perecederas, ¿Positivas o negativas? Razónalo.
fugaces, ya que las flores se marchitan pronto y las naturale-
zas muertas del bodegón se pudrirán también en poco tiem- c) Escribe tú una letrilla satírica sobre la importancia de las
po. Todo es bello (la flor, las manzanas, las granadas...), pero apariencias y de la libertad individual en la actualidad.
el tiempo acabará con ello, marchitándolo, ajando su lozana a), b) y c) Respuesta libre.

106 Lengua castellana y Literatura. 3.º ESO. Solucionario


TODOS A UNA 12
APERTURA DE UNIDAD madre muere en el parto (signo que parece demostrar los va-
ticinios), es encarcelado nada más nacer en una torre, para
1. ¿Qué es lo que pretende averiguar el juez? ¿A quién in- intentar evitar que se cumplan las predicciones.
terroga y por qué crees que lo hace?
3. ¿Con qué elementos se compara Segismundo? ¿De dón-
Pretende averiguar quién de los habitantes de Fuente Ovejuna de proceden tales elementos y qué tienen en común?
mató al comendador. Interroga a un niño porque espera que,
¿Por qué crees que ha elegido el autor esta comparación?
al tratarse de un ser más débil y al no estar instruido en el
arte de la mentira y el engaño, su pureza y debilidad harán Se compara con elementos de la naturaleza que inspiran
que diga la verdad. sensación de libertad: un ave, un pez y un bruto (las bestias
salvajes).
2. Explica cuál es la actitud de los habitantes de Fuente Ove-
4. ¿Ha cometido algún delito Segismundo? ¿Cuál es, según
juna durante el interrogatorio a que los somete el juez.
él? ¿Estás de acuerdo en que es un delito? Busca la palabra
Se trata de un comportamiento solidario y valiente, aunque en el diccionario y explica si Segismundo ha sido autor de
un tanto temerario. Se escudan en la máxima de que la unión
tal hecho.
hace la fuerza para proteger al verdadero ejecutor del cri-
men. Lo hacen porque, aunque uno ha sido el que ha llevado No, pero él se cuestiona si lo ha cometido y cree que sí. El
a cabo la acción, todos sentían el mismo odio hacia el Co- delito de que se acusa es de su propio nacimiento. Como no
mendador y estaban de acuerdo en cometer el crimen. Para ha tenido libertad nunca para cometer ninguna atrocidad,
todo el pueblo de Fuente Ovejuna ha resultado liberadora llega a la conclusión de que solo el hecho de existir debe ser
esta muerte. su delito.

3. Teniendo en cuenta dicha actitud, ¿qué queremos expre- El significado literal de la palabra delito es: culpa, crimen,
quebrantamiento de la ley.
sar cuando hoy en día usamos la expresión «Todos a una,
como Fuente Ovejuna»? ¿Recuerdas alguna ocasión en Según esta definición, podemos afirmar que Segismundo, al
que hayas obrado así junto con tus amigos o compañeros? carecer de libertad, no ha podido ser autor de ningún delito
en este sentido. Aquí delito se utiliza casi en el sentido de pe-
Queremos expresar que hay que permanecer unidos, actuan- cado, una especie de pecado original con el que el personaje
do de forma solidaria en una acción o proyecto. Esto signifi- ha nacido, pues no ha tenido ocasión de adquirirlo por sus
ca que no debe haber discrepancia o actitudes enfrentadas, propias obras.
sino que todo un grupo de personas, interesadas o afecta-
das por un mismo hecho, deben seguir las mismas directrices. 5. ¿Te resulta conmovedor este monólogo? ¿Conoces algún
Normalmente se emplea en contextos en los que varias perso- otro personaje de la literatura aquejado por la falta de
nas han sufrido lo que consideran una desgracia o injusticia libertad?
y deciden unirse para hacerse fuertes y encontrar apoyos en Respuesta libre.
dicha unión.
Otros personajes de la literatura aquejados de falta de libertad:
4. Puedes ver la versión cinematográfica de esta obra de • Cenicienta, en el cuento popular, carece de libertad para
Lope de Vega tecleando en YouTube «Fuente Ovejuna pe- actuar debido al estado de esclavitud a que la somete su
lícula» y compararla con la adaptación de Antonio Gades malvada madrastra.
(versión de ballet flamenco). • Rapunzel, protagonista de un cuento de los hermanos
Respuesta libre. Grimm, también sufre falta de libertad, pues vive encerra-
5. Debate y comenta con tus compañeros cuál de estas ver- da en una torre por culpa de un antojo que su madre tuvo
siones te ha gustado más aportando argumentos claros y antes de traerla al mundo. Una bruja la ha condenado a
ejemplos concretos que apoyen tu valoración. vivir presa en una torre en medio del bosque. Esta prisión
también recuerda a la de algunas damas de la literatura
Respuesta libre.
caballeresca, a las que los caballeros han de rescatar.
6. En el pueblo de Fuente Obejuna (Córdoba) se realiza • La Sachette de Nuestra Señora de Notre Dame de París,
anualmente un montaje teatral en torno a esta obra. In- de Victor Hugo, que sufre un patético e injusto encarcela-
vestiga sobre él y pon en común con tus compañeros la miento parecido o Jen Valjean en Los Miserables, también
información que hayas obtenido. de Victor Hugo.
Respuesta libre. • El reo de muerte en el relato de Espronceda.
• El protagonista de Cárcel de amor de Diego de San Pedro.
VIVE LA LECTURA En esta obra medieval, la cárcel es metafórica, pues se tra-
Comprensión lectora ta en realidad del sentimiento amoroso que hace preso al
protagonista.
1. ¿Cuál crees que es el tema general de este texto: una ala-
banza, una maldición, una declaración de amor, una expo- 6. Procura ponerte en la piel de este personaje y explica
sición de hechos, una queja o una narración? Justifica tu cómo crees que te sentirías tú en su misma situación.
respuesta. Respuesta libre.
La queja de Segismundo por su falta de libertad.
Vocabulario
Todas las comparaciones a las que recurre el protagonista
inciden en la libertad de que gozan los elementos natrales y la 7. Segismundo acusa al cielo —al poder divino— de su justicia
hace contrastar con su estado de riguroso encarcelamiento. y rigor. ¿Qué significa este último término? El adjetivo
2. ¿Qué le ocurre a Segismundo? ¿Por qué? riguroso deriva del sustantivo rigor; escribe dos oraciones
con él.
Está preso porque antes de su nacimiento, un adivino predijo
que el niño que iba a nacer causaría grandes daños y sería Rigor: excesiva y escrupulosa severidad.
una amenaza para su propio padre, el rey. Segismundo, cuya Oraciones libres.

Lengua castellana y Literatura. 3.º ESO. Solucionario 107


12 TODOS A UNA

8. ¿Qué quiere decir el protagonista con el verso «para apu- Procura vocalizar bien, marcar las pausas y darle a cada
rar mis desvelos»? oración el sentido y la intención que el autor pretendía
Para ahondar en la causa de su sufrimiento y llegar al fondo que transmitiesen. Si quieres, puedes consultar la repre-
de la cuestión, aunque sea para su mal. sentación que algún actor o actriz hayan realizado de este
fragmento. Una de las representaciones más recientes y
9. Busca en el diccionario las siguientes palabras y anota su
novedosas es la de la actriz Blanca Portillo. Puedes acceder
significado. Después construye una oración con cada una
a ella a través de YouTube. Teclea «Blanca Portillo» «La vida
de ellas:
es sueño» (RDE).
privilegio etéreas docto lamas albedrío Respuesta libre.
• Privilegio: exención de una obligación o ventaja exclusiva
o especial que goza alguien por concesión de un superior o LITERATURA
por determinada circunstancia propia.
1. Lee el siguiente texto de El capitán Alatriste, de Arturo
• Etéreas: que pertenecen al cielo. (Etéreas salas: es una me- Pérez-Reverte, y anota las características del teatro que
táfora de los distintos lugares del cielo por donde pasa el aparecen en él. Compara este espectáculo con otros ac-
ave).
tuales de semejante seguimiento e importancia:
• Docto: que a fuerza de estudiar ha adquirido más conoci- Desde el monarca hasta el último villano, la España del Cuar-
mientos que los comunes y ordinarios. Sabio. to Felipe amó con locura el teatro. Las comedias tenían tres
• Lamas: cieno blando, suelto y pegajoso, de color oscuro, jornadas o actos, y eran todas en verso, con diferentes metros
que se halla en algunos lugares del fondo del mar o de los y rimas. Sus autores consagrados, como hemos visto al re-
ríos, y en el de los recipientes o lugares en donde hay o ha ferirme a Lope, eran queridos y respetados por la gente; y la
habido agua largo tiempo. Alga u ova de los lamedales o popularidad de actores y actrices era inmensa. Cada estreno o
charcales. reposición de una obra famosa congregaba al pueblo y la corte,
• Albedrío: potestad o derecho de obrar por reflexión y elec- teniéndolos en suspenso, admirados, las casi tres horas que
ción. Dícese más ordinariamente libre albedrío. duraba la representación; que en aquel tiempo solía desarro-
llarse a la luz del día, por la tarde después de comer, en locales
Investigación y redacción al aire libre conocidos como corrales. Dos había en Madrid: el
del Príncipe, también llamado de La Pacheca, y el de la Cruz.
10. Indaga sobre dos aspectos debatidos en la época en que • Socialización del teatro (se extiende a todas las clases so-
se escribió esta obra: la predestinación y el libre albedrío. ciales): Desde el monarca hasta el último villano, la Es-
Explica tu opinión sobre si existe o no un destino fijo del paña del Cuarto Felipe amó con locura el teatro.
que el ser humano no puede huir.
• Estructuración de las obras en tres actos.
En el siglo de oro existe una polémica filosófica entre el con-
• Escrito en verso, con diversidad de estrofas.
cepto de la predestinación y el libre albedrío. Ante la duda
metafísica de si el hombre es libre y decide sobre su propia • Constituía un espectáculo de masas, muy querido por el
vida, los que defienden la idea de la predestinación opinan pueblo.
que es Dios el que marca los designios del hombre y que este • Se representaba en corrales de comedias al aire libre y se
en realidad no elige su destino, pues le viene dado desde el veían varias piezas.
momento del nacimiento. Los defensores del libre albedrío
2. Busca el esquema métrico de las estrofas que se mencio-
opinan, en cambio, que si Dios creó al hombre, lo creó libre
nan en el fragmento de El arte nuevo de Lope y anótalo
para que decidiera los derroteros de su propia vida y la direc-
ción de sus acciones. en tu cuaderno.
• Décima: estrofa de diez versos octosílabos cuya rima obe-
La vida es sueño recoge y analiza el conflicto entre ambas
dece al siguiente esquema: abbaaccddc.
interpretaciones, por eso, Segismundo comienza la obra pre-
destinado a la falta de libertad. Se le concede luego el libre • Soneto: poema estrófico de versos endecasílabos y
albedrío pero este es utilizado para obrar el mal y acaba de rima consonante formado por dos cuartetos (11A11B-
nuevo encarcelado. Parece así de nuevo predestinado al mal y 11B11A) y dos tercetos que pueden o no ser encadenados
a la prisión. Pero la reflexión final de la obra deja una puerta (11C11D11C-11D11C11D).
abierta a la idea del libre albedrío relacionada con la idea del • Romance: poema estrófico compuesto por versos octosíla-
bien. El hombre debe obrar bien, tanto si se siente libre para bos que riman en asonante los pares (8-8a8-8a).
elegir como si no lo hace, porque las buenas acciones nunca
• Octava: la llamada octava real es una estrofa de ocho ver-
están de más.
sos endecasílabos y rima consonante cuyo esquema métri-
11. La vida es sueño se divide en tres actos y en cada uno co es: ABABABCC.
de ellos hay un monólogo importante. ¿Recuerdas qué • Terceto: estrofa de tres versos endecasílabos que riman en
es un monólogo? Busca los del segundo y el tercer acto. asonante. Puede tener varios esquemas métricos, pero en
Encontrarás la obra completa en la biblioteca digital Ciu- el caso de los tercetos encadenados, la rima es: CDC, DCD.
dad Seva (ciudadseva.com). • Redondilla: estrofa de arte menor compuesta por versos
El monólogo es un tipo de texto teatral en el que interviene octosílabos, con rima asonante o consonante, y el siguiente
solo un personaje. Sus palabras dan a conocer al público sus esquema: abba.
pensamientos, reflexiones y sentimientos, pues expresa 3. Lee este fragmento de Tuerto, maldito y enamorado, de
sus ideas en voz alta. Rosa Huertas, y busca información sobre los datos bio-
12. Ahora que has trabajado el vocabulario, vuelve a leer el gráficos de Lope que menciona.
texto intentando comprenderlo por completo y realiza Le conté todos los amoríos de Lope, su juventud díscola y su
una lectura dramatizada para tus compañeros. madurez desquiciada; le hablé de sus magníficas obras, de

108 Lengua castellana y Literatura. 3.º ESO. Solucionario


TODOS A UNA 12
lo que significaron para la escena española, de su fama sin Los últimos años de la vida de Lope estarán llenos de amar-
límites, de su increíble capacidad creadora y de su concepción guras: el rechazo de los nuevos gobernantes en los que buscó
novedosa del teatro. el favor, la locura, ceguera y posterior muerte de Marta de
Pero le mentí. Le había dicho que iba a contarle todo lo que Nevares, la muerte de su hijo Lope Félix y el rapto de su hija
sabía y no lo hice. Mi narración de la biografía del Fénix de Antonia Clara.
los ingenios se detuvo en 1616, el año en que conoció a Marta El Fénix de los Ingenios, apelativo que se había ganado por lo
de Nevares. prolífico de su obra, murió en Madrid, el 27 de agosto de 1635
y fue enterrado solemnemente en la iglesia de San Sebastián.
—Háblame ahora tú de él —le pedí—. Cuéntame qué recuer-
das de Lope después de lo que acabas de escuchar. Quiero La novela de Rosa Huertas repasa, a través del protagonismo
saber cómo era. […] de un espíritu que sufre la maldición del propio Lope de Vega,
la vida del autor. Se repasan los lugares en los que vivió y las
—Recuerdo sus cabellos grises, su bigote recortado y su peri- mujeres a las que amó. Se recrea su carácter vital y su intensa
lla. Era obstinado y vanidoso, pero claro y natural, sin afeites vida amorosa.
ni rebozos. Se mostró siempre paternal conmigo, su afecto
era hondo y sincero y sé que me quiso tanto como después me 4. Reconstruye el argumento de El condenado por descon-
aborreció hasta maldecirme cruelmente. No era una actitud fiado de Tirso basándote en la intervención del Demonio
extraña en él, que amaba y odiaba con la intensidad de los y la última del protagonista, Paulo.
genios. DEMONIO.— Diez años ha que persigo
Resumimos los datos biográficos de Lope de Vega, su intensa a este monje en el desierto […].
vida amorosa y su relación con Marta de Nevares: Hoy duda de su fe; que es duda
de la fe lo que hoy ha hecho,
Félix Lope de Vega Carpio nació en Madrid a finales de 1562.
porque es la fe en el cristiano
Hay discusión acerca de la fecha exacta.
que, sirviendo a Dios y haciendo
En cuanto a su formación, Lope de Vega estudió primero en buenas obras, ha de ir
Madrid, en la escuela regentada por Vicente Espinel y poste- a gozar dél en muriendo.
riormente en la Compañía de Jesús. También cursó un año en PAULO.— ¡Malditos mis padres sean
Alcalá de Henares, aunque sin conseguir titulación y, solo tal
mil veces pues me engendraron!
vez, en Salamanca.
¡Y yo también sea maldito,
En 1583 zarpó a Lisboa y a su regreso, conoció a la primera de pues que fui desconfiado!
las muchas mujeres a las que amó: Elena Osorio (a la que llama Argumento de El condenado por desconfiado de Tirso: Las
Filis en su poesía), hija del empresario Jerónimo Velasquez, a vidas del soberbio monje Paulo y la del criminal Enrico que,
la que acabó destinando una carta ofensiva que le ocasionó el sin embargo, tiene esperanza en su salvación, se entrecruzan.
destierro. En 1588 contrajo matrimonio con Isabel de Urbina El criminal salvará su alma porque es capaz de amar, mientras
(Belisa) y se alistó en la Armada. que el monje, por desconfiado, acabará condenándose.
Tras finalizar el destierro, vivió en Toledo junto a su esposa 5. Busca información sobre la obra de Tirso titulada Don Gil
Isabel, quien murió al dar a luz.
de las calzas verdes y explica por qué la protagonista se
En 1595 obtiene el perdón (acaba el destierro de la corte) y se disfraza de hombre.
traslada a Madrid, en donde conoce a Micaela Luján (Camila
Juana, noble doncella de escasa fortuna, es seducida por
Lucinda en sus versos), mujer casada —bella pero inculta—
Martín, a quien su padre obliga a prometerse a una dama que
con quien mantendrá una relación amorosa hasta 1608. Pero
considera mejor partido: Inés.
en 1598 había contraído segundas nupcias con Juana de Guar-
do, hija de un rico empresario, que apenas encontró mención Como Juana carece de un caballero de su familia que pueda
en su literatura. vengar su honor perdido, será ella misma, disfrazada de hom-
bre (Don Gil de las calzas verdes) quien se dirija a Madrid,
En 1605 conoce y traba amistad con don Luis Fernández de acompañada de su criado fiel (Quintana) con el propósito de
Córdoba y de Aragón, duque de Sessa, con el que mantendrá rescatar su honor.
a lo largo de toda su vida una extraña relación en la que se
mezclan los papeles de secretario y confidente. 6. Lee este fragmento de El gran teatro del mundo, en el
que, de forma metafórica, el autor del mundo reparte los
En septiembre de 1610 Lope se traslada definitivamente a
papeles.
Madrid, en donde vivirá el resto de sus días. En 1609 había
ingresado en la Congregación de Esclavos del Santísimo a) Explica cómo recibe el pobre el papel que le ha toca-
Sacramento. do en suerte.
Pero no duró mucho esta experiencia plácida y sin contratiem- b) ¿Crees que está conforme? Explica por qué.
pos. Doña Juana sufre frecuentes enfermedades y en 1612 su ¿Por qué tengo de hacer yo
hijo Carlos Félix muere de unas calenturas. En agosto del año el pobre en esta comedia?
siguiente Juana de Guardo muere también, al dar a luz. Lope ¿Para mí ha de ser tragedia,
decide entonces ordenarse de sacerdote. La huella literaria de y para los otros no?
esta crisis y sus arrepentimientos irá a parar a las Rimas ¿Cuando este papel me dio
sacras, publicadas en 1614, tu mano, no me dio en él
El recién ordenado entró enseguida en la carrera de los be- igual alma a la de aquel
neficios eclesiásticos. Pero duró muy poco la castidad del que hace al rey? ¿Igual sentido?
nuevo sacerdote. Además de la relación con una comedianta a) Lo recibe con desagrado porque sabe que la pobreza será el
(«La loca») durante su viaje a Valencia de 1616, Lope tiene el origen de las desdichas por las que tendrá que pasar en su
último gran amor de su vida en otra mujer casada, Marta de vida. Por eso habla de su vida como tragedia. Si el teatro
Nevares, a la que en los textos literarios llamará Amarilis y es metáfora de la vida de estos personajes, la tragedia es
Marcia Leonarda. metáfora de una vida mísera.

Lengua castellana y Literatura. 3.º ESO. Solucionario 109


12 TODOS A UNA

b) No está conforme porque en otros aspectos se siente igual La ambigüedad se desprende del doble significado de soldados
al rey o al rico y no entiende qué criterio se ha usado para (participio del verbo soldar y sustantivo: luchadores a sueldo)
repartir los papeles, es decir, las diferentes formas de vida. y celos (sustantivo en su significado de recelo, desconfianza
No entiende que Dios le sitúe en inferioridad porque no o persona que está a tu cargo).
siente que su alma sea inferior.
Llevar los celos soldados significa que se lleva con él los celos
7. Escribe en verso o en prosa una respuesta para el pobre, o el recelo que le produce saber que Casilda se queda a merced
como si tú hubieses repartido los papeles. de los deseos del Comendador.
Respuesta libre. Llevar a los soldados que son sus celos tiene el significado
literal: va a llevar a Toledo a los soldados a los que tiene a su
8. Busca información sobre el mito grecolatino de Narciso cargo por orden del Comendador, encargo que este le ha dado
y Eco y lee luego este fragmento de la obra de Calderón para que no estorbe sus planes.
titulada Eco y Narciso. Explica a qué momento corres-
2. ¿Qué características de la comedia barroca aprecias en
ponden estas palabras que dice Narciso mirándose en
este fragmento?
una fuente.
Los personajes. Aparecen tres de los personajes que más se
NARCISO.— Cómo tú, hermoso prodigio, repiten en la comedia barroca: la dama, el villano (encarnado
solo me miras y callas, en la figura de Peribáñez) y el poderoso (encarnado por el
yo no hago más que mirarte, Comendador de Ocaña).
y callar; pero esto basta,
porque como yo te vea, Los juegos verbales como el que se aprecia en las palabras
¿qué más dicha? […] celos y soldados.
¡Qué divina O este otro juego con el pronombre vos:
eres, deidad soberana!
… vengo, y a decir que os dejo
Bella me pareció Eco
a vos de vos misma en guarda,
antes que a ti te mirara:
porque en vos y con vos quedo;
pero después que te vi
aún no es tu sombra. Hay otro juego verbal con el que Peribáñez da a entender a
Casilda que no tiene celos o recelo de ella sino de la causa por
Eco es una ninfa del bosque que protagonizó varios relatos la que lleva a los soldados a Toledo, es decir, de la persona
mitológicos. Era habladora y juguetona y solía entretener a la que le aleja de allí con malas artes. Se refiere al Comendador.
diosa Hera. Mientras, su esposo Zeus aprovechaba que Hera
estaba distraída para cometer sus infidelidades. Por ello, la El tema de la honra y los abusos de poder.
diosa condenó a Eco a no poder hablar por sí misma sino a re- 3. ¿En qué grupo de las comedias de Lope la incluirías?
petir tan solo la última palabra que otros dijesen. Asustada, En las de tema histórico, concretamente en el grupo de los
Eco huyó de los bosques en donde vivía y se refugió en una dramas del poder injusto en los que la figura del poderoso
cueva, cerca de un riachuelo. suele abusar de su poder, en perjuicio de algún villano hon-
Narciso era un joven de gran belleza al que, al nacer, el adivino rado (Peribáñez).
Tiresias le había predicho que el ver su propia imagen en un Tema y estructura. Lope trata en esta obra los temas más
espejo causaría su perdición. Por eso su madre se había encar- importantes en el siglo XVII. La obra consta de tres actos con
gado de destruir todos los espejos y de evitar que pudiese con-
una veintena de escenas cada uno.
templarse a sí mismo. Así creció, sin conocer su propia belleza,
y se convirtió en un joven introvertido que se distraía dando 4. ¿Cuáles de los temas más recurrentes de Lope y de la
largos paseos. Un día pasó cerca de la cueva en la que habitaba comedia barroca aparecen en este fragmento?
Eco, quien se enamoró perdidamente al verlo. La ninfa empezó El amor (y su variante de los celos) y la honra.
a observarlo de lejos, pero un día Narciso la descubrió espian-
do. Cuando el joven le preguntó qué hacía allí, ella no pudo sino 5. ¿En cuántas partes se divide este fragmento? Justifica
repetir sus últimas palabras. Esto le causó la risa a Narciso y tu respuesta.
el despecho a Eco, que se fue a su cueva repitiendo las últimas Hay tres partes. Una primera abarcaría la primera interven-
palabras de Narciso: Qué tonta…, Qué tonta… ción de Peribáñez y Casilda, en la que se saludan y manifiestan
Otra versión cuenta que Eco, que sí podía hablar, le pidió su amor.
ayuda a la diosa Afrodita. Esta le concedió un tiempo para Las dos siguientes intervenciones de cada uno de ellos consti-
que intentara conmover a Narciso, pero este estaba más tuyen la segunda parte, en la que tratan el tema de la partida
pendiente de mirar su reflejo en el agua. Pasado el tiem- de Peribáñez. Siempre usando un código cifrado que solo los
po estipulado, la diosa castigó a Narciso a enamorarse de enamorados entienden. Hablan del recelo que siente Peribáñez
su propio reflejo y a Eco a repetir las últimas palabras de al partir, no de Casilda, sino del Comendador y sus oscuras
cuanto oyese. intenciones.
El texto corresponde al momento en el que Narciso ve su re- La tercera y última parte es la despedida de ambos y comienza
flejo en el agua y se enamora profundamente de él. con la intervención de Casilda: Tomad… Contiene esta parte
el símbolo del listón negro con el que Casilda intenta preve-
nir a Peribáñez del peligro y que este sabrá interpretar, pues
COMENTARIO DE TEXTO
vuelve de improviso a Ocaña y salva a Casilda.
Localización. Los esposos, Casilda y Peribáñez, sospechan Análisis de la forma y el contenido. Lope, en su Arte nuevo de
que el comendador trata de distanciarlos, mandando al espo- hacer comedias, defiende la idoneidad de un tipo de estrofa
so a Toledo, porque está enamorado de Casilda. según el tema tratado.
1. Explica la ambigüedad que se crea entre llevar «los celos 6. Mide los versos. ¿Qué estrofa se usa en este texto? Ponla
soldados» y «llevar a los soldados que son sus celos». en relación con su contenido.

110 Lengua castellana y Literatura. 3.º ESO. Solucionario


TODOS A UNA 12
Se trata del romance, poema estrófico de versos octosílabos No los llevo tan soldados,
con rima asonante en los versos pares. En este fragmento, que no tengan mucho miedo,
efectivamente, los personajes tienen una relación amorosa y no de vos, mas de la causa
están dialogando sobre los obstáculos que le han surgido a por quien sabéis que los llevo.
la misma, para lo cual, según Lope, es adecuado emplear el
romance. EL JARDÍN DE LA LITERATURA
7. Aunque el personaje de Peribáñez se ha caracterizado por 1. Tirso de Molina presenta por primera vez en la literatura
usar un registro lingüístico más sencillo desde el principio española la figura del donjuán: hombre insensible y conquis-
de la obra, quizá por haber sido ordenado caballero su ex- tador. Con él nace un mito que encontrará su mejor versión,
presión se complica. Busca y explica los recursos retóricos ya en el siglo XIX, en la obra de José Zorrilla, Don Juan Teno-
más significativos en sus parlamentos. rio. Lee este monólogo y contesta a las preguntas:
Uno de los recursos en los que mejor se plasma esa complica- DON JUAN.— A mí el papel ha llegado
ción del lenguaje es la paronomasia, que consiste en jugar con por la estafeta del viento.
palabras parecidas pero de distinto significado. Así ocurre Sin duda que es de una dama
con las palabras soldados/soldándose y zelos. Cuando Peri- que el marqués me ha encarecido.
báñez habla de los soldados (sustantivo) que lleva a Toledo, Venturoso en esto he sido.
Casilda, que ha entendido el sentimiento de celos que Peribá- Sevilla a voces me llama
ñez se lleva consigo, usa otras palabras con igual sonido pero El Burlador, y el mayor
diferente significado: gusto que en mí puede haber
Si soldados los lleváis, es burlar una mujer
ya no ternéis pena de ellos; y dejarla sin honor. […]
que nunca el honor quebró Ya está abierto el tal papel
en soldándose los zelos. y que es suyo es cosa llana,
porque aquí firma: —Doña Ana—.
También encontramos exclamaciones retóricas con las que
Dice así: «Mi padre infiel
se nos da a conocer el hondo y sincero amor de Casilda y
en secreto me ha casado
Peribáñez: ¡Ah, gallardo capitán!, ¡Ah dama, la del balcón!
sin poderme resistir;
Otro recurso es la metáfora del honor como un objeto quebra- no sé si podré vivir,
dizo: pues nunca el honor quebró, metáfora muy usada en el porque la muerte me ha dado.
teatro barroco. Si estimas, como es razón,
Se recurre a la aliteración de la n y la anáfora (ni) para rei- mi amor y mi voluntad,
terar la idea de que Peribáñez está velando por su honor en muéstralo en esta ocasión.
los versos: Porque veas que te estimo
… ni ellos fueran donde van ven esta noche a la puerta,
ni yo, señora, con ellos. que estará a las once abierta,
donde tu esperanza, primo,
Por otro lado, la situación contradictoria que la intervención
goces, y el fin de tu amor. […]
del Comendador ha creado entre los enamorados se refleja
Mi amor todo de ti fío,
en antítesis como: La seguridad que es paz / De la guerra en
y adiós». Desdichado amante.
que me veo o juegos de palabras como a vos de vos misma
¿Hay suceso semejante?
en guarda, porque en vos y con vos quedo.
Ya de la burla me río.
Conclusiones. Peribáñez manifiesta en este texto que lo que Gozarela, vive Dios,
está en juego es su honor. con el engaño y cautela
8. ¿Cree que va a perderlo o confía en que Casilda sabrá que en Nápoles a Isabela.
ser fiel? Busca los versos en los que expresa esta opinión. a) Explica la actitud del personaje con las mujeres y el
Peribáñez es consciente del peligro, pero confía plenamente significado de la expresión «burlar a una mujer».
en Casilda. Se lo hace saber en los versos: b) Teclea en un buscador los primeros versos de este
Que si zelos fueran tales parlamento del Don Juan de Zorrilla; lee la descripción
que yo los llamara vuestros completa de sus hazañas y realiza una comparación de
ni ellos fueran donde van ambos personajes (el don Juan de Tirso y el don Juan
ni yo, señora, con ellos. de Zorrilla).
Con estos versos quiere decir que si creyera que tiene motivos DON JUAN.— Como gustéis, igual es,
para estar celoso de ella o desconfiar, no se iría a ningún sitio. que nunca me hago esperar.
Además le dice más adelante que la deja a cargo de sí misma, Pues, señor, yo desde aquí,
es decir, que sabe que ella sabe cuidarse y que su firmeza es buscando mayor espacio
para él una garantía de seguridad. Pero Casilda, que le ve para mis hazañas, di
confiado, le da a entender (con el símbolo del listón negro) sobre Italia, porque allí
que el peligro no está en ella, pero está, y él debe actuar en tiene el placer un palacio.
consecuencia. De la guerra y del amor
antigua y clásica tierra,
9. Localiza los versos donde Peribáñez manifiesta su temor,
y en ella el Emperador,
y explica a quién hacen referencia. con ella y con Francia en guerra,
Recela del Comendador porque se da cuenta de que le hace díjeme: «¿Dónde mejor?
salir de Ocaña con la excusa de llevar soldados a Toledo para Donde hay soldados hay juego,
mantenerlo lejos. Se aprecia en estos versos: hay pendencias y amoríos».

Lengua castellana y Literatura. 3.º ESO. Solucionario 111


12 TODOS A UNA

c) ¿Cómo crees que es Doña Ana? Describe su carácter. de los hermanos, después se transformará en un recurso
para aumentar el amor de Manuel, al que también ella ama.
a) Su actitud es deplorable, pues de ella se desprende una
concepción machista de la mujer como objeto. Don Juan c) Respuesta libre.
utiliza a las mujeres para el propio disfrute sin tener en
cuenta las expectativas amorosas que crea en ellas. No le LA FACTORÍA DE LOS TEXTOS
importa engañarlas, utilizarlas y herirlas con su egoísmo.
1. ¿Cuál de los siguientes actos de comunicación constituye
Burlar a una mujer significa hacer que no oponga re- un diálogo? Explica tu razonamiento.
sistencia para ser seducida. Por eso añade don Juan: es
burlar a una mujer / y dejarla sin honor. Deshonrar a
a) El texto de La vida es sueño con el que se abre la
una mujer burlando su vigilancia y venciendo su firmeza unidad.
constituía una gravísima ofensa en esta época. b) El expendedor de gasolina invitándote a servirte tú
b) Respuesta libre. mismo.
c) Es una joven cándida e inocente que no se imagina las c) Un interrogatorio de la policía.
intenciones de don Juan. Le abre las puertas de su casa d) Una entrevista realizada a un personaje famoso publi-
y de su corazón porque cree firmemente en el amor que cada en el periódico.
don Juan le ha prometido, por eso afirma: Mi amor todo
de ti fío.
e) Una conversación telefónica.
Es imprudente y apasionada, carne de cañón para el in-
f) Una escena teatral en la que intervienen dos o más
compasible seductor, que ya compara la burla que pla- personajes.
nea con otras realizadas en Italia, concretamente sobre La c) si el interrogado colabora, la d), la a) y la f).
Isabella. 2. Rellena en tu cuaderno la tira cómica que aparece en el
2. El siguiente texto de Calderón pertenece a La dama lateral con un diálogo.
duende, graciosa comedia en donde doña Ángela (joven Respuesta libre.
viuda) es custodiada rígidamente por sus hermanos. Con 3. En la página www.ivoox.com encontrarás un quiosco de
mucha astucia, encuentra la manera de escribir e intentar audios. Entra en ella y teclea en el buscador «Entrevista
conquistar al huésped don Manuel entrando y saliendo a García Márquez» «Crónica de una muerte anunciada».
por una alacena: Escucha la entrevista y explica luego de dónde nace la
BEATRIZ.— ¿Cómo este hombre inspiración para escribir la obra de que se habla y a qué
viendo que hay quien lleva y trae género pertenece.
papeles no te ha espïado
Respuesta libre.
y te ha cogido en el lance?
Á NGELA.— No está eso por prevenir 4. Inventa una conversación en la que acuerdas con tus ami-
porque tengo a sus umbrales gos un plan para el fin de semana.
un hombre yo que me avisa Respuesta libre.
de quien entra y de quien sale
y así no pasa Isabel
5. Escribe un diálogo poético entre dos elementos contra-
hasta saber que no hay nadie. rios (agua/fuego; guerra/paz; amor/odio; recuerdo/olvi-
do). Ilustra tu diálogo con alguna imagen representativa
a) ¿A qué tipo de obras de las que escribió Calderón y ponlo en común con tus compañeros.
pertenece esta? ¿Por qué lo has sabido?
Respuesta libre.
b) En esta obra Calderón utiliza el recurso de «la tapada»
6. Teclea en YouTube: «RTPA. Debate en 30. Jóvenes y
en el personaje de doña Ángela, es decir, convierte a
consumo de alcohol». Observa en Internet este debate
esta en un personaje invisible cuya identidad es desco-
y decide si se ha llevado a cabo correctamente. Señala
nocida. ¿Crees que este recurso favorece que el hués-
también con qué ideas estás de acuerdo y con cuáles no,
ped don Manuel, curioso por naturaleza, se enamore
justificando por qué en cada caso.
de la que considera una «dama duende»?
Respuesta libre.
c) Continúa el fragmento con una intervención de don
Manuel explicando su curiosidad y amor hacia la dama ACTIVIDADES FINALES
duende.
a) A las de capa y espada. Porque se ve el enredo, propio de Repasa lo que has aprendido
estas comedias, que lleva a cabo el personaje de Ángela. Se 1. Lee este fragmento y contesta a las preguntas:
aprecia también en la presencia de un objeto (la alacena)
Y cuando he de escribir una comedia,
que propicia este enredo.
encierro los preceptos1 con seis llaves;
Es habitual en estas comedias encontrar conflictos amo- saco a Terencio y a Plauto2 de mi estudio,
rosos y, en esta, el conflicto está servido por el hecho de para que no me den voces; que suele
que la protagonista desafía la vigilancia de los hermanos dar gritos la verdad en libros mudos;
para jugar al amor con quien no debe. y escribo por el arte que inventaron
b) Por supuesto. El hecho de que el personaje femenino apa- los que el vulgar aplauso pretendieron;
rezca y desaparezca de manera misteriosa en la habitación porque, como las paga el vulgo, es justo
de un curioso por naturaleza hace que Manuel se sienta hablarle en necio para darle gusto. […]
doblemente atraído por la dama. Doña Ángela represen- Lo trágico y lo cómico mezclado,
ta el amor, pero también un apetitoso misterio. En prin- Y Terencio con Séneca3, aunque sea
cipio ella recurre a la alacena para huir de la vigilancia como otro minotauro de Pasifae4,

112 Lengua castellana y Literatura. 3.º ESO. Solucionario


TODOS A UNA 12
harán grave una parte, otra ridícula; Estos preceptos grecolatinos durante el Barroco dejan de
Que aquesta variedad deleita mucho. ser el modelo literario. La comedia barroca mezcla per-
Lope DE VEGA: El arte nuevo sonajes y géneros (lo cómico y lo trágico), y desarrolla
de hacer comedias (Planeta). paralelamente varias acciones.
2. Lope dice en su Arte nuevo que al «vulgo hay que hablarle
1
preceptos: reglas. 2 Plauto y Terencio: autores más importan- en necio para darle gusto». Reflexiona y responde:
tes del género teatral que destaca en Roma: la comedia. 3 Sé- a) ¿A qué clase social se refiere con «el vulgo»?
neca: filósofo y escritor romano de tragedias. 4 Según un mito
clásico, Pasifae fue raptada por un minotauro (ser mitológico, b) ¿Crees que los espectadores se identificaban con per-
mitad toro, mitad hombre). sonajes de las comedias barrocas? ¿Por qué y en qué
aspectos?
a) ¿Cuál es el tema de este texto? a) Cuando hablamos de un fenómeno de popularización del
teatro barroco nos referimos a que el público, que en prin-
b) Explica los versos que están en negrita y conecta su
cipio pertenecía a la clase media (funcionarios, soldados,
significado con las claves del éxito teatral de Lope de comerciantes, nobles) empezó a incorporar personas de
Vega. la clases más bajas. Este nuevo público estaba formado
c) ¿Cuáles son los preceptos que dice Lope encerrar con principalmente por trabajadores manuales como zapate-
seis llaves? Explica en qué consisten. Analiza su origen ros, artesanos, cocineros, etc. y que recibieron el nombre
y lo que ocurre con ellos en el Barroco. de mosqueteros, un público poco culto e irrespetuoso, que
resultaba especialmente ruidoso. Esa incultura es la
a) El tema del texto es la escritura misma. Lope explica cómo
que lleva al autor a tildar a este público con el apelativo
es el proceso de escritura de sus comedias.
de vulgar, aunque son los principales consumidores del
b) Lope dice que encierra los preceptos con seis llaves para teatro de la época.
referirse a que no sigue las reglas clásicas ni ninguna otra
b) Seguramente este público se sentía identificado en unos
regla. En sus comedias busca encontrar argumentos que
tipos de comedia más que en otros, pero era un público afi-
lleguen a un público variopinto, para lo cual escoge tam-
cionado a las comedias, pues este era el mejor espectáculo
bién el lenguaje que todo el mundo puede entender. Sabe
a su alcance para llenar sus ratos de ocio.
que su arte no es el arte refinado y sujeto a reglas que
siguieron los clásicos porque prefiere que el público no La obras con las que más podía identificarse esta clase
lo formen solo personas cultas. A un público que no es social son la de enredo y de costumbres, las de tema his-
culto no se le puede ofrecer un teatro en el que prime un tórico (muy del gusto del público), más concretamente los
registro culto. Por ello dice que sigue el arte que inventa- dramas del poder injusto de Lope, especialmente las que
ron los que el vulgar aplauso pretendieron (aquellos que cuentan con la figura del villano. Algunos personajes co-
no escribían solo para un público culto) y que al vulgo hay tidianos se prestan más a dicha identificación, como el
que hablarle en necio para darle gusto. villano (labrador honrado, humilde y trabajador) y el ca-
ballero (padre, hermano o marido de una dama, que vela
Con la expresión Lo trágico y lo cómico mezclado se re-
por ella y que dicta las normas y se opone a los amoríos
fiere a que, para los clásicos, el teatro no debe mezclar la
entre el galán y la dama). En este último caso, también
tragedia y la comedia; deben ser géneros distintos. Pero
contribuye a la empatía con el personaje del caballero la
Lope no respetó las reglas clásicas y, precisamente, en
importancia que el concepto de la honra alcanza en esta
ello residió su éxito. Logró temas y tramas que atrajeron
época. Cualquier padre, marido o hermano que asistiese
la atención de un público variopinto. Se trataba de obras
a una representación podía sentirse retratado en el perso-
cercanas, en las que, debido a la mezcla de personajes y
naje del caballero, así como las mujeres podían sentirse
tonos, todos los espectadores se veían reflejados e implica-
liberadas al contemplar cómo en las comedias de enredo,
dos. Por eso habla también de esta variedad: que aquesta
las damas logran burlar dicha vigilancia.
variedad deleita mucho.
c) Se refiere a los preceptos clásicos, es decir, la regla de Podemos decir que los autores del teatro barroco, especial-
las tres unidades de la literatura grecorromana. Los es- mente Lope de Vega, supieron ganarse a los distintos secto-
tableció Aristóteles en su Poética. Estas reglas son bási- res (de ambos sexos) que componían su variopinto público.
camente tres: 3. Lee el siguiente fragmento de la Poética de Aristóteles y
• Unidad de lugar: las obras deben desarrollarse en el contesta después a las cuestiones.
menor número posible de escenarios distintos. La perfección del estilo es que sea claro y no bajo. El que se
• Unidad de tiempo: la acción debe abarcar el menor tiem- compone de palabras comunes es sin duda clarísimo, pero
po posible, a poder ser un día. bajo. […] Será noble y superior al vulgar el que usa de palabras
extrañas. Por extrañas entiendo el dialecto, la metáfora, la
• Unidad de acción: la obra debe abarcar una única trama
prolongación y cualquiera que no sea ordinaria.
principal. No es aconsejable que se desarrollen varias
tramas diferentes para que no se disperse la atención a) Analiza si se trata de la misma concepción del estilo
del espectador en varios sucesos. que la que plantea Lope de Vega en El arte nuevo.
Además, se defiende la separación de géneros, tonos, re- b) Escribe un diálogo donde Lope y Aristóteles discrepen
gistros y personajes. sobre algún aspecto teatral, como el uso de las reglas o
La tragedia es un género de tono serio, donde se desa- la mezcla de géneros.
rrollan sucesos graves y que tiene final desgraciado. Los a) No. Aristóteles defiende aspectos del arte de la escritura
personajes son de origen noble y el tono elevado. radicalmente opuestos a los que defiende Lope. Aristó-
La comedia es un género cómico, donde se desarrollan teles propugna huir del estilo bajo, de la lengua común
asuntos humorísticos, el tono es desenfadado y los per- y de la sencillez. Para Aristóteles, el lenguaje y la comedia
sonajes tienen origen humilde. de Lope hubiesen resultado bajos, pues Lope buscaba ante

Lengua castellana y Literatura. 3.º ESO. Solucionario 113


12 TODOS A UNA

todo la sencillez, ser claro. Con su política de hablar clara- en cuando de Rojas Zorrilla sea Entre bobos anda el juego y no
mente a gente vulgar, Lope le resultaría a Aristóteles un Donde hay agravios no hay celos, que es, en mi concepto, una
escritor vulgar, alejado del lenguaje perfecto y exquisito propuesta infinitamente más perfecta y actual. […] Al margen
al que se refiere en su Poética. de esto hay unas decenas de comedias y tragedias de prime-
El arte que defiende Lope es una literatura para las masas rísimo orden que rara vez se ven en los escenarios. Pienso,
y el arte que estipula Aristóteles es una literatura para una por ejemplo, en Los cabellos de Absalón o Los malcasados
minoría selecta, un público culto. de Valencia o Eco y Narciso o El marqués de Mantua o La
prueba de las promesas y un largo etcétera.
b) Respuesta libre.
Revista Anagnórisis, n.º 2, diciembre de 2010.
4. Investiga sobre las compañías de actores. Por ejemplo,
el bululú era una compañía de un único actor, ñaque es el
nombre de la compañía formada por dos actores que lleva- 1
áureo y aurisecular: del Siglo de Oro.
ban piezas breves. Define las siguientes: gangarilla, camba-
leo, garnacha, bojiganga, farándula y compañía. a) Busca información sobre el autor y el argumento de las
Cada compañía recibe un nombre según sus integrantes: obras teatrales que menciona el entrevistado.
• Bululú: un solo actor. b) Explica por qué este fragmento es un texto dialogado.
• Ñaque: dos actores que llevaban piezas muy breves.
c) ¿Es un texto dialogado planificado o espontáneo, oral
• Gangarilla: dos hombres y un muchacho que hace de dama. o escrito?
• Cambaleo: una mujer que canta y cinco hombres. a) Respuesta libre.
• Garnacha: cinco o seis hombres, una mujer y un muchacho. b) Es un texto dialogado porque en él dos personas intercam-
• Bojiganga: dos mujeres, un muchacho y seis o siete hombres. bian información respetando el turno de palabra.
• Farándula: siete u ocho hombres, dos mujeres y un muchacho. c) Se trata de una entrevista, por lo que es un diálogo plani-
• Compañía: unos dieciocho actores y algunos auxiliares. ficado. Aunque se ha recogido por escrito, esta entrevista
se ha realizado seguramente de forma oral. Es probable
5. Recoge fragmentos de diálogos espontáneos y planifica- que al dejarla por escrito se hayan abreviado o retocado
dos (conversaciones con tu familia o tus amigos, fragmen- partes, pues no apreciamos vacilaciones, repeticiones, mu-
tos de chats, entrevistas de radio o televisión, debates, etc.) letillas, etc., bastante habituales en la lengua oral.
y analiza sus características. Expón oralmente tu trabajo a
tus compañeros. Recuerda lo que ya sabías
Respuesta libre. 8. Recita en voz alta este fragmento de La vida es sueño y
6. Analiza esta estrofa de El perro del hortelano, de Lope de da respuesta a las cuestiones que se formulan sobre él:
Vega. ¿Qué tipo de estrofa es? ¿Para qué se debe emplear, Cuentan de un sabio que un día
según Lope? tan pobre y mísero estaba,
DIANA.— Hame dicho cierta amiga que solo se sustentaba
que desconfía de sí de unas hierbas que cogía.
que el papel que traigo aquí ¿Habrá otro, entre sí decía,
le escriba. A hacerlo me obliga más pobre y triste que yo?;
la amistad, aunque yo ignoro, y cuando el rostro volvió
Teodoro, cosas de amor; halló la respuesta, viendo
y que le escribas mejor que otro sabio iba cogiendo
vengo a decirte, Teodoro. las hierbas que él arrojó.
La estrofa que usa es la redondilla, que Lope recomienda para a) Realiza su esquema métrico e indica la estrofa que
las cosas de amor. utiliza.
7. Este fragmento pertenece a una entrevista realizada a uno b) ¿Te parece que el contenido es más propio de la lírica
de los directores de las Jornadas de Teatro Clásico de Al- barroca o de la renacentista? ¿Por qué?
magro. Léelo y contesta: c) La estructura de este fragmento es la misma que la de
P REGUNTA . Si nos fijamos en la cartelera teatral, vemos los los cuentos. Establece la correspondencia entre los ver-
mismos títulos de Lope y Calderón de toda la vida. Teniendo sos y las tres partes de un cuento.
en cuenta el gran corpus teatral áureo, ¿por qué cree que no d) Explica qué moraleja se puede extraer del fragmento.
se apuesta por ofrecer más variedad? ¿Es una cuestión «co- a) La décima: 8a8b8b8a8a8c8c8d8d8c.
mercial»? ¿Lo «nuevo» es arriesgado, no vende? Nos gustaría
saber cuál es su opinión sobre el repertorio actual del teatro b) De la renacentista, por el pesimismo que encierra. La vi-
aurisecular1. sión de la existencia humana durante el Renacimiento es
más optimista y no hay alusiones tan claras a las miserias
RESPUESTA DE FELIPE B. PEDRAZA. Mi opinión es que muchas
humanas.
de las obras que están en repertorio reúnen méritos más que
sobrados para que fijemos nuestra atención en ellas. La vida c) Las tres partes propias de un cuento o medieval, concreta-
es sueño, El alcalde de Zalamea, El perro del hortelano o Don mente los que se incluyen en El conde Lucanor son: marco
Gil de las calzas verdes son obras maestras del teatro univer- narrativo, el cuento propiamente dicho y una moraleja que
sal. Está plenamente justificada la atención que les pueden se desprende del cuento.
prestar los cómicos y los espectadores. Es verdad también d) En el fragmento poético se observa la parte central, la
que una parte del actual repertorio es fruto de ciertos aza- del cuento. La moraleja que se extrae es que siempre que
res del mundo de la crítica y la edición. Así, por ejemplo, me alguien se queja hay alguien en una situación peor, pero
parece un dislate que la única obra que se representa de vez no está explícita.

114 Lengua castellana y Literatura. 3.º ESO. Solucionario


TODOS A UNA 12
9. El argumento de este fragmento retoma el de un cuento TEODORO.— Yo me voy, señora mía;
de El conde Lucanor de Don Juan Manuel, titulado «Los Yo me voy, el alma no. […]
altramuces». Léelo y resuelve las preguntas. A NARDA.— Perdidos los dos están.
DOROTEA.— ¡Qué mal se encubre el amor!
a) Repasa el argumento de ambos textos y establece si- A NARDA.— Quedarse fuera mejor.
militudes y diferencias. Manos y prendas se dan.
b) Recuerda que todos los cuentos de El conde Lucanor DOROTEA.— Diana ha venido a ser.
tienen las mismas partes. Señálalas en «Los altramuces». Lope DE VEGA: El perro del hortelano.

a) Similitudes: la situación relatada según la cual un hombre a) ¿Es este un texto dialogado? Razona tu respuesta y, si
come lo que para otro son desechos. Diferencias: en el cuen- es afirmativa, indica de qué tipo.
to de los altramuces, el alimento despreciado son las pieles b) Valora qué te ha parecido la adaptación cinematográ-
de los altramuces y los dos hombres dialogan. fica. ¿Se adecúa la representación de la actriz Emma
b) Al principio, el conde Lucanor le plantea su problema Suárez al carácter variable de Diana?
a Patronio (que aun teniendo bienes, a veces se siente
c) Escribe unos versos más en los que Diana convenza a
acosado por la pobreza); Patronio le cuenta la historia
del hombre que comía la piel de los altramuces de otro
Teodoro para que no se vaya.
hombre que la despreciaba; luego le dice al conde que a) En realidad hay dos diálogos. El de Teodoro y Diana por
reflexione sobre los motivos que él puede tener para sen- un lado. En él los personajes intercambian información,
tirse desesperado y que piense en personas que están en aunque si bien es cierto, la información más importante
peor situación. El narrador nos indica que al conde le que debieran ofrecerse (que están enamorados el uno del
sirvió la lección. Luego, se alude a que don Juan Manuel otro) no llegan a manifestarla.
vio que el cuento era interesante y lo mandó escribir. Por otro lado, está el diálogo de dos personajes que espían
Por último, aparece la moraleja: Por padecer pobreza la escena sin ser vistos: Dorotea y Anarda. Este es un diá-
nunca os desaniméis porque otros más pobres un día logo más acertado, pues no se ocultan la información que
encontraréis. saben, como sí ocurre en el caso anterior.
b) Respuesta libre.
MIRA A TU ALREDEDOR Y…
c) Respuesta libre.
… ve más allá 3. En Inglaterra, el Barroco también es una época de auge
1. El teatro del Siglo de Oro ha dejado su huella en expre- para el teatro. Realiza un mural en el que compares el
siones populares, y algunas expresiones populares se han teatro barroco en España y en Inglaterra (espacios de
inmortalizado gracias a él. Explica estas: representación, compañías, autores, etc.). Aquí tienes
algunas imágenes de los lugares de representación en
a) Hacer algo todos a una.
ambos países:
b) Ser (como) el perro del hortelano. Respuesta libre.
c) Ser un donjuán. 4. El autor más famoso del teatro inglés es William Shakes-
a) Actuar de forma unánime. peare y una de sus obras más conocidas es Romeo y Julieta.
b) Impedir que otros disfruten de algo aunque no se esté dis- Busca información sobre el argumento y haz un resumen
puesto a disfrutarlo uno mismo. en tu cuaderno.
c) Se dice de un hombre que es un seductor y tiene éxito con Romeo y Julieta, dos jóvenes de la Verona medieval, pertene-
las mujeres, aunque hoy en día se usa de forma intras- cen a familias encontradas, cuyo odio es el mayor impedimen-
cendente, sin la carga negativa de la que sí está dotado el to que encuentran los muchachos para disfrutar de su amor.
personaje de Tirso. Para poder escapar de esta situación, en la que se debaten
entre el respeto a sus familias y el amor que les une, idean
2. El cine ha sabido recoger las tramas barrocas. Un ejemplo un plan que consiste en que Romeo fingirá su muerte para
de ello es la versión cinematográfica dirigida por Pilar huir con Julieta. La casualidad hace que Julieta carezca de la
Miró de El perro del hortelano de Lope, parte de cuyo información de que la muerte de Romeo es fingida. Por eso,
rodaje se realizó en un escenario de lujo: el castillo de cuando ve a su amado aparentemente muerto, saca una daga
Sintra (Portugal). y se la clava. Al despertar Romeo de su letargo, contempla a
Julieta, muerta a su lado, y se da muerte también.
Lee este fragmento y compáralo con la escena cinemato-
gráfica. (Teclea en YouTube «El perro del hortelano ¿Llo- … encuentra la clave
ras?»). Después, resuelve las cuestiones:
1. Te recomendamos que veas la película titulada Shakes-
DIANA.— Teodoro.
peare in love, de John Madden. Además de reflejar cómo
Tú te partes; yo te adoro.
funcionaban las compañías teatrales incluye la represen-
TEODORO.— Por tus crueldades me voy.
DIANA.— Soy quien sabes, ¿qué he de hacer? tación de fragmentos de Romeo y Julieta.
TEODORO.— ¿Lloras? Observa en ella el papel de la mujer en el teatro barroco
DIANA.— No, que me ha caído como personaje y como actriz.
algo en los ojos. Respuesta libre.
TEODORO.— ¿Si ha sido
amor? 2. Lee este fragmento de La dama duende, de Calderón de
DIANA.— Sí debe de ser; la Barca, y valora si la actitud de los hermanos de doña
pero mucho antes cayó, Ángela te parece la correcta o constituye un caso de dis-
y agora salir querría. criminación sexista.

Lengua castellana y Literatura. 3.º ESO. Solucionario 115


12 TODOS A UNA

Á NGELA.— ¡Válgame el cielo, que yo 4. Busca en Internet figurines femeninos del Siglo de Oro,
entre dos paredes muera, conviértelos en personajes femeninos de la época que rei-
donde apenas el sol sabe vindiquen, en prosa o en verso, el derecho a la igualdad.
quién soy! Pues la pena mía
en el término del día Respuesta libre.
ni se contiene, ni cabe […] 5. Escucha la canción de Nach e Ismael Serrano titulada Ellas,
donde, en efecto, encerrada, un bello y poético tributo a las mujeres. En esta canción,
sin libertad he vivido, el autor habla de los sentimientos como si fuesen figuras
porque enviudé de un marido,
femeninas. ¿Qué crees que quiere expresar con estas ale-
con dos hermanos casada.
gorías? Relaciona el contenido de la canción con su estri-
Y luego delito sea
sin que toque en liviandad, billo y su título.
depuesta la autoridad Esta canción habla de distintos sentimientos personificados
ir donde tapada vea en distintas mujeres.
un teatro en quien la fama Tanto las distintas personificaciones y metáforas de que se
para su aplauso inmortal compone la canción como el estribillo ensalzan el papel que
con acentos de metal tiene la mujer en la vida del hombre. Cada una representa un
a voces de bronce llama. sentimiento nuevo y cada uno hace evolucionar al hombre y
¡Suerte injusta! ¡Dura estrella! todas dejan su huella imborrable:
ISABEL.— Señora, no tiene duda Luz (la pureza), Ilusión (el amor adolescente, la belleza, el
de que mirándote viuda, idealismo), Indiferencia (el apartamiento, la intromisión),
tan moza, bizarra y bella, Melancolía (el desencuentro y la tristeza se personifica aquí
tus hermanos cuidadosos en una especie de mujer fatal), Pasión (el amor carnal) Cons-
te celen, porque este estado tancia (representa el amor constante, la promesa), Envidia (el
es el más ocasionado fracaso, la envidia misma), Nostalgia (la cobardía), Soledad
a delitos amorosos. (el abandono), Esperanza (la esperanza misma), Libertad (la
Respuesta libre. fidelidad).
3. Debate sobre la consideración de la mujer en el siglo XVII La enumeración de todas ellas habla de la intensa vida senti-
y en la actualidad. El trabajo de actor era exclusivo de los mental en la que representan hitos importantes.
hombres. ¿Hay profesiones en la actualidad que le están El estribillo ensalza el valor de cada una de ellas, es decir, de
vedadas a la mujer? Tomad posición a favor o en contra de cada una de las mujeres que pasaron por la vida del hombre
que la mujer acceda a cualquier tipo de profesión. Nombrad y el sentimiento que despertaron; la huella que dejaron. Para
un moderador y entablad un debate. Recordad que debéis conferirles ese valor recurre a una serie de metáforas lumino-
sas muy logradas (bálsamo para tu herida, la vida tiritando
respetar todas las normas estudiadas para que este diálogo
en una estrella, los restos de un naufragio, el cielo en el que
lo sea realmente. sueñan los cautivos, brillaban como diamantes). Todo eso y
Respuesta libre. mucho más son ellas, las mujeres.

116 Lengua castellana y Literatura. 3.º ESO. Solucionario


ANEXO: LAS TIPOLOGÍAS TEXTUALES EN LA LENGUA ORAL

1. ¿Conoces el canal 24 Horas en directo? Búscalo en www. (El personaje hace una pausa y modifica la expresión del
rtve.es. ¿Cuál es su contenido? ¿Qué tipologías textuales rostro, silencio atento entre los oyentes).
aparecen ahí? … siento, como siempre, un respeto pavoroso hacia todo aquel
Noticias y temas de actualidad. Las tipologías textuales son que realiza un compromiso como el que Angust y Laura han
informativos, entrevistas, tertulias y reportajes. sellado hoy, yo no sería capaz de hacerlo y creo que es mara-
villoso que ellos sí hayan sido capaces.
2. Busca la página del programa Informe semanal en la web
mencionada. Visítala, escoge un reportaje que te interese y a) ¿De qué recursos se vale el hablante para llamar la
realiza una exposición que presentarás oralmente en clase, atención de los oyentes?
con los siguientes puntos: rasgos que lo caracterizan como b) ¿Cuál es la finalidad comunicativa que persigue con
texto oral, rasgos del reportaje, tema tratado, vigencia o su intervención?
interés de este. c) ¿Hay alusiones a la situación comunicativa? ¿Cuáles?
Respuesta libre. d) ¿Intenta el emisor conectar emocionalmente con los
3. Tecleando «Carmen Vaquero» «Garcilaso de la Vega» en oyentes? ¿Cómo?
el buscador de la página march.es, encontrarás una con- e) ¿En cuál de las tipologías textuales que has estudiado
ferencia de la profesora Carmen Vaquero sobre Garcilaso lo situarías? Justifica tu respuesta.
de la Vega. Escucha los primeros cinco minutos y comenta a) De recursos no verbales, como los toques en las copas de
los rasgos de este tipo de discurso que encuentres. cristal; de expresiones apelativas (vocativos como damas
En ese fragmento pueden comentarse los siguientes rasgos: y caballeros); se pone también de manifiesto el intento de
presentación de la conferenciante, alusión al contexto co- conectar con los receptores, primero por medio del hu-
municativo, captación de la atención de los oyentes (función mor (la pareja en cuestión aún me habla) y después en
apelativa del lenguaje), utilización de elementos visuales que el aspecto emocional (siento como siempre un respeto
sirven de complemento a la información transmitida, función pavoroso hacia todo aquel que realiza un compromiso…).
representativa del lenguaje. b) La finalidad es homenajear a los protagonistas de la ce-
4. ¿Escuchas la radio? ¿Qué emisoras conoces? Elige una y lebración, resaltar un momento especial. El emisor pre-
busca un programa de tertulia radiofónica. ¿Qué temas tende expresar sus ideas y sentimientos hacia ellos, a
tratan? ¿Cómo se dirigen unos a otros los tertulianos? ¿Hay modo de dedicatoria.
intervenciones de los oyentes? Explica el modo en que el c) Sí, al principio: Siento interrumpirles el delicioso postre.
moderador les da la palabra a estos últimos y cómo se ex- d) Sí, al expresar sus ideas y sentimientos, su punto de vista
presan cuando intervienen. personal.
Respuesta libre. e) Es un discurso, pues pretende resaltar una ocasión espe-
5. Aquí tienes un fragmento de una humorística intervención cial, y se trata de una exposición en la que la función ex-
del protagonista de la película Cuatro bodas y un funeral presiva se superpone a la representativa.
(Mike Newell, 1994). 6. Cread un anuncio publicitario utilizando diversos recur-
(La intervención comienza con unos suaves toques del hablan- sos comunicativos (verbales y no verbales). Vais a lanzar
te sobre las copas de cristal). una nueva aplicación para móviles que vosotros mismos
Damas y caballeros, siento interrumpirles el delicioso postre. inventaréis. Convenced a los compañeros de su utilidad.
Ya que soy el padrino me gustaría decirles un par de cosillas. Respuesta libre.
Esta es la segunda vez que hago de padrino y espero haberlo 7. ¿Has asistido alguna vez a una sesión de cuentacuentos?
hecho bien la primera. La pareja en cuestión aún me habla,
¿En qué consiste? ¿Qué recursos emplean los narradores
desgraciadamente en la actualidad no se hablan entre ellos, se
para conectar con el auditorio? El lenguaje verbal pre-
divorciaron hace ya un par de meses, pero les aseguro que yo
no tuve nada que ver. […] domina, pero ¿hay elementos paralingüísticos? ¿Cuáles?
¿Qué elementos acústicos o visuales emplea?
(Rostros estupefactos y desaprobadores entre los miembros
más maduros del auditorio). Respuesta libre. No obstante, el alumno deberá aludir a la
combinación de elementos verbales y no verbales, donde
Mi misión es la de hablar de Angust y él no tiene ningún lío
la paralingüística tiene una importancia clave (gestos, ento-
escabroso, o al menos eso creo…
nación, movimientos corporales). Como apoyo el cuentacuen-
(Risas de los oyentes). tos recurre a la música y, en algunos casos, a la proyección
Luego volveré sobre el tema, ahora querría decir que… de imágenes.

Lengua castellana y Literatura. 3.º ESO. Solucionario 117


GUÍAS DE LECTURA

1 Don Juan Manuel. El conde Lucanor. Intención


«De lo que aconteció a una mujer que 9. Recuerda la biografía de don Juan Manuel y luego indica
dónde se situaba él en la organización social de la Edad
le decían doña Truhana» Media.
Localización En el estamento de los privilegiados, concretamente en la
nobleza.
1. ¿Recuerdas en qué consiste la «literatura de exempla»?
Busca información sobre otras colecciones de «exempla» 10. ¿Crees que la posición social de don Juan Manuel deter-
que se difundiesen en la Edad Media y anótalas. minaba las conductas morales que transmitió en su obra?
Son colecciones de cuentos de origen folclórico que un autor
¿En qué sentido?
recopila para utilizarlos como ejemplos con objetivos morali- Sí. Las conductas que aconseja contribuyen a mantener el
zantes o didácticos. Obras muy destacadas son colecciones: orden social establecido en su época.
Calila e Dimna y Sendebar. 11. Señala qué importancia tiene el didactismo en la literatura
2. ¿Qué es un apólogo? Busca información sobre este con- medieval. A continuación, indica qué otros autores y obras
cepto y anota su definición. de la Edad Media se definen por su intención didáctica.
Narración en prosa con finalidad didáctica o moralizante, y Es uno de los rasgos más llamativos de la literatura de esta
que suele acompañarse al final de una moraleja. época. Ejemplos de didactismo los encontramos en las colec-
ciones de exempla, pero también en obras del mester de cle-
3. Investiga qué lugar ocupa el cuento de doña Truhana en-
recía, como las de Gonzalo de Berceo.
tre los cincuenta y un cuentos de esta obra.
El número siete. Contenido
4. Busca información sobre las otras cuatro partes que, junto 12. En efecto, muchos de los cuentos de El conde Lucanor pre-
a los cincuenta y un cuentos, constituyen el conjunto de sentan dificultades o conflictos frecuentes entre los nobles
El conde Lucanor. del siglo XIV. ¿Ocurre esto en el cuento de doña Truhana?
Segunda parte: enseñanza moral a partir de cien aforismos. ¿Cuál es en este caso la situación que preocupa al conde?
Don Juan Manuel declara su intención de seguir abordando Al conde le han propuesto un negocio que, según parece,
los temas que puedan contribuir a la salvación de las almas podría conllevar consecuencias muy positivas, que irían des-
de los hombres a la vez que a conservar su riqueza, su posición gajándose unas de las otras.
y su dignidad. 13. Una vez expuesto el conflicto, se produce una elipsis narra-
Tercera parte: semejante a la anterior, basada en cincuenta pro- tiva. Esto es: se dice que el conde explicó los detalles de
verbios. Persigue los mismos objetivos que en las dos partes la situación que le inquietaba, pero no se exponen dichos
anteriores pero la comprensión es más difícil que en ellas. detalles. Localiza el momento en que se produce dicha elip-
Cuarta parte: prevención de Patronio sobre la complejidad con sis. ¿Por qué crees que se realiza?
que abordará los temas que le ocupan ahora, y que exigirá del
Al comienzo del tercer párrafo, cuando el narrador dice:
conde la mayor concentración.
E contó a Patronio la manera cómmo podría seer. La elipsis
Quinta parte: Patronio explica los requisitos necesarios para evita detalles innecesarios, aporta sencillez y permite centrar-
la salvación del alma: la fe sin dudas, la práctica de bue- se en el asunto que realmente interesa: el apólogo.
nas obras con la finalidad de merecer el cielo, evitar las malas
14. Antes de narrar su cuento, Patronio anticipa una opinión
acciones.
al conde sobre el problema que le ha planteado. ¿Cuál es?
5. Investiga sobre Calila e Dimna y explica las relaciones
Que lo sensato es atenerse a las cosas ciertas y razonables, y
que existen entre este libro y El conde Lucanor. no a las esperanzas vanas.
La estructura es muy semejante, así como el marco narrativo.
15. Redacta un resumen de lo que le ocurrió a doña Truhana.
En ambos casos se parte del motivo del mentor que instruye
al joven. Además, algunos cuentos de El conde Lucanor están Debe ocuparte entre cinco y ocho líneas.
tomados del Calila e Dimna, como el de doña Truhana. Doña Truhana se disponía a vender una olla de miel en el
mercado, e iba pensando en cómo invertiría sus ganancias, de
6. El argumento del cuento de doña Truhana, sin duda, debe
forma que se multiplicaría sucesivamente hasta hacerla rica.
de resultarte familiar. ¿Con qué otro relato podemos aso- Pero sus ilusiones terminan al rompérsele la olla.
ciarlo? Justifica tu respuesta.
16. Al terminar el relato, Patronio expone claramente su con-
El cuento de la lechera, que se centra en el tema de la joven
sejo. ¿Cuál es?
fantasiosa que concibe ilusiones vanas y las pierde antes de
haberlas rozado siquiera. Hay que centrarse en las cosas reales y razonables, y no en
los sueños vanos.
7. Indica en qué estamentos estaba organizada la sociedad
medieval. 17. Delimita el tema central de este cuento y responde: ¿cuál
es la conducta sobre la que se pretende moralizar?
Nobleza, clero (alto y bajo) y pueblo llano (campesinos, villa-
nos, artesanos y burgueses). El tema central de este cuento es la importancia de las cosas
tangibles y la fragilidad de las ilusiones. La enseñanza advier-
8. Señala en manos de quiénes estaban la cultura y el conoci- te sobre las conductas excesivamente fantasiosas.
miento en la Edad Media. Reflexiona sobre esto y respon-
de: ¿a qué podía deberse el interés de don Juan Manuel 18. ¿Crees que la enseñanza de este cuento tiene vigencia en
por transmitir enseñanzas a otros? la actualidad y en nuestra época? Justifica tu respuesta.
Respuesta libre.
De los nobles y, sobre todo, del clero. Don Juan Manuel pretende
mantener el orden social establecido pero también considera 19. ¿Conoces la expresión «vender la piel del oso antes de
que el hecho de ser un hombre formado y culto le confiere la res- cazarlo»? Explica su significado y relaciónalo con la con-
ponsabilidad de instruir a otros sobre las buenas costumbres. ducta de doña Truhana.

118 Lengua castellana y Literatura. 3.º ESO. Solucionario


GUÍAS DE LECTURA

La expresión describe perfectamente la conducta de doña Los personajes son Patronio y el conde, en toda la colección,
Truhana. Se refiere a las personas que se dejan llevar por y los personajes de cada cuento en particular.
fantasías poco razonables sobre proyectos que aún ni siquiera Discurso propio de la narración, con elementos espaciotem-
han comenzado a poner en práctica, viviéndolos como si ya porales mínimamente necesarios, verbos en pasado e intro-
estuviesen cumplidos. ducción de diálogos y descripciones cuando son requeridos.
20. En el refranero español existen enseñanzas semejantes a 29. ¿Qué palabras y expresiones aluden al tiempo en el cuen-
la que se contiene en este cuento. Cita algún ejemplo y to? ¿Y al espacio?
explica su significado. Alusiones temporales mediante locuciones como otra vez o
Más vale pájaro en mano que ciento volando. Este refrán un día. Al espacio se alude en el relato de Patronio al hablar
previene sobre el peligro de forjar demasiadas ilusiones sobre del mercado y del camino.
proyectos o deseos que aún no tienen visos de materializarse.
30. Justifica por qué podemos adscribir el relato de doña
Personajes Truhana al subgénero narrativo del cuento.
21. Patronio es el ayo del conde Lucanor. Investiga y explica Por su brevedad y por la simplicidad de la trama. No interesan
qué era un ayo en la Edad Media. los detalles ni hay descripciones prolongadas, sino el narrador
El ayo era un sirviente que, en las casas de los nobles, se dedi- que se centra en lo que se relaciona directamente con el tema
caba a instruir y educar a los niños y jóvenes. central. De los personajes no hay una caracterización profun-
da, sino plana, porque su función es, estrictamente, presentar
22. Señala a qué estamento de la Edad Media pertenece el el tema que se quiere abordar.
conde. Ahora reflexiona e indica qué edad crees que tie-
ne, dado que necesita un ayo. Estilo
Pertenece al estamento de la nobleza. Es un hombre joven, que 31. Relaciona la sencillez del estilo con la intención didáctica
está aprendiendo aún a desempeñar las responsabilidades de don Juan Manuel.
que le depara su posición social. La sencillez del estilo contribuye a la eficacia comunicativa,
23. ¿Qué datos se nos dan sobre doña Truhana? ¿Son físicos o garantiza la comprensión.
psicológicos? ¿Hay alguna descripción de ella o podemos 32. Cita frases o partes del cuento que ejemplifiquen dicha
caracterizarla a través de sus conductas? sencillez.
Se nos dan pocos rasgos y ninguno es físico. Se nos dice que Señor conde Lucanor, siempre oí decir que era buen seso
era pobre y muy fantasiosa. atenerse omne a las cosas çiertas e non a las vanas fuças;
24. Escribe tú ahora una caracterización del personaje de doña E pensando en esto començó a reir con grand placer que avía
Truhana de acuerdo con la información que el narrador de la su buena andança, e, en riendo, dio con la mano en su
proporciona. fruente e entonçe cayó la olla de la miel en tierra, e quebrose;
Cuidat siempre todas las cosas tales que sean aguisadas e
Respuesta libre. non fuzas dubdosas e vanas.
25. Teniendo en cuenta dicha información, ¿cuál es el grupo 33. Localiza los elementos dialogados que existen en el cuen-
social de la Edad Media al que pertenece doña Truhana? to de doña Truhana.
Justifica tu respuesta.
El diálogo se introduce en la primera parte, cuando el conde
Pueblo llano, puesto que se nos dice que es pobre y su medio expone su problema a Patronio; se da por segunda vez cuan-
de vida es vender sus productos en el mercado. do el ayo, tras escucharle, expone su opinión, justo antes de
Estructura hacer su relato.
26. Recuerda qué es un pareado y explícalo. 34. Busca en el cuento ejemplos de estilo directo e indirecto.
Estrofa de dos versos que riman entre sí. Estilo directo: Fablava el conde Lucanor con Patronio en esta
guisa: —Patronio, un omne me dixo…; Respondió al conde
27. Diferencia esas cinco partes en el cuento de doña Truha- en esta manera: —Señor conde Lucanor, siempre oí que…
na y enuncia las ideas que se recogen en cada una.
Estilo indirecto: E contó a Patronio la manera cómmo podría
Primera parte, párrafo segundo. El conde explica a Patronio seer.
la propuesta que le han hecho y que, aparentemente, le traerá
tantos beneficios. 35. Observa los tiempos y modos verbales utilizados por
Segunda parte: narración del cuento de doña Truhana por
Patronio en el cuento de doña Truhana. ¿Cuáles emplea
parte de Patronio. para contar el relato? ¿Y para aconsejar al conde?
Tercera parte: Patronio explica la aplicación práctica de su En el relato utiliza el pretérito perfecto simple y el imperfecto
relato (E vos, señor conde, si queredes que lo que vos dixieren de indicativo. Cuando aconseja al conde utiliza sobre todo el
e lo que vos cuidardes sea todo cosa çierta). imperativo.
Cuarta parte: el conde acepta y sigue el consejo (Al conde 36. Ya sabemos que todos los cuentos del libro terminan con
plogo de lo que Patronio le dixo). una moraleja en forma de pareado. ¿Por qué crees que
Quinta parte: pareado final con la moraleja del relato (A las terminan así? ¿Tiene alguna utilidad práctica?
cosas çciertas vos comendat / e las fuizas vanas vos dexat). La versificación, el ritmo y la rima contribuyen a la memo-
rización.
Género
28. Localiza en el cuento de doña Truhana todos los elemen- El texto como acto comunicativo
tos característicos de la narración y explícalos. 37. Teniendo en cuenta lo anterior, ¿quiénes son los recep-
El marco tiene un narrador omnisciente que organiza la reco- tores en cada caso?
pilación de cuentos. En cada uno de ellos, a su vez, hay otro El receptor del texto global es el lector de cualquier época. El
narrador: Patronio. del relato de Patronio es el conde Lucanor.

Lengua castellana y Literatura. 3.º ESO. Solucionario 119


GUÍAS DE LECTURA

38. ¿Recuerdas la diferencia entre autor y narrador? Explícala Recuerde el alma dormida / avive el seso y despierte (copla I);
a partir de este cuento. Indica si aparece el autor en el Ved de cuán poco valor / son las cosas tras que andamos / y
texto y explica dónde. corremos (copla VIII); Decidme: la hermosura, / la gentil fres-
cura y tez / de la cara, / el color y la blancura, / cuando viene
El autor de la obra es el infante don Juan Manuel. El narra-
la vejez, / ¿cuál se para? (copla IX); Los estados y riqueza /
dor del texto global es un narrador omnisciente y externo a
que nos dejan a deshora / ¿quién lo duda? / no les pidamos
la narración. El narrador de cada cuento de la colección es
firmeza (copla XI).
Patronio. El autor aparece mencionado al final, tras el relato
de Patronio. Es el narrador omnisciente quien lo menciona Contenido
cuando dice E porque son Johan se pagó deste exiemplo, fízo- 5. En la primera copla el poeta presenta el alma humana
lo poner en este libro e fizo estos viessos. como un ser dormido que debe despertar. ¿Para qué?
39. A su vez, Patronio y el conde son interlocutores de un acto ¿De qué debe tomar conciencia?
comunicativo. Explica sus elementos a partir del cuento de Para que se haga consciente de que su estancia en el mundo
doña Truhana. es provisional, un mero tránsito que pasa rápido, trayendo la
Patronio y el conde establecen un acto comunicativo que con- muerte sin remedio.
siste en el problema que expone el conde y el consejo que le da 6. ¿Qué quiere decir el poeta en la segunda copla cuando
Patronio, utilizando el cuento como ejemplo. En el relato de afirma que «si juzgamos sabiamente, daremos lo no veni-
dicho cuento, Patronio es el emisor, el cuento es el mensaje y do por pasado»?
el conde es el receptor.
Se refiere a la fugacidad de las cosas. Todo pasa rápidamente.
Temas para la reflexión Igual que el presente se marcha en un instante, el futuro será
40. Si en la actualidad se escribiese este libro, ¿qué clase pasado antes de que nos demos cuenta.
de autor tendría? ¿Quiénes crees que deben transmitir 7. En la Copla III se presenta el río como metáfora de la vida.
enseñanzas a otros en la sociedad actual? Explica cómo esta metáfora le sirve al poeta para hablar
Respuesta libre. del poder igualatorio de la muerte.
41. ¿Sobre qué costumbres crees que podría asentarse en la Igual que todos los ríos van al mar, sea cual sea su caudal,
todas las trayectorias humanas acaban en la muerte, sea cual
actualidad una enseñanza para ser un buen ciudadano?
sea la importancia o condición de la persona.
¿Morales, religiosas, cívicas? Redacta un escrito de veinte
líneas sobre el tema. 8. ¿A quién invoca el yo poético en la cuarta copla?
A Cristo.
En la actualidad, el didactismo suele orientarse al fomento de
conductas cívicas. 9. En la Copla V, nuevamente se habla de la vida mediante
42. Busca otros ejemplos en la literatura o en el cine que bus- un símbolo. ¿Cuál?
quen enseñar entreteniendo. El del camino.
Respuesta libre. 10. La idea que se recoge en dicha copla es típicamente
medieval. Explícala.

2 Jorge Manrique. Coplas a la muerte


La vida solo es un tránsito hacia la muerte, que nos dará el
merecimiento por nuestros actos. Ese tránsito debe hacerse
de su padre. Coplas I-XIII con acierto y bondad.
11. En la Copla VI el poeta insiste en la idea anterior. Cita los
Localización versos que lo demuestran.
1. La poesía culta del siglo XV supone una novedad en el Este mundo bueno fue / si bien usáramos de él / como debe-
panorama poético porque hasta el momento solo encon- mos / porque, según nuestra fe / es para ganar aquel / que
tramos casos de poesía narrativa y didáctica. Explica las atendemos.
diferencias entre la poesía lírica y la poesía narrativa. 12. Explica la reflexión sobre la belleza espiritual y material
La poesía lírica aborda temas relacionados con los sentimien- de la Copla VII.
tos y su expresión. La poesía narrativa se conforma de relatos La belleza espiritual depende de nosotros. La material o físi-
versificados. ca, no. Si dependiese, nos dedicaríamos solo a ella en vez de
2. En las Coplas de Manrique conviven la visión medieval del atender a la del alma.
mundo y la renacentista. El poeta nos habla del escaso valor 13. Señala el valor que tienen las cosas mundanas. Resume
de las cosas terrenales, y también de la preocupación huma- el contenido de la Copla VIII.
na por dejar memoria nuestra en este mundo. ¿Cuál de Las cosas mundanas que nos preocupan tienen escaso valor,
estas dos ideas es medieval y cuál anticipa el Renacimiento? son frágiles y poco duraderas, se pierden incluso antes de
El escaso valor de las cosas terrenales es una idea medie- morir.
val. El deseo de dejar memoria en este mundo tras nuestra 14. En la Copla IX, ¿cómo se habla del paso de la juventud
muerte forma parte de una nueva visión que viene con el a la vejez?
Renacimiento. Habla de la transformación que sufren la belleza y la fuerza de
3. ¿Cuál de esos temas se trata en las trece primeras coplas la juventud, que se convierten en gravedad al llegar la vejez.
de la composición? 15. En la Copla X se alude a la nobleza, cuya elevada condi-
La vanidad de las cosas terrenales. ción no siempre se corresponde con el nivel que alcanza
en esta vida. ¿Cuáles son las dos razones por las que pue-
Intención de ocurrir esto, según Manrique?
4. Observa el tono exhortativo de las trece coplas que has Unos porque no tienen la valía que corresponde a su elevada
leído: enumera las llamadas de atención y recomendacio- posición; otros porque no tienen medios para vivir de acuerdo
nes que hace Manrique. con esta y deben desempeñar oficios impropios de la nobleza.

120 Lengua castellana y Literatura. 3.º ESO. Solucionario


GUÍAS DE LECTURA

16. En la Copla XI aparece la Fortuna como símbolo de lo la fama). Esta idea puede hacer más llevadero el trance de
precario y lo voluble. Explica esta idea relacionándola con la muerte, en el que deberá renunciar al mundo vano y pere-
la mentalidad medieval. cedero (vida terrenal). La vida de la fama (vida de honor la
llama en la copla XXXV) es mucho más larga y sólida que la
Las cosas mundanas son frágiles, pueden ser ganadas o per-
terrenal, pero también acabará extinguiéndose. La celestial
didas azarosamente. No puede confiarse en su firmeza o su
es la única eterna.
persistencia, hay que asumir la volubilidad de lo material.
17. La Copla XII sigue hablando de los estados y la riqueza Personajes
que, según dice, se van a la tumba con su dueño. ¿En qué Copla XXVI Copla XXXIX
momento se hace esta afirmación? Amigo de sus amigos, No dejó grandes thesoros
En los tres primeros versos. ¡qué señor para criados ni alcanzó grandes riquezas
y parientes! ni baxillas3,
18. También en la Copla XII se presenta una contraposición
¡Qué enemigo de enemigos! mas hizo guerra a los moros
entre lo efímero y lo eterno. Localízala y explica el sentido
Qué maestro de esforçados1 anando sus fortalezas
cristiano que tienen estos versos. y valientes! y sus villas;
El placer terrenal es breve; los tormentos de la condenación, ¡Qué seso para discretos! y en las lides que venció,
eternos. E los deleites d´acá / son, en que nos deleitamos, / ¡Qué gracia para donosos! muchos moros y cavallos
temporales, / e los tormentos d´allá, / que por ellos esperamos, ¡Qué razón! se perdieron,
/ eternales. ¡Qué benigno a los subjetos2! y en este oficio ganó
19. ¿Qué conducta humana se describe en la Copla XIII? ¿Tie- Y a los bravos y dañosos, las rentas y los vasallos
ne sentido, según el poeta? ¡un león! que le dieron.
La tendencia a caer en la trampa del placer mundano, sin pen-
sar en las consecuencias hasta que es demasiado tarde y ya 1
esforçados: aguerridos. 2
subjetos: vencidos. 3
baxillas: ajuar.
no es posible rectificar.
20. Investiga ahora sobre el contenido de las Coplas XIV-XXIV 22. A partir del contenido de esas estrofas, describe la perso-
y enumera los personajes y hechos a los que Manrique nalidad de este caballero y la importancia de sus hechos.
alude en ellas.
Hombre generoso con sus amigos, buen señor para sus sir-
Copla XXXIII Y pues de vida y salud vientes, prudente y sensato, magnánimo, valiente y aguerrido.
Después que puso la vida hezistes tan poca cuenta Sus riquezas no fueron muchas, pero sus hazañas contra los
tantas veces por su ley por la fama, moros fueron notables y le permitieron forjar su patrimonio.
al tablero, esfuércese la virtud
después de tan bien servida para sofrir esta afruenta Estructura
la corona de su rey que os llama. 23. ¿Qué medida tienen los versos utilizados? ¿Cuál es la
verdadero, organización de la rima?
Copla XXXV
después de tanta hazaña
—No se os haga tan amarga Ocho y cuatro sílabas. En cada copla, la rima duplica el
a que no puede bastar
la batalla temerosa esquema 8a8b4c8a8b4c. La composición se llama copla de
cuenta cierta
que esperáis, pie quebrado.
en la su villa de Ocaña,
pues otra vida más larga 24. Localiza en el conjunto de las trece coplas iniciales estos
vino la muerte a llamar
de fama tan gloriosa contenidos: una exhortación, una invocación a Cristo y
a su puerta,
acá dexáis;
Copla XXXIV una reflexión sobre la vanidad de las cosas terrenales.
aunque esta vida de honor
diciendo: —Buen caballero, tampoco no es eternal Exhortación, coplas I-III. Invocación a Cristo, coplas IV-VI.
dexad el mundo engañoso ni verdadera, Reflexión sobre la vanidad de las cosas terrenales, coplas
y su halago; mas con todo es muy mejor VII-XIII.
vuestro coraçon de azero que la otra temporal
muestre su esfuerço famoso Género
pereçedera.
en este trago. 25. Explica los elementos del género lírico a partir de las
Copla XIV: los antiguos reyes, papas, emperadores. Copla XV: coplas que has leído.
los héroes de Troya y los romanos. Copla XVI: el rey Juan II, El uso del verso, la rima y el ritmo. Se puede comentar el
los infantes de Aragón (hijos del rey Fernando). Copla XVII: las esquema métrico, el arte menor y la rima consonante. Asimis-
damas lujosamente vestidas, los amantes que las cortejaban mo, el empleo del lenguaje figurado mediante recursos como
con música. Copla XVIII: Enrique IV, sucesor del rey don Juan. la metáfora (por ejemplo, los ríos y el mar para referirse a la
Copla XIX: las riquezas de los palacios. Copla XX: don Alfonso, vida y a la muerte respectivamente) o el símbolo (el camino,
hermano de Enrique IV que, aunque fue proclamado rey siendo que representa la vida). Se puede hablar también de recursos
aún niño, no llegó a reinar nunca y murió a los catorce años. expresivos como la interrogación retórica, en las coplas IX
Copla XXI: el condestable Álvaro de Luna, que asumió el poder y XI, o el paralelismo sintáctico, que favorece el ritmo y que
durante el reinado de Juan II y murió decapitado. Copla XXII: el se utiliza con mucha frecuencia (copla I, copla III, copla IV,
maestre de Santiago, don Juan Pacheco, marqués de Villena y copla XII, etcétera).
el maestre de Calatrava, don Pedro Girón. Copla XXIII: nobles 26. Podríamos decir que este poema es también una expo-
que realizaron notables hazañas en la guerra. Copla XXIV: los
sición de ideas. Argumenta esta afirmación con citas del
grandes ejércitos y las fortalezas inexpugnables.
texto.
21. Explica cómo se alude a las tres vidas en las Coplas XXXIII, Ejemplos de argumentación que pueden citarse: copla V, donde
XXXIV y XXXV. presenta la idea de la vida terrenal como un simple tránsito;
Don Rodrigo llevó a cabo honrosas y elevadas hazañas. De coplas VII-XII, donde habla del escaso valor de las cosas mate-
ello quedará memoria para las futuras generaciones (vida de riales y de la vida terrenal.

Lengua castellana y Literatura. 3.º ESO. Solucionario 121


GUÍAS DE LECTURA

Estilo c) Divinización de Melibea por parte de Calisto.


27. Localiza ejemplos de exhortaciones en las coplas que has d) Planteamiento del tópico del mundo como valle de
leído y relaciónalos con el tono didáctico de la obra. ¿Qué lágrimas por parte de Pleberio, padre de Melibea, al
tiempos y modos verbales predominan? final de la obra.
Recuerde el alma dormida / avive el seso e despierte… a) Medieval.
(copla I); Ved de cuán poco valor / son las cosas tras que
b) Renacentista.
andamos… (copla VIII); Dezidme: la hermosura, la gentil
frescura y tez… (copla IX). Verbos en imperativo. c) Renacentista.
28. Localiza en las Coplas I-XIII ejemplos de metáfora, com- d) Medieval.
paración, paralelismo sintáctico, interrogación retórica. Intención
¿Cómo contribuyen a la intención del poeta? Síguese la comedia o tragicomedia de Calisto y Melibea, com-
Metáfora: entre otros ejemplos, se pueden citar los de la puesta en reprehensión de los locos enamorados que, vencidos
copla III, cuando se identifica a las vidas con los ríos, y a en su desordenado apetito, a sus amigas llaman y dicen ser su
la muerte con el mar; copla XIII, cuando se identifican los Dios. Asimismo hecha en aviso de los engaños de las alcahue-
placeres de la vida terrenal con soldados de avanzadilla tas y malos y lisonjeros sirvientes.
(corredores). Comparación: en la copla XII se dice que se va
la vida apriessa como sueño. Paralelismo sintáctico: véanse
2. Según esta declaración de Rojas, ¿qué clase de amor se
las coplas I, III, IV o XII. Interrogación retórica: hay ejem- condenaría en la obra? ¿Tiene relación con el amor cortés?
plos en las coplas IX y XI. Las metáforas y comparaciones El amor loco, el pasional, el carnal. No tiene que ver con el
incrementan la expresividad y, a la vez, afianzan la idea que amor cortés.
pretende transmitir al lector, pues presentan una simbología 3. ¿Por qué se advierte contra la ayuda de las alcahuetas y
propia de la cultura cristiana (por ejemplo, el planteamiento los sirvientes?
de la vida como un camino) que resulta perfectamente enten-
dible por el receptor, que comparte con Manrique esos plantea- Porque solo pretenden engañar con el fin de obtener beneficio
mientos culturales. Los paralelismos sintácticos contribuyen propio.
al ritmo y, a la vez, al didactismo. La finalidad didáctica tam- Por Dios, dejemos enojo y al tiempo el consejo. Hayamos
bién queda muy reforzada con las interrogaciones retóricas. mucho placer. Mientras hoy tuviéremos de comer, no pen-
semos en mañana. También se muere el que mucho allega
El texto como acto comunicativo como el que pobremente vive, y el doctor como el pastor y el
29. ¿Por qué crees que un poeta que desea hacer una alabanza papa como el sacristán y el señor como el siervo y el de alto
a la memoria de su padre se detiene tanto en esas consi- linaje como el bajo, y tú con oficio como yo sin ninguno. No
deraciones previas y generales? habremos de vivir para siempre. Gocemos y holguemos, que
Pretende transmitirnos su visión de la vida y de la muerte, su la vejez pocos la ven, y de los que la ven ninguno murió de
idea sobre el proceder correcto de un buen cristiano, tanto en hambre.
el desarrollo de su vida como en el momento de afrontar la 4. Esta declaración de Elicia alude al poder igualatorio de
muerte. La figura de su padre se convierte en un modelo de la muerte. También Manrique había tratado este asunto
conducta, en ese ideario que ha expuesto. en el siglo XV, aunque con intención distinta. Compara
Temas para la reflexión ambos enfoques.
30. Dos ideas destacan en las trece coplas iniciales: la de que En ambos casos se afirma que la muerte aguarda a todos, sea
cual sea la condición o posición, es lo único que iguala a todos
cualquier tiempo pasado fue mejor, y la de que esta vida solo
los humanos. Sin embargo, mientras que Manrique hablaba de
es un tránsito para la eterna, que es la que importa. Analiza
la vida como tránsito perecedero hacia la eternidad, que era lo
y valora estas dos ideas desde la perspectiva del siglo XXI. que verdaderamente importaba, aquí se plantea que hay que
Posiblemente la idea de que cualquier tiempo pasado fue mejor intentar vivir gozando, precisamente por el carácter efímero
sea propia de la naturaleza humana, pues en nuestro tiempo de la existencia. Es decir, si para Manrique ese carácter pere-
sigue ocurriendo que a veces se idealiza el pasado. No ocurre cedero era el argumento que restaba valor a la vida, ahora es
lo mismo con la idea de que la vida es un tránsito hacia la eter- precisamente lo que se lo confiere.
nidad, esta idea no está presente ya en el siglo XXI, sino todo lo
contrario. En nuestro tiempo y en nuestra sociedad existe el Contenido
principio de que hay que intentar alcanzar la felicidad porque 5. Resume en tres o cuatro líneas las palabras de Calisto a
esta vida es lo único que tenemos. Melibea al comienzo de la obra.
Calisto alaba a Melibea por su perfecta hermosura, a la que
3 Fernando de Rojas. La Celestina. considera el mayor regalo divino. Se mezclan sentimientos
carnales con espirituales y la ausencia de Melibea le provo-
Acto primero: enamoramiento de Calisto cará un gran dolor.
y conversación con Sempronio 6. ¿Cómo reacciona Melibea ante esa declaración? ¿Por qué
crees que reacciona así?
Localización
Con enojo. Considera que Calisto es un atrevido y que pone
1. Indica si los siguientes elementos de la obra son medie- en peligro su virtud.
vales o renacentistas:
7. Al ver el estado en que se encuentra su señor, ¿qué dudas
a) Intención moralizante del autor al advertir contra los tiene Sempronio?
peligros del amor y el servicio de las alcahuetas. No sabe si dejarle solo con su dolor o acompañarle. Si le deja
b) Exaltación de los placeres, del amor, de la juventud y solo, teme que Calisto se mate. Si le acompaña, teme que le
de la belleza. mate a él.

122 Lengua castellana y Literatura. 3.º ESO. Solucionario


GUÍAS DE LECTURA

8. Calisto le pide a Sempronio que cante con un laúd y hace 18. Explica cómo se contrapone el hecho de que Calisto acu-
una intervención que comienza diciendo: «¿Cómo templa- da a Celestina para conseguir a Melibea con el concepto
rá el destemplado?». ¿Cómo describe el joven su estado del amor en la tradición cortés.
de ánimo en dicha intervención? Comenta su contenido. En la tradición cortés, la amada se presentaba como inalcan-
Está desasosegado. No es capaz de conciliar su voluntad con zable y así era aceptado por el amante. No podía ser de otro
su razón. Tiene diversos sentimientos, todos atormentados. modo, ese era el código de conducta. Además, en la tradición
9. Explica el sentido que tiene en este contexto la declaración cortés nos encontramos ante un sentimiento idealizado. El de
aquí es carnal y pasional.
de Calisto: «Melibeo soy y a Melibea adoro y en Melibea
creo y a Melibea amo». Oh, duro corazón de padre, ¿cómo no te quiebras de dolor,
que ya quedas sin tu amada heredera? ¿Para quién edifiqué
Previamente Sempronio le ha dicho: ¿Tú no eres cristiano?
torres; para quién adquirí honras; para quién planté árboles;
Calisto proclama su amor por Melibea como su única fe, su
para quién fabriqué navíos? […]
única devoción. El amor es su religión.
Oh, fortuna variable, ministra y mayordoma de los temporales
10. Cuando Calisto confiesa a Sempronio su estado de ánimo, bienes, ¿por qué no ejecutaste tu cruel ira, tus mudables ondas,
este le hace unas advertencias sobre lo poco cristianas en aquello que a ti es sujeto? ¿Por qué no destruiste mi patri-
que resultan algunas de sus afirmaciones. ¿Qué es exac- monio; por qué no quemaste mi morada; por qué no asolaste
tamente lo que le advierte? mis grandes heredamientos? Dejárasme aquella florida plan-
Que lo que dice es contrario a los fundamentos cristianos, que ta, en quien tú poder no tenías; diérasme, fortuna fluctuosa,
es una especie de herejía. triste la mocedad con vejez alegre; no pervertieras la orden.
11. El asunto de la blasfemia y la herejía continúa presente […]
en la conversación. Explícalo. ¡Oh, amor, amor, que no pensé que tenías fuerza ni poder de
Calisto dice que Melibea es su dios, y Sempronio le responde matar a tus sujetos! […] ¿Quién te dio tanto poder? ¿Quién te
que eso es una blasfemia y que su pecado es peor que el de puso nombre que no te conviene? Si amor fueses, amarías a
Sodoma. Esto último, sin embargo, hace reír a Calisto. Sem- tus sirvientes. Si los amases, no les darías pena. Si alegres
pronio, en su discurso, se mueve entre la advertencia sobre viviesen, no se matarían, como agora mi amada hija. ¿En qué
los excesos emocionales de su señor y la broma o la ironía. pararon tus sirvientes y tus ministros? La falsa alcahueta
Celestina murió a manos de los más fieles compañeros que
12. Sempronio previene a su señor contra las mujeres en una ella para tu servicio empozoñado jamás halló. Ellos murieron
actitud claramente misógina. Localiza los pasajes en los degollados; Calisto, despeñado. Mi triste hija quiso tomar la
que pueda percibirse esta actitud. misma muerte por seguirle. Esto todo causas.
¿Quién te contaría sus mentiras, sus tráfagos, sus cambios, 19. ¿Cuáles son los sentimientos que alberga Pleberio como
su liviandad, sus lagrimillas, sus alteraciones, sus osadías?
padre?
[…] ¿Sus disimulaciones, su lengua, su engaño, su olvido,
su desamor, su ingratitud, su inconstancia, su testimoniar, Cualquier desgracia, por grande que hubiese sido, habría sido
su negar, su revolver, su presunción, su vanagloria, su aba- menor que la que vive ahora. Se ha invertido el orden natural,
timiento, su locura, su desdén, su soberbia…?; No tienen que manda que los padres mueran antes que los hijos. Además,
modo, no razón, no intención […] ¡Oh, qué plaga, oh, qué su vida ahora no tiene sentido: su hija era la heredera de todo
enojo, oh, qué hastío es conferir con ellas más de aquel breve lo que él había logrado a lo largo de su existencia.
tiempo que aparejadas son a deleite! 20. ¿Qué reproches le hace a la fortuna? ¿Y al amor?
13. ¿Tienen efecto en Calisto las palabras de Sempronio? La fortuna ha trastocado el orden debido, pues él tendría que
¿Cómo reacciona a ellas? haber muerto antes que su hija. Al amor le reprocha su tiranía
con quienes lo albergan, pues los destruye.
Calisto persiste en su obsesión por Melibea, y se considera el
más desgraciado de los hombres porque ella le rechaza. 21. ¿Qué sentido crees que tiene el hecho de que aluda expre-
14. Calisto hace una descripción de Melibea. ¿Con qué acti- samente al triste final que han tenido todos los personajes?
tud la escucha Sempronio? Un sentido moralizante, como si mostrase las consecuencias
Con absoluto desacuerdo ante todo lo que Calisto expo- terribles que tienen los errores que han cometido Calisto y
ne. También experimenta desprecio, aunque esto lo oculta. Melibea.
E incluso, en algunos momentos, podemos ver que se burla Personajes
de su señor.
22. Melibea aparece al principio tratada como una dama del
15. Finalmente, Sempronio ofrece a Calisto su ayuda. ¿Lo amor cortés. Justifica esta afirmación de acuerdo con la
hace desinteresadamente? Justifica tu respuesta. intervención de ella que has leído.
No. Piensa en el beneficio que podría obtener si Calisto con- Se indigna al ver que Calisto se atreve a dirigirse a ella, como
sigue sus fines. si cuestionase su virtud y el hecho de que es inalcanzable
16. Explica cómo se refleja en Calisto esa vivencia del amor para él.
como enfermedad. 23. Calisto, conforme a lo elevado de su condición social, se
Está desasosegado, triste y de mal humor. Habla también de expresa en términos caballerescos. Sin embargo, en su
un fuego interior que le consume, como una fiebre intensa. fuero interno es egoísta y está tan obsesionado con satis-
17. Al comienzo de la obra, cuando Calisto expresa su admi- facer su deseo que impone sus aspiraciones a cualquier
ración a Melibea, alude a la mezcla de sentimientos espi- aspecto moral o ético. Argumenta esta caracterización de
rituales y carnales que experimenta. Localiza esa alusión. Calisto atendiendo a lo que has leído y a lo que conoces
Ellos [los santos] se glorifican sin temor de caer de tal bien- sobre el resto de la trama.
aventuranza, y yo, mixto, me alegro con recelo del esquivo Su condición de caballero no le impide mostrar su deseo a
tormento que tu ausencia me ha de causar. Melibea al comienzo de la obra, provocando las iras de esta.

Lengua castellana y Literatura. 3.º ESO. Solucionario 123


GUÍAS DE LECTURA

Tampoco duda en pedir ayuda a una mujer como Celestina, Sempronio, no al lisonjero: en esta intervención se presentan
hechicera y alcahueta. En su contexto social, un caballero no ideas misóginas bastante extendidas en la Edad Media.
debería poner en peligro su honra ni la de su amada, pero él
lo hace sin sentir culpabilidad siquiera. En cuanto al honor, Género
en este no piensa tampoco. Además, cuando se entera de la 30. Localiza y comenta las dificultades para poner en escena
muerte de Pármeno y Sempronio el dolor no le dura dema- el fragmento que has leído.
siado, pues enseguida se centra nuevamente en su obsesión. Cambios en el espacio (balcón de Melibea, diversas dependen-
24. Hay una caracterización paralela de Calisto, que realiza cias de la casa de Calisto). No hay acotaciones.
Sempronio en la conversación que ambos mantienen. Nos 31. ¿Qué rasgos de la comedia humanística observas en ese
referimos a la intervención del criado que comienza así: fragmento?
«Lo primero que eres hombre y de claro ingenio». ¿Crees Aparecen registros distintos: Calisto emplea un estilo galante
que Sempronio está siendo sincero con Calisto en este y caballeresco para dirigirse a Melibea, pero cambia el tono
momento? ¿Qué intención persigue al exponer todas las con Sempronio; este, por su parte, tiene ciertos usos popula-
virtudes que le alaba? res. Estilo proverbial: puede apreciarse en el tono sentencioso
Pretende adularle. Sempronio es uno de esos lisonjeros cria- de Sempronio, y en el recurso a los refranes y dichos.
dos contra los que previene el autor al comienzo de la obra. ¿Qué, hijo? ¡Una docena de agujetas1 y un torce2 para el bonete
25. Sempronio es un personaje sin escrúpulos y lleno de codi- y un arco para andarte de casa en casa tirando a pájaros y
cia. ¿En qué momentos del fragmento que has leído crees aojando3 pájaras a las ventanas! Mochachas digo, bobo, de las
que se perciben estos rasgos? que no saben volar, que bien me entiendes. Que no hay mejor
alcahuete para ellas que un arco, que se puede entrar cada
En los apartes de Sempronio podemos ver el desprecio que
uno hecho mostrenco4, como dicen: en achaque de trama5, etc.
siente hacia su amo, y el modo en que lo disimula con sus
¡Más, ay, Sempronio, de quien tiene que mantener honra y se
hipócritas lisonjas. También en esos partes el criado alude
va haciendo vieja como yo!
a los beneficios que espera obtener si apoya a su amo en
sus pretensiones.
26. En la conversación entre Calisto y Sempronio, este pro-
1
agujetas: correas. 2 torce: collar. 3 aojando: echando el
ojo. 4 hecho mostrenco: sin pedir permiso. 5 en achaque de tra-
pone a su señor que pida ayuda a Celestina y le habla de
ma: es la primera parte del dicho «En achaque de trama, ¿viene acá
ella. ¿Cómo describe a la alcahueta?
nuestra ama?». Se empleaba para aludir a las viejas que entraban
Astuta y hechicera, sagaz, componedora de virgos y capaz de sin permiso en las casas con intención de robar y, si se encontraban
provocar la lujuria en quien se proponga. con alguien, decían que buscaban al ama.
27. Localiza pasajes o citas en los que pueda comentarse la
mencionada dualidad entre lo que los personajes dicen 32. Localiza y explica los elementos propios del lenguaje
ser y lo que son en realidad. coloquial y las expresiones populares que aparecen en
En Calisto, la dualidad se manifiesta si comparamos su caba- esta intervención de Celestina.
lleresco modo de expresarse con Melibea con las pretensio-
nes que manifiesta. En cuanto a Sempronio, pueden citarse Son usos propios de la lengua oral y coloquial las exclamacio-
la mayoría de sus apartes, pues en ellos siempre se contienen nes (¡Una docena de agujetas…!) y los vocativos (¿Qué, hijo?;
ideas y propósitos opuestos a los que expone ante Calisto. Mochachas digo, bobo; ¡Más, ay, Sempronio…!), así como el
léxico (por ejemplo pájaras con sentido de mozas). Hay expre-
Estructura siones populares como hecho mostrenco (sin pedir permiso)
28. Divide el pasaje que has leído en partes de acuerdo con o en achaque de trama, que alude a un dicho: En achaque de
su contenido. trama, ¿viene acá nuestra ama?
Primera parte: intervención de Calisto ante Melibea y rechazo Amiga y señora mía, no te maravilles, porque estos fines con
de esta. efecto me dan osadía a sufrir los ásperos y escrupulosos des-
Segunda parte: llegada de Calisto a su casa, malhumorado, y víos de las encerradas doncellas como tú. Verdad es que ante
monólogo de Sempronio al ver el estado en que viene su amo. que me determinase, así por el camino, como en tu casa, estuve
en grandes dudas si te descubriría mi petición. Visto el gran
Tercera parte: conversación con Sempronio, en la que a su
poder de tu padre, temía; mirando la gentileza de Calisto, osa-
vez tenemos que diferenciar apartados; en primer lugar, la
ba; vista tu discreción, me recelaba; mirando tu virtud y huma-
exposición de Calisto sobre su estado y sus sentimientos; en
nidad, me esforzaba. En lo uno hallaba el miedo y en lo otro la
segundo lugar, las advertencias de Sempronio sobre las muje-
seguridad. Y pues así, señora, has querido descubrir la gran
res; en tercer lugar, la descripción que hace Calisto de Melibea.
merced que nos has hecho, declara tu voluntad […]. Yo daré
Por último, la propuesta de Sempronio de acudir a Celestina. forma como tu deseo y el de Calisto sean en breve cumplidos.
29. Teniendo presente que el texto es dialogado, diferencia 33. Comenta el uso de los adjetivos en esa intervención de
los pasajes puramente dialógicos de aquellos que pue- Celestina. ¿Son explicativos o especificativos? ¿Qué valor
dan considerarse monólogos. ¿Qué función tienen estos expresivo aportan al texto?
últimos?
Son adjetivos explicativos, su finalidad es enfática, retórica.
La estructura dialógica se rompe con los monólogos siguien- Aportan al discurso la emotividad con la que Celestina pretende
tes de Sempronio: el que pronuncia cuando ve llegar a Calis- embaucar a Melibea. También contribuyen a elevar el registro.
to de mal humor (No creo, según pienso, ir conmigo el que
contigo queda…) y la exposición que hace sobre las muje- 34. Compara esta intervención con la anterior, en la que se
res, atacándolas (¿Escociote? Lee los historiales, estudia los dirigía a Sempronio.
filósofos…). En el primer caso, está a solas con sus propios Celestina prescinde aquí de las exclamaciones, su tono es
pensamientos y el fragmento nos permite conocer sus temo- mucho más mesurado. Cuando se dirigía a Sempronio, logra-
res y conclusiones. En el segundo, es Calisto el destinatario ba la expresividad mediante los coloquialismos, los juegos
de sus afirmaciones, pero estamos escuchando al verdadero de palabras (aojando pájaras) y la gracia de los dichos

124 Lengua castellana y Literatura. 3.º ESO. Solucionario


GUÍAS DE LECTURA

populares. Al dirigirse a Melibea la expresividad la persigue A comienzos del siglo XVI nos encontramos en un momento de
mediante la adjetivación con epítetos, o ciertos recursos retó- esplendor: es la España imperial. Sin embargo, el Lazarillo
ricos como el paralelismo sintáctico (visto el gran poder de tu muestra los contrastes que existen en una sociedad marcada
padre, temía; mirando la gentileza de Calisto, osaba; vista por el orgullo imperialista pero, a la vez, por las desigualdades
tu discreción, me recelaba…). Pero, en todo caso, obsérvese sociales, el hambre y la miseria en muchos casos. El idealismo
cómo en los dos fragmentos intenta poner al interlocutor de su de la literatura renacentista cede paso a una visión más crítica
parte, con cierta actitud de victimismo. Por ejemplo, al criado sobre esos contrastes.
le dice: ¡Mas, ay, Sempronio de quien tiene que mantener la 3. ¿Cómo es la sociedad que se presenta en el relato, de
honra y se va haciendo vieja como yo! A Melibea: Estos fines acuerdo con el Tratado I?
con efecto me dan osadía a sufrir los ásperos y escrupulosos
Una sociedad en la que existen el hambre y la miseria, pese al
desvíos de las encerradas doncellas.
triunfalismo existente en tiempos de Carlos I.
El texto como acto comunicativo 4. Explica el sentido que tienen estas palabras de Lázaro
CELESTINA.— ¡Oh mi señor Calisto! ¿Y aquí estás? […] ¿Con qué en el prólogo:
pagarás a la vieja, que hoy ha puesto su vida al tablero por
Y también porque consideren los que heredaron nobles esta-
tu servicio? ¿Cuál mujer se vido en tan estrecha afrenta
dos cuán poco se les debe, pues Fortuna fue con ellos parcial,
como yo, que en tornallo a pensar se me menguan y vacían
y cuánto más hicieron los que, siéndoles contraria, con fuer-
todas las venas de mi cuerpo, de sangre? Mi vida diera por
za y maña remando salieron a buen puerto.
menor precio que agora daría este manto raído y viejo.
Lázaro diferencia entre aquellos que heredaron sus bienes y
PÁRMENO.— (Tú dirás lo tuyo: «entre col y col, lechuga». Subido
los que han tenido que salir adelante por sus propios medios.
has un escalón; más adelante te espero a la saya. Todo
para ti y no nada de que puedas dar parte. Pelechar quiere 5. Busca información sobre la presencia de los ciegos en el
la vieja. Tú me sacarás a mí verdadero, y a mi amo loco. folclore español y redacta un escrito de veinte líneas con
No le pierdas palabra, Sempronio, y verás como no quiere los datos que encuentres.
pedir dinero, porque es divisible. En la literatura oral de la Edad Media era habitual la pareja
SEMPRONIO.— Calla, hombre desesperado, que te matará Calisto de ciego y mozo, entre los que se establecía una rivalidad de
si te oye). ingenio y artimañas para afrontar el hambre. Exceptuando el
35. Lee atentamente el fragmento anterior (perteneciente al episodio de las uvas, todos los que se narran entre Lázaro y
el ciego tienen antecedentes en el folclore.
acto VI) y contesta: ¿quién es el interlocutor de Celesti-
na?; ¿y los de Pármeno y Sempronio? Intención
El interlocutor de Celestina es Calisto. Pármeno y Sempronio 6. Investiga sobre la obra Cuarto libro del esforzado caballe-
hablan entre ellos, sin ser oídos por los otros dos. ro Reinaldos de Montalbán. ¿Qué elementos hay en ella
36. ¿Cuál es la intención de la alcahueta? ¿Cómo la esconde? que podríamos asociar al Lazarillo?
¿Quién la conoce? Se trata de una obra de 1542 con ciertas influencias de Apu-
Celestina pretende que Calisto se sienta deudor con ella, leyo. Es un relato autobiográfico y hay un personaje (Cíngar)
haciéndole creer que ha puesto su vida en peligro por ayu- que proviene de la clase baja y es criado de un ciego.
darle. Pármeno se da cuenta de que la vieja quiere mejorar de 7. Busca también información sobre El asno de oro de Apu-
estado y obtener un beneficio que no sea posible compartir, leyo. ¿Encuentras alguna semejanza entre este relato y el
para quedárselo ella sola. de Lázaro de Tormes?
Temas para la reflexión Es un relato de aventuras del siglo II a. de C. que también pre-
senta la sucesión de diversos amos y tiene tono satírico.
37. ¿Crees que la condición de converso de Rojas influyó en
su obra? ¿Cómo? 8. Explica el sentido que tienen las palabras del ciego en
Respuesta libre.
este pasaje del Tratado I:
Comenzamos nuestro camino, y en muy pocos días me mos-
38. Teniendo en cuenta el contexto del siglo XV, reflexiona tró jerigonza1; y, como me viese de buen ingenio, holgábase
sobre el valor que tiene esa declaración de Areúsa y mucho y decía:
compáralo con las posibilidades de independencia que —Yo oro ni plata no te lo puedo dar; mas avisos para vivir
tiene una mujer en el siglo XXI. Redacta un escrito con muchos te mostraré.
tus conclusiones. Y fue ansí, que, después de Dios, este me dio la vida y, siendo
Respuesta libre. ciego, me alumbró y adestró en la carrera de vivir.

4 Lazarillo de Tormes. Tratado I


1
jerigonza: jerga que utilizaban los ciegos para entenderse entre
ellos.
Localización
1. Investiga y explica cuáles son las referencias históricas El ciego es pobre, pero tiene larga experiencia. Aprecia ense-
que aparecen en esta obra. guida la inteligencia de Lázaro y se propone sacarle de la ino-
En el Tratado I se menciona la batalla de los Gelves. Hubo dos: cencia y enseñarle lecciones realistas para sobrevivir.
puede referirse tanto a la de 1510 como a la de 1520. En el Tra- 9. Explica cómo aparece el problema del hambre en el Tra-
tado VII se habla de las Cortes que Carlos I celebra en Toledo. tado I.
También en este caso hubo dos (1525 y 1538-39) y el narrador
El hambre es un tema muy importante en la novela, pero es
puede referirse a cualquiera de ellas.
con el ciego con quien Lázaro conoce esta experiencia. Pasa
2. Recuerda el contexto social y cultural que comienza a hambre porque el ciego es mezquino y apenas le da migajas
darse a mediados de siglo y relaciónalo con el novedoso al niño. Pero esto pone en marcha su inventiva y su ingenio
realismo del Lazarillo de Tormes. para afrontar la necesidad.

Lengua castellana y Literatura. 3.º ESO. Solucionario 125


GUÍAS DE LECTURA

10. Busca información sobre el resto de los capítulos de la 18. Explica cómo entra Lázaro al servicio del ciego.
novela e indica en qué otros momentos de la trayectoria El ciego lo conoce en el mesón en el que trabaja la madre de
del personaje se presenta el motivo temático del hambre. Lázaro y le parece que puede ser buen criado. La madre se lo
En los Tratados II y III el hambre va en aumento. Después, encomienda. Cuando el ciego decide abandonar Salamanca,
esta situación va mejorando pero Lázaro no llega a olvidarla el niño se marcha con él.
en toda su vida: al final de la novela, ya convertido en adul- 19. Localiza el momento en que se despiden Lázaro y su
to, le vemos triunfar sobre el hambre al precio de aceptar su madre. ¿Qué palabras le dirige ella?
deshonra.
Hijo, ya sé que no te veré más Procura de ser bueno, y Dios te
11. Investiga sobre el contenido del Tratado III y explica cómo guíe. Criado te he y con buen amo te he puesto; válete por ti.
aparece ahí el tratamiento del conflicto entre apariencia
20. Al marcharse con el ciego, saliendo de Salamanca, Lázaro
y realidad.
aprende pronto su primera lección. Resume ese pasaje y
El escudero está absolutamente decidido a ocultar su verdade- explica cuál es la enseñanza que aprende el niño.
ra situación. Tanto es así que incluso Lázaro se deja engañar
por su aspecto al principio. La presunción del escudero es En el puente romano a la salida de Salamanca, el ciego le dice
el único defecto que el protagonista deplora de este amo: su que acerque el oído al toro de piedra y escuchará un ruido. El
empeño en aparentar lo que no es, por un orgullo fantasioso. niño lo cree y, al aproximarse, el ciego le coge la cabeza y se
la golpea fuertemente contra el toro. El niño aprende que debe
12. La honra era un concepto que en el Siglo de Oro estaba desconfiar de todos, pues está solo.
relacionado con la consideración social que se tenía sobre
una persona. Este tema se trata con frecuencia a lo largo 21. ¿Cómo es la vida que lleva Lázaro con el ciego? ¿Qué
de la obra. Al final, en el Tratado VII, Lázaro se casa con la recursos emplea para aliviar su necesidad?
criada de un arcipreste. Busca información sobre el argu- Pasa mucha hambre, pues el ciego es avaro y no comparte
mento de este capítulo y explica cómo se enfoca ahí el nada de lo que gana con él. Lázaro despliega todo su ingenio
para engañarle y poder así alimentarse.
tema de la honra y el sentido que tienen estas palabras
del arcipreste: 22. El episodio del jarro de vino supone un cambio impor-
—Lázaro de Tormes, quien ha de mirar a dichos de malas tante en la relación de Lázaro con el ciego. ¿Qué senti-
lenguas nunca medrará1. Digo esto porque no me maravillaría mientos empieza a albergar el niño hacia su amo? ¿Qué
alguno, viendo entrar en mi casa a tu mujer y salir della. Ella opinión se crea sobre él?
entra muy a tu honra y suya; y esto te lo prometo. Por tanto, Empieza a tenerle animadversión. Se da cuenta de su crueldad
no mires a lo que pueden decir, sino a lo que te toca: digo a y de que disfrutó castigándole.
tu provecho.
23. Resume el episodio del racimo de uvas.
Lázaro vive en una situación deshonrosa porque acepta que
El ciego le propone compartir un racimo de uvas que les han
su mujer tenga relaciones con el arcipreste, algo que es un
dado: irán comiendo las uvas de una en una por turnos. Pero
secreto a voces para todo su entorno. Acepta esta situación
el ciego empieza a tomarlas de dos en dos y entonces Lázaro
porque le resulta provechosa. Como le dice el arcipreste, debe
decide comerlas de tres en tres. El ciego descubre que el niño
olvidarse de los rumores, y pensar solamente en su provecho,
le ha engañado porque, cuando él empezó a coger dos de gol-
es decir, en su prosperidad. El arcipreste dice, acudiendo al
pe, Lázaro no protestó.
doble sentido de sus palabras, que el bienestar importa mucho
más que la honra. 24. ¿Qué acontecimientos tienen lugar en Escalona?
El episodio de la longaniza.
Contenido
13. Al comienzo del relato, Lázaro explica las circunstancias 25. El ciego acostumbra a hacer mofas en público de las des-
de su nacimiento. ¿Por qué tiene el sobrenombre «de venturas de Lázaro. Cita algunos ejemplos.
Tormes»? El ciego cuenta públicamente en el mesón las desventuras de
Lázaro: la del jarro, la del racimo y la actual, la de la longani-
Porque nació en la ribera de ese río. Su madre se puso de parto
za. Habla de cuánto le gusta el vino.
estando en el molino que proveían a orillas del Tormes.
14. ¿Quiénes eran sus padres? ¿A qué se dedicaba su padre? 26. Lázaro toma la decisión de dejar al ciego. ¿Cuándo y
cómo lleva a cabo su propósito? ¿Cuál es el desenlace del
Tomé González y Antona Pérez, naturales de Tejares. Su padre
era molinero.
capítulo?
Salen a mendigar por Escalona, pero está lloviendo y el ciego
15. Tras su viudez, la madre de Lázaro se empareja de nuevo.
decide volver a la posada. Han de pasar por un arroyo. Lázaro
¿Con quién? le engaña sobre el lugar por el que debe pasar, supuestamente
Con un hombre morisco. donde el arroyo se estrecha. Le dice que dé un buen salto para
16. La humorística anécdota del hermanastro de Lázaro que no mojarse los pies, pero lo que provoca es que se estrelle
tiene miedo de su propio padre por el color de la piel contra un poste y caiga al suelo. Aprovecha el momento en
formaba parte de la tradición oral de la época. ¿En qué que acude gente a socorrer a su amo para escaparse a Torrijos.
se basa la comicidad de dicha anécdota? ¿Qué reflexión Personajes
suscita en Lázaro?
27. En este aprendizaje, ¿son rápidos los progresos de Lázaro
El niño tiene miedo de su padre negro, pero él también es en el Tratado I? Justifica tu respuesta.
negro. Lázaro reflexiona sobre las gentes que no saben mirar-
se a sí mismas. Sí. El primer incidente (el golpetazo contra el toro en el puen-
te de Salamanca) hace que Lázaro comprenda que debe ser
17. ¿Qué castigo sufre la madre de Lázaro por tener relacio- desconfiado y le saca de su inocencia de niño. Al final del
nes con un hombre de otra religión? tratado incluso vemos cierta crueldad en el modo en que el
Recibe cien azotes. protagonista abandona al ciego.

126 Lengua castellana y Literatura. 3.º ESO. Solucionario


GUÍAS DE LECTURA

28. ¿Qué idea sobre Lázaro y el ciego podemos hacernos a 33. ¿A quién se dirige Lázaro en la carta? ¿Qué sabemos de
partir del siguiente pasaje? ¿Y sobre la relación que hay este destinatario?
entre ellos? No sabemos nada de él, excepto que tiene un rango superior
Y, aunque yo quisiera asentar mi corazón y perdonalle el jarra- al de Lázaro, lo cual no implica que se trate de una persona
zo, no daba lugar el maltratamiento que el mal ciego dende especialmente importante. El Vuestra Merced con que Lázaro
allí en adelante me hacía, que sin causa ni razón me hería, se dirige a él solo implica una actitud respetuosa.
dándome coxcorrones1 y repelándome2. Y, si alguno le decía 34. Divide en partes el Tratado I de acuerdo con el conte-
por qué me trataba mal, luego contaba el cuento del jarro, nido del capítulo. ¿Podemos hablar en dicho tratado de
diciendo: una estructura organizada en planteamiento, nudo y
—¿Pensaréis que este mi mozo es algún inocente? Pues oíd si desenlace?
el demonio ensayara otra tal hazaña3.
Sí. El planteamiento abarca el relato de la infancia de Lázaro
cuando aún vivía con su madre hasta que conoce al ciego.
1
coxcorrones: coscorrones. 2 repelándome: arrancándo- El nudo ocupa los diversos hechos que se van encadenando
me el pelo. 3 ensayara otra tal hazaña: maquinara una hazaña mientras permanece con su amo (episodio del toro a la salida
semejante. de Salamanca, racimo de uvas, jarro de vino, longaniza, etc.).
El desenlace se da al narrar el día lluvioso en que Lázaro
Lázaro tiene buen fondo y quisiera perdonar al ciego por su abandonó al ciego y el modo en que lo hizo.
crueldad, pero es imposible pues este es un hombre violento
y cruel. La relación entre ellos se hace inaguantable para el Género
protagonista: el ciego desconfía del niño, lo humilla y lo gol- 35. ¿Qué subgéneros narrativos se cultivaron en el Renaci-
pea delante de todo el mundo, y el niño llega a tomarle mucha miento? ¿En qué época se originaron?
aversión.
Las novelas sentimentales y las de caballerías, que se habían
29. El ciego es un hombre desconfiado, y ni siquiera se fía de originado en el siglo XV, siguen teniendo éxito en el XVI. En el
su propio mozo. Cita partes del Tratado I que justifiquen Renacimiento surgen la novela bizantina, la pastoril, la moris-
esta afirmación. ca y la picaresca.
En el episodio del jarro de vino: Mas como fuese el traidor 36. Recuerda los rasgos y la temática de los subgéneros
tan astuto, pienso que me sintió, y dende en adelante mudó narrativos del Renacimiento y explica las diferencias que
propósito asentaba su jarro entre las piernas y atapábale con el Lazarillo tiene con ellos.
la mano, y ansí bebía seguro; Tantas vueltas y tientos dio al Las diferencias del Lazarillo —y de la novela picaresca— con
jarro que halló la fuente. respecto a los demás subgéneros narrativos son el realismo y,
En el episodio del racimo de uvas: Acabado el racimo, estuvo sobre todo, el hecho de que el protagonista sea un antihéroe.
un poco con el escobajo en la mano y, meneando la cabeza,
37. Localiza en el Tratado I los elementos propios de la corrien-
dijo: —Lázaro, engañado me has. Juraré yo a Dios que has
tú comido las uvas tres a tres.
te picaresca y explícalos.
La forma autobiográfica y epistolar; el realismo de la narra-
En el episodio de la longaniza: Yo torné a jurar y perjurar
ción, que además está ambientada en un espacio contempo-
que estaba libre de aquel trueco y cambio; mas poco me
ráneo (véanse las alusiones históricas, además del retrato de
aprovechó, pues a las astucias del maldito ciego nada se le
la sociedad española del siglo XVI); la procedencia humilde del
escondía.
protagonista; el planteamiento de su evolución psicológica y
30. Además de su desconfianza, ¿cómo se describe al ciego? su aprendizaje vital, que viene apuntado ya desde la primera
Resume su caracterización. de sus aventuras.
Mezquino y avariento. Cruel. Con muchas habilidades para 38. Explica también los elementos del género narrativo, a
sacarle dinero a la gente y supuestos conocimientos curati- partir de tu lectura de dicho tratado.
vos. Sagaz.
La presencia del narrador personaje (protagonista), la locali-
31. Ateniéndote al argumento de la obra, explica cuáles son zación espacial y temporal (hay claras referencias en ambos
las conclusiones a las que llega Lázaro al final, y en las que aspectos), la inserción de la descripción y el diálogo en el
se basa para vivir feliz y tranquilo. discurso narrativo, la presentación de unos personajes y su
La tranquilidad, para él, significa no pasar hambre y haber caracterización.
alcanzado una posición social con su matrimonio, aunque esta 39. Recuerda lo que has estudiado sobre el desarrollo de la
posición sea humilde. Está dispuesto a mantener ese estatus corriente picaresca después de la publicación del Lazarillo.
aunque deba pagar el precio de su honra. ¿En qué época se produjo este desarrollo? Busca informa-
32. Recuerda lo que has estudiado sobre el argumento com- ción y cita algunas obras y autores de esta corriente.
pleto de esta obra e indica a qué niveles sociales perte- La novela picaresca se desarrollaría durante el siglo XVII. Otras
necen sus personajes. obras picarescas de interés: Guzmán de Alfarache, de Mateo
A las clases bajas: los orígenes de Lázaro son muy humildes, Alemán; La vida del Buscón llamado Pablos, de Quevedo;
su primer amo es un mendigo, tendrá también cinco amos La pícara Justina, de Francisco López de Úbeda; Vida del
que pertenecen al nivel inferior del clero. Al final de su vida escudero Marcos de Obregón, de Vicente Espinel.
alcanza un mínimo grado de prosperidad al entrar en el oficio Estilo
de pregonero, pero es un oficio mal considerado.
40. En el Tratado I, busca citas en las que pueda observarse
Estructura el tono del narrador y la naturalidad del estilo.
Y, pues Vuestra Merced escribe se le escriba y relate el caso Comenzamos nuestro camino, y en muy pocos días me mostró
muy por extenso, paresciome no tomalle por el medio, sino jerigonza; y como me viese de buen ingenio, holgábase mucho
del principio, porque se tenga entera noticia de mi persona. y decía: —Yo oro ni plata no te lo puedo dar; mas avisos para

Lengua castellana y Literatura. 3.º ESO. Solucionario 127


GUÍAS DE LECTURA

vivir muchos te mostraré. Y fue ansí, que, después de Dios, una discapacidad? ¿Tienen menos oportunidades que el
este me dio la vida y, siendo ciego, me alumbró y adestró en la resto de la población? ¿Existen medios para paliar sus
carrera de vivir. Huelgo de contar a Vuestra Merced estas niñe- limitaciones en la vida social y laboral?
rías, para mostrar cuánta virtud sea saber los hombres subir
En la sociedad actual, los discapacitados no son marginados,
siendo bajos, y dejarse bajar siendo altos cuánto vicio; Mas,
como ocurría hace siglos. Además, la sociedad actual dispone
por no ser prolijo, dejo de contar muchas cosas, así graciosas
de recursos y legislación para garantizar sus derechos y com-
como de notas, que con este mi primer amo me acaescieron, y
pensar sus limitaciones.
quiero decir el despidiente y, con él, acabar.
41. ¿Qué palabras y usos lingüísticos propios del ambiente 48. ¿Sería hoy posible que un niño de ocho años tuviese unas
marginal del protagonista aparecen en ese capítulo? andanzas como las que Lázaro vive con el ciego? Redacta
un escrito en el que expliques las diferencias entre la for-
Hideputa; al triste de mi padrastro azotaron y pringaron (tor-
ma de vida de un niño de clase social baja en el siglo XVI
mento que consistía en echar tocino derretido al fuego sobre
las heridas de los azotes previos); centenario (castigo de cien
y en nuestro tiempo.
azotes); jerigonza (jerga de los ciegos); besos (para referirse a Respuesta libre.
tragos de vino); tío (es el tratamiento que Lázaro da al ciego, y 49. El aprendizaje a partir de las experiencias vitales supone a
responde a la fórmula de respeto que solía dirigirse a las per- veces un paso de la inocencia a la desconfianza, que mar-
sonas mayores en el ambiente popular), etcétera. ca un cambio entre la vida infantil y la adulta. ¿Compartes
42. Localiza también ejemplos de dichos populares, refranes esta afirmación? Expón tus opiniones sobre este asunto en
y lenguaje proverbial. un texto de unas veinte líneas.
Refranes y dichos populares: Echar la soga tras el caldero Respuesta libre.
(echarlo todo a perder); Más da el duro que el desnudo; No
paresciendo ellas, pudiera negar la demanda (al no aparecer
el cuerpo del delito —las narices— podría evitar las respon- 5 Miguel de Cervantes. Don Quijote
sabilidades, la «demanda criminal»), etc. El tono sentencioso, de la Mancha. Capítulos I-VI
de estilo proverbial, puede también comentarse: El mozo del
ciego un punto ha de saber más que el diablo; Cuánta virtud Localización
sea saber los hombres subir siendo bajos, y dejarse bajar sien- 1. Resume el argumento de los seis primeros capítulos.
do altos cuánto vicio; Lo que te enfermó te sana y da salud, Alonso Quijano, un viejo hidalgo que lleva una vida tranquila
etcétera. en una aldea manchega, pasa el día leyendo obsesivamente
43. ¿Has observado lo frecuentes que son las interjecciones libros de caballerías, lo cual le lleva a la locura. Decide enton-
y exclamaciones en los diálogos? Cita algunos ejemplos. ces convertirse en caballero andante y adopta el nombre de
¿Qué finalidad crees que se persigue con la abundancia Don Quijote de la Mancha. Se prepara unas armas y un caba-
de exclamaciones? llo, sale a buscar aventuras, se aloja en una venta que toma
por castillo y allí toma la orden de caballería. Corre entonces
¡Lacerado de mí!, ¡Oh, gran Dios!, ¡Sús! El objetivo es el rea-
diversas aventuras: libera a un criado que está siendo azota-
lismo, la expresividad y la naturalidad.
do por su amo, se encuentra con unos mercaderes a los que
El texto como acto comunicativo se enfrenta, acaba maltrecho y lo encuentra un vecino de su
44. Busca y anota citas en las que se aluda a este receptor. pueblo, quien lo devuelve a su hogar. Allí, el ama y la sobrina
le cuidan y encomiendan al cura y al barbero que quemen los
Pues sepa Vuestra Merced, ante todas las cosas; Mas también libros que han provocado la locura del hidalgo. Estos hacen un
quiero que sepa Vuestra Merced; Y porque vea Vuestra Merced escrutinio en el que deciden qué libros merecen ser salvados y
a cuánto se extendía el ingenio deste astuto ciego; Conside- cuáles quemados y encienden una hoguera en el patio.
rando lo que aquel día me dijo salirme tan verdadero como
adelante Vuestra Merced oirá, etc. 2. Repasa lo que has estudiado sobre esta obra y contesta:
¿qué aventuras son las más destacadas en la segunda
45. El emisor ficticio (Lázaro) se dirige a un lector implícito: es
salida?
decir, presupone que este conocerá el asunto del que va a
La de los molinos de viento, la de los galeotes, la del yelmo de
hablarle, que tiene ciertos datos sobre él y sobre su vida.
Mambrino (bacía de barbero que don Quijote toma por yel-
¿Podrías citar algún pasaje en el que se observe esto? mo legendario), el encuentro con otros personajes en la venta
En el prólogo Lázaro escribe: Y pues Vuestra Merced escribe (Fernando y Dorotea, Luscinda y Cardenio), etcétera.
se le escriba y relate el caso muy por extenso… No se aclara a
qué caso se refiere, pero es evidente que esta omisión corres-
3. Recuerda lo que has aprendido sobre la segunda parte
ponde a un contexto compartido por Lázaro y el narratario. del Quijote e indica qué nuevos personajes aparecen en
el Quijote de 1615. ¿Cuáles son en esta segunda parte las
Temas para la reflexión aventuras más destacadas?
46. Explica las dificultades para la comprensión del mensaje Es muy importante la figura de Sansón Carrasco, el bachiller
que puede conllevar el hecho de que el lector pertenezca que decide hacerse pasar por caballero andante para desafiar a
a una época distinta de la del autor. don Quijote y, venciéndole, ponerle la condición de que regrese
Cambios en el idioma, tanto en su morfología como en el léxico. a casa (Sansón Carrasco tomará la identidad del Caballero de
Contexto social y cultural muy diferente; alusiones relaciona- Los Espejos y de La Blanca Luna), los duques, Álvaro Tarfe
das con costumbres e ideología que necesitarán aclaraciones (personaje de la novela de Avellaneda, que Cervantes utilizará
para el lector actual. La visión del mundo, la forma de vida, para desautorizar a los falsos don Quijote y Sancho), etcétera.
las ideas, las conductas, etc., del lector del siglo XXI son muy 4. ¿Qué obras pertenecientes a dicha corriente conoces?
diferentes de las del siglo XVI. ¿Quiénes son sus autores?
47. ¿Cómo ha cambiado la sociedad en el siglo XXI? ¿Qué Entre otras, Amadís de Gaula, editada en 1508 (aunque es
tratamiento reciben los ciegos y las personas que tienen una refundición firmada por Garci Rodríguez de Montalvo de

128 Lengua castellana y Literatura. 3.º ESO. Solucionario


GUÍAS DE LECTURA

ediciones anteriores), Palmerín de Oliva, publicada en 1511, los más del año) se daba a leer libros de caballerías con tanta
seguidas ambas por sus continuaciones: las Sergas de Esplan- afición y gusto, que olvidó casi de todo punto el ejercicio de
dián, el Palmerín de Inglaterra, etc. la caza, y aun la administración de su hacienda; Con estas
5. ¿Cuáles eran los elementos argumentales y temáticos razones perdía el pobre caballero el juicio, y desvelábase por
propios de los libros de caballerías? entenderlas y desentrañarles el sentido.
Las aventuras de un caballero andante, enamorado de una 12. ¿Cuáles eran sus costumbres? ¿Cuáles son sus armas?
dama idealizada a la que dedica todas sus hazañas. El prota- ¿Cómo escoge a su dama?
gonista se rige de acuerdo con el código de la caballería, que Tenía las costumbres propias de un hidalgo ya anciano, aco-
implica fidelidad y devoción a su amada, honestidad, honor y modado y de vida monótona, entregado a la lectura hasta que
aceptación de las condiciones que le imponga otro caballero perdió el juicio. Sus armas habían pertenecido a sus bisabue-
que le derrote en justa lid. En los relatos caballerescos hay los, estaban oxidadas y enmohecidas y olvidadas en un rin-
siempre elementos y seres fantásticos, ambientes guerreros cón. Las limpió y aderezó la celada con cartones y barras de
y, a la vez, corteses, donde se celebran justas y combates por- hierro. A su dama la escoge recordando a una labradora del
tentosos. Las narraciones sobre caballeros andantes siempre Toboso de la que estuvo enamorado antaño, y que se llamaba
son presentadas como si fuesen recopilaciones de supuestos Aldonza Lorenzo. Le puso de nombre Dulcinea del Toboso.
cronistas, como si se tratase de hechos históricos. 13. En el capítulo II, don Quijote emprende su aventura y
6. Recuerda lo que sabes sobre el contexto histórico de Cer- se imagina que, algún día, los historiadores contarán sus
vantes. ¿Por qué definimos ese tiempo como un periodo hazañas. Localiza el pasaje concreto en el que fantasea
de crisis? ¿En qué terrenos? con estas ideas.
La época imperial ha entrado en decadencia, es ya un pasado ¿Quién duda sino que en los venideros tiempos, cuando salga
glorioso que suscita el orgullo de muchos, pero no puede evi- a la luz la verdadera historia de mis famosos hechos, que el
tar una realidad de crisis económica y social y de corrupción sabio que los escribiere no ponga, cuando llegue a contar esta
política. mi primera salida tan de mañana, desta manera?: Apenas
7. Las etapas de crisis suelen dar lugar a expresiones artísti- había el rubicundo Apolo; Dichosa edad y siglo dichoso aquel
cas orientadas a la sátira, la caricatura, la ironía y, en gene- adonde saldrán a luz las famosas hazañas mías, dignas de
ral, al humor crítico. ¿Por qué crees que los artistas acuden entallarse en bronces; ¡Oh, tú, sabio encantador, quienquiera
tan a menudo al humor para expresar sus desengaños o que seas, a quien ha de tocar ser el coronista desta peregrina
sus críticas? historia, ruégote que no te olvides de mi buen Rocinante.
La ironía e incluso el sarcasmo, la caricatura y la sátira son 14. Don Quijote llega a una venta. ¿Dónde imagina que se
vehículos que hacen llegar la crítica más lejos y suscitan halla? ¿Qué personajes encuentra allí?
reflexiones en los receptores de estas obras. En un castillo. Dos mozas, un porquero, el ventero, un castra-
dor de puercos.
Intención
15. ¿Cómo se comporta don Quijote con ellos? ¿Y ellos con él?
8. ¿Qué queremos decir cuando hablamos de «tema meta-
literario»? Don Quijote se comporta con ellos de acuerdo con la idea que
se forma: piensa que está en un castillo, que ellos le ven como
La literatura adopta como tema la literatura misma.
un caballero andante. Ellos le tratan con una mezcla de asom-
9. Como novela de caballerías, el Quijote es una parodia. bro y sorna, y le siguen la corriente.
¿Por qué? 16. Explica las diferencias que existen entre la imagen que
Porque presenta todos los elementos de los libros de caballería don Quijote se forja de dichos personajes y la verdadera
deformándolos con la técnica de la caricatura, a través de la condición que estos tienen.
ironía y la sátira.
Ha tomado a las mozas por bellas damas; al porquero que
10. Los héroes de la narrativa anterior a Cervantes eran perso- toca un cuerno, por enano que le recibe haciendo señal de su
najes heroicos e idealizados. Localiza en los seis primeros venida; al ventero, por alcaide de la fortaleza, y al castrador
capítulos tres momentos que nos muestren el contraste de puercos que toca un silbato, por alguien que ameniza su
entre don Quijote y aquellos personajes literarios. cena con música.
La propia caracterización que se nos hace al principio del relato: 17. Indica qué acontecimientos tienen lugar en el capítulo III
el caballero manchego es un hombre ya anciano, desconocido, mientras don Quijote está velando sus armas.
no es un héroe sino un hombre destinado a una vida monótona
El ventero confirma que don Quijote está loco de remate pero
e invisible y de él nos dice Cervantes que se le secó el cerebro.
le sigue la corriente y le dice que oficiará la ceremonia para
En segundo lugar puede citarse todo el pasaje en que recibe la
que sea armado caballero. Explica a los demás personajes la
orden de caballería, y se describen el desconcierto y las mofas
locura del hidalgo y todos le observan mientras este vela sus
de los personajes que contemplan la escena. Otro pasaje sig-
armas en el patio. Durante la noche, unos arrieros quieren ir
nificativo es el que narra el estado lamentable en que queda el
a dar agua a su recua y necesitan retirar las armas de don
protagonista tras el encuentro con los mercaderes.
Quijote, que están apoyadas en una pila. Se originan enfren-
Contenido tamientos sucesivos, el protagonista golpea primero a uno
11. En el capítulo I se nos presenta al personaje. ¿Cómo se y luego a otro con la lanza en la cabeza. Sus compañeros le
apedrean aunque al final le acaban tomando miedo.
le caracteriza? Contesta a la pregunta citando pasajes
concretos. 18. El curso de los acontecimientos crea ciertos temores en
Un hidalgo de los de lanza en astillero, adarga antigua, rocín el ventero. ¿Cuáles? Para evitar lo que teme, ¿qué con-
flaco y galgo corredor; Era de complexión recia, seco de car- ducta adopta?
nes, enjuto de rostro, gran madrugador y amigo de la caza. Teme que se produzca otro altercado semejante y decide acor-
Quieren decir que tenía el sobrenombre de Quijada o Quesada; tar la ceremonia y darle la orden de caballería a don Quijote
Este sobredicho hidalgo, los ratos que estaba ocioso (que eran cuanto antes.

Lengua castellana y Literatura. 3.º ESO. Solucionario 129


GUÍAS DE LECTURA

19. Resume la ceremonia en que don Quijote es armado 28. Explica quiénes llevan a cabo el escrutinio de libros en
caballero. ¿Quiénes intervienen en ella? el capítulo VI. ¿Qué actitud tienen el ama y la sobrina
El ventero trae un libro de asientos, hace arrodillar a don ante lo que le ocurre al protagonista? ¿Qué opiniones
Quijote, finge pronunciar una oración y le da el consabido manifiestan sobre el destino que han de correr los libros?
espaldarazo. Ordena a una de las mozas que le ciña la espada El cura y el barbero. El ama tiene miedo de los encantadores
y ella lo hace, muerta de risa. La ceremonia se desarrolla con que ella cree que hay en los libros. Tanto ella como la sobrina
tremenda rapidez. Don Quijote trata con galanura a las mozas, son partidarias de quemarlos todos porque todos les parecen
les pone sobrenombres (doña Tolosa y doña Molinera), se des- causa del daño.
pide con gratitud del ventero y se marcha.
29. ¿Qué libros se quemaron? ¿Cuáles se salvaron? ¿Por qué?
20. Cuando don Quijote abandona la venta, el ventero no le
Se queman: las Sergas de Esplandián, Amadís de Grecia,
cobra los servicios que le ha ofrecido. ¿Por qué? porque no les agradan las continuaciones del Amadís de
Porque quiere evitar nuevos conflictos y desea que don Qui- Gaula; Don Olivante de Laura, Florismarte de Hircania,
jote se marche lo antes posible. El Caballero Platir, El Caballero de la Cruz, Bernardo del
21. Explica qué sucede en el capítulo IV cuando don Quijote Carpio, Roncesvalles, Palmerín de Oliva, La Diana segunda
se encuentra con el labrador que está azotando a su criado. del Salmantino, El pastor de iberia, Ninfas de Henares, y
Don Quijote ordena al labrador (Juan Haldudo) que libere a su Desengaños de celos.
criado (Andrés) y que le pague su sueldo. El labrador responde Se salvan: Amadís de Gaula, porque es el mejor y primer
que Andrés vaya a su casa, que allí le pagará, y don Quijote orde- libro de caballerías; Espejo de Caballerías, aunque lo que
na al mozo que así lo haga. Este se resiste, pero el caballero le parece bien al cura es el original en italiano (Orlando Furio-
asegura que basta el juramento de Haldudo para saber que cum- so) y no la traducción; Palmerín de Inglaterra, porque es muy
plirá, pues así lo mandan las leyes de la caballería. Cuando don bueno y lo compuso un rey de Portugal; Don Belianís, porque,
Qujiote se marcha, el labrador continúa castigando a su criado. si se eliminan algunas cosas que deberían enmendarse, mere-
ce la pena conservarlo; Tirante el Blanco, de Johanot Marto-
22. Explica el contraste que se observa en este episodio
rell, porque es el mejor libro del mundo; La Diana, de Jorge
entre el mundo que concibe don Quijote —idealizado— y de Montemayor, por su parte en prosa y por ser el primero en
el real, en el que viven los demás personajes. su género, aunque recomiendan eliminar la parte en verso;
En el mundo literario de don Quijote, el mozo azotado está Diana enamorada, de Gil Polo; Los diez libros de Fortuna de
sufriendo un agravio y el labrador es alguien que abusa de un amor, de Antonio de Lofraso, porque al cura le parece lo mejor
ser indefenso. En el mundo real, el criado es un bellaco des- de su género; El pastor de Fílida, de Luis Gálvez de Montalvo,
cuidado que recibe el castigo de su amor. Además, el idealismo porque al cura le parece una joya preciosa; Tesoro de varias
del protagonista le lleva a creer que el labrador actuará como poesías, porque su autor —Pedro de Padilla— es amigo del
él le ha ordenado porque le obliga el juramento de la caballe- cura y aunque ese libro no le parece de los mejores, siente un
ría. Para don Quijote ese juramento es la única garantía que gran respeto hacia otras obras suyas; El Cancionero de López
necesita. El mozo Andrés le advierte que Juan Haldudo no es Maldonado, porque su autor es amigo del cura y es un gran
un caballero, le describe la realidad, pero don Quijote no la ve. poeta; La Galatea, porque Miguel de Cervantes ha prometido
23. En cierto momento, don Quijote llega a una encrucijada. una segunda parte y es amigo del cura; La Araucana, La
¿Cómo decide el camino que toma en ella? Austríada y El Monserrate, de Cristóbal de Virués, porque
los consideran las más ricas muestras de poesía en España; y
Se queda pensativo un buen rato, imitando lo que ha visto
Las lágrimas de Angélica, porque su autor (Luis Barahona de
hacer a los héroes de sus libros. Después, suelta la rienda de
Soto) es uno de los más famosos poetas del mundo.
Rocinante, dejando que este tome el rumbo que quiera.
24. Tras la encrucijada, don Quijote se encuentra con unos 30. En el escrutinio de los libros, aparece el propio Cervan-
mercaderes. ¿Por quiénes los toma el hidalgo? ¿Qué les tes y se menciona una obra suya. ¿Cuál? ¿Se salva de la
reclama? quema? ¿Qué se dice sobre esta obra y sobre su autor?
Por caballeros andantes. Les reclama que confiesen que Dul- La Galatea. Dice el barbero que Cervantes es amigo suyo, que
cinea del Toboso es la doncella más hermosa del mundo. le han ocurrido muchas desdichas, que el libro no es una gran
obra pero que merece la pena esperar a la segunda parte que
25. ¿Cómo reaccionan ellos? ¿Cómo termina este incidente? el autor ha prometido.
Responden que para hacer ese reconocimiento tienen que ver
a la dama, lo cual enoja a don Quijote. Ofendido, arremete Personajes
contra ellos con su lanza, pero se cae del caballo y un mozo 31. Teniendo en cuenta esta idea de la reinvención, explica el
de mulas comienza a apalearle. contenido del siguiente fragmento del capítulo V.
26. Señala con quién se encuentra el viejo caballero en el A esto respondió el labrador:
capítulo V. —Mire vuestra merced, señor, pecador de mí, que yo no soy
Se encuentra con un labrador y vecino de su pueblo, que al don Rodrigo de Narváez, ni el marqués de Mantua, sino Pedro
verle molido en el suelo, le ayuda y le lleva de vuelta a su casa. Alonso, su vecino; ni vuestra merced es Valdovinos, ni Abin-
27. Explica cómo y por qué regresa don Quijote a casa. darráez, sino el honrado hidalgo del señor Quijana.
Regresa ayudado por el vecino, que espera a que se haga de —Yo sé quién soy —respondió don Quijote—, y sé que pue-
noche para que la gente del pueblo no vea al viejo hidalgo en do ser no solo los que he dicho, sino todos los doce Pares de
tan deplorable estado. Al llegar, oyen cómo el ama y la sobrina Francia, y aun todos los nueve de la Fama, pues a todas las
les cuentan al cura y al barbero que don Quijote lleva tres días hazañas que ellos todos juntos y cada uno por sí hicieron, se
fuera de casa y que antes de marcharse no dejaba de decir dis- aventajarán las mías.
parates. El labrador entonces comprende la locura del hidalgo Alonso Quijano decide convertirse en don Quijote, es decir,
y llama para que les abran, hablando y comportándose como decide ser quien él quiere ser. Salta por encima de su vida
un personaje caballeresco para seguirle la corriente al anciano. anónima y anodina para llevar adelante grandes hazañas.

130 Lengua castellana y Literatura. 3.º ESO. Solucionario


GUÍAS DE LECTURA

32. A partir de tu lectura de los seis primeros capítulos, loca- tales, Sancho Panza, que así se llamaba el labrador, dejó su
liza y explica los elementos concretos que contraponen la mujer e hijos y asentó por escudero de su vecino.
caracterización de Alonso Quijano con la de don Quijote 38. Siempre se ha presentado a Sancho Panza como el perso-
de la Mancha. Acude a citas concretas de la obra para naje realista por oposición con el idealismo de don Qui-
responder. jote. Investiga sobre el argumento de la novela y justifica
Alonso Quijano era un hidalgo que se dedicaba a la caza y a esta afirmación.
su hacienda y llevaba una existencia doméstica y rutinaria Sancho siempre expone ante su señor la realidad en que se
(véase la descripción que se hace al comienzo de la novela). mueven, advirtiéndole que él no ve gigantes, sino molinos, ni
Don Quijote no quiere estar en el hogar, sino al aire libre, y ha encontrado a Dulcinea, sino a una tosca labradora. Ade-
vivir aventuras, no en las rutinas (casi todo aquel día caminó más, siempre está pensando en cosas materiales, especial-
sin acontecerle cosa que de contar fuese, de lo cual se deses- mente en la comida.
peraba, porque quisiera topar luego luego con quien hacer 39. Pese a dicho realismo, Sancho Panza es incondicional de su
experiencia del valor de su fuerte brazo). Quiere vivir una
señor, le acompaña fielmente, confía siempre en su palabra
vida digna de ser narrada en una gran crónica.
y su criterio y evoluciona junto a él, contagiándose de su
33. ¿Qué otros personajes aparecen en estos capítulos ini- idealismo. Busca información sobre esta evolución de San-
ciales? ¿Qué relación tienen con don Quijote? ¿Cuál es cho y redacta un escrito con los datos que encuentres.
su función en la trama de dichos capítulos? Pese a que él no ve el mundo como don Quijote, Sancho consi-
Ama, sobrina, cura, barbero, Pedro Alonso (su vecino del dera que es él el equivocado, porque no tiene el elevado inge-
pueblo): representan la vida que don Quijote ha dejado atrás, nio de su señor. Para él don Quijote es un modelo. Así, poco a
la vida de Alonso Quijano. Ventero y demás personajes de la poco va aprendiendo a ver la realidad desde la perspectiva del
venta, labrador Juan Haldudo y mozo Andrés, mercaderes: caballero, va conociendo el código y el contexto de ese mundo
marcan la realidad contra la que choca el mundo imaginario literario. En la segunda parte esta evolución es decisiva. Por
de don Quijote, contribuyen así a la parodia y la comicidad. ello, al final de la novela, cuando don Quijote agoniza, el fiel
34. ¿Qué personajes y libros de la época se citan? Entre ellos, escudero intenta convencerle para que se levante de la cama
¿quién constituye el ideal para don Quijote? y se disponga nuevamente a vivir en libertad la vida de un
caballero andante.
Sobre todo, Amadís de Gaula.
35. ¿Qué información se nos da acerca de los personajes Estructura
secundarios? ¿Se les caracteriza psicológicamente? 40. De acuerdo con el concepto tradicional de narración,
No. De estos personajes solo conocemos el contraste que pro- ¿podrían ser estos primeros seis capítulos una novelita
vocan con respecto a don Quijote. independiente? Justifica tu respuesta.
36. Entre los diversos personajes con quienes don Quijote se Sí. En estos seis capítulos encontramos una presentación de la
topa en estos seis capítulos, encontramos muy diversas situación y los personajes, un motivo temático central (la sali-
conductas: unos se burlan de él, otros le siguen la corrien- da de don Quijote), un desarrollo de una trama y un desenlace
te, otros intentan ayudarle, otros responden agresiva- que devuelve al protagonista al punto de partida.
mente a lo que consideran ataques del caballero. Pon un 41. Normalmente, una narración suele estructurarse en tres
ejemplo de cada caso. partes: planteamiento, nudo y desenlace. Divide la trama
Se burlan de él las mozas de la venta. Le sigue la corriente el de los seis primeros capítulos en estas tres partes.
ventero. Intenta ayudarle Pedro Alonso, su vecino. Responde Planteamiento: primer capítulo, donde se presenta la situación
agresivamente a don Quijote el mozo de mulas que iba con y a los personajes. Nudo: capítulos II-V, que recogen las aven-
los mercaderes. turas del caballero, desde su salida hasta su regreso, y las
consecuencias a que dichas aventuras le conducen. De sen-
37. ¿Cómo es Dulcinea? Localiza en estos capítulos iniciales
lace: capítulo VI, en el que el escrutinio de libros constituye
los pasajes que muestran la imagen que don Quijote tiene
toda una reflexión metaliteraria que explica el sentido que
de ella. Cervantes da a su relato.
No existe, es solo un ser imaginario, una invención de don Qui-
jote. Por ello es una dama idealizada, típica doncella del amor Género
cortés. Y fue, a lo que se cree, que en un lugar cerca del suyo 42. Indica qué subgéneros narrativos aparecen en el Quijote
había una moza labradora de muy buen parecer, de quien él de 1605.
un tiempo anduvo enamorado (aunque, según se entiende, La novela pastoril (episodio de la pastora Marcela), la sen-
ella jamás lo supo ni se dio cata dello). Llamábase Aldonza timental (narración de El curioso impertinente), la novela
Lorenzo, y a esta le pareció bien darle título de señora de bizantina y la morisca (relatos de aventuras, como el del cau-
sus pensamientos; y buscándole nombre que no desdijese tivo en la venta).
mucho del suyo, y que tirase y se encaminase al de princesa
gran señora, vino a llamarla Dulcinea del Toboso, porque era 43. Teniendo en cuenta dicho tratamiento del narrador, expli-
natural del Toboso. ca el sentido que adquiere el siguiente fragmento del capí-
En este tiempo solicitó don Quijote a un labrador vecino suyo, tulo I:
hombre de bien (si es que este título se puede dar al que es Quieren decir que tenía el sobrenombre de Quijada, o Quesa-
pobre), pero de muy poca sal en la mollera. En resolución, tan- da, que en esto hay alguna diferencia entre los autores que
to le dijo, tanto le persuadió y prometió, que el pobre villano deste caso escriben; aunque por conjeturas verosímiles se deja
se determinó de salirse con él y servirle de escudero. Decía- entender que se llamaba Quejana. Pero esto importa poco a
le, entre otras cosas, don Quijote que se dispusiese a ir con nuestro cuento; basta que en la narración dél no se salga un
él de buena gana, porque tal vez le podía suceder aventura punto de la verdad.
que ganase, en quítame allá esas pajas, alguna ínsula, y le El narrador se limita a recoger la información que encuentra
dejase a él por gobernador della. Con estas promesas y otras en las crónicas, incluso cuando es imprecisa (como ocurre

Lengua castellana y Literatura. 3.º ESO. Solucionario 131


GUÍAS DE LECTURA

con el nombre del hidalgo). Las inexactitudes no le preocupan la linda Dulcinea del Toboso, por quien yo he hecho, hago y
porque lo que interesa es la veracidad de los hechos narrados. haré los más famosos hechos de caballerías que se han visto,
44. Cita otros momentos en los seis primeros capítulos en vean ni verán en el mundo.
que se pueda hablar de alusiones semejantes. 48. Busca ahora citas donde se aprecie el realismo de los
Al cabo se vino a llamar don Quijote; de donde, como queda personajes secundarios.
dicho, tomaron ocasión los autores desta tan verdadera his- En el capítulo IV: ¿Irme yo con él —dijo el muchacho— más?
toria que sin duda se debía llamar Quijada, y no Quesada, ¡Mal año! No, señor, ni por pienso; ¿porque en viéndose solo
como otros quisieron decir; Y fue, a lo que se cree, que en un me desuelle como a un San Bartolomé?
lugar cerca del suyo había una moza labradora de muy buen En el capítulo V: Señor Quijana (que así se debía de llamar
parecer, de quien él un tiempo anduvo enamorado (aunque, cuando él tenía juicio y no había pasado de hidalgo sosegado
según se entiende, ella jamás lo supo): obsérvese cómo en esta a caballero andante), ¿quién ha puesto a vuestra merced desta
alusión el narrador afirma que ni siquiera el origen de la inven- suerte?; Mire vuestra merced, señor, pecador de mí, que yo no
ción de Dulcinea parece ser del todo seguro para los cronistas. soy don Rodrigo de Narváez, ni el marqués de Mantua, sino
45. Investiga y explica quién es ese cronista que aparecerá a Pedro Alonso, su vecino; ni vuestra merced es Valdovinos, ni
partir del capítulo IX. Abindarráez, sino el honrado hidalgo del señor Quijana; ¿Qué
le parece a vuestra merced, señor licenciado Pero Pérez (que
Cide Hamete Benengeli.
así se llamaba el cura), de la desgracia de mi señor? Tres días
46. En todo momento, se persigue transmitir al lector la idea ha que no parecen él, ni el rocín, ni la adarga, ni la lanza, ni
de que se narra una historia real. Localiza pasajes en los las armas. ¡Desventurada de mí!, que me doy a entender, y
que se aprecie este objetivo. así es ello la verdad como nací para morir, que estos malditos
Puede observarse en todas las alusiones a los cronistas, o en el libros de caballerías que él tiene y suele leer tan de ordinario
uso de expresiones como según se cree, según se entiende, que le han vuelto el juicio; que ahora me acuerdo haberle oído
dan a entender que el narrador está recopilando unos hechos decir muchas veces, hablando entre sí, que quería hacerse
reales de los que hablan otros. A esta impresión contribuye caballero andante; ¡Mirá, en hora maza —dijo a este punto el
definitivamente el realismo del relato, con sus topónimos y ama—, si me decía a mí bien mi corazón del pie que cojeaba
tópicos totalmente reconocibles para el lector de la época. mi señor! Suba vuestra merced en buen hora, que sin que
venga esa hurgada le sabremos aquí curar; ¡Ta, ta! —dijo el
Estilo cura— ¿Jayanes hay en la danza? Para mi santiguada que
47. Localiza pasajes donde se observe el estilo literario y alti- yo los queme mañana antes que llegue la noche.
sonante del protagonista. 49. Localiza y explica el uso de la ironía por parte del narrador.
Hay muchos. Entre otros, pueden citarse los siguientes: La ironía está presente ya desde la primera línea: En un lugar
En el capítulo II: ¡Oh princesa Dulcinea, señora deste cautivo de La Mancha, de cuyo nombre no quiero acordarme. En todo
corazón! Mucho agravio me habedes fecho en despedirme momento el narrador da la impresión de estar divirtiéndose
y reprocharme con el riguroso afincamiento de mandarme con su propio relato. Por ejemplo, obsérvese el pasaje en que
no parecer ante la vuestra fermosura. Plégaos, señora, de cuenta cómo don Quijote arregla sus viejas armas y se prepara
membraros desde vuestro sujeto corazón, que tantas cuitas una celada de encaje con cartón y unas barras de hierro; o el
por vuestro amor padece! momento en que sale para correr sus aventuras y va pensando
en cómo los cronistas contarán algún día su historia (con estos
En el capítulo III: No esperaba yo menos de la gran magnifi-
iba ensartando otros disparates); o cuando ha sido armado
cencia vuestra, señor mío —respondió don Quijote—; y así,
caballero y sale de la venta (el gozo le reventaba por las cin-
os digo que el don que os he pedido y de vuestra liberalidad
chas del caballo).
me ha sido otorgado es que mañana en aquel día me habéis de
armar caballero. Puede comentarse también la propia ironía de los persona-
jes. Por ejemplo, las palabras de Juan Haldudo cuando se ha
—¡Oh, tú, quienquiera que seas, atrevido caballero, que llegas
marchado don Quijote y el labrador vuelve a encontrarse a
a tocar las armas del más valeroso andante que jamás se ciñó
solas con su criado: Llamad, señor Andrés, ahora —decía el
espada! Mira lo que haces, y no las toques, si no quieres dejar
labrador— al desfacedor de agravios.
la vida en pago de tu atrevimiento.
También contribuyen a la ironía los frecuentes dobles sentidos
¡Oh señora de la fermosura, esfuerzo y vigor del debilitado cora- que se aprecian en algunas palabras. Por ejemplo, de Roci-
zón mío! Ahora es tiempo que vuelvas los ojos de tu grandeza a nante se dice que tenía más cuartos que un real, dándole a
este tu cautivo caballero, que tamaña aventura está atendiendo. la palabra cuartos a la vez el significado de moneda y el de
En el capítulo IV: Por el sol que nos alumbra que estoy por enfermedad de los caballos. O cuando se habla de las dis-
pasaros de parte a parte con esta lanza. Pagadle luego sin traídas mozas de la venta, que don Quijote toma por nobles
más réplica; si no, por el Dios que nos rige que os concluya doncellas, se alude a su ligereza moral.
y aniquile en este punto. Por último, hay que comentar el doble plano en que se articula
Del sahumerio os hago gracia —dijo don Quijote—; dádselos siempre el relato, de manera que se presenta el contraste entre
en reales, que con eso me contento; y mirad que lo cumpláis la visión del mundo de don Quijote y su correlato humorístico
como lo habéis jurado; si no, por el mismo juramento os juro con los elementos de la realidad tosca y obvia. El narrador
de volver a buscaros y castigaros; y que os tengo que hallar incide con marcado empeño en ese contraste, lo cual supone
aunque os escondáis más que una lagartija. Y si queréis la clave más importante para la ironía.
saber quién os manda esto, para quedar con más veras obli-
gado a cumplirlos, sabed que yo soy el valeroso don Quijote El texto como acto comunicativo
de la Mancha, el desfacedor de agravios y sinrazones, y a 50. Localiza en los seis primeros capítulos pasajes en los que
Dios quedad, y no se os parta de las mientes lo prometido y se pueda apreciar el conocimiento que tienen los perso-
jurado, so pena de la pena pronunciada. najes sobre las costumbres de los caballeros andantes.
En el capítulo V: —Sepa vuestra merced, señor don Rodrigo El ventero es un buen ejemplo. Obsérvese el pasaje en que dice
de Narváez, que esta hermosa Jarifa que he dicho es ahora a don Quijote que él había ejercido la caballería andante en

132 Lengua castellana y Literatura. 3.º ESO. Solucionario


GUÍAS DE LECTURA

su juventud, y luego explica al protagonista los recursos que elevadas y de las populares, con estilos lingüísticos que se
puede poner en práctica para no descuidar las cuestiones mate- ajustan a la condición de cada uno; división de la obra en tres
riales (dinero, ropa limpia, etc.). Y, por supuesto, la ceremonia actos o jornadas; ruptura de la regla de las tres unidades.
en que don Quijote es armado caballero. Estos pasajes están 2. ¿Cómo relacionas el título de la obra y su tema filosófico
llenos de alusiones a elementos y motivos temáticos que solo
con la concepción barroca de la vida?
pueden ser citados por quien conoce los lugares comunes de la
literatura de aventuras y caballeresca. También las interven- Se relaciona con el pesimismo de la época, que concibe la vida
ciones del cura y del barbero (sobre todo del primero) demues- como algo inconsistente.
tran ese conocimiento de las costumbres de la caballería. 3. Investiga sobre el argumento de la obra y responde. ¿Por
Temas para la reflexión qué Rosaura ha emprendido ese viaje? ¿Por qué Segis-
mundo vive encerrado? ¿Quién heredará el trono de
51. ¿Qué géneros literarios o cinematográficos podríamos Basilio?
parodiar si hoy día quisiésemos contar una historia como
Porque busca a Astolfo, que la ha deshonrado, para reparar
la de don Quijote? ¿Quiénes serían los protagonistas?
su honra.
Podríamos parodiar las películas americanas producidas
para la televisión con personajes y argumentos tópicos Intención
(a veces lacrimógenos, a veces de terror fácil, a menudo con 4. Explica cómo se presenta el tema del libre albedrío en
historias sentimentales muy manidas) cuyo devenir es previ-
el monólogo de Segismundo que aparece en la escena
sible desde el primer momento. También podríamos parodiar
las interminables sagas televisivas, que van complicando de
que has leído.
forma inverosímil las tramas de los personajes a lo largo En este monólogo se presenta el dolor de Segismundo al verse
de muchísimos capítulos, y que han llegado a constituir el en inferioridad con respecto a las criaturas de la naturaleza,
género que popularmente se conoce como «culebrón». Los pro- que son libres pese a no tener alma ni albedrío, al contra-
tagonistas de las parodias contemporáneas serían los seres rio que él.
menos idealizados de nuestra sociedad: por ejemplo, los adoles- Sueña el rey que es rey, y vive
centes feos y marginados; las amas de casa que no participan con este engaño mandando,
de la igualdad de oportunidades; los personajes fracasados en disponiendo y gobernando;
el trabajo y en los negocios, etcétera. y este aplauso, que recibe
52. El Quijote es consecuencia de un momento en el que se prestado, en el viento escribe
consideran agotados ciertos valores y esquemas no solo y en cenizas le convierte
artísticos, sino también sociales. ¿Crees que el contexto la muerte (¡desdicha fuerte!):
¡que hay quien intente reinar
que vivimos en el siglo XXI propiciaría el relato de una his-
viendo que ha de despertar
toria así? Busca ejemplos de ello en la literatura contem- en el sueño de la muerte!
poránea o en el cine. Sueña el rico en su riqueza,
Vivimos también una época de crisis, de transformación de que más cuidados le ofrece;
los valores. En consecuencia, también los arquetipos y los sueña el pobre que padece
tópicos de la ficción quedan cuestionados. Falta dar el paso su miseria y su pobreza;
de cuestionar la corrección política, esquema de pensamien- sueña el que medrar empieza,
to que se impone en nuestros días, tanto en lo social como sueña el que afana y pretende,
en lo artístico, y que no puede obviarse fácilmente sin sufrir sueña el que agravia y ofende,
duras críticas o provocar fuertes polémicas. En la literatura y y en el mundo, en conclusión,
en el cine contemporáneos a veces encontramos historias que todos sueñan lo que son,
les dan la vuelta a los arquetipos y presentan protagonistas aunque ninguno lo entiende.
antiheroicos. Las novelas de Luis Landero son un buen ejem- Yo sueño que estoy aquí,
plo (Juegos de la edad tardía, El mágico aprendiz, además destas prisiones cargado;
Landero rinde un claro homenaje a Cervantes en buena parte y soñé que en otro estado
de su obra). En el cine, se llevan cuestionando muchos de los más lisonjero me vi.
modelos desde hace años. Sin perdón (Clint Eastwood, 1992) ¿Qué es la vida? Un frenesí.
presentaba al típico forajido de los westerns como un persona- ¿Qué es la vida? Una ilusión,
je ya mayor, venido a menos, convertido en padre de familia y una sombra, una ficción,
granjero. En la actualidad, películas como Birdman (Alejan- y el mayor bien es pequeño;
dro González Iñárritu, 2014) nos ofrecen también un ejemplo que toda la vida es sueño,
de antihéroe cuyas andanzas se desarrollan en una sociedad y los sueños, sueños son.
sumida en la crisis.
5. ¿Cuál es el sentido que tiene la existencia humana para
Segismundo? ¿Cómo relacionas estas palabras con la
6 Pedro Calderón de la Barca. mentalidad barroca?
La vida es sueño. Escena II de la jornada I: No tiene más sentido que el de un vano sueño. Estas palabras
se relacionan con la idea barroca de la inconsistencia de la
Rosaura conoce a Segismundo existencia humana.

Localización Contenido
1. ¿Cuáles eran las características generales del teatro ba- 6. Antes de escuchar el monólogo de Segismundo, Rosaura
rroco? describe la torre. ¿Qué dice acerca de ella?
Mezcla de elementos trágicos y cómicos; utilización del ver- Que es una prisión oscura y tenebrosa, sepultura de un cadá-
so; polimetría; personajes provenientes de las clases sociales ver vivo. Que su breve luz la hace más tenebrosa todavía.

Lengua castellana y Literatura. 3.º ESO. Solucionario 133


GUÍAS DE LECTURA

7. Tras este momento, se inicia el monólogo del protago- 16. Dicha historia puedes relacionarla con uno de los cuentos
nista, quien dice que desea saber algo. ¿Qué es eso que de El conde Lucanor: «De lo que aconteció a un hombre
desea saber? que por pobreza y mengua de otra vianda comía altramu-
Qué delito ha cometido contra los cielos con el mero hecho ces». Investiga sobre el contenido de ese cuento y rela-
de nacer, pues no ha podido hacer ninguna otra cosa, ya que ciónalo con el relato que hace Rosaura.
lleva toda su vida encerrado. En ese cuento se habla de un hombre que solo tenía altramu-
8. Segismundo entiende que su existencia es un castigo. ces para comer y se lamentaba por ello, hasta que descubrió
¿Por qué cree que ha sido castigado? ¿Piensa que es jus- a otro que iba comiéndose las cáscaras de los altramuces que
ta su situación? él tiraba. Es el mismo planteamiento que tiene la narración
de Rosaura.
Su nacimiento debe de ser un delito puesto que recibe el cas-
tigo de esa dura prisión. 17. Al final de la escena, Rosaura se dispone a narrar su his-
toria, pues dice que esta puede tener cierta utilidad para
9. El protagonista comienza una exposición en la que va pre-
Segismundo. ¿Cuál es esa utilidad?
sentando diversos seres vivos y objetos con los que se
compara. Indica de qué seres se trata y qué comparación Ha comprendido que es consuelo de un desdichado ver que
existe otro que es más desdichado aún. Ahora piensa que sus
establece Segismundo en cada caso.
penas tal vez puedan aliviar en parte a Segismundo de las
El ave, que es bella y libre, pero no tiene alma. El bruto, que suyas.
tiene el instinto de la crueldad. El pez, que no tiene albedrío.
El arroyo, que tiene el campo abierto para su carrera. Segis-
18. Teniendo en cuenta el argumento global de La vida es
mundo, en cambio, tiene alma, buen instinto, albedrío, pero sueño, ¿en cuál de estas dos posturas crees que se sitúa
no tiene libertad. Calderón? ¿El destino del ser humano está decidido de
10. Esa comparación suscita invariablemente en Segismundo antemano por las estrellas o puede este vencer a cualquier
una pregunta: explica cuál es. designio con su prudencia?
Calderón apuesta por la importancia de la prudencia. El hombre
Qué leyes o argumentos confieren a esos seres más libertad
puede decidir su propio destino al elegir entre el bien y el mal.
que a los hombres.
19. Los conflictos sentimentales que se han planteado en la
11. Al terminar su monólogo, el joven se da cuenta de que
trama quedan resueltos al final, sin transgredir el orden
alguien le ha escuchado. ¿Cuál es su primer impulso
social de la época. Investiga y aclara cómo se resuelven
entonces? ¿Cómo relacionarías ese impulso con la situa-
dichos conflictos en el desenlace.
ción en la que vive el personaje?
Los conflictos sentimentales venían dados por las diferencias
El impulso de matar al que le ha oído. Es un impulso lógico
sociales entre Astolfo y Rosaura: él es un príncipe, y no se siente
en un hombre que siempre ha vivido privado de relación con
obligado a cumplir su palabra con ella, porque ella no conoce
otros seres humanos.
padre. Al final se descubre que Rosaura es hija de Clotaldo y
12. La presencia de Rosaura modifica el impulso de Segis- entonces Astolfo se casa con ella. La infanta Estrella no queda
mundo. ¿Qué sentimientos se despiertan en este? desairada, pues se casa con Segismundo, que es rey, y en con-
Se enternece y se sobrecoge. Siente fascinación al mirarla. secuencia la unión entre ambos entra de lleno en el orden social.
13. Para expresar esos sentimientos, Segismundo alude antes Personajes
a una serie de circunstancias que podrían obstaculizar lo 20. ¿A qué clases sociales pertenecen los diferentes perso-
que experimenta, encadenando una larga hilera de oracio- najes de este drama?
nes concesivas («aunque yo aquí tan poco del mundo sé…»; A la nobleza, exceptuando a Clarín.
«aunque nunca vi ni hablé…»; «Y aunque en desdichas tan
graves…»). ¿Cuáles son esas circunstancias? Pese a ellas, 21. ¿Crees que Rosaura podría responder al tipo de la dama
¿qué efecto ha conseguido provocar Rosaura en él? ¿Tie- habitual en el teatro del XVII? ¿Cómo la caracterizarías tú,
nen algo de contradictorio estos nuevos sentimientos? teniendo en cuenta la acción con que se inicia la obra?
No es lo habitual en la época que una mujer se convierta en
No sabe nada del mundo; siempre ha vivido encerrado en esa
artífice de su propio destino. Sin embargo, tuvo éxito en el
torre como un muerto viviente; nunca ha tenido más compañía
siglo XVII cierto modelo de dama hermosa y decidida, al que
que la de un solo un ser humano, que es quien le trae noticias del
responde Rosaura.
mundo; las aves y las bestias han sido sus maestros. Rosaura le
provoca fascinación y admiración y mientras más la mira, más 22. Investiga sobre los rasgos que definen al tipo del gracioso
desea mirarla. Sí son contradictorios sus sentimientos: Viendo y relaciónalos con el personaje de Clarín.
que el ver me da muerte / estoy muriendo por ver. Se convierte en contrapunto de los momentos de tensión y
14. También Rosaura explica lo que siente al escuchar el dramatismo, introduce la comicidad. Es un personaje realista
lamento de Segismundo. ¿Qué dice? y conformista, que no está dispuesto a dar la vida por nadie,
sino a guardarla.
Que ella, siendo muy desdichada, ha conocido en Segismundo
a alguien mucho más desventurado que ella. 23. Busca también información sobre el resto de los perso-
15. En dicha intervención de Rosaura, esta alude a una histo- najes de la obra y relaciónalos con los diversos tipos que
ria popular. ¿Cuál es? ¿Qué pretende transmitir Rosaura hemos mencionado.
al contarla? Basilio se identifica con el poderoso, Astolfo con el galán,
Estrella responde al arquetipo de la dama, Clotaldo es el viejo.
Habla de un hombre que se lamentaba de su pobreza porque
solo tenía para comer las yerbas que cogía. Pero al mirar atrás 24. A tu juicio, ¿podríamos caracterizar a Segismundo como
vio a otro que comía lo que él iba arrojando. Pretende trans- un personaje simbólico? Justifica tu respuesta.
mitir que, por muy desafortunado que se sienta uno, siempre Sí, pues encierra el conflicto que preocupa a Calderón: la pre-
hay otro que lo es más. destinación o el libre albedrío.

134 Lengua castellana y Literatura. 3.º ESO. Solucionario


GUÍAS DE LECTURA

Estructura entre los personajes, hasta casi el final de la escena, es decir,


entre los versos 173 y 272. En el 273 comienza el esquema
25. Investiga y explica el argumento de cada una de las jor- métrico del romance (versos octosílabos con rima asonante
nadas de La vida es sueño. en los pares), que continuará en la escena siguiente.
Jornada primera: Rosaura, que ha emprendido un viaje para
recuperar su honor, conoce a Segismundo, que vive encerrado Género
y no sabe por qué. Clotaldo reconoce a Rosaura y comprende 30. A partir de la lectura de la escena II, justifica la adscrip-
que es su hija, pero mantiene aún el secreto. El rey Basilio ción del texto al género dramático localizando en él sus
revela la existencia de su hijo, Segismundo, a quien encerró elementos.
para evitar el augurio que dictaba que el heredero habría de
ser un tirano. Pero se siente culpable y decide darle una opor- La acción se presenta íntegramente a través del diálogo,
tunidad, pensando que tal vez los augurios fuesen erróneos. predomina la función apelativa del lenguaje, se introduce el
Si la prueba sale bien, Segismundo heredará su trono y si no, monólogo, se emplean las acotaciones.
le sucederá Astolfo, que se casará con Estrella. 31. ¿Qué elementos de la tragedia y de la comedia se mez-
Jornada segunda: Segismundo, dormido, es llevado a palacio. clan aquí?
Allí despierta y, desconcertado, se conduce como un salva- De la tragedia, el hecho de que los personajes sean reyes, prín-
je: intenta forzar a Rosaura (que ha entrado en palacio como cipes o nobles, la gravedad del tema filosófico. De la come-
doncella de Estrella, ocultando su identidad), se enfrenta con dia, los elementos populares que vienen introducidos por el
Clotaldo y Astolfo, agrede a un criado. De esta conducta el rey gracioso, Clarín.
Basilio concluye que los augurios eran ciertos y ordena dormir
de nuevo a Segismundo y devolverlo a la torre. En cuanto a la 32. ¿Cómo se enfocan las tres unidades en La vida es sueño?
acción secundaria, Astolfo corteja a Estrella, y se descubre No se respetan. Hay dos espacios distintos: la torre y el pala-
la identidad de Rosaura. Por su parte, Segismundo despierta cio. Hay dos acciones: la central (conflicto de Segismundo) y
de nuevo en la torre y piensa que todo lo que ha vivido ha una secundaria, que se superpone a ella (historia de Rosau-
sido un sueño. ra y triángulo sentimental entre ella, Astolfo y Estrella). En
Jornada tercera: el pueblo, al saber que existe un príncipe cuanto al tiempo, hay saltos temporales, aunque no abarcan
heredero, se revuelve y libera a Segismundo. Este se enfren- un segmento amplio.
ta a su padre y vence, pero hace gala de prudencia y honor al 33. A Calderón le interesaban mucho la escenografía y la tra-
decirle a su padre que está dispuesto a aceptar que lo mate, moya. ¿Qué elementos escenográficos y especialmente
puesto que se ha levantado contra él. Basilio comprende espectaculares citarías en el comienzo de La vida es sue-
entonces que su hijo es sensato y sabe comportarse como ño? (Recuerda la entrada de Rosaura en la acción).
un verdadero príncipe. Segismundo establece la unión entre
Astolfo y Rosaura y, ante las dudas de Astolfo, Clotaldo desve- La obra tiene un arranque espectacular y de complicada pues-
la entonces ante todos que la joven es su hija. El protagonista ta en escena: Rosaura va bajando a caballo desde lo alto de
se promete en matrimonio con Estrella. La obra termina con un monte.
la admiración de todos por la conducta de Segismundo, que Estilo
recupera sus derechos.
34. En el monólogo en que Segismundo expresa los senti-
26. ¿Constituye la primera jornada una presentación de los mientos nuevos que le inspira la presencia de Rosaura
temas, conflictos y personajes de la obra? ¿Por qué? (versos 190-242) tenemos un buen ejemplo del estilo
Sí, se presentan los conflictos de los personajes, el pasado de calderoniano. Localiza y comenta en dicho monólogo la
Basilio que ha provocado la situación de Segismundo. Quedan presencia de los recursos que hemos mencionado.
planteadas las dos acciones de la obra.
Es muy visible la estructura paralelística en ese fragmento,
27. Localiza en ese monólogo las mencionadas partes. donde se suceden las oraciones concesivas: aunque yo aquí
En las dos primeras estrofas se presenta el conflicto del per- tan poco del mundo sé; aunque desde que nací solo advier-
sonaje y su lamento. La argumentación se desarrolla entre las to…; aunque nunca vi ni hablé…; Y aunque en desdichas tan
estrofas tres y seis. La séptima constituye la recapitulación graves… A su vez, en ese largo periodo sintáctico hay otros
y la conclusión. paralelismos: siendo un esqueleto vivo / siendo un animado
muerto; la pasión a mis enojos / la suspensión a mis ojos,
28. Explica cómo se produce la recapitulación final y a qué
/ la admiración al oído. Además, encontramos un particular
conclusión llega Segismundo en ella. uso del paralelismo en el retruécano: soy un hombre de las
Segismundo recapitula mencionando todos los seres en los que fieras / y una fiera de los hombres.
ha ido deteniéndose anteriormente, aunque en orden inverso a
Se juega con el sentido de las palabras y con su reiteración:
como los ha mencionado. Los recoge en los dos últimos versos
viendo que el ver me da muerte / estoy muriendo por ver.
(a un cristal / a un pez, a un bruto y a un ave). La conclusión,
en realidad, es la pregunta sin respuesta del personaje: en La paradoja puede ejemplificarse en esqueleto vivo y animado
nombre de qué ley todos esos seres gozan del privilegio de la muerto.
libertad, que se les niega a los hombres. 35. ¿Sabes lo que es un retruécano? Infórmate sobre este
29. ¿Qué tipo de versos se emplean en la escena II de la segun- recurso y explica su uso en los versos «soy un hombre de
da jornada? ¿Cuál es la rima? las fieras / y una fiera de los hombres». ¿Qué sentido tienen
En el primer diálogo (entre Clarín y Rosaura, vv. 80-101) se estas palabras de Segismundo?
alternan versos de siete y de once sílabas, que se distribu- El retruécano es un paralelismo cruzado, en el que además se
yen en pareados rimando en consonante. En el monólogo repiten las palabras, como en el ejemplo mencionado. La afir-
de Segismundo (vv. 102-172), los versos son octosílabos, se mación de Segismundo tiene el sentido de que no pertenece a
agrupan en décimas (siete en total) cuyo esquema métrico es ninguno de los dos mundos. Vive como las fieras, pero es un
abbaaccddc. Continúan las décimas en el posterior diálogo hombre. No tiene sitio en ninguna parte.

Lengua castellana y Literatura. 3.º ESO. Solucionario 135


GUÍAS DE LECTURA

36. ¿Qué es la paronomasia? Explícala en el verso: «fuera más Ese planteamiento habría resultado provocador y no habría
que muerte fiera». sido comprendido por la mayoría en el siglo XVII. Hoy día, por
el contrario, lo que resultaría inadmisible sería que un hom-
Es la repetición de palabras semejantes en cuanto a su forma
bre dejase a su prometida por el mero hecho de que no tie-
y su fonética. En el citado ejemplo existe esa semejanza entre
ne padre conocido, para prometerse en matrimonio con una
fuera y fiera, y además se reitera el diptongo —ue— en fuera
mujer de su posición social.
y muerte.
43. Reflexiona: teniendo en cuenta lo particular que es el géne-
37. El verbo morir se ha utilizado desde siempre en la poe-
ro teatral como acto comunicativo, ¿por qué crees que Cal-
sía con muy diversas connotaciones, especialmente en la
derón inserta una trama de este tipo en medio de un drama
temática amorosa. Explica el juego de significados con
tan grave y filosófico como La vida es sueño?
que se utiliza este verbo y el sustantivo muerte en los
Para que el argumento tenga más interés para el público.
versos 229-242.
44. Al hablar del estilo calderoniano, hemos mencionado
Segismundo, mediante la metáfora de la hidropesía, habla
de cómo sus ojos desean beber esa belleza que, sin embargo, el gusto de este escritor por la artificiosidad ingeniosa
le hace enfermar (de amor, de fascinación). El verbo morir se del lenguaje. ¿Crees que este rasgo contribuye a la bue-
emplea con el sentido del amor cortés: sufrir por lo inalcan- na acogida de la obra entre el público o que la dificulta?
zable que, a la vez, provoca deleite. Con la palabra muerte se Razona tu respuesta.
refiere al afán que experimenta ante la contemplación de la Seguramente contribuye a la buena acogida, porque al público
dama y al dolor que provoca su ausencia, un dolor que sería le deslumbran los usos ingeniosos y los hábiles giros del estilo
mucho peor que la muerte, ahora en sentido literal. calderoniano. El estilo de Calderón, por artificioso que sea,
no es enrevesado ni hermético.
38. Comenta los elementos culteranos que aparecen en la
intervención de Rosaura al comienzo de la escena (versos 45. También hemos hablado del gusto de Calderón por los
85-90). monólogos. ¿Qué función comunicativa tienen estos en
Obsérvense los adjetivos (explicativos en su mayoría): breve,
el conjunto de una obra de teatro? ¿Y en La vida es sueño,
caduca, pálida, trémulos, oscura, que inciden en el efecto concretamente?
sensorial de la luz que se describe. Por otro lado, el enuncia- Transmitirnos los pensamientos y sentimientos de un perso-
do que abarcan esos versos está estructurado en hipérbaton. naje, o su historia y sus circunstancias. En La vida es sueño,
los monólogos tienen varias funciones: las dos que acabamos
39. Localiza los hipérbatos que aparecen en el fragmento que
de mencionar, y también la de exponer reflexiones didácticas
has leído. sobre las ideas de la obra.
Además del que se produce en los versos mencionados en la
actividad anterior, hay hipérbatos en los versos 94, 97, 98, 99, Temas para la reflexión
103, 106, 135, 137-138, 157-158, 168, 173-174, 183-185, 200, 205, 46. En la experiencia del siglo XXI y en la sociedad en la que
206-207, 214-218, 224, 226, 250-251, 263-264, 268, 271-272. vivimos, ¿cómo responderíamos a la gran pregunta sobre el
40. Indica con qué reiteración sella el personaje cada una de destino del ser humano? Es decir, ¿somos libres para elegir
esas razones. Explica qué figura retórica encontramos en entre el bien y el mal o estamos predestinados?
esa reiteración. En el siglo XXI prevalece la idea de la libertad para elegir entre
el bien y el mal.
¿Y teniendo yo más alma, / tengo menos libertad? […] ¿Y yo,
con mejor instinto, / tengo menos libertad?) […] ¿Y yo, con 47. En la obra de Calderón, el mensaje final es que la prudencia
más albedrío, / tengo menos libertad? […] ¿Y teniendo debe imponerse a los instintos y a la pasión. ¿Compartes
yo más vida / tengo menos libertad? Son interrogaciones esta idea? Redacta un texto expositivo sobre este particu-
retóricas. lar, en el que expongas tus opiniones.
41. ¿Qué elementos lingüísticos dan al monólogo el tono de Respuesta libre.
lamento? 48. ¿Se corresponde esa caracterización del personaje con la
Las exclamaciones, la interjección ¡ay!, la interrogación retó- conducta ideal de la mujer en el siglo XVII? ¿Y en nuestra
rica, el léxico que connota compasión (mísero, infelice, des- época? ¿Supone alguna novedad que una mujer se con-
velos, castigo). duzca de esta forma?
El texto como acto comunicativo No era lo habitual en el siglo XVII . Normalmente, eran los
padres, maridos o hermanos los que reparaban el honor de
42. ¿Crees que el espectador de aquella época habría aco- las mujeres cuando había alguna afrenta, por considerarlo
gido una obra en la que Astolfo se casase con una joven parte de su propio honor. En nuestra época, por el contrario,
que no perteneciese a la nobleza? ¿Cómo contrasta esto no es ninguna novedad que una mujer resuelva por sí misma
con la mentalidad de hoy? sus propios problemas.

136 Lengua castellana y Literatura. 3.º ESO. Solucionario

También podría gustarte